Π­Π”Π‘ ΠΈΠ½Π΄ΡƒΠΊΡ†ΠΈΠΈ. ΠŸΡ€ΠΈΠΌΠ΅Ρ€Ρ‹ Ρ€Π΅ΡˆΠ΅Π½ΠΈΡ Π·Π°Π΄Π°Ρ‡ ΠΏΠΎ Ρ„ΠΈΠ·ΠΈΠΊΠ΅. 10-11 класс

Π­Π”Π‘ ΠΈΠ½Π΄ΡƒΠΊΡ†ΠΈΠΈ. ΠŸΡ€ΠΈΠΌΠ΅Ρ€Ρ‹ Ρ€Π΅ΡˆΠ΅Π½ΠΈΡ Π·Π°Π΄Π°Ρ‡ ΠΏΠΎ Ρ„ΠΈΠ·ΠΈΠΊΠ΅. 10-11 класс

Π—Π°Π΄Π°Ρ‡ΠΈ ΠΏΠΎ Ρ„ΠΈΠ·ΠΈΠΊΠ΅ — это просто!

НС Π·Π°Π±Ρ‹Π²Π°Π΅ΠΌ, Ρ‡Ρ‚ΠΎ Ρ€Π΅ΡˆΠ°Ρ‚ΡŒ Π·Π°Π΄Π°Ρ‡ΠΈ Π½Π°Π΄ΠΎ всСгда Π² систСмС БИ!


А Ρ‚Π΅ΠΏΠ΅Ρ€ΡŒ ΠΊ Π·Π°Π΄Π°Ρ‡Π°ΠΌ!

Π­Π»Π΅ΠΌΠ΅Π½Ρ‚Π°Ρ€Π½Ρ‹Π΅ Π·Π°Π΄Π°Ρ‡ΠΈ ΠΈΠ· курса школьной Ρ„ΠΈΠ·ΠΈΠΊΠΈ Π½Π° вычислСниС Π­Π”Π‘ ΠΈΠ½Π΄ΡƒΠΊΡ†ΠΈΠΈ.

Π—Π°Π΄Π°Ρ‡Π° 1

Π—Π° врСмя 5 мс Π² солСноидС, содСрТащСм 500 Π²ΠΈΡ‚ΠΊΠΎΠ² ΠΏΡ€ΠΎΠ²ΠΎΠ΄Π°, ΠΌΠ°Π³Π½ΠΈΡ‚Π½Ρ‹ΠΉ ΠΏΠΎΡ‚ΠΎΠΊ Ρ€Π°Π²Π½ΠΎΠΌΠ΅Ρ€Π½ΠΎ ΡƒΠ±Ρ‹Π²Π°Π΅Ρ‚ ΠΎΡ‚ 7 ΠΌΠ’Π± Π΄ΠΎ 3 ΠΌΠ’Π±.
НайдитС Π­Π”Π‘ ΠΈΠ½Π΄ΡƒΠΊΡ†ΠΈΠΈ Π² солСноидС.


Π—Π°Π΄Π°Ρ‡Π° 2

Какой ΠΌΠ°Π³Π½ΠΈΡ‚Π½Ρ‹ΠΉ ΠΏΠΎΡ‚ΠΎΠΊ ΠΏΡ€ΠΎΠ½ΠΈΠ·Ρ‹Π²Π°Π΅Ρ‚ ΠΊΠ°ΠΆΠ΄Ρ‹ΠΉ Π²ΠΈΡ‚ΠΎΠΊ ΠΊΠ°Ρ‚ΡƒΡˆΠΊΠΈ, ΠΈΠΌΠ΅ΡŽΡ‰Π΅ΠΉ 1000 Π²ΠΈΡ‚ΠΊΠΎΠ², Ссли ΠΏΡ€ΠΈ Ρ€Π°Π²Π½ΠΎΠΌΠ΅Ρ€Π½ΠΎΠΌ исчСзновСнии ΠΌΠ°Π³Π½ΠΈΡ‚Π½ΠΎΠ³ΠΎ поля Π² Ρ‚Π΅Ρ‡Π΅Π½ΠΈΠ΅ 0,1 с Π² ΠΊΠ°Ρ‚ΡƒΡˆΠΊΠ΅ индуцируСтся Π­Π”Π‘ равная 10 Π’ ?

Π—Π°Π΄Π°Ρ‡Π° 3

Π’ΠΈΡ‚ΠΎΠΊ ΠΏΡ€ΠΎΠ²ΠΎΠ΄Π½ΠΈΠΊΠ°Β Β  ΠΏΠ»ΠΎΡ‰Π°Π΄ΡŒΡŽ 2 см2 располоТСн пСрпСндикулярно Π²Π΅ΠΊΡ‚ΠΎΡ€Ρƒ ΠΌΠ°Π³Π½ΠΈΡ‚Π½ΠΎΠΉ ΠΈΠ½Π΄ΡƒΠΊΡ†ΠΈΠΈ.
Π§Π΅ΠΌΡƒ Ρ€Π°Π²Π½Π° Π­Π”Π‘ ΠΈΠ½Π΄ΡƒΠΊΡ†ΠΈΠΈ Π² Π²ΠΈΡ‚ΠΊΠ΅, Ссли Π·Π° врСмя 0,05 сСкунд магнитная индукция Ρ€Π°Π²Π½ΠΎΠΌΠ΅Ρ€Π½ΠΎ ΡƒΠ±Ρ‹Π²Π°Π΅Ρ‚ с 0,5 Π’Π» Π΄ΠΎ 0,1 Π’Π»?


Π—Π°Π΄Π°Ρ‡Π° 4

Π’ ΠΎΠ΄Π½ΠΎΡ€ΠΎΠ΄Π½ΠΎΠΌ ΠΌΠ°Π³Π½ΠΈΡ‚Π½ΠΎΠΌ ΠΏΠΎΠ»Π΅ пСрпСндикулярно ΠΊ Π½Π°ΠΏΡ€Π°Π²Π»Π΅Π½ΠΈΡŽ Π²Π΅ΠΊΡ‚ΠΎΡ€Π° ΠΈΠ½Π΄ΡƒΠΊΡ†ΠΈΠΈ , ΠΌΠΎΠ΄ΡƒΠ»ΡŒ ΠΊΠΎΡ‚ΠΎΡ€ΠΎΠ³ΠΎ 0,1 Π’Π», двиТСтся ΠΏΡ€ΠΎΠ²ΠΎΠ΄ Π΄Π»ΠΈΠ½ΠΎΠΉ 2 ΠΌΠ΅Ρ‚Ρ€Π° со ΡΠΊΠΎΡ€ΠΎΡΡ‚ΡŒΡŽ 5 ΠΌ/с, пСрпСндикулярной ΠΏΡ€ΠΎΠ²ΠΎΠ΄Π½ΠΈΠΊΡƒ.
Какая Π­Π”Π‘ индуцируСтся Π² этом ΠΏΡ€ΠΎΠ²ΠΎΠ΄Π½ΠΈΠΊΠ΅?



Π—Π°Π΄Π°Ρ‡Π° 5

ΠŸΠ΅Ρ€ΠΏΠ΅Π½Π΄ΠΈΠΊΡƒΠ»ΡΡ€Π½ΠΎ Π²Π΅ΠΊΡ‚ΠΎΡ€Ρƒ ΠΌΠ°Π³Π½ΠΈΡ‚Π½ΠΎΠΉ ΠΈΠ½Π΄ΡƒΠΊΡ†ΠΈΠΈ пСрСмСщаСтся ΠΏΡ€ΠΎΠ²ΠΎΠ΄Π½ΠΈΠΊ Π΄Π»ΠΈΠ½ΠΎΠΉ 1,8 ΠΌΠ΅Ρ‚Ρ€Π° со ΡΠΊΠΎΡ€ΠΎΡΡ‚ΡŒΡŽ 6 ΠΌ/c. Π­Π”Π‘ ΠΈΠ½Π΄ΡƒΠΊΡ†ΠΈΠΈ Ρ€Π°Π²Π½Π° 1,44 Π’.
Найти ΠΌΠ°Π³Π½ΠΈΡ‚Π½ΡƒΡŽ ΠΈΠ½Π΄ΡƒΠΊΡ†ΠΈΡŽΒ  ΠΌΠ°Π³Π½ΠΈΡ‚Π½ΠΎΠ³ΠΎ поля.


Π—Π°Π΄Π°Ρ‡Π° 6

Π‘Π°ΠΌΠΎΠ»Π΅Ρ‚ ΠΈΠΌΠ΅Π΅Ρ‚ Ρ€Π°Π·ΠΌΠ°Ρ… ΠΊΡ€Ρ‹Π»ΡŒΠ΅Π² 15 ΠΌΠ΅Ρ‚Ρ€ΠΎΠ². Π“ΠΎΡ€ΠΈΠ·ΠΎΠ½Ρ‚Π°Π»ΡŒΠ½Π°Ρ ΡΠΊΠΎΡ€ΠΎΡΡ‚ΡŒ ΠΏΠΎΠ»Π΅Ρ‚Π° Ρ€Π°Π²Π½Π°720 ΠΊΠΌ/час.
ΠžΠΏΡ€Π΅Π΄Π΅Π»ΠΈΡ‚ΡŒ Ρ€Π°Π·Π½ΠΎΡΡ‚ΡŒ ΠΏΠΎΡ‚Π΅Π½Ρ†ΠΈΠ°Π»ΠΎΠ², Π²ΠΎΠ·Π½ΠΈΠΊΠ°ΡŽΡ‰ΠΈΡ… ΠΌΠ΅ΠΆΠ΄Ρƒ ΠΊΠΎΠ½Ρ†Π°ΠΌΠΈ ΠΊΡ€Ρ‹Π»ΡŒΠ΅Π². Π’Π΅Ρ€Ρ‚ΠΈΠΊΠ°Π»ΡŒΠ½Π°Ρ ΡΠΎΡΡ‚Π°Π²Π»ΡΡŽΡ‰Π°Ρ ΠΌΠ°Π³Π½ΠΈΡ‚Π½ΠΎΠΉ ΠΈΠ½Π΄ΡƒΠΊΡ†ΠΈΠΈ (пСрпСндикулярно повСрхности Π—Π΅ΠΌΠ»ΠΈ) Ρ€Π°Π²Π½Π° 50 ΠΌΠΊΠ’Π».

Π—Π°Π΄Π°Ρ‡Π° 7

ΠœΠ°Π³Π½ΠΈΡ‚Π½Ρ‹ΠΉ ΠΏΠΎΡ‚ΠΎΠΊ Ρ‡Π΅Ρ€Π΅Π· ΠΊΠΎΠ½Ρ‚ΡƒΡ€ ΠΏΡ€ΠΎΠ²ΠΎΠ΄Π½ΠΈΠΊΠ° сопротивлСниСм 0,03 Ом Π·Π° 2 сСкунды  измСнился Π½Π° 0,012 Π’Π±.
НайдитС  силу Ρ‚ΠΎΠΊΠ° Π² ΠΏΡ€ΠΎΠ²ΠΎΠ΄Π½ΠΈΠΊΠ΅ Ссли ΠΈΠ·ΠΌΠ΅Π½Π΅Π½ΠΈΠ΅ ΠΏΠΎΡ‚ΠΎΠΊΠ° происходило Ρ€Π°Π²Π½ΠΎΠΌΠ΅Ρ€Π½ΠΎ.

Π—Π°Π΄Π°Ρ‡Π° 8

Π’ ΠΎΠ΄Π½ΠΎΡ€ΠΎΠ΄Π½ΠΎΠΌ ΠΌΠ°Π³Π½ΠΈΡ‚Π½ΠΎΠΌ ΠΏΠΎΠ»Π΅ находится плоский Π²ΠΈΡ‚ΠΎΠΊ ΠΏΠ»ΠΎΡ‰Π°Π΄ΡŒΡŽ 10 см2, располоТСнный пСрпСндикулярно Π²Π΅ΠΊΡ‚ΠΎΡ€Ρƒ ΠΌΠ°Π³Π½ΠΈΡ‚Π½ΠΎΠΉ ΠΈΠ½Π΄ΡƒΠΊΡ†ΠΈΠΈ.
Какой Ρ‚ΠΎΠΊ Ρ‚Π΅Ρ‡Π΅Ρ‚ ΠΏΠΎ Π²ΠΈΡ‚ΠΊΡƒ, Ссли ΠΏΠΎΠ»Π΅ Π±ΡƒΠ΄Π΅Ρ‚ ΡƒΠ±Ρ‹Π²Π°Ρ‚ΡŒ с постоянной ΡΠΊΠΎΡ€ΠΎΡΡ‚ΡŒΡŽ 0,5 Π’Π»/с?

Π—Π°Π΄Π°Ρ‡Π° 9

Π‘ΠΎΠΏΡ€ΠΎΡ‚ΠΈΠ²Π»Π΅Π½ΠΈΠ΅ Π·Π°ΠΌΠΊΠ½ΡƒΡ‚ΠΎΠ³ΠΎ ΠΊΠΎΠ½Ρ‚ΡƒΡ€Π° Ρ€Π°Π²Π½ΠΎ 0,5 Ом. ΠŸΡ€ΠΈ ΠΏΠ΅Ρ€Π΅ΠΌΠ΅Ρ‰Π΅Π½ΠΈΠΈ ΠΊΠΎΠ»ΡŒΡ†Π° Π² ΠΌΠ°Π³Π½ΠΈΡ‚Π½ΠΎΠΌ ΠΏΠΎΠ»Π΅ ΠΌΠ°Π³Π½ΠΈΡ‚Π½Ρ‹ΠΉ ΠΏΠΎΡ‚ΠΎΠΊ Ρ‡Π΅Ρ€Π΅Π· ΠΊΠΎΠ»ΡŒΡ†ΠΎ измСнился Π½Π° 5×10-3 Π’Π±.
Какой Π·Π° это врСмя ΠΏΡ€ΠΎΡˆΠ΅Π» заряд Ρ‡Π΅Ρ€Π΅Π· ΠΏΠΎΠΏΠ΅Ρ€Π΅Ρ‡Π½ΠΎΠ΅ сСчСниС ΠΏΡ€ΠΎΠ²ΠΎΠ΄Π½ΠΈΠΊΠ°?



class-fizika.ru

Π­Π”Π‘ ΠΈΠ½Π΄ΡƒΠΊΡ†ΠΈΠΈ Π² двиТущихся ΠΏΡ€ΠΎΠ²ΠΎΠ΄Π½ΠΈΠΊΠ°Ρ…

Π­Π”Π‘ – это Π°Π±Π±Ρ€Π΅Π²ΠΈΠ°Ρ‚ΡƒΡ€Π° Ρ‚Ρ€Π΅Ρ… слов: элСктродвиТущая сила. Π­Π”Π‘ ΠΈΠ½Π΄ΡƒΠΊΡ†ΠΈΠΈ () появляСтся Π² проводящСм Ρ‚Π΅Π»Π΅, ΠΊΠΎΡ‚ΠΎΡ€ΠΎΠ΅ находится Π² ΠΏΠ΅Ρ€Π΅ΠΌΠ΅Π½Π½ΠΎΠΌ ΠΌΠ°Π³Π½ΠΈΡ‚Π½ΠΎΠΌ ΠΏΠΎΠ»Π΅. Если проводящим Ρ‚Π΅Π»ΠΎΠΌ являСтся, Π½Π°ΠΏΡ€ΠΈΠΌΠ΅Ρ€, Π·Π°ΠΌΠΊΠ½ΡƒΡ‚Ρ‹ΠΉ ΠΊΠΎΠ½Ρ‚ΡƒΡ€, Ρ‚ΠΎ Π² Π½Π΅ΠΌ Ρ‚Π΅Ρ‡Π΅Ρ‚ элСктричСский Ρ‚ΠΎΠΊ, ΠΊΠΎΡ‚ΠΎΡ€Ρ‹ΠΉ Π½Π°Π·Ρ‹Π²Π°ΡŽΡ‚ Ρ‚ΠΎΠΊΠΎΠΌ ΠΈΠ½Π΄ΡƒΠΊΡ†ΠΈΠΈ.

Π—Π°ΠΊΠΎΠ½ ЀарадСя для элСктромагнитной ΠΈΠ½Π΄ΡƒΠΊΡ†ΠΈΠΈ

ΠžΡΠ½ΠΎΠ²Π½Ρ‹ΠΌ Π·Π°ΠΊΠΎΠ½ΠΎΠΌ, ΠΊΠΎΡ‚ΠΎΡ€Ρ‹ΠΉ ΠΈΡΠΏΠΎΠ»ΡŒΠ·ΡƒΡŽΡ‚ ΠΏΡ€ΠΈ расчСтах, связанных с элСктромагнитной ΠΈΠ½Π΄ΡƒΠΊΡ†ΠΈΠ΅ΠΉ являСтся Π·Π°ΠΊΠΎΠ½ ЀарадСя. Он Π³ΠΎΠ²ΠΎΡ€ΠΈΡ‚ ΠΎ Ρ‚ΠΎΠΌ, Ρ‡Ρ‚ΠΎ элСктродвиТущая сила элСктромагнитной ΠΈΠ½Π΄ΡƒΠΊΡ†ΠΈΠΈ Π² ΠΊΠΎΠ½Ρ‚ΡƒΡ€Π΅ Ρ€Π°Π²Π½Π° ΠΏΠΎ Π²Π΅Π»ΠΈΡ‡ΠΈΠ½Π΅ ΠΈ ΠΏΡ€ΠΎΡ‚ΠΈΠ²ΠΎΠΏΠΎΠ»ΠΎΠΆΠ½Π° ΠΏΠΎ Π·Π½Π°ΠΊΡƒ скорости измСнСния ΠΌΠ°Π³Π½ΠΈΡ‚Π½ΠΎΠ³ΠΎ ΠΏΠΎΡ‚ΠΎΠΊΠ° () сквозь ΠΏΠΎΠ²Π΅Ρ€Ρ…Π½ΠΎΡΡ‚ΡŒ, ΠΊΠΎΡ‚ΠΎΡ€ΡƒΡŽ ΠΎΠ³Ρ€Π°Π½ΠΈΡ‡ΠΈΠ²Π°Π΅Ρ‚ рассматриваСмый ΠΊΠΎΠ½Ρ‚ΡƒΡ€:

Β  Β 

Π—Π°ΠΊΠΎΠ½ ЀарадСя (1) записан для систСмы БИ. Надо ΡƒΡ‡ΠΈΡ‚Ρ‹Π²Π°Ρ‚ΡŒ, Ρ‡Ρ‚ΠΎ ΠΈΠ· ΠΊΠΎΠ½Ρ†Π° Π²Π΅ΠΊΡ‚ΠΎΡ€Π° Π½ΠΎΡ€ΠΌΠ°Π»ΠΈ ΠΊ ΠΊΠΎΠ½Ρ‚ΡƒΡ€Ρƒ ΠΎΠ±Ρ…ΠΎΠ΄ ΠΊΠΎΠ½Ρ‚ΡƒΡ€Π° Π΄ΠΎΠ»ΠΆΠ΅Π½ ΠΏΡ€ΠΎΡ…ΠΎΠ΄ΠΈΡ‚ΡŒ ΠΏΡ€ΠΎΡ‚ΠΈΠ² часовой стрСлки. Если ΠΈΠ·ΠΌΠ΅Π½Π΅Π½ΠΈΠ΅ ΠΏΠΎΡ‚ΠΎΠΊΠ° происходит Ρ€Π°Π²Π½ΠΎΠΌΠ΅Ρ€Π½ΠΎ, Ρ‚ΠΎ Π­Π”Π‘ ΠΈΠ½Π΄ΡƒΠΊΡ†ΠΈΠΈ находят ΠΊΠ°ΠΊ:

Β  Β 

ΠœΠ°Π³Π½ΠΈΡ‚Π½Ρ‹ΠΉ ΠΏΠΎΡ‚ΠΎΠΊ, ΠΊΠΎΡ‚ΠΎΡ€Ρ‹ΠΉ ΠΎΡ…Π²Π°Ρ‚Ρ‹Π²Π°Π΅Ρ‚ проводящий ΠΊΠΎΠ½Ρ‚ΡƒΡ€, ΠΌΠΎΠΆΠ΅Ρ‚ ΠΈΠ·ΠΌΠ΅Π½ΡΡ‚ΡŒΡΡ Π² связи с Ρ€Π°Π·Π½Ρ‹ΠΌΠΈ ΠΏΡ€ΠΈΡ‡ΠΈΠ½Π°ΠΌΠΈ. Π­Ρ‚ΠΎ ΠΌΠΎΠΆΠ΅Ρ‚ Π±Ρ‹Ρ‚ΡŒ ΠΈ ΠΈΠ·ΠΌΠ΅Π½ΡΡŽΡ‰Π΅Π΅ΡΡ Π²ΠΎ Π²Ρ€Π΅ΠΌΠ΅Π½ΠΈ ΠΌΠ°Π³Π½ΠΈΡ‚Π½ΠΎΠ΅ ΠΏΠΎΠ»Π΅ ΠΈ дСформация самого ΠΊΠΎΠ½Ρ‚ΡƒΡ€Π°, ΠΈ ΠΏΠ΅Ρ€Π΅ΠΌΠ΅Ρ‰Π΅Π½ΠΈΠ΅ ΠΊΠΎΠ½Ρ‚ΡƒΡ€Π° Π² ΠΏΠΎΠ»Π΅. Полная производная ΠΎΡ‚ ΠΌΠ°Π³Π½ΠΈΡ‚Π½ΠΎΠ³ΠΎ ΠΏΠΎΡ‚ΠΎΠΊΠ° ΠΏΠΎ Π²Ρ€Π΅ΠΌΠ΅Π½ΠΈ ΡƒΡ‡ΠΈΡ‚Ρ‹Π²Π°Π΅Ρ‚ дСйствиС всСх ΠΏΡ€ΠΈΡ‡ΠΈΠ½.

Π­Π”Π‘ ΠΈΠ½Π΄ΡƒΠΊΡ†ΠΈΠΈ Π² двиТущСмся ΠΏΡ€ΠΎΠ²ΠΎΠ΄Π½ΠΈΠΊΠ΅

Допустим, Ρ‡Ρ‚ΠΎ проводящий ΠΊΠΎΠ½Ρ‚ΡƒΡ€ пСрСмСщаСтся Π² постоянном ΠΌΠ°Π³Π½ΠΈΡ‚Π½ΠΎΠΌ ΠΏΠΎΠ»Π΅. Π­Π”Π‘ ΠΈΠ½Π΄ΡƒΠΊΡ†ΠΈΠΈ Π²ΠΎΠ·Π½ΠΈΠΊΠ°Π΅Ρ‚ Π²ΠΎ всСх частях ΠΊΠΎΠ½Ρ‚ΡƒΡ€Π°, ΠΊΠΎΡ‚ΠΎΡ€Ρ‹Π΅ ΠΏΠ΅Ρ€Π΅ΡΠ΅ΠΊΠ°ΡŽΡ‚ силовыС Π»ΠΈΠ½ΠΈΠΈ ΠΌΠ°Π³Π½ΠΈΡ‚Π½ΠΎΠ³ΠΎ поля. ΠŸΡ€ΠΈ этом, Ρ€Π΅Π·ΡƒΠ»ΡŒΡ‚ΠΈΡ€ΡƒΡŽΡ‰Π°Ρ Π­Π”Π‘, ΠΏΠΎΡΠ²Π»ΡΡŽΡ‰Π°ΡΡΡ Π² ΠΊΠΎΠ½Ρ‚ΡƒΡ€Π΅ Π±ΡƒΠ΄Π΅Ρ‚ Ρ€Π°Π²Π½Π° алгСбраичСской суммС Π­Π”Π‘ ΠΊΠ°ΠΆΠ΄ΠΎΠ³ΠΎ участка. Π’ΠΎΠ·Π½ΠΈΠΊΠ½ΠΎΠ²Π΅Π½ΠΈΠ΅ Π­Π”Π‘ Π² рассматриваСмом случаС ΠΎΠ±ΡŠΡΡΠ½ΡΡŽΡ‚ Ρ‚Π΅ΠΌ, Ρ‡Ρ‚ΠΎ Π½Π° любой свободный заряд, ΠΊΠΎΡ‚ΠΎΡ€Ρ‹ΠΉ двиТСтся вмСстС с ΠΏΡ€ΠΎΠ²ΠΎΠ΄Π½ΠΈΠΊΠΎΠΌ Π² ΠΌΠ°Π³Π½ΠΈΡ‚Π½ΠΎΠΌ ΠΏΠΎΠ»Π΅, Π±ΡƒΠ΄Π΅Ρ‚ Π΄Π΅ΠΉΡΡ‚Π²ΠΎΠ²Π°Ρ‚ΡŒ сила Π›ΠΎΡ€Π΅Π½Ρ†Π°. ΠŸΡ€ΠΈ воздСйствии сил Π›ΠΎΡ€Π΅Π½Ρ†Π° заряды двиТутся ΠΈ ΠΎΠ±Ρ€Π°Π·ΡƒΡŽΡ‚ Π² Π·Π°ΠΌΠΊΠ½ΡƒΡ‚ΠΎΠΌ ΠΏΡ€ΠΎΠ²ΠΎΠ΄Π½ΠΈΠΊΠ΅ Ρ‚ΠΎΠΊ ΠΈΠ½Π΄ΡƒΠΊΡ†ΠΈΠΈ.

Рассмотри случай, ΠΊΠΎΠ³Π΄Π° Π² ΠΎΠ΄Π½ΠΎΡ€ΠΎΠ΄Π½ΠΎΠΌ ΠΌΠ°Π³Π½ΠΈΡ‚Π½ΠΎΠΌ ΠΏΠΎΠ»Π΅ находится ΠΏΡ€ΡΠΌΠΎΡƒΠ³ΠΎΠ»ΡŒΠ½Π°Ρ проводящая Ρ€Π°ΠΌΠΊΠ° (рис.1). Одна сторона Ρ€Π°ΠΌΠΊΠΈ ΠΌΠΎΠΆΠ΅Ρ‚ Π΄Π²ΠΈΠ³Π°Ρ‚ΡŒΡΡ. Π”Π»ΠΈΠ½Π° этой стороны Ρ€Π°Π²Π½Π° l. Π­Ρ‚ΠΎ ΠΈ Π±ΡƒΠ΄Π΅Ρ‚ наш двиТущийся ΠΏΡ€ΠΎΠ²ΠΎΠ΄Π½ΠΈΠΊ. ΠžΠΏΡ€Π΅Π΄Π΅Π»ΠΈΠΌ, ΠΊΠ°ΠΊ ΠΌΠΎΠΆΠ½ΠΎ Π²Ρ‹Ρ‡ΠΈΡΠ»ΠΈΡ‚ΡŒ Π­Π”Π‘ ΠΈΠ½Π΄ΡƒΠΊΡ†ΠΈΠΈ, Π² нашСм ΠΏΡ€ΠΎΠ²ΠΎΠ΄Π½ΠΈΠΊΠ΅, Ссли ΠΎΠ½ пСрСмСщаСтся со ΡΠΊΠΎΡ€ΠΎΡΡ‚ΡŒΡŽ v. Π’Π΅Π»ΠΈΡ‡ΠΈΠ½Π° ΠΈΠ½Π΄ΡƒΠΊΡ†ΠΈΠΈ ΠΌΠ°Π³Π½ΠΈΡ‚Π½ΠΎΠ³ΠΎ поля Ρ€Π°Π²Π½Π° B. ΠŸΠ»ΠΎΡΠΊΠΎΡΡ‚ΡŒ Ρ€Π°ΠΌΠΊΠΈ пСрпСндикулярна Π²Π΅ΠΊΡ‚ΠΎΡ€Ρƒ ΠΌΠ°Π³Π½ΠΈΡ‚Π½ΠΎΠΉ ΠΈΠ½Π΄ΡƒΠΊΡ†ΠΈΠΈ. ВыполняСтся условиС .

Π­Π”Π‘ ΠΈΠ½Π΄ΡƒΠΊΡ†ΠΈΠΈ Π² рассматриваСмом Π½Π°ΠΌΠΈ ΠΊΠΎΠ½Ρ‚ΡƒΡ€Π΅ Π±ΡƒΠ΄Π΅Ρ‚ Ρ€Π°Π²Π½Π° Π­Π”Π‘, которая Π²ΠΎΠ·Π½ΠΈΠΊΠ°Π΅Ρ‚ Ρ‚ΠΎΠ»ΡŒΠΊΠΎ Π² ΠΏΠΎΠ΄Π²ΠΈΠΆΠ½ΠΎΠΉ Π΅Π³ΠΎ части. Π’ стационарных частях ΠΊΠΎΠ½Ρ‚ΡƒΡ€Π° Π² постоянном ΠΌΠ°Π³Π½ΠΈΡ‚Π½ΠΎΠΌ ΠΏΠΎΠ»Π΅ ΠΈΠ½Π΄ΡƒΠΊΡ†ΠΈΠΈ Π½Π΅Ρ‚.

Для нахоТдСния Π­Π”Π‘ ΠΈΠ½Π΄ΡƒΠΊΡ†ΠΈΠΈ Π² Ρ€Π°ΠΌΠΊΠ΅ Π²ΠΎΡΠΏΠΎΠ»ΡŒΠ·ΡƒΠ΅ΠΌΡΡ основным Π·Π°ΠΊΠΎΠ½ΠΎΠΌ (1). Но для Π½Π°Ρ‡Π°Π»Π° опрСдСлимся с ΠΌΠ°Π³Π½ΠΈΡ‚Π½Ρ‹ΠΌ ΠΏΠΎΡ‚ΠΎΠΊΠΎΠΌ. По ΠΎΠΏΡ€Π΅Π΄Π΅Π»Π΅Π½ΠΈΡŽ ΠΏΠΎΡ‚ΠΎΠΊ ΠΌΠ°Π³Π½ΠΈΡ‚Π½ΠΎΠΉ ΠΈΠ½Π΄ΡƒΠΊΡ†ΠΈΠΈ Ρ€Π°Π²Π΅Π½:

Β  Β 

Π³Π΄Π΅ , Ρ‚Π°ΠΊ ΠΊΠ°ΠΊ ΠΏΠΎ ΡƒΡΠ»ΠΎΠ²ΠΈΡŽ ΠΏΠ»ΠΎΡΠΊΠΎΡΡ‚ΡŒ Ρ€Π°ΠΌΠΊΠΈ пСрпСндикулярна Π½Π°ΠΏΡ€Π°Π²Π»Π΅Π½ΠΈΡŽ Π²Π΅ΠΊΡ‚ΠΎΡ€Π° ΠΈΠ½Π΄ΡƒΠΊΡ†ΠΈΠΈ поля, ΡΠ»Π΅Π΄ΠΎΠ²Π°Ρ‚Π΅Π»ΡŒΠ½ΠΎ, Π½ΠΎΡ€ΠΌΠ°Π»ΡŒ ΠΊ Ρ€Π°ΠΌΠΊΠ΅ ΠΈ Π²Π΅ΠΊΡ‚ΠΎΡ€ ΠΈΠ½Π΄ΡƒΠΊΡ†ΠΈΠΈ ΠΏΠ°Ρ€Π°Π»Π»Π΅Π»ΡŒΠ½Ρ‹. ΠŸΠ»ΠΎΡ‰Π°Π΄ΡŒ, ΠΊΠΎΡ‚ΠΎΡ€ΡƒΡŽ ΠΎΠ³Ρ€Π°Π½ΠΈΡ‡ΠΈΠ²Π°Π΅Ρ‚ Ρ€Π°ΠΌΠΊΠ°, Π²Ρ‹Ρ€Π°Π·ΠΈΠΌ ΡΠ»Π΅Π΄ΡƒΡŽΡ‰ΠΈΠΌ ΠΎΠ±Ρ€Π°Π·ΠΎΠΌ:

Β  Β 

Π³Π΄Π΅ – расстояниС, Π½Π° ΠΊΠΎΡ‚ΠΎΡ€ΠΎΠ΅ пСрСмСщаСтся двиТущийся ΠΏΡ€ΠΎΠ²ΠΎΠ΄Π½ΠΈΠΊ. ΠŸΠΎΠ΄ΡΡ‚Π°Π²ΠΈΠΌ Π²Ρ‹Ρ€Π°ΠΆΠ΅Π½ΠΈΠ΅ (2), с ΡƒΡ‡Π΅Ρ‚ΠΎΠΌ (3) Π² Π·Π°ΠΊΠΎΠ½ ЀарадСя, ΠΏΠΎΠ»ΡƒΡ‡ΠΈΠΌ:

Β  Β 

Π³Π΄Π΅ v – ΡΠΊΠΎΡ€ΠΎΡΡ‚ΡŒ двиТСния ΠΏΠΎΠ΄Π²ΠΈΠΆΠ½ΠΎΠΉ стороны Ρ€Π°ΠΌΠΊΠΈ ΠΏΠΎ оси X.

Если ΡƒΠ³ΠΎΠ» ΠΌΠ΅ΠΆΠ΄Ρƒ Π½Π°ΠΏΡ€Π°Π²Π»Π΅Π½ΠΈΠ΅ΠΌ Π²Π΅ΠΊΡ‚ΠΎΡ€Π° ΠΌΠ°Π³Π½ΠΈΡ‚Π½ΠΎΠΉ ΠΈΠ½Π΄ΡƒΠΊΡ†ΠΈΠΈ () ΠΈ Π²Π΅ΠΊΡ‚ΠΎΡ€ΠΎΠΌ скорости двиТСния ΠΏΡ€ΠΎΠ²ΠΎΠ΄Π½ΠΈΠΊΠ° () составляСт ΡƒΠ³ΠΎΠ» , Ρ‚ΠΎ ΠΌΠΎΠ΄ΡƒΠ»ΡŒ Π­Π”Π‘ Π² ΠΏΡ€ΠΎΠ²ΠΎΠ΄Π½ΠΈΠΊΠ΅ ΠΌΠΎΠΆΠ½ΠΎ Π²Ρ‹Ρ‡ΠΈΡΠ»ΠΈΡ‚ΡŒ ΠΏΡ€ΠΈ ΠΏΠΎΠΌΠΎΡ‰ΠΈ Ρ„ΠΎΡ€ΠΌΡƒΠ»Ρ‹:

Β  Β 

ΠŸΡ€ΠΈΠΌΠ΅Ρ€Ρ‹ Ρ€Π΅ΡˆΠ΅Π½ΠΈΡ Π·Π°Π΄Π°Ρ‡

ru.solverbook.com

Π¦Π”Πž_2semestr_1 — Π‘Ρ‚Ρ€ 2

[task#2]

ЭнСргия ΠΌΠ°Π³Π½ΠΈΡ‚Π½ΠΎΠ³ΠΎ поля ΠΊΠΎΠ½Ρ‚ΡƒΡ€Π° с Ρ‚ΠΎΠΊΠΎΠΌ ΠΏΡ€ΠΎΠΏΠΎΡ€Ρ†ΠΈΠΎΠ½Π°Π»ΡŒΠ½Π°…

1=…ΠΏΠ΅Ρ€Π²ΠΎΠΉ стСпСни силы Ρ‚ΠΎΠΊΠ°.

2=…Π²Ρ‚ΠΎΡ€ΠΎΠΉ стСпСни силы Ρ‚ΠΎΠΊΠ°.

3=…Ρ‚Ρ€Π΅Ρ‚ΡŒΠ΅ΠΉ стСпСни силы Ρ‚ΠΎΠΊΠ°. 4=…ΠΊΠΎΡ€Π½ΡŽ ΠΊΠ²Π°Π΄Ρ€Π°Ρ‚Π½ΠΎΠΌΡƒ ΠΈΠ· силы Ρ‚ΠΎΠΊΠ°. 5=…ΠΊΠΎΡ€Π½ΡŽ кубичСскому ΠΈΠ· силы Ρ‚ΠΎΠΊΠ°. [task#3]

Π—Π° 2 с ΠΌΠ°Π³Π½ΠΈΡ‚Π½Ρ‹ΠΉ ΠΏΠΎΡ‚ΠΎΠΊ, ΠΏΡ€ΠΎΠ½ΠΈΠ·Ρ‹Π²Π°ΡŽΡ‰ΠΈΠΉ ΠΊΠΎΠ½Ρ‚ΡƒΡ€, Ρ€Π°Π²Π½ΠΎΠΌΠ΅Ρ€Π½ΠΎ ΡƒΠΌΠ΅Π½ΡŒΡˆΠΈΠ»ΡΡ с 8 Π΄ΠΎ 2 Π’Π±. Π§Π΅ΠΌΡƒ ΠΏΡ€ΠΈ этом Π±Ρ‹Π»ΠΎ Ρ€Π°Π²Π½ΠΎ Π·Π½Π°Ρ‡Π΅Π½ΠΈΠ΅ (Π² Π²ΠΎΠ»ΡŒΡ‚Π°Ρ…) Π­Π”Π‘ ΠΈΠ½Π΄ΡƒΠΊΡ†ΠΈΠΈ Π² ΠΊΠΎΠ½Ρ‚ΡƒΡ€Π΅? answer1=3.0 % 4

[task#4]

ΠšΠΎΡΡ„Ρ„ΠΈΡ†ΠΈΠ΅Π½Ρ‚ самоиндукции ΠΊΠΎΠ½Ρ‚ΡƒΡ€Π° с Ρ‚ΠΎΠΊΠΎΠΌ Π² Π²Π°ΠΊΡƒΡƒΠΌΠ΅ зависит Ρ‚ΠΎΠ»ΡŒΠΊΠΎ ΠΎΡ‚…

1=… Ρ€Π°Π·ΠΌΠ΅Ρ€ΠΎΠ², Ρ„ΠΎΡ€ΠΌΡ‹ ΠΏΡ€ΠΎΠ²ΠΎΠ΄Π½ΠΈΠΊΠ° ΠΈ Π΅Π³ΠΎ ΡƒΠ΄Π΅Π»ΡŒΠ½ΠΎΠ³ΠΎ сопротивлСния. 2=… Ρ„ΠΎΡ€ΠΌΡ‹ ΠΏΡ€ΠΎΠ²ΠΎΠ΄Π½ΠΈΠΊΠ° ΠΈ силы Ρ‚ΠΎΠΊΠ°.

3=… Ρ€Π°Π·ΠΌΠ΅Ρ€ΠΎΠ² ΠΈ Ρ„ΠΎΡ€ΠΌΡ‹ ΠΏΡ€ΠΎΠ²ΠΎΠ΄Π½ΠΈΠΊΠ°. 4=… Ρ€Π°Π·ΠΌΠ΅Ρ€ΠΎΠ² ΠΏΡ€ΠΎΠ²ΠΎΠ΄Π½ΠΈΠΊΠ°.

5=… Ρ€Π°Π·ΠΌΠ΅Ρ€ΠΎΠ² ΠΏΡ€ΠΎΠ²ΠΎΠ΄Π½ΠΈΠΊΠ° ΠΈ Π΅Π³ΠΎ ΡƒΠ΄Π΅Π»ΡŒΠ½ΠΎΠ³ΠΎ сопротивлСния. [task#5]

ΠŸΡ€Π΅Π΄ΡΡ‚Π°Π²Π»Π΅Π½Π½Π°Ρ Π½Π° рисункС систСма ΡƒΡ€Π°Π²Π½Π΅Π½ΠΈΠΉ МаксвСлла справСдлива для элСктромагнитного поля…

1=…Ρ‚ΠΎΠ»ΡŒΠΊΠΎ, Ссли ΠΏΠΎΠ»Π΅ стационарно. 2=…Ρ‚ΠΎΠ»ΡŒΠΊΠΎ Π² отсутствиС Ρ‚ΠΎΠΊΠΎΠ² проводимости. 3=…Ρ‚ΠΎΠ»ΡŒΠΊΠΎ Π² отсутствиС заряТСнных Ρ‚Π΅Π».

4=…Ρ‚ΠΎΠ»ΡŒΠΊΠΎ Π² отсутствиС заряТСнных Ρ‚Π΅Π» ΠΈ Ρ‚ΠΎΠΊΠΎΠ² проводимости.

5=…ΠΏΡ€ΠΈ Π½Π°Π»ΠΈΡ‡ΠΈΠΈ заряТСнных Ρ‚Π΅Π» ΠΈ Ρ‚ΠΎΠΊΠΎΠ² проводимости.

variant_number=04

[task#1]

Π‘ΠΎΠ»Π΅Π½ΠΎΠΈΠ΄ X β€”Yс ΠΏΡ€ΠΎΠ²ΠΎΠ»ΠΎΠΊΠΎΠΉ, Π½Π°ΠΌΠΎΡ‚Π°Π½Π½ΠΎΠΉ Π½Π° ΠΊΠ°Ρ€Ρ‚ΠΎΠ½Π½ΡƒΡŽ Ρ‚Ρ€ΡƒΠ±ΠΊΡƒ, располоТСн Π½Π° ΠΎΠ΄Π½ΠΎΠΉ оси с Ρ†ΠΈΠ»ΠΈΠ½Π΄Ρ€ΠΎΠΌ pβ€”qΠΈΠ· магнитомягкого ΠΆΠ΅Π»Π΅Π·Π°. Π’ΠΎΠΊ I Π² солСноидС ΠΏΠΎΡ€ΠΎΠΆΠ΄Π°Π΅Ρ‚ ΠΌΠ°Π³Π½ΠΈΡ‚Π½ΠΎΠ΅ ΠΏΠΎΠ»Π΅. Π£ΠΊΠ°ΠΆΠΈΡ‚Π΅ Π²Π΅Ρ€Π½Ρ‹ΠΉ Π²Π°Ρ€ΠΈΠ°Π½Ρ‚ располоТСния полюсов солСноида ΠΈ Ρ†ΠΈΠ»ΠΈΠ½Π΄Ρ€Π°.

1=X β€”ΠΏΠΎΠ»ΡŽΡN,Yβ€”ΠΏΠΎΠ»ΡŽΡS.pβ€”ΠΏΠΎΠ»ΡŽΡN,qβ€”ΠΏΠΎΠ»ΡŽΡS.

2=X β€”ΠΏΠΎΠ»ΡŽΡS,Yβ€”ΠΏΠΎΠ»ΡŽΡN.pβ€”ΠΏΠΎΠ»ΡŽΡN,qβ€”ΠΏΠΎΠ»ΡŽΡS.3=Xβ€”ΠΏΠΎΠ»ΡŽΡN,Yβ€”ΠΏΠΎΠ»ΡŽΡS.pβ€”ΠΏΠΎΠ»ΡŽΡS,qβ€”ΠΏΠΎΠ»ΡŽΡN.4=Xβ€”ΠΏΠΎΠ»ΡŽΡS,Yβ€”ΠΏΠΎΠ»ΡŽΡN.pβ€”ΠΏΠΎΠ»ΡŽΡS,qβ€”ΠΏΠΎΠ»ΡŽΡN.5=Xβ€”ΠΏΠΎΠ»ΡŽΡN,Yβ€”ΠΏΠΎΠ»ΡŽΡS. Π£ Ρ†ΠΈΠ»ΠΈΠ½Π΄Ρ€Π° полюсов Π½Π΅Ρ‚. [task#2]

ЭнСргия ΠΌΠ°Π³Π½ΠΈΡ‚Π½ΠΎΠ³ΠΎ поля, создаваСмого ΠΊΠΎΠ½Ρ‚ΡƒΡ€ΠΎΠΌ с Ρ‚ΠΎΠΊΠΎΠΌ, ΠΏΡ€ΠΎΠΏΠΎΡ€Ρ†ΠΈΠΎΠ½Π°Π»ΡŒΠ½Π°…

1=… ΠΊΠΎΡ€Π½ΡŽ ΠΊΠ²Π°Π΄Ρ€Π°Ρ‚Π½ΠΎΠΌΡƒ ΠΈΠ· индуктивности ΠΊΠΎΠ½Ρ‚ΡƒΡ€Π°. 2=… индуктивности ΠΊΠΎΠ½Ρ‚ΡƒΡ€Π° Π² минус Π²Ρ‚ΠΎΡ€ΠΎΠΉ стСпСни. 3=… индуктивности ΠΊΠΎΠ½Ρ‚ΡƒΡ€Π° Π² минус ΠΏΠ΅Ρ€Π²ΠΎΠΉ стСпСни.

4=… ΠΏΠ΅Ρ€Π²ΠΎΠΉ стСпСни индуктивности ΠΊΠΎΠ½Ρ‚ΡƒΡ€Π°.

5=… Π²Ρ‚ΠΎΡ€ΠΎΠΉ стСпСни индуктивности ΠΊΠΎΠ½Ρ‚ΡƒΡ€Π°. [task#3]

По ΠΏΡ€ΠΎΠ²ΠΎΠ΄Π½ΠΈΠΊΠ°ΠΌ, располоТСнным Π² ΠΎΠ΄Π½ΠΎΡ€ΠΎΠ΄Π½ΠΎΠΌ ΠΌΠ°Π³Π½ΠΈΡ‚Π½ΠΎΠΌ ΠΏΠΎΠ»Π΅ пСрпСндикулярно Π²Π΅ΠΊΡ‚ΠΎΡ€Ρƒ Π’, Ρ€Π°Π²Π½ΠΎΠΌΠ΅Ρ€Π½ΠΎ со ΡΠΊΠΎΡ€ΠΎΡΡ‚ΡŒΡŽ V двиТСтся ΠΏΠ΅Ρ€Π΅ΠΌΡ‹Ρ‡ΠΊΠ°. Π’Ρ‹Π±Π΅Ρ€ΠΈΡ‚Π΅ Π³Ρ€Π°Ρ„ΠΈΠΊ измСнСния Π²ΠΎ Π²Ρ€Π΅ΠΌΠ΅Π½ΠΈ ΠΏΠΎΠΊΠ°Π·Π°Π½ΠΈΠΉ Π°ΠΌΠΏΠ΅Ρ€ΠΌΠ΅Ρ‚Ρ€Π° ΠΈ ΡƒΠΊΠ°ΠΆΠΈΡ‚Π΅ Π½Π°ΠΏΡ€Π°Π²Π»Π΅Π½ΠΈΠ΅ Ρ‚ΠΎΠΊΠ° Π² ΠΏΠ΅Ρ€Π΅ΠΌΡ‹Ρ‡ΠΊΠ΅. Π‘ΠΎΠΏΡ€ΠΎΡ‚ΠΈΠ²Π»Π΅Π½ΠΈΠ΅ ΠΏΡ€ΠΎΠ²ΠΎΠ΄Π½ΠΈΠΊΠΎΠ² Ρ€Π°Π²Π½ΠΎ Π½ΡƒΠ»ΡŽ.

1=Π“Ρ€Π°Ρ„ΠΈΠΊ 1, Π½Π°ΠΏΡ€Π°Π²Π»Π΅Π½ΠΈΠ΅ Ρ‚ΠΎΠΊΠ° свСрху Π²Π½ΠΈΠ·.

2=Π“Ρ€Π°Ρ„ΠΈΠΊ 2, Π½Π°ΠΏΡ€Π°Π²Π»Π΅Π½ΠΈΠ΅ Ρ‚ΠΎΠΊΠ° свСрху Π²Π½ΠΈΠ·. 3=Π“Ρ€Π°Ρ„ΠΈΠΊ 3, Π½Π°ΠΏΡ€Π°Π²Π»Π΅Π½ΠΈΠ΅ Ρ‚ΠΎΠΊΠ° снизу Π²Π²Π΅Ρ€Ρ…. 4=Π“Ρ€Π°Ρ„ΠΈΠΊ 4, Π½Π°ΠΏΡ€Π°Π²Π»Π΅Π½ΠΈΠ΅ Ρ‚ΠΎΠΊΠ° снизу Π²Π²Π΅Ρ€Ρ….

5=ΠŸΡ€Π°Π²ΠΈΠ»ΡŒΠ½Ρ‹ΠΉ Π³Ρ€Π°Ρ„ΠΈΠΊ Π½Π΅ ΠΏΡ€ΠΈΠ²Π΅Π΄Π΅Π½.Π½Π°ΠΏΡ€Π°Π²Π»Π΅Π½ΠΈΠ΅ Ρ‚ΠΎΠΊΠ° зависит ΠΎΡ‚ Π²Π΅Π»ΠΈΡ‡ΠΈΠ½Ρ‹ скорости. [task#4]

Π‘ΠΎΠ»Π΅Π½ΠΎΠΈΠ΄ большой индуктивности подсоСдиняСтся ΠΊ сСти постоянного Ρ‚ΠΎΠΊΠ°. Как Π²Π΅Π΄Π΅Ρ‚ сСбя Ρ‚ΠΎΠΊ Π² солСноидС Π² ΠΌΠΎΠΌΠ΅Π½Ρ‚ Π²ΠΊΠ»ΡŽΡ‡Π΅Π½ΠΈΡ?

1=Π’ΠΎΠΊ устанавливаСтся ΠΌΠ³Π½ΠΎΠ²Π΅Π½Π½ΠΎ Π² соотвСтствии с Π·Π°ΠΊΠΎΠ½ΠΎΠΌ Ома.

2=Π’ΠΎΠΊ нарастаСт постСпСнно.

3=УстанавливаСтся Ρ‚ΠΎΠΊ мСньший, Ρ‡Π΅ΠΌ ΠΏΠΎ Π·Π°ΠΊΠΎΠ½Ρƒ Ома. 4=УстанавливаСтся Ρ‚ΠΎΠΊ больший, Ρ‡Π΅ΠΌ ΠΏΠΎ Π·Π°ΠΊΠΎΠ½Ρƒ Ома. 5=Π’ ΠΌΠΎΠΌΠ΅Π½Ρ‚ Π²ΠΊΠ»ΡŽΡ‡Π΅Π½ΠΈΡ Π²ΠΎΠ·Π½ΠΈΠΊΠ°Π΅Ρ‚ ΠΊΠΎΡ€ΠΎΡ‚ΠΊΠΎΠ΅ Π·Π°ΠΌΡ‹ΠΊΠ°Π½ΠΈΠ΅. [task#5]

ΠŸΡ€Π΅Π΄ΡΡ‚Π°Π²Π»Π΅Π½Π½Π°Ρ Π½Π° рисункС систСма ΡƒΡ€Π°Π²Π½Π΅Π½ΠΈΠΉ МаксвСлла справСдлива для элСктромагнитного поля…

1=…Ρ‚ΠΎΠ»ΡŒΠΊΠΎ, Ссли ΠΏΠΎΠ»Π΅ стационарно. 2=…Ρ‚ΠΎΠ»ΡŒΠΊΠΎ Π² отсутствиС Ρ‚ΠΎΠΊΠΎΠ² проводимости.

3=…Ρ‚ΠΎΠ»ΡŒΠΊΠΎ Π² отсутствиС заряТСнных Ρ‚Π΅Π».

4=…Ρ‚ΠΎΠ»ΡŒΠΊΠΎ Π² отсутствиС заряТСнных Ρ‚Π΅Π» ΠΈ Ρ‚ΠΎΠΊΠΎΠ² проводимости. 5=…ΠΏΡ€ΠΈ Π½Π°Π»ΠΈΡ‡ΠΈΠΈ заряТСнных Ρ‚Π΅Π» ΠΈ Ρ‚ΠΎΠΊΠΎΠ² проводимости.

3=… Н ΠΈ ΞΌΠΎΒ·Π’.

4=… Ниμо·J.

variant_number=05

[task#1]

ΠœΠ°Π³Π½ΠΈΡ‚Π½Π°Ρ Π²ΠΎΡΠΏΡ€ΠΈΠΈΠΌΡ‡ΠΈΠ²ΠΎΡΡ‚ΡŒ «hi» являСтся коэффициСнтом ΠΏΡ€ΠΎΠΏΠΎΡ€Ρ†ΠΈΠΎΠ½Π°Π»ΡŒΠ½ΠΎΡΡ‚ΠΈ ΠΌΠ΅ΠΆΠ΄Ρƒ физичСскими Π²Π΅Π»ΠΈΡ‡ΠΈΠ½Π°ΠΌΠΈ (Н β€” Π²Π΅ΠΊΡ‚ΠΎΡ€ напряТСнности ΠΌΠ°Π³Π½ΠΈΡ‚Π½ΠΎΠ³ΠΎ поля, Π’ β€”Π²Π΅ΠΊΡ‚ΠΎΡ€ΠΌΠ°Π³Π½ΠΈΡ‚Π½ΠΎΠΉ ΠΈΠ½Π΄ΡƒΠΊΡ†ΠΈΠΈ, J—вСкторнамагничСнности,μо—магнитнаяпостоянная)…

1=…НиВ. 2=… J иН. 5=… JΠΈΠ’. [task#2]

ΠŸΡ€ΠΈ ΡƒΠ²Π΅Π»ΠΈΡ‡Π΅Π½ΠΈΠΈ напряТСнности ΠΌΠ°Π³Π½ΠΈΡ‚Π½ΠΎΠ³ΠΎ поля Π² Π½Π΅ΠΊΠΎΡ‚ΠΎΡ€ΠΎΠΉ области Π² Ρ‡Π΅Ρ‚Ρ‹Ρ€Π΅ Ρ€Π°Π·Π° объСмная ΠΏΠ»ΠΎΡ‚Π½ΠΎΡΡ‚ΡŒ энСргии ΠΌΠ°Π³Π½ΠΈΡ‚Π½ΠΎΠ³ΠΎ поля…

1=…. ΡƒΠ±Ρ‹Π²Π°Π΅Ρ‚Π²16 Ρ€Π°Π·. 2=…. ΡƒΠ±Ρ‹Π²Π°Π΅Ρ‚ Π² 4 Ρ€Π°Π·Π°. 3=… остаСтся Π½Π΅ΠΈΠ·ΠΌΠ΅Π½Π½ΠΎΠΉ. 4=…. возрастаСт Π² 4 Ρ€Π°Π·Π°.

5=… возрастаСт Π² 16 Ρ€Π°Π·.

[task#3]

ΠŸΡ€ΠΈ ΠΏΠΎΠ΄ΠΊΠ»ΡŽΡ‡Π΅Π½ΠΈΠΈ источника Ρ‚ΠΎΠΊΠ° ΠΊ ΠΊΠ°Ρ‚ΡƒΡˆΠΊΠ΅ стрСлка Π³Π°Π»ΡŒΠ²Π°Π½ΠΎΠΌΠ΅Ρ‚Ρ€Π° отклоняСтся Π²ΠΏΡ€Π°Π²ΠΎ. Как ΠΏΠΎΠ²Π΅Π΄Π΅Ρ‚ сСбя стрСлка Π³Π°Π»ΡŒΠ²Π°Π½ΠΎΠΌΠ΅Ρ‚Ρ€Π°, Ссли источник ΠΎΡ‚ΠΊΠ»ΡŽΡ‡ΠΈΡ‚ΡŒ ΠΈ Π½Π°Ρ‡Π°Ρ‚ΡŒ Π²Π΄Π²ΠΈΠ³Π°Ρ‚ΡŒ Π² ΠΊΠ°Ρ‚ΡƒΡˆΠΊΡƒ постоянный ΠΌΠ°Π³Π½ΠΈΡ‚ сСвСрным полюсом Π²ΠΏΠ΅Ρ€Π΅Π΄?

1=ΠžΡ‚ΠΊΠ»ΠΎΠ½ΠΈΡ‚ΡΡΠ²Π»Π΅Π²ΠΎ.

2=НС отклонится. 3=ΠžΡ‚ΠΊΠ»ΠΎΠ½ΠΈΡ‚ΡΡ Π²ΠΏΡ€Π°Π²ΠΎ.

4=Π‘Π½Π°Ρ‡Π°Π»Π° отклонится Π²ΠΏΡ€Π°Π²ΠΎ, Π·Π°Ρ‚Π΅ΠΌ Π²Π»Π΅Π²ΠΎ. 5=Π‘Π½Π°Ρ‡Π°Π»Π° отклонится Π²Π»Π΅Π²ΠΎ, Π·Π°Ρ‚Π΅ΠΌ β€”Π²ΠΏΡ€Π°Π²ΠΎ.[task#4]

Π§Π΅ΠΌΡƒ Ρ€Π°Π²Π½Π° (Π² Π²ΠΎΠ»ΡŒΡ‚Π°Ρ…) Π­Π”Π‘ самоиндукции Π² ΠΊΠ°Ρ‚ΡƒΡˆΠΊΠ΅ с ΠΈΠ½Π΄ΡƒΠΊΡ‚ΠΈΠ²Π½ΠΎΡΡ‚ΡŒΡŽ 2 Π“Π½, Ссли сила Ρ‚ΠΎΠΊΠ° Π² Π½Π΅ΠΉ Π·Π° 0.1 с Ρ€Π°Π²Π½ΠΎΠΌΠ΅Ρ€Π½ΠΎ ΡƒΠΌΠ΅Π½ΡŒΡˆΠΈΠ»Π°ΡΡŒ с 5 Π΄ΠΎ 3 А?answer1=40.0 % 3

[task#5]

ΠŸΡ€Π΅Π΄ΡΡ‚Π°Π²Π»Π΅Π½Π½Π°Ρ Π½Π° рисункС систСма ΡƒΡ€Π°Π²Π½Π΅Π½ΠΈΠΉ МаксвСлла справСдлива для элСктромагнитного поля…

1=…Ρ‚ΠΎΠ»ΡŒΠΊΠΎ, Ссли ΠΏΠΎΠ»Π΅ стационарно.

2=…Ρ‚ΠΎΠ»ΡŒΠΊΠΎ Π² отсутствиС Ρ‚ΠΎΠΊΠΎΠ² проводимости. 3=..Ρ‚ΠΎΠ»ΡŒΠΊΠΎ Π² отсутствиС заряТСнных Ρ‚Π΅Π».

4=…Ρ‚ΠΎΠ»ΡŒΠΊΠΎ Π² отсутствиС заряТСнных Ρ‚Π΅Π» ΠΈ Ρ‚ΠΎΠΊΠΎΠ² проводимости. 5=…ΠΏΡ€ΠΈ Π½Π°Π»ΠΈΡ‡ΠΈΠΈ заряТСнных Ρ‚Π΅Π» ΠΈ Ρ‚ΠΎΠΊΠΎΠ² проводимости.

variant_number=06

[task#1]

Π’Π΅ΠΌΠΏΠ΅Ρ€Π°Ρ‚ΡƒΡ€Π° ΠšΡŽΡ€ΠΈ-этотСмпСратура, Π²Ρ‹ΡˆΠ΅ ΠΊΠΎΡ‚ΠΎΡ€ΠΎΠΉ…

1=… ΠΏΠ°Ρ€Π°ΠΌΠ°Π³Π½Π΅Ρ‚ΠΈΠΊΠΈ Ρ‚Π΅Ρ€ΡΡŽΡ‚ свои свойства ΠΈ становятся Ρ„Π΅Ρ€Ρ€ΠΎΠΌΠ°Π³Π½Π΅Ρ‚ΠΈΠΊΠ°ΠΌΠΈ. 2=… Ρ„Π΅Ρ€Ρ€ΠΎΠΌΠ°Π³Π½Π΅Ρ‚ΠΈΠΊΠΈ Ρ‚Π΅Ρ€ΡΡŽΡ‚ свои свойства ΠΈ становятся Π΄ΠΈΠ°ΠΌΠ°Π³Π½Π΅Ρ‚ΠΈΠΊΠ°ΠΌΠΈ. 3=… ΠΏΠ°Ρ€Π°ΠΌΠ°Π³Π½Π΅Ρ‚ΠΈΠΊΠΈ Ρ‚Π΅Ρ€ΡΡŽΡ‚ свои свойства ΠΈ становятся Π΄ΠΈΠ°ΠΌΠ°Π³Π½Π΅Ρ‚ΠΈΠΊΠ°ΠΌΠΈ.

4=… Ρ„Π΅Ρ€Ρ€ΠΎΠΌΠ°Π³Π½Π΅Ρ‚ΠΈΠΊΠΈ Ρ‚Π΅Ρ€ΡΡŽΡ‚ свои свойства ΠΈ становятся ΠΏΠ°Ρ€Π°ΠΌΠ°Π³Π½Π΅Ρ‚ΠΈΠΊΠ°ΠΌΠΈ.

5=… Π΄ΠΈΠ°ΠΌΠ°Π³Π½Π΅Ρ‚ΠΈΠΊΠΈ Ρ‚Π΅Ρ€ΡΡŽΡ‚ свои свойства ΠΈ становятся ΠΏΠ°Ρ€Π°ΠΌΠ°Π³Π½Π΅Ρ‚ΠΈΠΊΠ°ΠΌΠΈ. [task#2]

Π’ области «A» магнитная ΠΏΡ€ΠΎΠ½ΠΈΡ†Π°Π΅ΠΌΠΎΡΡ‚ΡŒ Π² Ρ‚Ρ€ΠΈ Ρ€Π°Π·Π° большС Ρ‡Π΅ΠΌ Π² области «B» ΠΏΡ€ΠΈ ΠΎΠ΄ΠΈΠ½Π°ΠΊΠΎΠ²ΠΎΠΉ Π²Π΅Π»ΠΈΡ‡ΠΈΠ½Π΅ напряТСнности ΠΌΠ°Π³Π½ΠΈΡ‚Π½ΠΎΠ³ΠΎ поля. Π‘Ρ€Π°Π²Π½ΠΈΡ‚Π΅ ΠΎΠ±ΡŠΠ΅ΠΌΠ½Ρ‹Π΅ плотности энСргии Π² этих областях.

1=Π’ области «A» Π² 9 Ρ€Π°Π·Π° большС. 2=Π’ области «B» Π² 9 Ρ€Π°Π·Π° большС.

3=Π’ области «A» Π² 3 Ρ€Π°Π·Π° большС.

4=Π’ области «B» Π² 3 Ρ€Π°Π·Π° большС. 5=Одинакова.

[task#3]

ΠšΠΎΠ½Ρ‚ΡƒΡ€ΠΏΠ»ΠΎΡ‰Π°Π΄ΡŒΡŽ50 см2 находится Π² ΠΎΠ΄Π½ΠΎΡ€ΠΎΠ΄Π½ΠΎΠΌ ΡƒΠ±Ρ‹Π²Π°ΡŽΡ‰Π΅ΠΌ ΠΌΠ°Π³Π½ΠΈΡ‚Π½ΠΎΠΌ ΠΏΠΎΠ»Π΅. ΠœΠ°Π³Π½ΠΈΡ‚Π½Π°Ρ индукция измСняСтся ΠΏΠΎ ΡƒΠΊΠ°Π·Π°Π½Π½ΠΎΠΉ Ρ„ΠΎΡ€ΠΌΡƒΠ»Π΅, Π³Π΄Π΅ константы Π’ΠΎ = 0,1 Π’Π», А = 10 Π’Π»/с. Π£Π³ΠΎΠ» ΠΌΠ΅ΠΆΠ΄Ρƒ Π²Π΅ΠΊΡ‚ΠΎΡ€ΠΎΠΌ Π’ ΠΈ Π½ΠΎΡ€ΠΌΠ°Π»ΡŒΡŽ n ΠΊ повСрхности ΠΊΠΎΠ½Ρ‚ΡƒΡ€Π° составляСт 60Β°. Π£ΠΊΠ°ΠΆΠΈΡ‚Π΅ Π²Π΅Π»ΠΈΡ‡ΠΈΠ½Ρƒ ΠΈΠ½Π΄ΡƒΡ†ΠΈΡ€ΡƒΠ΅ΠΌΠΎΠΉ Π² ΠΊΠΎΠ½Ρ‚ΡƒΡ€Π΅ Π­Π”Π‘ (Π² ΠΌΠΈΠ»Π»ΠΈΠ²ΠΎΠ»ΡŒΡ‚Π°Ρ…)?answer1=25.0 % 2

[task#4]

Β 

Β 

Β 

Β 

Β 

Π•Π΄ΠΈΠ½ΠΈΡ†Π΅ΠΉ измСрСния индуктивности являСтся…

Β 

Β 

Β 

1=… ВСсла.

Β 

2=… Π“Π΅Π½Ρ€ΠΈ.

3=… Π’Π΅Π±Π΅Ρ€.

4=… БимСнс.

5=… Ом.

[task#5]

Β 

Β 

Β 

Β 

Β 

Для ΠΊΠΎΠ½Ρ‚ΡƒΡ€Π° L, ΠΈΠ·ΠΎΠ±Ρ€Π°ΠΆΠ΅Π½Π½ΠΎΠ³ΠΎ Π½Π° рисункС циркуляция Π²Π΅ΠΊΡ‚ΠΎΡ€Π° ΠΌΠ°Π³Π½ΠΈΡ‚Π½ΠΎΠΉ напряТСнности Ρ€Π°Π²Π½Π°…

Β 

1=… β€”J1β€”J2β€”J3.

Β 

Β 

Β 

Β 

2=… J1 + J2 + J3.

Β 

Β 

Β 

Β 

3=… β€”J1+ J2β€”J3.

Β 

Β 

Β 

Β 

4=… J1 β€”J2+ J3.

Β 

Β 

Β 

Β 

5=ΠžΡ‚Π²Π΅Ρ‚ зависит ΠΎΡ‚ Ρ‚ΠΈΠΏΠ° ΠΌΠ°Π³Π½Π΅Ρ‚ΠΈΠΊΠ°.

Β 

Β 

Β 

[task#1]

Β 

Β 

variant_number=07

Β 

Β 

Β 

Β 

Β 

Β 

На рисункС прСдставлСны Ρ‡Π΅Ρ‚Ρ‹Ρ€Π΅ Π²Π°Ρ€ΠΈΠ°Π½Ρ‚Π° Π²Π·Π°ΠΈΠΌΠ½ΠΎΠ³ΠΎ размСщСния солСноида S ΠΈ Ρ†ΠΈΠ»ΠΈΠ½Π΄Ρ€Π° F ΠΈΠ· магнитомягкого ΠΆΠ΅Π»Π΅Π·Π°.

Π’Ρ‹Π±Π΅Ρ€ΠΈΡ‚Π΅ Π²Π°Ρ€ΠΈΠ°Π½Ρ‚Ρ‹, ΠΊΠΎΡ‚ΠΎΡ€Ρ‹Π΅ ΡΠΎΠΎΡ‚Π²Π΅Ρ‚ΡΡ‚Π²ΡƒΡŽΡ‚ Π²Ρ‚ΡΠ³ΠΈΠ²Π°Π½ΠΈΡŽ Ρ†ΠΈΠ»ΠΈΠ½Π΄Ρ€Π° Π² солСноид.

Β 

1=A ΠΈB.

2=A ΠΈC.

3=B ΠΈD.

4=CΠΈD.

5=A, B, CΠΈD.

Β 

[task#2]

ΠŸΡ€ΠΈ ΠΏΠΎΠ΄ΠΊΠ»ΡŽΡ‡Π΅Π½ΠΈΠΈ ΠΊΠ°Ρ‚ΡƒΡˆΠΊΠΈ с ΠΈΠ½Π΄ΡƒΠΊΡ‚ΠΈΠ²Π½ΠΎΡΡ‚ΡŒΡŽ L ΠΈ сопротивлСниСм R ΠΊ источнику Ρ‚ΠΎΠΊ Π² ΠΊΠ°Ρ‚ΡƒΡˆΠΊΠ΅ возрастаСт ΠΎΡ‚ нуля Π΄ΠΎ I. ΠŸΡ€ΠΈ этом сторонниС силы Π² источникС ΡΠΎΠ²Π΅Ρ€ΡˆΠ°ΡŽΡ‚ Ρ€Π°Π±ΠΎΡ‚Ρƒ Аст ΠΈ Π² схСмС выдСляСтсятСплота Q. Π£ΠΊΠ°ΠΆΠΈΡ‚Π΅ Π½ΠΎΠΌΠ΅Ρ€ ΠΏΡ€Π°Π²ΠΈΠ»ΡŒΠ½ΠΎΠ³ΠΎ выраТСния для разности Аст β€”Q.answer1=3

[task#3]

ΠžΠΏΡ€Π΅Π΄Π΅Π»ΠΈΡ‚Π΅ Π°ΠΌΠΏΠ»ΠΈΡ‚ΡƒΠ΄Π½ΠΎΠ΅ Π·Π½Π°Ρ‡Π΅Π½ΠΈΠ΅ (Π² Π²ΠΎΠ»ΡŒΡ‚Π°Ρ…) ΠΏΠ΅Ρ€Π΅ΠΌΠ΅Π½Π½ΠΎΠΉ Π­Π”Π‘, Π²ΠΎΠ·Π½ΠΈΠΊΠ°ΡŽΡ‰Π΅ΠΉ Π² Ρ€Π°ΠΌΠΊΠ΅ ΠΏΡ€ΠΈ Π΅Π΅ Ρ€Π°Π²Π½ΠΎΠΌΠ΅Ρ€Π½ΠΎΠΌ Π²Ρ€Π°Ρ‰Π΅Π½ΠΈΠΈ Π² ΠΎΠ΄Π½ΠΎΡ€ΠΎΠ΄Π½ΠΎΠΌ ΠΌΠ°Π³Π½ΠΈΡ‚Π½ΠΎΠΌ ΠΏΠΎΠ»Π΅, Ссли ΠΏΡ€ΠΈ ΡƒΠ³Π»Π΅ «fi» = 45Β° ΠΌΠ³Π½ΠΎΠ²Π΅Π½Π½ΠΎΠ΅ Π·Π½Π°Ρ‡Π΅Π½ΠΈΠ΅ Π­Π”Π‘ Ρ€Π°Π²Π½ΠΎ 100 Π’.answer1=141.0 % 1

[task#4]

Какая ΠΈΠ· Π»Π°ΠΌΠΏ Π² прСдставлСнной схСмС загорится ΠΏΠΎΠ·ΠΆΠ΅ всСх ΠΎΡΡ‚Π°Π»ΡŒΠ½Ρ‹Ρ… ΠΏΡ€ΠΈ Π·Π°ΠΌΡ‹ΠΊΠ°Π½ΠΈΠΈ ΠΊΠ»ΡŽΡ‡Π°? Π‘ΠΎΠΏΡ€ΠΎΡ‚ΠΈΠ²Π»Π΅Π½ΠΈΠ΅ R2 Π²Π΄Π²ΠΎΠ΅

большС R1.

Β 

3=3

Β 

Β 

1=1

2=2

4=4

5=ВсС Π»Π°ΠΌΠΏΡ‹ загорятсяодноврСмСнно.

[task#5]

Β 

Β 

Β 

Β 

Циркуляция напряТСнности элСктричСского поля ΠΏΠΎ ΠΏΡ€ΠΎΠΈΠ·Π²ΠΎΠ»ΡŒΠ½ΠΎΠΌΡƒ Π½Π΅ΠΏΠΎΠ΄Π²ΠΈΠΆΠ½ΠΎΠΌΡƒ Π·Π°ΠΌΠΊΠ½ΡƒΡ‚ΠΎΠΌΡƒ ΠΊΠΎΠ½Ρ‚ΡƒΡ€Ρƒ всСгда Ρ€Π°Π²Π½Π° …

1=… Π½ΡƒΠ»ΡŽ.

2=… ΠΏΠΎΡ‚ΠΎΠΊΡƒ плотности Ρ‚ΠΎΠΊΠ° Ρ‡Π΅Ρ€Π΅Π· ΠΏΠΎΠ²Π΅Ρ€Ρ…Π½ΠΎΡΡ‚ΡŒ, Π½Π°Ρ‚ΡΠ½ΡƒΡ‚ΡƒΡŽ Π½Π° этот ΠΊΠΎΠ½Ρ‚ΡƒΡ€.

3=… взятому с ΠΎΠ±Ρ€Π°Ρ‚Π½Ρ‹ΠΌ Π·Π½Π°ΠΊΠΎΠΌ ΠΏΠΎΡ‚ΠΎΠΊΡƒ плотности Ρ‚ΠΎΠΊΠ° Ρ‡Π΅Ρ€Π΅Π· ΠΏΠΎΠ²Π΅Ρ€Ρ…Π½ΠΎΡΡ‚ΡŒ, Π½Π°Ρ‚ΡΠ½ΡƒΡ‚ΡƒΡŽ Π½Π° этот ΠΊΠΎΠ½Ρ‚ΡƒΡ€. 4=… скорости измСнСния ΠΌΠ°Π³Π½ΠΈΡ‚Π½ΠΎΠ³ΠΎ ΠΏΠΎΡ‚ΠΎΠΊΠ° Ρ‡Π΅Ρ€Π΅Π· ΠΏΠΎΠ²Π΅Ρ€Ρ…Π½ΠΎΡΡ‚ΡŒ, Π½Π°Ρ‚ΡΠ½ΡƒΡ‚ΡƒΡŽ Π½Π° этот ΠΊΠΎΠ½Ρ‚ΡƒΡ€.

5=… взятой с ΠΎΠ±Ρ€Π°Ρ‚Π½Ρ‹ΠΌ Π·Π½Π°ΠΊΠΎΠΌ скорости измСнСния ΠΌΠ°Π³Π½ΠΈΡ‚Π½ΠΎΠ³ΠΎ ΠΏΠΎΡ‚ΠΎΠΊΠ° Ρ‡Π΅Ρ€Π΅Π· ΠΏΠΎΠ²Π΅Ρ€Ρ…Π½ΠΎΡΡ‚ΡŒ, Π½Π°Ρ‚ΡΠ½ΡƒΡ‚ΡƒΡŽ Π½Π° этот ΠΊΠΎΠ½Ρ‚ΡƒΡ€.

variant_number=08

[task#1]

Π’Ρ‹Π±Π΅Ρ€ΠΈΡ‚Π΅ ΠΏΡ€Π°Π²ΠΈΠ»ΡŒΠ½ΠΎΠ΅ Π²Ρ‹Ρ€Π°ΠΆΠ΅Π½ΠΈΠ΅ для Π²Π΅ΠΊΡ‚ΠΎΡ€Π° намагничСнности. answer1=2

[task#2]

К источнику напряТСния U ΠΏΠΎΠ΄ΠΊΠ»ΡŽΡ‡Π΅Π½Π° ΠΊΠ°Ρ‚ΡƒΡˆΠΊΠ° индуктивности L. Как измСнится полная энСргия, запасСнная Π² ΠΌΠ°Π³Π½ΠΈΡ‚Π½ΠΎΠΌ ΠΏΠΎΠ»Π΅ систСмы, Ссли ΠΏΠΎΡΠ»Π΅Π΄ΠΎΠ²Π°Ρ‚Π΅Π»ΡŒΠ½ΠΎ с ΠΏΠ΅Ρ€Π²ΠΎΠΉ ΠΏΠΎΠ΄ΠΊΠ»ΡŽΡ‡ΠΈΡ‚ΡŒ Π²Ρ‚ΠΎΡ€ΡƒΡŽ Ρ‚Π°ΠΊΡƒΡŽ ΠΆΠ΅ ΠΊΠ°Ρ‚ΡƒΡˆΠΊΡƒ ?

1=УвСличится Π² 2 Ρ€Π°Π·Π°.

2=Π£ΠΌΠ΅Π½ΡŒΡˆΠΈΡ‚ΡΡ Π² 2 Ρ€Π°Π·Π°.

3=УвСличится Π² 4 Ρ€Π°Π·Π°. 4=Π£ΠΌΠ΅Π½ΡŒΡˆΠΈΡ‚ΡΡ Π² 4 Ρ€Π°Π·Π°. 5=ЭнСргия останСтся Ρ‚Π°ΠΊΠΎΠΉ ΠΆΠ΅. [task#3]

ΠœΠ°Π³Π½ΠΈΡ‚Π½Ρ‹ΠΉ ΠΏΠΎΡ‚ΠΎΠΊ, ΠΏΡ€ΠΎΠ½ΠΈΠ·Ρ‹Π²Π°ΡŽΡ‰ΠΈΠΉ ΠΊΠ°Ρ‚ΡƒΡˆΠΊΡƒ, измСняСтся со Π²Ρ€Π΅ΠΌΠ΅Π½Π΅ΠΌ Π² соотвСтствии с Π³Ρ€Π°Ρ„ΠΈΠΊΠΎΠΌ. НайдитС (Π² Π²ΠΎΠ»ΡŒΡ‚Π°Ρ…) максимальноС Π·Π½Π°Ρ‡Π΅Π½ΠΈΠ΅ модуля ΠΈΠ½Π΄ΡƒΠΊΡ†ΠΈΠΎΠ½Π½ΠΎΠΉ Π­Π”Π‘. answer1=4.0 % 3

[task#4]

Π˜Π½Π΄ΡƒΠΊΡ‚ΠΈΠ²Π½ΠΎΡΡ‚ΡŒΡŽ Π·Π°ΠΌΠΊΠ½ΡƒΡ‚ΠΎΠ³ΠΎ проводящСго ΠΊΠΎΠ½Ρ‚ΡƒΡ€Π° называСтся Π²Π΅Π»ΠΈΡ‡ΠΈΠ½Π°, равная …

1=… ΠΎΡ‚Π½ΠΎΡˆΠ΅Π½ΠΈΡŽ циркуляции ΠΌΠ°Π³Π½ΠΈΡ‚Π½ΠΎΠΉ ΠΈΠ½Π΄ΡƒΠΊΡ†ΠΈΠΈ ΠΊ силС Ρ‚ΠΎΠΊΠ° Π² этом ΠΊΠΎΠ½Ρ‚ΡƒΡ€Π΅. 2=…ΠΏΡ€ΠΎΠΈΠ·Π²Π΅Π΄Π΅Π½ΠΈΡŽ потокосцСплСния самоиндукции ΠΊΠΎΠ½Ρ‚ΡƒΡ€Π° Π½Π° силу Ρ‚ΠΎΠΊΠ° Π² этом ΠΊΠΎΠ½Ρ‚ΡƒΡ€Π΅ .

3=… взятому с ΠΎΠ±Ρ€Π°Ρ‚Π½Ρ‹ΠΌ Π·Π½Π°ΠΊΠΎΠΌ ΠΏΡ€ΠΎΠΈΠ·Π²Π΅Π΄Π΅Π½ΠΈΡŽ потокосцСплСния самоиндукции ΠΊΠΎΠ½Ρ‚ΡƒΡ€Π° Π½Π° силу Ρ‚ΠΎΠΊΠ° Π² этом ΠΊΠΎΠ½Ρ‚ΡƒΡ€Π΅ .

4=…ΠΎΡ‚Π½ΠΎΡˆΠ΅Π½ΠΈΡŽ потокосцСплСния самоиндукции ΠΊΠΎΠ½Ρ‚ΡƒΡ€Π° ΠΊ силС Ρ‚ΠΎΠΊΠ° Π² этом ΠΊΠΎΠ½Ρ‚ΡƒΡ€Π΅ .

5=… взятому с ΠΎΠ±Ρ€Π°Ρ‚Π½Ρ‹ΠΌ Π·Π½Π°ΠΊΠΎΠΌ ΠΎΡ‚Π½ΠΎΡˆΠ΅Π½ΠΈΡŽ потокосцСплСния самоиндукции ΠΊΠΎΠ½Ρ‚ΡƒΡ€Π° ΠΊ силС Ρ‚ΠΎΠΊΠ° Π² этом ΠΊΠΎΠ½Ρ‚ΡƒΡ€Π΅ . [task#5]

ΠŸΠΎΠ²Π΅Ρ€Ρ…Π½ΠΎΡΡ‚ΡŒ S ΠΎΠ³Ρ€Π°Π½ΠΈΡ‡Π΅Π½Π° Π½Π΅ΠΏΠΎΠ΄Π²ΠΈΠΆΠ½Ρ‹ΠΌ Π·Π°ΠΌΠΊΠ½ΡƒΡ‚Ρ‹ΠΌ ΠΊΠΎΠ½Ρ‚ΡƒΡ€ΠΎΠΌ L. Циркуляция напряТСнности элСктричСского поля ΠΏΠΎ ΠΊΠΎΠ½Ρ‚ΡƒΡ€Ρƒ L всСгда Ρ€Π°Π²Π½Π° …

1=… Π½ΡƒΠ»ΡŽ.

2=… скорости измСнСния ΠΌΠ°Π³Π½ΠΈΡ‚Π½ΠΎΠ³ΠΎ ΠΏΠΎΡ‚ΠΎΠΊΠ° Ρ‡Π΅Ρ€Π΅Π· ΠΏΠΎΠ²Π΅Ρ€Ρ…Π½ΠΎΡΡ‚ΡŒ S.

3=… взятому с ΠΎΠ±Ρ€Π°Ρ‚Π½Ρ‹ΠΌ Π·Π½Π°ΠΊΠΎΠΌ ΠΏΠΎΡ‚ΠΎΠΊΡƒ плотности Ρ‚ΠΎΠΊΠ° Ρ‡Π΅Ρ€Π΅Π· ΠΏΠΎΠ²Π΅Ρ€Ρ…Π½ΠΎΡΡ‚ΡŒ S 4=… ΠΏΠΎΡ‚ΠΎΠΊΡƒ плотности Ρ‚ΠΎΠΊΠ° Ρ‡Π΅Ρ€Π΅Π· ΠΏΠΎΠ²Π΅Ρ€Ρ…Π½ΠΎΡΡ‚ΡŒ S.

5=…взятому с ΠΎΠ±Ρ€Π°Ρ‚Π½Ρ‹ΠΌ Π·Π½Π°ΠΊΠΎΠΌ ΠΏΠΎΡ‚ΠΎΠΊΡƒ Π²Π΅ΠΊΡ‚ΠΎΡ€Π° скорости измСнСния ΠΌΠ°Π³Π½ΠΈΡ‚Π½ΠΎΠΉ ΠΈΠ½Π΄ΡƒΠΊΡ†ΠΈΠΈ Ρ‡Π΅Ρ€Π΅Π· ΠΏΠΎΠ²Π΅Ρ€Ρ…Π½ΠΎΡΡ‚ΡŒ S.

variant_number=09

[task#1]

Π’ ΠΊΠ°ΠΊΠΎΠΉ строкС ΠΏΡ€Π°Π²ΠΈΠ»ΡŒΠ½ΠΎ ΠΎΡ‚Ρ€Π°ΠΆΠ΅Π½Ρ‹ свойства ΠΏΠ°Ρ€Π°ΠΌΠ°Π³Π½Π΅Ρ‚ΠΈΠΊΠΎΠ² ΠΈ ΡΠΎΡΡ‚Π°Π²Π»ΡΡŽΡ‰ΠΈΡ… ΠΈΡ… ΠΌΠΎΠ»Π΅ΠΊΡƒΠ»?(«hi» β€”ΠΌΠ°Π³Π½ΠΈΡ‚Π½Π°ΡΠ²ΠΎΡΠΏΡ€ΠΈΠΈΠΌΡ‡ΠΈΠ²ΠΎΡΡ‚ΡŒ)?

1=Π’Π΅Π»ΠΈΡ‡ΠΈΠ½Π° «hi» малСнькая ΠΈ ΠΏΠΎΠ»ΠΎΠΆΠΈΡ‚Π΅Π»ΡŒΠ½Π°Ρ, собствСнный ΠΌΠ°Π³Π½ΠΈΡ‚Π½Ρ‹ΠΉ ΠΌΠΎΠΌΠ΅Π½Ρ‚ ΠΌΠΎΠ»Π΅ΠΊΡƒΠ» ΠΎΡ‚Π»ΠΈΡ‡Π΅Π½ ΠΎΡ‚ нуля.

2=Π’Π΅Π»ΠΈΡ‡ΠΈΠ½Π° «hi» малСнькая ΠΈ ΠΎΡ‚Ρ€ΠΈΡ†Π°Ρ‚Π΅Π»ΡŒΠ½Π°Ρ, собствСнный ΠΌΠ°Π³Π½ΠΈΡ‚Π½Ρ‹ΠΉ ΠΌΠΎΠΌΠ΅Π½Ρ‚ ΠΌΠΎΠ»Π΅ΠΊΡƒΠ» ΠΎΡ‚Π»ΠΈΡ‡Π΅Π½ ΠΎΡ‚ нуля. 3=Π’Π΅Π»ΠΈΡ‡ΠΈΠ½Π° «hi» большая ΠΈ ΠΏΠΎΠ»ΠΎΠΆΠΈΡ‚Π΅Π»ΡŒΠ½Π°Ρ, собствСнный ΠΌΠ°Π³Π½ΠΈΡ‚Π½Ρ‹ΠΉ ΠΌΠΎΠΌΠ΅Π½Ρ‚ ΠΌΠΎΠ»Π΅ΠΊΡƒΠ» ΠΎΡ‚Π»ΠΈΡ‡Π΅Π½ ΠΎΡ‚ нуля. 4=Π’Π΅Π»ΠΈΡ‡ΠΈΠ½Π° «hi» малСнькая ΠΈ ΠΎΡ‚Ρ€ΠΈΡ†Π°Ρ‚Π΅Π»ΡŒΠ½Π°Ρ, собствСнный ΠΌΠ°Π³Π½ΠΈΡ‚Π½Ρ‹ΠΉ ΠΌΠΎΠΌΠ΅Π½Ρ‚ΠΌΠΎΠ»Π΅ΠΊΡƒΠ» Ρ€Π°Π²Π΅Π½ Π½ΡƒΠ»ΡŽ.

5=Π’Π΅Π»ΠΈΡ‡ΠΈΠ½Π° «hi» большая ΠΈ ΠΏΠΎΠ»ΠΎΠΆΠΈΡ‚Π΅Π»ΡŒΠ½Π°Ρ, собствСнный ΠΌΠ°Π³Π½ΠΈΡ‚Π½Ρ‹ΠΉ ΠΌΠΎΠΌΠ΅Π½Ρ‚ ΠΌΠΎΠ»Π΅ΠΊΡƒΠ» Ρ€Π°Π²Π΅Π½ Π½ΡƒΠ»ΡŽ. [task#2]

БистСма ΠΏΡ€ΠΎΠ²ΠΎΠ΄Π½ΠΈΠΊΠΎΠ² находится Π² ΠΏΠ°Ρ€Π°ΠΌΠ°Π³Π½ΠΈΡ‚Π½ΠΎΠΉ срСдС. Как измСнится энСргия, запасСнная Π² ΠΌΠ°Π³Π½ΠΈΡ‚Π½ΠΎΠΌ ΠΏΠΎΠ»Π΅ , Ссли силу Ρ‚ΠΎΠΊΠ° Π²ΠΎ всСх ΠΏΡ€ΠΎΠ²ΠΎΠ΄Π½ΠΈΠΊΠ°Ρ… ΡƒΠ²Π΅Π»ΠΈΡ‡ΠΈΡ‚ΡŒ Π²Π΄Π²ΠΎΠ΅?

1=НикакнС измСнится. 2=УвСличится Π² Π΄Π²Π° Ρ€Π°Π·Π°. 3=Π£ΠΌΠ΅Π½ΡŒΡˆΠΈΡ‚ΡΡ Π² Π΄Π²Π° Ρ€Π°Π·Π°.

4=УвСличится Π² Ρ‡Π΅Ρ‚Ρ‹Ρ€Π΅ Ρ€Π°Π·Π°.

5=Π£ΠΌΠ΅Π½ΡŒΡˆΠΈΡ‚ΡΡ Π² Ρ‡Π΅Ρ‚Ρ‹Ρ€Π΅ Ρ€Π°Π·Π°.

[task#3]

ΠœΠ΅Ρ‚Π°Π»Π»ΠΈΡ‡Π΅ΡΠΊΠ°Ρ Ρ€Π°ΠΌΠΊΠ° пСрСмСщаСтся Π² ΠΎΠ΄Π½ΠΎΡ€ΠΎΠ΄Π½ΠΎΠΌ ΠΌΠ°Π³Π½ΠΈΡ‚Π½ΠΎΠΌ ΠΏΠΎΠ»Π΅. Π’Ρ‹Π±Π΅Ρ€ΠΈΡ‚Π΅ ΠΠΠ˜Π‘ΠžΠ›Π•Π• ΠΏΠΎΠ»Π½ΠΎΠ΅ описаниС Π΄Π²ΠΈΠΆΠ΅Π½ΠΈΠΉ Ρ€Π°ΠΌΠΊΠΈ, ΠΏΡ€ΠΈ ΠΊΠΎΡ‚ΠΎΡ€Ρ‹Ρ… Π² Π½Π΅ΠΉ НЕ Π²ΠΎΠ·Π½ΠΈΠΊΠ°ΡŽΡ‚ ΠΈΠ½Π΄ΡƒΠΊΡ†ΠΈΠΎΠ½Π½Ρ‹Π΅ Ρ‚ΠΎΠΊΠΈ ?

1.Π’ΠΎΠΊΠΈ наводятся ΠΏΡ€ΠΈ любом ΠΏΠ΅Ρ€Π΅ΠΌΠ΅Ρ‰Π΅Π½ΠΈΠΈ Ρ€Π°ΠΌΠΊΠΈ.

2.Π’Ρ€Π°Ρ‰Π΅Π½ΠΈΠ΅ Π²ΠΎΠΊΡ€ΡƒΠ³ Π»ΠΈΠ½ΠΈΠΉ ΠΈΠ½Π΄ΡƒΠΊΡ†ΠΈΠΈ Π’.

3.ΠŸΠ°Ρ€Π°Π»Π»Π΅Π»ΡŒΠ½ΠΎ линиям ΠΈΠ½Π΄ΡƒΠΊΡ†ΠΈΠΈ Π’.

4.ΠŸΠ΅Ρ€ΠΏΠ΅Π½Π΄ΠΈΠΊΡƒΠ»ΡΡ€Π½ΠΎ линиям ΠΈΠ½Π΄ΡƒΠΊΡ†ΠΈΠΈ Π’.

5.ΠŸΠ°Ρ€Π°Π»Π»Π΅Π»ΡŒΠ½ΠΎ самой сСбС ΠΏΠΎ любой Ρ‚Ρ€Π°Π΅ΠΊΡ‚ΠΎΡ€ΠΈΠΈ.

[task#4]

Π£ΠΊΠ°ΠΆΠΈΡ‚Π΅ Π²Π΅Π»ΠΈΡ‡ΠΈΠ½Ρƒ Π­Π”Π‘ самоиндукции ΠΈ Π½Π°ΠΏΡ€Π°Π²Π»Π΅Π½ΠΈΠ΅ сторонних сил Π² ΠΊΠ°Ρ‚ΡƒΡˆΠΊΠ΅ с ΠΈΠ½Π΄ΡƒΠΊΡ‚ΠΈΠ²Π½ΠΎΡΡ‚ΡŒΡŽ 2 Π“Π½, Ссли сила Ρ‚ΠΎΠΊΠ° Π² Π½Π΅ΠΉ Π·Π° 0.2 с Ρ€Π°Π²Π½ΠΎΠΌΠ΅Ρ€Π½ΠΎ ΡƒΠ²Π΅Π»ΠΈΡ‡ΠΈΠ»Π°ΡΡŒ с 3 Π΄ΠΎ 5 А?

1.Π­Π”Π‘ = 0.8 Π’, Π½Π°ΠΏΡ€Π°Π²Π»Π΅Π½ΠΈΠ΅ сторонних сил совпадаСт с Π½Π°ΠΏΡ€Π°Π²Π»Π΅Π½ΠΈΠ΅ΠΌ Ρ‚ΠΎΠΊΠ°. 2.Π­Π”Π‘ = 0.8 Π’, Π½Π°ΠΏΡ€Π°Π²Π»Π΅Π½ΠΈΠ΅ сторонних сил совпадаСт с Π½Π°ΠΏΡ€Π°Π²Π»Π΅Π½ΠΈΠ΅ΠΌ Ρ‚ΠΎΠΊΠ°. 3.Π­Π”Π‘ = 20 Π’, Π½Π°ΠΏΡ€Π°Π²Π»Π΅Π½ΠΈΠ΅ сторонних сил совпадаСт с Π½Π°ΠΏΡ€Π°Π²Π»Π΅Π½ΠΈΠ΅ΠΌ Ρ‚ΠΎΠΊΠ°.

4.Π­Π”Π‘ = 20 Π’, Π½Π°ΠΏΡ€Π°Π²Π»Π΅Π½ΠΈΠ΅ сторонних сил ΠΏΡ€ΠΎΡ‚ΠΈΠ²ΠΎΠΏΠΎΠ»ΠΎΠΆΠ½ΠΎ Π½Π°ΠΏΡ€Π°Π²Π»Π΅Π½ΠΈΡŽ Ρ‚ΠΎΠΊΠ°.

5.Π­Π”Π‘ = 40 Π’, Π½Π°ΠΏΡ€Π°Π²Π»Π΅Π½ΠΈΠ΅ сторонних сил Π½Π΅ ΠΎΠΏΡ€Π΅Π΄Π΅Π»Π΅Π½ΠΎ. [task#5]

ΠŸΠΎΠ²Π΅Ρ€Ρ…Π½ΠΎΡΡ‚ΡŒ S ΠΎΠ³Ρ€Π°Π½ΠΈΡ‡Π΅Π½Π° Π½Π΅ΠΏΠΎΠ΄Π²ΠΈΠΆΠ½Ρ‹ΠΌ Π·Π°ΠΌΠΊΠ½ΡƒΡ‚Ρ‹ΠΌ ΠΊΠΎΠ½Ρ‚ΡƒΡ€ΠΎΠΌ L. Циркуляция напряТСнности ΠΌΠ°Π³Π½ΠΈΡ‚Π½ΠΎΠ³ΠΎ поля ΠΏΠΎ ΠΊΠΎΠ½Ρ‚ΡƒΡ€Ρƒ L всСгда Ρ€Π°Π²Π½Π° …

1=…взятому с ΠΎΠ±Ρ€Π°Ρ‚Π½Ρ‹ΠΌ Π·Π½Π°ΠΊΠΎΠΌ ΠΏΠΎΡ‚ΠΎΠΊΡƒ Π²Π΅ΠΊΡ‚ΠΎΡ€Π° скорости измСнСния элСктричСской ΠΈΠ½Π΄ΡƒΠΊΡ†ΠΈΠΈ Ρ‡Π΅Ρ€Π΅Π· ΠΏΠΎΠ²Π΅Ρ€Ρ…Π½ΠΎΡΡ‚ΡŒ S. 2=… ΠΏΠΎΡ‚ΠΎΠΊΡƒ плотности Ρ‚ΠΎΠΊΠ° Ρ‡Π΅Ρ€Π΅Π· ΠΏΠΎΠ²Π΅Ρ€Ρ…Π½ΠΎΡΡ‚ΡŒ S.

3=… суммС ΠΏΠΎΡ‚ΠΎΠΊΠ° плотности элСктричСского Ρ‚ΠΎΠΊΠ° ΠΈ ΠΏΠΎΡ‚ΠΎΠΊΠ° Π²Π΅ΠΊΡ‚ΠΎΡ€Π° скорости измСнСния элСктричСской ΠΈΠ½Π΄ΡƒΠΊΡ†ΠΈΠΈ Ρ‡Π΅Ρ€Π΅Π· ΠΏΠΎΠ²Π΅Ρ€Ρ…Π½ΠΎΡΡ‚ΡŒ S.

4=… разности ΠΏΠΎΡ‚ΠΎΠΊΠ° плотности элСктричСского Ρ‚ΠΎΠΊΠ° ΠΈ ΠΏΠΎΡ‚ΠΎΠΊΠ° Π²Π΅ΠΊΡ‚ΠΎΡ€Π° скорости измСнСния элСктричСской ΠΈΠ½Π΄ΡƒΠΊΡ†ΠΈΠΈ Ρ‡Π΅Ρ€Π΅Π· ΠΏΠΎΠ²Π΅Ρ€Ρ…Π½ΠΎΡΡ‚ΡŒ S.

5=… Π½ΡƒΠ»ΡŽ.

variant_number=10

[task#1]

Π£ΠΊΠ°ΠΆΠΈΡ‚Π΅ ΠΏΡ€Π°Π²ΠΈΠ»ΡŒΠ½ΠΎΠ΅ Π²Ρ‹Ρ€Π°ΠΆΠ΅Π½ΠΈΠ΅ для Π²Π΅ΠΊΡ‚ΠΎΡ€Π° напряТСнности ΠΌΠ°Π³Π½ΠΈΡ‚Π½ΠΎΠ³ΠΎ поля.answer1=1

[task#2]

Π’ΠΎΠΊΠΈ, Ρ‚Π΅ΠΊΡƒΡ‰ΠΈΠ΅ ΠΏΠΎ ΠΏΡ€ΠΎΠ²ΠΎΠ΄Π½ΠΈΠΊΠ°ΠΌ, ΡΠΎΠ·Π΄Π°ΡŽΡ‚ ΠΌΠ°Π³Π½ΠΈΡ‚Π½ΠΎΠ΅ ΠΏΠΎΠ»Π΅ с ΠΈΠ½Π΄ΡƒΠΊΡ†ΠΈΠ΅ΠΉ Π’ ΠΈ Π½Π°ΠΏΡ€ΡΠΆΠ΅Π½Π½ΠΎΡΡ‚ΡŒΡŽ Н. Π£ΠΊΠ°ΠΆΠΈΡ‚Π΅ Π²Ρ‹Ρ€Π°ΠΆΠ΅Π½ΠΈΠ΅, ΠΈΠ½Ρ‚Π΅Π³Ρ€Π°Π» ΠΎΡ‚ ΠΊΠΎΡ‚ΠΎΡ€ΠΎΠ³ΠΎ ΠΏΠΎ ΠΎΠ±ΡŠΠ΅ΠΌΡƒ Ρ€Π°Π²Π΅Π½ энСргии, запасСнной Π² ΠΌΠ°Π³Π½ΠΈΡ‚Π½ΠΎΠΌ ΠΏΠΎΠ»Π΅ ΠΏΡ€ΠΎΠ²ΠΎΠ΄Π½ΠΈΠΊΠΎΠ².answer1=3

[task#3]

ΠœΠ°Π³Π½ΠΈΡ‚Π½Ρ‹ΠΉ ΠΏΠΎΡ‚ΠΎΠΊ, ΠΏΡ€ΠΎΠ½ΠΈΠ·Ρ‹Π²Π°ΡŽΡ‰ΠΈΠΉ ΠΊΠ°Ρ‚ΡƒΡˆΠΊΡƒ, измСняСтся со Π²Ρ€Π΅ΠΌΠ΅Π½Π΅ΠΌ Π² соотвСтствии с Π³Ρ€Π°Ρ„ΠΈΠΊΠΎΠΌ. Π’ ΠΊΠ°ΠΊΠΎΠΉ ΠΏΡ€ΠΎΠΌΠ΅ΠΆΡƒΡ‚ΠΎΠΊ

Π²Ρ€Π΅ΠΌΠ΅Π½ΠΈ ΠΌΠΎΠ΄ΡƒΠ»ΡŒ Π­Π”Π‘ ΠΈΠ½Π΄ΡƒΠΊΡ†ΠΈΠΈ ΠΈΠΌΠ΅Π΅Ρ‚ минимальноС Π·Π½Π°Ρ‡Π΅Π½ΠΈΠ΅?

Β 

Β 

1=0 β€”t1

2=t1 β€”t2

3=t2 β€”t3

4=t3 β€”t4

5=t4 β€”t5

[task#4]

Π’Ρ‹Π±Π΅Ρ€ΠΈΡ‚Π΅ ΠΏΡ€Π°Π²ΠΈΠ»ΡŒΠ½ΠΎΠ΅ Π²Ρ‹Ρ€Π°ΠΆΠ΅Π½ΠΈΠ΅ для модуля Π­Π”Π‘ самоиндукции Π² ΠΌΠΎΠΌΠ΅Π½Ρ‚ Π²Ρ€Π΅ΠΌΠ΅Π½ΠΈ t Π² ΠΊΠΎΠ½Ρ‚ΡƒΡ€Π΅ с сопротивлСниСм R ΠΈ ΠΈΠ½Π΄ΡƒΠΊΡ‚ΠΈΠ²Π½ΠΎΡΡ‚ΡŒΡŽ L, ΠΏΠΎ ΠΊΠΎΡ‚ΠΎΡ€ΠΎΠΌΡƒ Ρ‚Π΅Ρ‡Π΅Ρ‚ Ρ‚ΠΎΠΊ I(t).answer1=6

[task#5]

ΠžΠ±ΡŠΡ‘ΠΌ V ΠΎΠ³Ρ€Π°Π½ΠΈΡ‡Π΅Π½ Π½Π΅ΠΏΠΎΠ΄Π²ΠΈΠΆΠ½ΠΎΠΉ Π·Π°ΠΌΠΊΠ½ΡƒΡ‚ΠΎΠΉ ΠΏΠΎΠ²Π΅Ρ€Ρ…Π½ΠΎΡΡ‚ΡŒΡŽ S. Для любого элСктричСского поля ΠΏΠΎΡ‚ΠΎΠΊ смСщСния (элСктричСской ΠΈΠ½Π΄ΡƒΠΊΡ†ΠΈΠΈ) Ρ‡Π΅Ρ€Π΅Π· ΠΏΠΎΠ²Π΅Ρ€Ρ…Π½ΠΎΡΡ‚ΡŒ S Ρ€Π°Π²Π΅Π½…

1=… скорости измСнСния ΠΌΠ°Π³Π½ΠΈΡ‚Π½ΠΎΠ³ΠΎ ΠΏΠΎΡ‚ΠΎΠΊΠ° Ρ‡Π΅Ρ€Π΅Π· ΠΏΠΎΠ²Π΅Ρ€Ρ…Π½ΠΎΡΡ‚ΡŒ S. 2=… ΠΏΠΎΡ‚ΠΎΠΊΡƒ плотности Ρ‚ΠΎΠΊΠ° Ρ‡Π΅Ρ€Π΅Π· ΠΏΠΎΠ²Π΅Ρ€Ρ…Π½ΠΎΡΡ‚ΡŒ S.

3=… суммарному связанному заряду, ΠΊΠΎΡ‚ΠΎΡ€Ρ‹ΠΉ находится Π²Π½ΡƒΡ‚Ρ€ΠΈ объСма V,

4=… суммарному свободному заряду, ΠΊΠΎΡ‚ΠΎΡ€Ρ‹ΠΉ находится Π²Π½ΡƒΡ‚Ρ€ΠΈ объСма V.

5=… Π½ΡƒΠ»ΡŽ.

variant_number=11

[task#1]

ΠšΠ°Ρ‚ΡƒΡˆΠΊΠ° солСноида А создаСт ΠΌΠ°Π³Π½ΠΈΡ‚Π½ΠΎΠ΅ ΠΏΠΎΠ»Π΅, Π² ΠΊΠΎΡ‚ΠΎΡ€ΠΎΠ΅ ΠΏΠΎΠΌΠ΅Ρ‰Π°ΡŽΡ‚ ΠΎΠ΄ΠΈΠ½Π°ΠΊΠΎΠ²Ρ‹Π΅ ΠΏΠΎ количСству вСщСства ΠΎΠ±Ρ€Π°Π·Ρ†Ρ‹ S ΠΏΠ°Ρ€Π°ΠΌΠ°Π³Π½Π΅Ρ‚ΠΈΠΊΠ° ΠΈ Π΄ΠΈΠ°ΠΌΠ°Π³Π½Π΅Ρ‚ΠΈΠΊΠ°. Π’Ρ‹Π±Π΅Ρ€ΠΈΡ‚Π΅ ΠΏΡ€Π°Π²ΠΈΠ»ΡŒΠ½Ρ‹Π΅ ΡΠΎΠΎΡ‚Π½ΠΎΡˆΠ΅Π½ΠΈΡ для FΠΏΠΈ FΠ΄β€”ΠΏΡ€ΠΎΠ΅ΠΊΡ†ΠΈΠΉΠ½Π° ось Z ΠΌΠ°Π³Π½ΠΈΡ‚Π½ΠΎΠΉ силы,

Π΄Π΅ΠΉΡΡ‚Π²ΡƒΡŽΡ‰Π΅ΠΉ Π½Π° ΠΊΠ°ΠΆΠ΄Ρ‹ΠΉ ΠΈΠ· ΠΎΠ±Ρ€Π°Π·Ρ†ΠΎΠ².

Β 

Β 

Β 

1=FΠ΄= 0; FΠΏ> 0.

2=FΠ΄< 0; FΠΏ> 0.

3=FΠ΄> 0; FΠΏ< 0.

4=FΠ΄= 0; FΠΏ< 0.

5=FΠΏ> FΠ΄> 0.

[task#2]

Β 

Β 

Β 

Β 

ΠšΠ°ΠΊΡƒΡŽ Ρ€Π°Π±ΠΎΡ‚Ρƒ (Π² миллидТоулях) ΡΠΎΠ²Π΅Ρ€ΡˆΠ°ΡŽΡ‚ сторонниС силы самоиндукции Π² Π·Π°ΠΌΠΊΠ½ΡƒΡ‚ΠΎΠΌ проводящСм ΠΊΠΎΠ½Ρ‚ΡƒΡ€Π΅ с ΠΈΠ½Π΄ΡƒΠΊΡ‚ΠΈΠ²Π½ΠΎΡΡ‚ΡŒΡŽ 0,1 Π“Π½ ΠΏΡ€ΠΈ ΡƒΠΌΠ΅Π½ΡŒΡˆΠ΅Π½ΠΈΡ силы элСктричСского Ρ‚ΠΎΠΊΠ° ΠΎΡ‚ 1А Π΄ΠΎ нуля ?answer1=50.0 % 1

[task#3]

ΠŸΠΎΡΡ‚ΠΎΡΠ½Π½Ρ‹ΠΉ ΠΌΠ°Π³Π½ΠΈΡ‚ Π²Π΄Π²ΠΈΠ³Π°ΡŽΡ‚ Π² алюминиСвоС ΠΊΠΎΠ»ΡŒΡ†ΠΎ сСвСрным полюсом. ΠŸΡ€ΠΈΡ‚ΡΠ³ΠΈΠ²Π°Π΅Ρ‚ΡΡ ΠΊΠΎΠ»ΡŒΡ†ΠΎ ΠΊ ΠΌΠ°Π³Π½ΠΈΡ‚Ρƒ ΠΈΠ»ΠΈ отталкиваСтся ΠΎΡ‚ Π½Π΅Π³ΠΎ? Как Π½Π°ΠΏΡ€Π°Π²Π»Π΅Π½ ΠΈΠ½Π΄ΡƒΠΊΡ†ΠΈΠΎΠ½Π½Ρ‹ΠΉ Ρ‚ΠΎΠΊ Π² ΠΊΠΎΠ»ΡŒΡ†Π΅?

1=ΠŸΡ€ΠΈΡ‚ΡΠ³ΠΈΠ²Π°Π΅Ρ‚ΡΡ. По Π½Π°ΠΏΡ€Π°Π²Π»Π΅Π½ΠΈΡŽ А. 2=ΠŸΡ€ΠΈΡ‚ΡΠ³ΠΈΠ²Π°Π΅Ρ‚ΡΡ. По Π½Π°ΠΏΡ€Π°Π²Π»Π΅Π½ΠΈΡŽ Π’.

3=ΠžΡ‚Ρ‚Π°Π»ΠΊΠΈΠ²Π°Π΅Ρ‚ΡΡ. По Π½Π°ΠΏΡ€Π°Π²Π»Π΅Π½ΠΈΡŽ А.

4=ΠžΡ‚Ρ‚Π°Π»ΠΊΠΈΠ²Π°Π΅Ρ‚ΡΡ. По Π½Π°ΠΏΡ€Π°Π²Π»Π΅Π½ΠΈΡŽ Π’.

5=НС притягиваСтся ΠΈ Π½Π΅ отталкиваСтся. Π‘ΠΈΠ»Π° Ρ‚ΠΎΠΊΠ° Ρ€Π°Π²Π½Π° Π½ΡƒΠ»ΡŽ. [task#4]

ЭлСктричСский Ρ‚ΠΎΠΊ 2 А создаСт Π² ΠΊΠΎΠ½Ρ‚ΡƒΡ€Π΅ ΠΌΠ°Π³Π½ΠΈΡ‚Π½Ρ‹ΠΉ ΠΏΠΎΡ‚ΠΎΠΊ Π€ = 4 Π’Π±. НайдитС (Π² Π³Π΅Π½Ρ€ΠΈ) ΠΈΠ½Π΄ΡƒΠΊΡ‚ΠΈΠ²Π½ΠΎΡΡ‚ΡŒ ΠΊΠΎΠ½Ρ‚ΡƒΡ€Π°? answer1=2.0 % 6

[task#5]

ΠžΠ±ΡŠΡ‘ΠΌ V ΠΎΠ³Ρ€Π°Π½ΠΈΡ‡Π΅Π½ Π½Π΅ΠΏΠΎΠ΄Π²ΠΈΠΆΠ½ΠΎΠΉ Π·Π°ΠΌΠΊΠ½ΡƒΡ‚ΠΎΠΉ ΠΏΠΎΠ²Π΅Ρ€Ρ…Π½ΠΎΡΡ‚ΡŒΡŽ S. Для любого ΠΌΠ°Π³Π½ΠΈΡ‚Π½ΠΎΠ³ΠΎ поля ΠΏΠΎΡ‚ΠΎΠΊ ΠΈΠ½Π΄ΡƒΠΊΡ†ΠΈΠΈ Ρ‡Π΅Ρ€Π΅Π· ΠΏΠΎΠ²Π΅Ρ€Ρ…Π½ΠΎΡΡ‚ΡŒ S Ρ€Π°Π²Π΅Π½…

1=… ΠΏΠΎΡ‚ΠΎΠΊΡƒ плотности Ρ‚ΠΎΠΊΠ° смСщСния Ρ‡Π΅Ρ€Π΅Π· ΠΏΠΎΠ²Π΅Ρ€Ρ…Π½ΠΎΡΡ‚ΡŒ S.

2=… скорости измСнСния суммарного заряда, находящСгося Π²Π½ΡƒΡ‚Ρ€ΠΈ объСма V. 3=… ΠΏΠΎΡ‚ΠΎΠΊΡƒ плотности Ρ‚ΠΎΠΊΠ° Ρ‡Π΅Ρ€Π΅Π· ΠΏΠΎΠ²Π΅Ρ€Ρ…Π½ΠΎΡΡ‚ΡŒ S.

4=… суммарному свободному заряду, ΠΊΠΎΡ‚ΠΎΡ€Ρ‹ΠΉ находится Π²Π½ΡƒΡ‚Ρ€ΠΈ объСма V.

5=…Π½ΡƒΠ»ΡŽ.

variant_number=12

[task#1]

ΠšΠ°Ρ‚ΡƒΡˆΠΊΠ° солСноида А создаСт ΠΌΠ°Π³Π½ΠΈΡ‚Π½ΠΎΠ΅ ΠΏΠΎΠ»Π΅, Π² ΠΊΠΎΡ‚ΠΎΡ€ΠΎΠ΅ ΠΏΠΎΠΌΠ΅Ρ‰Π°ΡŽΡ‚ ΠΎΠ΄ΠΈΠ½Π°ΠΊΠΎΠ²Ρ‹Π΅ ΠΏΠΎ количСству вСщСства ΠΎΠ±Ρ€Π°Π·Ρ†Ρ‹ S ΠΏΠ°Ρ€Π°ΠΌΠ°Π³Π½Π΅Ρ‚ΠΈΠΊΠ° ΠΈ Ρ„Π΅Ρ€Ρ€ΠΎΠΌΠ°Π³Π½Π΅Ρ‚ΠΈΠΊΠ°. Π’Ρ‹Π±Π΅Ρ€ΠΈΡ‚Π΅ ΠΏΡ€Π°Π²ΠΈΠ»ΡŒΠ½Ρ‹Π΅ ΡΠΎΠΎΡ‚Π½ΠΎΡˆΠ΅Π½ΠΈΡ для FΠΏ ΠΈ FΡ„β€”ΠΏΡ€ΠΎΠ΅ΠΊΡ†ΠΈΠΉΠ½Π° ось Z ΠΌΠ°Π³Π½ΠΈΡ‚Π½ΠΎΠΉ силы,

Π΄Π΅ΠΉΡΡ‚Π²ΡƒΡŽΡ‰Π΅ΠΉ Π½Π° ΠΊΠ°ΠΆΠ΄Ρ‹ΠΉ ΠΈΠ· ΠΎΠ±Ρ€Π°Π·Ρ†ΠΎΠ².

3=FΡ„>FΠΏ> 0.

Β 

Β 

1=FΠΏ= 0; FΡ„> 0.

2=FΡ„= 0; FΠΏ> 0.

4=FΠΏ< 0; FΡ„> 0.

5=FΡ„< 0; FΠΏ> 0.

[task#2]

Π’ ΠΏΠ°Ρ€Π°ΠΌΠ°Π³Π½Π΅Ρ‚ΠΈΠΊΠ΅ с ΠΌΠ°Π³Π½ΠΈΡ‚Π½ΠΎΠΉ ΠΏΡ€ΠΎΠ½ΠΈΡ†Π°Π΅ΠΌΠΎΡΡ‚ΡŒΡŽ ΠΏΡ€ΠΈΠ±Π»ΠΈΠ·ΠΈΡ‚Π΅Π»ΡŒΠ½ΠΎ Ρ€Π°Π²Π½ΠΎΠΉ Π΅Π΄ΠΈΠ½ΠΈΡ†Π΅ создано ΠΎΠ΄Π½ΠΎΡ€ΠΎΠ΄Π½ΠΎΠ΅ ΠΌΠ°Π³Π½ΠΈΡ‚Π½ΠΎΠ΅ ΠΏΠΎΠ»Π΅ с Π½Π°ΠΏΡ€ΡΠΆΠ΅Π½Π½ΠΎΡΡ‚ΡŒΡŽ 10000 А/ΠΌ. Бколько ΠΌΠΈΠΊΡ€ΠΎΠ΄ΠΆΠΎΡƒΠ»Π΅ΠΉ ΠΌΠ°Π³Π½ΠΈΡ‚Π½ΠΎΠΉ энСргии запасСно Π² ΠΎΠ΄Π½ΠΎΠΌ ΠšΠ£Π‘Π˜Π§Π•Π‘ΠšΠžΠœ Π‘ΠΠΠ’Π˜ΠœΠ•Π’Π Π•

ΠΏΠ°Ρ€Π°ΠΌΠ°Π³Π½Π΅Ρ‚ΠΈΠΊΠ°? ΠŸΡ€ΠΈΠ½ΡΡ‚ΡŒ, Ρ‡Ρ‚ΠΎ магнитная постоянная ΞΌΠΎ = 0,00000126 Π“Π½/ΠΌ.

answer1=63.0 % 4

[task#3]

Β 

Π”Π²Π° ΠΊΠΎΠ½Ρ‚ΡƒΡ€Π° располоТСны Ρ‚Π°ΠΊ, Ρ‡Ρ‚ΠΎ ΠΈΡ… плоскости ΠΏΠ°Ρ€Π°Π»Π»Π΅Π»ΡŒΠ½Ρ‹ Π΄Ρ€ΡƒΠ³ Π΄Ρ€ΡƒΠ³Ρƒ. По ΠΏΠ΅Ρ€Π²ΠΎΠΌΡƒ Ρ‚Π΅Ρ‡Π΅Ρ‚ Ρ‚ΠΎΠΊ Π² Π½Π°ΠΏΡ€Π°Π²Π»Π΅Π½ΠΈΠΈ ПО Π§ΠΠ‘ΠžΠ’ΠžΠ™ Π‘Π’Π Π•Π›ΠšΠ•. Как Π½Π°ΠΏΡ€Π°Π²Π»Π΅Π½ ΠΈΠ½Π΄ΡƒΠΊΡ†ΠΈΠΎΠ½Π½Ρ‹ΠΉ Ρ‚ΠΎΠΊ Π²ΠΎ Π²Ρ‚ΠΎΡ€ΠΎΠΌ ΠΊΠΎΠ½Ρ‚ΡƒΡ€Π΅, Ссли ΠΎΠ½ΠΈ расходятся (А) ΠΈΠ»ΠΈ ΡΠ±Π»ΠΈΠΆΠ°ΡŽΡ‚ΡΡ (Π’)?

1=(А) ΠΏΠΎ час.стрСлкС; (Π’) ΠΏΡ€ΠΎΡ‚ΠΈΠ² час. стрСлки. 2=(А) ΠΏΡ€ΠΎΡ‚ΠΈΠ² час. стрСлки; (Π’) ΠΏΠΎ час.стрСлкС. 3=Π’ ΠΎΠ±ΠΎΠΈΡ… случаях ΠΏΠΎ часовой стрСлкС.

4=Π’ ΠΎΠ±ΠΎΠΈΡ… случаях ΠΏΡ€ΠΎΡ‚ΠΈΠ² часовой стрСлки. 5=ΠžΡ‚Π²Π΅Ρ‚ зависит ΠΎΡ‚ ускорСния двиТСния ΠΊΠΎΠ½Ρ‚ΡƒΡ€ΠΎΠ². [task#4]

ΠœΠ°Π³Π½ΠΈΡ‚Π½Ρ‹ΠΉ ΠΏΠΎΡ‚ΠΎΠΊ, создаваСмый Ρ‚ΠΎΠΊΠΎΠΌ, Ρ‚Π΅ΠΊΡƒΡ‰ΠΈΠΌ Π² проводящСм ΠΊΠΎΠ½Ρ‚ΡƒΡ€Π΅, ΠΏΡ€ΠΎΠΏΠΎΡ€Ρ†ΠΈΠΎΠ½Π°Π»Π΅Π½…

1=… силС Ρ‚ΠΎΠΊΠ° Π² стСпСни 3/2

2=… ΠΏΠ΅Ρ€Π²ΠΎΠΉ стСпСни силы Ρ‚ΠΎΠΊΠ°.

3=… Π²Ρ‚ΠΎΡ€ΠΎΠΉ стСпСни силы Ρ‚ΠΎΠΊΠ°.

4=… ΠΊΠΎΡ€Π½ΡŽ ΠΊΠ²Π°Π΄Ρ€Π°Ρ‚Π½ΠΎΠΌΡƒ ΠΈΠ· силы Ρ‚ΠΎΠΊΠ°. 5=… силС Ρ‚ΠΎΠΊΠ° Π² стСпСни 2/3.

[task#5]

ΠŸΠΎΠ²Π΅Ρ€Ρ…Π½ΠΎΡΡ‚ΡŒ S ΠΎΠ³Ρ€Π°Π½ΠΈΡ‡Π΅Π½Π° Π½Π΅ΠΏΠΎΠ΄Π²ΠΈΠΆΠ½Ρ‹ΠΌ Π·Π°ΠΌΠΊΠ½ΡƒΡ‚Ρ‹ΠΌ ΠΊΠΎΠ½Ρ‚ΡƒΡ€ΠΎΠΌ L. ΠŸΡ€ΠΈ отсутствии Ρ‚ΠΎΠΊΠΎΠ² проводимости циркуляция напряТСнности ΠΌΠ°Π³Π½ΠΈΡ‚Π½ΠΎΠ³ΠΎ поля ΠΏΠΎ ΠΊΠΎΠ½Ρ‚ΡƒΡ€Ρƒ L Ρ€Π°Π²Π½Π° …

1=… Π½ΡƒΠ»ΡŽ.

2=…взятому с ΠΎΠ±Ρ€Π°Ρ‚Π½Ρ‹ΠΌ Π·Π½Π°ΠΊΠΎΠΌ ΠΏΠΎΡ‚ΠΎΠΊΡƒ Π²Π΅ΠΊΡ‚ΠΎΡ€Π° скорости измСнСния ΠΌΠ°Π³Π½ΠΈΡ‚Π½ΠΎΠΉ ΠΈΠ½Π΄ΡƒΠΊΡ†ΠΈΠΈ Ρ‡Π΅Ρ€Π΅Π· ΠΏΠΎΠ²Π΅Ρ€Ρ…Π½ΠΎΡΡ‚ΡŒ S. 3=… скорости измСнСния ΠΌΠ°Π³Π½ΠΈΡ‚Π½ΠΎΠ³ΠΎ ΠΏΠΎΡ‚ΠΎΠΊΠ° Ρ‡Π΅Ρ€Π΅Π· ΠΏΠΎΠ²Π΅Ρ€Ρ…Π½ΠΎΡΡ‚ΡŒ S.

4=… ΠΏΠΎΡ‚ΠΎΠΊΡƒ Π²Π΅ΠΊΡ‚ΠΎΡ€Π° скорости измСнСния элСктричСской ΠΈΠ½Π΄ΡƒΠΊΡ†ΠΈΠΈ Ρ‡Π΅Ρ€Π΅Π· ΠΏΠΎΠ²Π΅Ρ€Ρ…Π½ΠΎΡΡ‚ΡŒ S.

5=…взятому с ΠΎΠ±Ρ€Π°Ρ‚Π½Ρ‹ΠΌ Π·Π½Π°ΠΊΠΎΠΌ ΠΏΠΎΡ‚ΠΎΠΊΡƒ Π²Π΅ΠΊΡ‚ΠΎΡ€Π° скорости измСнСния элСктричСской ΠΈΠ½Π΄ΡƒΠΊΡ†ΠΈΠΈ Ρ‡Π΅Ρ€Π΅Π· ΠΏΠΎΠ²Π΅Ρ€Ρ…Π½ΠΎΡΡ‚ΡŒ S.

variant_number=13

[task#1]

Π’Ρ‹Π±Π΅Ρ€ΠΈΡ‚Π΅ ΠΏΡ€Π°Π²ΠΈΠ»ΡŒΠ½ΠΎΠ΅ ΡΠΎΠΎΡ‚Π½ΠΎΡˆΠ΅Π½ΠΈΠ΅ для Π½Π°ΠΏΡ€Π°Π²Π»Π΅Π½ΠΈΠΉ Π²Π΅ΠΊΡ‚ΠΎΡ€ΠΎΠ² напряТСнности H, ΠΌΠ°Π³Π½ΠΈΡ‚Π½ΠΎΠΉ ΠΈΠ½Π΄ΡƒΠΊΡ†ΠΈΠΈ B, ΠΈ намагничСности J Π² ΠΎΠ΄Π½ΠΎΡ€ΠΎΠ΄Π½ΠΎΠΌ ΠΈΠ·ΠΎΡ‚Ρ€ΠΎΠΏΠ½ΠΎΠΌ Π”Π˜ΠΠœΠΠ“ΠΠ•Π’Π˜ΠšΠ•.

1=Одинаково Π½Π°ΠΏΡ€Π°Π²Π»Π΅Π½Ρ‹ Ρ‚ΠΎΠ»ΡŒΠΊΠΎ H ΠΈ J.

2=H ΠΈ B Π½Π°ΠΏΡ€Π°Π²Π»Π΅Π½Ρ‹ Π² ΠΏΡ€ΠΎΡ‚ΠΈΠ²ΠΎΠΏΠΎΠ»ΠΎΠΆΠ½Ρ‹Π΅ стороны.

3=H ΠΈ B Π½Π°ΠΏΡ€Π°Π²Π»Π΅Π½Ρ‹ ΠΎΠ΄ΠΈΠ½Π°ΠΊΠΎΠ²ΠΎ, J β€”Π²ΠΏΡ€ΠΎΡ‚ΠΈΠ²ΠΎΠΏΠΎΠ»ΠΎΠΆΠ½ΡƒΡŽ сторону.

4=H ΠΈ B Π½Π°ΠΏΡ€Π°Π²Π»Π΅Π½Ρ‹ ΠΎΠ΄ΠΈΠ½Π°ΠΊΠΎΠ²ΠΎ, Π° J —пСрпСндикулярноим. 5=ВсС Π²Π΅ΠΊΡ‚ΠΎΡ€Ρ‹ Π½Π°ΠΏΡ€Π°Π²Π»Π΅Π½Ρ‹ ΠΎΠ΄ΠΈΠ½Π°ΠΊΠΎΠ²ΠΎ.

[task#2]

ЭнСргия ΠΌΠ°Π³Π½ΠΈΡ‚Π½ΠΎΠ³ΠΎ поля ΠΊΠΎΠ½Ρ‚ΡƒΡ€Π° с Ρ‚ΠΎΠΊΠΎΠΌ ΠΏΡ€ΠΎΠΏΠΎΡ€Ρ†ΠΈΠΎΠ½Π°Π»ΡŒΠ½Π°…

1=…ΠΏΠ΅Ρ€Π²ΠΎΠΉ стСпСни силы Ρ‚ΠΎΠΊΠ°.

2=…Π²Ρ‚ΠΎΡ€ΠΎΠΉ стСпСни силы Ρ‚ΠΎΠΊΠ°.

3=…Ρ‚Ρ€Π΅Ρ‚ΡŒΠ΅ΠΉ стСпСни силы Ρ‚ΠΎΠΊΠ°. 4=…ΠΊΠΎΡ€Π½ΡŽ ΠΊΠ²Π°Π΄Ρ€Π°Ρ‚Π½ΠΎΠΌΡƒ ΠΈΠ· силы Ρ‚ΠΎΠΊΠ°. 5=…ΠΊΠΎΡ€Π½ΡŽ кубичСскому ΠΈΠ· силы Ρ‚ΠΎΠΊΠ°. [task#3]

Из Π΄Π²ΡƒΡ… соосных ΠΊΠ°Ρ‚ΡƒΡˆΠ΅ΠΊ ΠΎΠ΄Π½Π° соСдинСна с источником Ρ‚ΠΎΠΊΠ° Π• , другая β€”ΡΠ³Π°Π»ΡŒΠ²Π°Π½ΠΎΠΌΠ΅Ρ‚Ρ€ΠΎΠΌ G. ΠžΠΏΡ€Π΅Π΄Π΅Π»ΠΈΡ‚Π΅ ΠΏΠΎΠ²Π΅Π΄Π΅Π½ΠΈΠ΅ стрСлки Π³Π°Π»ΡŒΠ²Π°Π½ΠΎΠΌΠ΅Ρ‚Ρ€Π° ΠΏΡ€ΠΈ Π·Π°ΠΌΡ‹ΠΊΠ°Π½ΠΈΠΈ ΠΊΠ»ΡŽΡ‡Π° К (1), Π΅Π³ΠΎ ΡƒΠ΄Π΅Ρ€ΠΆΠ°Π½ΠΈΠΈ (2), ΠΈ Π΅Π³ΠΎ Ρ€Π°Π·ΠΌΡ‹ΠΊΠ°Π½ΠΈΠΈ (3), Π‘Ρ‚Ρ€Π΅Π»ΠΊΠ° отклоняСтся Π² Π½Π°ΠΏΡ€Π°Π²Π»Π΅Π½ΠΈΠΈ Ρ‚ΠΎΠΊΠ°.

answer1=1.—отклоняСтсявправо.2.—сохраняСтпоказания.3.—возвращаСтсяв 0.answer2=1.—отклоняСтся Π²ΠΏΡ€Π°Π²ΠΎ. 2.—удСрТиваСтся Π² 0. 3.—отклоняСтся Π²Π»Π΅Π²ΠΎ. answer3=1.—отклоняСтсявлСво.2.—сохраняСтпоказания.3.—возвращаСтсяв 0.answer4=1.—отклоняСтсявправо.2.—удСрТиваСтсяв 0.3.—отклоняСтсявправо.answer5=1—отклоняСтсявлСво.2—удСрТиваСтсяв 0.3—отклоняСтсявлСво. [task#4]

Π’Ρ‹Π±Π΅Ρ€ΠΈΡ‚Π΅ ΠΏΡ€Π°Π²ΠΈΠ»ΡŒΠ½ΠΎΠ΅ Π²Ρ‹Ρ€Π°ΠΆΠ΅Π½ΠΈΠ΅ для индуктивности Π΄Π»ΠΈΠ½Π½ΠΎΠ³ΠΎ солСноида. ΞΌ -ΠΌΠ°Π³Π½ΠΈΡ‚Π½Π°ΡΠΏΡ€ΠΎΠ½ΠΈΡ†Π°Π΅ΠΌΠΎΡΡ‚ΡŒ вСщСства, ΞΌΠΎ — магнитная постоянная, n-числовитков Π½Π° Π΅Π΄ΠΈΠ½ΠΈΡ†ΡƒΠ΄Π»ΠΈΠ½Ρ‹, L-длинасолСноида, S-ΠΏΠ»ΠΎΡ‰Π°Π΄ΡŒΠ²ΠΈΡ‚ΠΊΠ°.answer1=5

[task#5]

ΠžΠ±ΡŠΡ‘ΠΌ V ΠΎΠ³Ρ€Π°Π½ΠΈΡ‡Π΅Π½ Π½Π΅ΠΏΠΎΠ΄Π²ΠΈΠΆΠ½ΠΎΠΉ Π·Π°ΠΌΠΊΠ½ΡƒΡ‚ΠΎΠΉ ΠΏΠΎΠ²Π΅Ρ€Ρ…Π½ΠΎΡΡ‚ΡŒΡŽ S. ΠŸΠΎΡ‚ΠΎΠΊ элСктричСского смСщСния Ρ‡Π΅Ρ€Π΅Π· ΠΏΠΎΠ²Π΅Ρ€Ρ…Π½ΠΎΡΡ‚ΡŒ S Ρ€Π°Π²Π΅Π½…

1=…Π½ΡƒΠ»ΡŽ.

2=… разности ΠΏΠΎΡ‚ΠΎΠΊΠ° плотности элСктричСского Ρ‚ΠΎΠΊΠ° ΠΈ ΠΏΠΎΡ‚ΠΎΠΊΠ° Π²Π΅ΠΊΡ‚ΠΎΡ€Π° скорости измСнСния элСктричСской ΠΈΠ½Π΄ΡƒΠΊΡ†ΠΈΠΈ Ρ‡Π΅Ρ€Π΅Π· ΠΏΠΎΠ²Π΅Ρ€Ρ…Π½ΠΎΡΡ‚ΡŒ S

3=… скорости измСнСния суммарного заряда, находящСгося Π²Π½ΡƒΡ‚Ρ€ΠΈ объСма V. 4=… ΠΏΠΎΡ‚ΠΎΠΊΡƒ плотности Ρ‚ΠΎΠΊΠ° смСщСния Ρ‡Π΅Ρ€Π΅Π· ΠΏΠΎΠ²Π΅Ρ€Ρ…Π½ΠΎΡΡ‚ΡŒ S.

5=… ΠΈΠ½Ρ‚Π΅Π³Ρ€Π°Π»Ρƒ ΠΎΡ‚ объСмной плотности свободных зарядов ΠΏΠΎ ΠΎΠ±ΡŠΠ΅ΠΌΡƒ V.

variant_number=14

[task#1]

Как измСнится индукция ΠΌΠ°Π³Π½ΠΈΡ‚Π½ΠΎΠ³ΠΎ поля Π²Π½ΡƒΡ‚Ρ€ΠΈ ΠΊΠ°Ρ‚ΡƒΡˆΠΊΠΈ с Π·Π°Π΄Π°Π½Π½Ρ‹ΠΌ Ρ‚ΠΎΠΊΠΎΠΌ, Ссли Π΅Π΅

Π²Π½ΡƒΡ‚Ρ€Π΅Π½Π½Π΅Π΅ пространство Π·Π°ΠΏΠΎΠ»Π½ΠΈΡ‚ΡŒ: А) Ρ„Π΅Ρ€Ρ€ΠΎΠΌΠ°Π³Π½Π΅Ρ‚ΠΈΠΊΠΎΠΌ; Π‘) ΠΏΠ°Ρ€Π°ΠΌΠ°Π³Π½Π΅Ρ‚ΠΈΠΊΠΎΠΌ? 1=А) сильно ΡƒΠΌΠ΅Π½ΡŒΡˆΠΈΡ‚ΡΡ; Π‘) Π½Π΅ΠΌΠ½ΠΎΠ³ΠΎ увСличится.

2=А) сильно ΡƒΠΌΠ΅Π½ΡŒΡˆΠΈΡ‚ΡΡ; Π‘) Π½Π΅ΠΌΠ½ΠΎΠ³ΠΎ ΡƒΠΌΠ΅Π½ΡŒΡˆΠΈΡ‚ΡΡ. 3=А) сильно увСличится; Π‘) сильно ΡƒΠΌΠ΅Π½ΡŒΡˆΠΈΡ‚ΡΡ.

4=А) сильно увСличится; Π‘) Π½Π΅ΠΌΠ½ΠΎΠ³ΠΎ увСличится.

5=А) сильно увСличится; Π‘) Π½Π΅ΠΌΠ½ΠΎΠ³ΠΎ ΡƒΠΌΠ΅Π½ΡŒΡˆΠΈΡ‚ΡΡ. [task#2]

Π§Π΅ΠΌΡƒ Ρ€Π°Π²Π½Π° (Π² дТоулях) энСргия ΠΌΠ°Π³Π½ΠΈΡ‚Π½ΠΎΠ³ΠΎ поля ΠΊΠ°Ρ‚ΡƒΡˆΠΊΠΈ ΠΈΠ½Π΄ΡƒΠΊΡ‚ΠΈΠ²Π½ΠΎΡΡ‚ΡŒΡŽ 2 Π“Π½ ΠΏΡ€ΠΈ силС Ρ‚ΠΎΠΊΠ° Π² Π½Π΅ΠΉ 3 А?answer1=9.0 % 2

[task#3]

Π£ΠΊΠ°ΠΆΠΈΡ‚Π΅ Π½ΠΎΠΌΠ΅Ρ€ Ρ„ΠΎΡ€ΠΌΡƒΠ»Ρ‹, которая, Π²Ρ‹Ρ€Π°ΠΆΠ°Π΅Ρ‚ Π·Π°ΠΊΠΎΠ½ элСктромагнитной ΠΈΠ½Π΄ΡƒΠΊΡ†ΠΈΠΈ?answer1=4

[task#4]

Π—Π°ΠΌΠΊΠ½ΡƒΡ‚Ρ‹ΠΉ плоский ΠΊΠΎΠ½Ρ‚ΡƒΡ€ L, состоящий ΠΈΠ· Π΄Π²ΡƒΡ… окруТностСй, ΠΏΠ΅Ρ€Π΅ΠΊΡ€ΡƒΡ‚ΠΈΠ»ΠΈ Ρ‚Π°ΠΊ, Ρ‡Ρ‚ΠΎ ΠΎΠ½ прСвратился Π² плоский ΠΊΠΎΠ½Ρ‚ΡƒΡ€ L’, ΠΈΠΌΠ΅ΡŽΡ‰ΠΈΠΉ пСрСхлСст посСрСдинС. Как ΠΏΡ€ΠΈ этом измСнилась ΠΈΠ½Π΄ΡƒΠΊΡ‚ΠΈΠ²Π½ΠΎΡΡ‚ΡŒ ΠΊΠΎΠ½Ρ‚ΡƒΡ€Π°?

1=Π˜Π½Π΄ΡƒΠΊΡ‚ΠΈΠ²Π½ΠΎΡΡ‚ΡŒ ΠΊΠΎΠ½Ρ‚ΡƒΡ€Π° Π½Π΅ измСнилась.

2=Π˜Π½Π΄ΡƒΠΊΡ‚ΠΈΠ²Π½ΠΎΡΡ‚ΡŒ ΠΊΠΎΠ½Ρ‚ΡƒΡ€Π° Π±Ρ‹Π»Π° ΠΎΡ‚Π»ΠΈΡ‡Π½ΠΎΠΉ ΠΎΡ‚ нуля стала Π½ΡƒΠ»Π΅Π²ΠΎΠΉ.

3=Π˜Π½Π΄ΡƒΠΊΡ‚ΠΈΠ²Π½ΠΎΡΡ‚ΡŒ ΠΊΠΎΠ½Ρ‚ΡƒΡ€Π° ΡƒΠ²Π΅Π»ΠΈΡ‡ΠΈΠ»Π°ΡΡŒ.

4=Π˜Π½Π΄ΡƒΠΊΡ‚ΠΈΠ²Π½ΠΎΡΡ‚ΡŒ ΠΊΠΎΠ½Ρ‚ΡƒΡ€Π° ΡƒΠ²Π΅Π»ΠΈΡ‡ΠΈΠ»Π°ΡΡŒ.

5=Π˜Π½Π΄ΡƒΠΊΡ‚ΠΈΠ²Π½ΠΎΡΡ‚ΡŒ ΠΊΠΎΠ½Ρ‚ΡƒΡ€Π° Π±Ρ‹Π»Π° Π½ΡƒΠ»Π΅Π²ΠΎΠΉ стала ΠΎΡ‚Π»ΠΈΡ‡Π½ΠΎΠΉ ΠΎΡ‚ нуля. [task#5]

ΠŸΠΎΠ²Π΅Ρ€Ρ…Π½ΠΎΡΡ‚ΡŒ S ΠΎΠ³Ρ€Π°Π½ΠΈΡ‡Π΅Π½Π° Π½Π΅ΠΏΠΎΠ΄Π²ΠΈΠΆΠ½Ρ‹ΠΌ Π·Π°ΠΌΠΊΠ½ΡƒΡ‚Ρ‹ΠΌ ΠΊΠΎΠ½Ρ‚ΡƒΡ€ΠΎΠΌ L. Если ΠΌΠ°Π³Π½ΠΈΡ‚Π½ΠΎΠ΅ ΠΏΠΎΠ»Π΅ стационарно, Ρ‚ΠΎ циркуляция Π²Π΅ΠΊΡ‚ΠΎΡ€Π° элСктричСской напряТСнности ΠΏΠΎ ΠΊΠΎΠ½Ρ‚ΡƒΡ€Ρƒ L Ρ€Π°Π²Π½Π° …

1=… Π½ΡƒΠ»ΡŽ.

2=…взятому с ΠΎΠ±Ρ€Π°Ρ‚Π½Ρ‹ΠΌ Π·Π½Π°ΠΊΠΎΠΌ ΠΏΠΎΡ‚ΠΎΠΊΡƒ Π²Π΅ΠΊΡ‚ΠΎΡ€Π° скорости измСнСния элСктричСской ΠΈΠ½Π΄ΡƒΠΊΡ†ΠΈΠΈ Ρ‡Π΅Ρ€Π΅Π· ΠΏΠΎΠ²Π΅Ρ€Ρ…Π½ΠΎΡΡ‚ΡŒ S. 3=… ΠΏΠΎΡ‚ΠΎΠΊΡƒ Π²Π΅ΠΊΡ‚ΠΎΡ€Π° скорости измСнСния элСктричСской ΠΈΠ½Π΄ΡƒΠΊΡ†ΠΈΠΈ Ρ‡Π΅Ρ€Π΅Π· ΠΏΠΎΠ²Π΅Ρ€Ρ…Π½ΠΎΡΡ‚ΡŒ S.

4=… ΠΌΠ°Π³Π½ΠΈΡ‚Π½ΠΎΠΌΡƒ ΠΏΠΎΡ‚ΠΎΠΊΡƒ Ρ‡Π΅Ρ€Π΅Π· ΠΏΠΎΠ²Π΅Ρ€Ρ…Π½ΠΎΡΡ‚ΡŒ S.

5=… ΠΏΠΎΡ‚ΠΎΠΊΡƒ ΠΌΠ°Π³Π½ΠΈΡ‚Π½ΠΎΠΉ напряТСнности Ρ‡Π΅Ρ€Π΅Π· ΠΏΠΎΠ²Π΅Ρ€Ρ…Π½ΠΎΡΡ‚ΡŒ S.

studfiles.net

51.2 ДвиТущийся ΠΏΡ€ΠΎΠ²ΠΎΠ΄Π½ΠΈΠΊ Π² ΠΌΠ°Π³Π½ΠΈΡ‚Π½ΠΎΠΌ ΠΏΠΎΠ»Π΅.

51.2 ДвиТущийся ΠΏΡ€ΠΎΠ²ΠΎΠ΄Π½ΠΈΠΊ Π² ΠΌΠ°Π³Π½ΠΈΡ‚Π½ΠΎΠΌ ΠΏΠΎΠ»Π΅.

 БСгодня ΠΏΠΎΡ‡Ρ‚ΠΈ ΠΎΡ‡Π΅Π²ΠΈΠ΄Π½ΠΎ, никакая конфигурация постоянного ΠΌΠ°Π³Π½ΠΈΡ‚Π½ΠΎΠ³ΠΎ поля Π½Π΅ ΠΌΠΎΠΆΠ΅Ρ‚ привСсти ΠΊ возникновСнию постоянного элСктричСского Ρ‚ΠΎΠΊΠ°. Для поддСрТания Ρ‚ΠΎΠΊΠ° Π² элСктричСской Ρ†Π΅ΠΏΠΈ, ΠΊΠ°ΠΊ ΠΌΡ‹ Π·Π½Π°Π΅ΠΌ, Π΄ΠΎΠ»ΠΆΠ΅Π½ Π±Ρ‹Ρ‚ΡŒ источник сторонних сил, ΠΊΠΎΡ‚ΠΎΡ€Ρ‹ΠΉ ΡΠΎΠ²Π΅Ρ€ΡˆΠ°Π΅Ρ‚ Ρ€Π°Π±ΠΎΡ‚Ρƒ ΠΏΠΎ ΠΏΡ€Π΅ΠΎΠ΄ΠΎΠ»Π΅Π½ΠΈΡŽ сил сопротивлСния. ΠœΠ°Π³Π½ΠΈΡ‚Π½ΠΎΠ΅ ΠΏΠΎΠ»Π΅ дСйствуСт Ρ‚ΠΎΠ»ΡŒΠΊΠΎ Π½Π° двиТущиСся заряды, ΠΏΡ€ΠΈΡ‡Π΅ΠΌ силы Π΄Π΅ΠΉΡΡ‚Π²ΡƒΡŽΡ‰Π°Ρ Π½Π° заряд (сила Π›ΠΎΡ€Π΅Π½Ρ†Π°) пСрпСндикулярна Π²Π΅ΠΊΡ‚ΠΎΡ€Ρƒ скорости частицы, поэтому ΠΎΠ½Π° Ρ€Π°Π±ΠΎΡ‚Ρ‹ Π½Π΅ ΡΠΎΠ²Π΅Ρ€ΡˆΠ°Π΅Ρ‚. НаконСц, Ссли Π±Ρ‹ стационарноС ΠΌΠ°Π³Π½ΠΈΡ‚Π½ΠΎΠ΅ ΠΏΠΎΠ»Π΅ ΠΌΠΎΠ³Π»ΠΎ ΠΏΠΎΠ΄Π΄Π΅Ρ€ΠΆΠΈΠ²Π°Ρ‚ΡŒ элСктричСский Ρ‚ΠΎΠΊ, Ρ‚ΠΎ это Π±Ρ‹Π» прямой ΠΏΡƒΡ‚ΡŒ ΠΊ созданию Β«Π²Π΅Ρ‡Π½ΠΎΠ³ΠΎ двигатСля», Ρ‚ΠΎ Π΅ΡΡ‚ΡŒ ΠΊ «бСсплатному» ΠΏΠΎΠ»ΡƒΡ‡Π΅Π½ΠΈΡŽ энСргии. Π”Π΅ΠΉΡΡ‚Π²ΠΈΡ‚Π΅Π»ΡŒΠ½ΠΎ, Ссли ΠΏΠΎΠ»Π΅ стационарно, Ρ‚ΠΎ Π΅Π³ΠΎ энСргия Π½Π΅ измСняСтся, Π° гипотСтичСский элСктричСский Ρ‚ΠΎΠΊ ΠΎΠ±Π»Π°Π΄Π°Π΅Ρ‚ энСргиСй ΠΈ способСн ΡΠΎΠ²Π΅Ρ€ΡˆΠ°Ρ‚ΡŒ Ρ€Π°Π±ΠΎΡ‚Ρƒ. Π‘Π»Π΅Π΄ΠΎΠ²Π°Ρ‚Π΅Π»ΡŒΠ½ΠΎ, для возникновСния Π­Π”Π‘ Π² ΠΊΠΎΠ½Ρ‚ΡƒΡ€Π΅, Π΄ΠΎΠ»ΠΆΠ΅Π½ ΡΡƒΡ‰Π΅ΡΡ‚Π²ΠΎΠ²Π°Ρ‚ΡŒ внСшний источник энСргии. ЭнСргия Π² ΠΊΠΎΠ½Ρ‚ΡƒΡ€ ΠΌΠΎΠΆΠ΅Ρ‚ ΠΏΠΎΡΡ‚ΡƒΠΏΠ°Ρ‚ΡŒ благодаря Ρ€Π°Π±ΠΎΡ‚Π΅ Π²Π½Π΅ΡˆΠ½ΠΈΡ… сил. Рассмотрим Π³Ρ€ΡƒΠΏΠΏΡƒ простых мыслСнных экспСримСнтов, Π΄ΠΎΠΏΡƒΡΠΊΠ°ΡŽΡ‰ΠΈΡ… простоС тСорСтичСскоС описаниС. ΠŸΡƒΡΡ‚ΡŒ цилиндричСский ΠΏΡ€ΠΎΠ²ΠΎΠ΄Π½ΠΈΠΊ двиТСтся Π² постоянном ΠΌΠ°Π³Π½ΠΈΡ‚Π½ΠΎΠΌ ΠΏΠΎΠ»Π΅, Ρ‚Π°ΠΊ Ρ‡Ρ‚ΠΎ Π²Π΅ΠΊΡ‚ΠΎΡ€ скорости v пСрпСндикулярСн оси Ρ†ΠΈΠ»ΠΈΠ½Π΄Ρ€Π°, Π° Π²Π΅ΠΊΡ‚ΠΎΡ€ ΠΈΠ½Π΄ΡƒΠΊΡ†ΠΈΠΈ ΠΌΠ°Π³Π½ΠΈΡ‚Π½ΠΎΠ³ΠΎ поля B пСрпСндикулярСн, ΠΊΠ°ΠΊ оси ΠΏΡ€ΠΎΠ²ΠΎΠ΄Π½ΠΈΠΊΠ°, Ρ‚Π°ΠΊ ΠΈ Π΅Π³ΠΎ скорости (рис. 509).


рис. 509
 ВмСстС с ΠΏΡ€ΠΎΠ²ΠΎΠ΄Π½ΠΈΠΊΠΎΠΌ двиТутся ΠΈ свободныС заряды, находящиСся Π²Π½ΡƒΡ‚Ρ€ΠΈ Π½Π΅Π³ΠΎ. Π‘ΠΎ стороны ΠΌΠ°Π³Π½ΠΈΡ‚Π½ΠΎΠ³ΠΎ поля Π½Π° эти заряды Π±ΡƒΠ΄ΡƒΡ‚ Π΄Π΅ΠΉΡΡ‚Π²ΠΎΠ²Π°Ρ‚ΡŒ силы Π›ΠΎΡ€Π΅Π½Ρ†Π°, Π½Π°ΠΏΡ€Π°Π²Π»Π΅Π½Π½Ρ‹Π΅, Π² соотвСтствии ΠΏΡ€Π°Π²ΠΈΠ»Π° Π»Π΅Π²ΠΎΠΉ Ρ€ΡƒΠΊΠΈ, вдоль оси ΠΏΡ€ΠΎΠ²ΠΎΠ΄Π½ΠΈΠΊΠ°.
 НаиболСС извСстными ΠΏΡ€ΠΎΠ²ΠΎΠ΄Π½ΠΈΠΊΠ°ΠΌΠΈ ΡΠ²Π»ΡΡŽΡ‚ΡΡ ΠΌΠ΅Ρ‚Π°Π»Π»Ρ‹, Π³Π΄Π΅ свободными зарядами ΡΠ²Π»ΡΡŽΡ‚ΡΡ ΠΎΡ‚Ρ€ΠΈΡ†Π°Ρ‚Π΅Π»ΡŒΠ½ΠΎ заряТСнныС частицы βˆ’ элСктроны. Однако здСсь ΠΈ Π² дальнСйшСм ΠΌΡ‹ Π±ΡƒΠ΄Π΅ΠΌ Ρ€Π°ΡΡΠΌΠ°Ρ‚Ρ€ΠΈΠ²Π°Ρ‚ΡŒ Π΄Π²ΠΈΠΆΠ΅Π½ΠΈΠ΅ ΠΏΠΎΠ»ΠΎΠΆΠΈΡ‚Π΅Π»ΡŒΠ½ΠΎ заряТСнных частиц, ΠΏΠΎΡ‚ΠΎΠΌΡƒ, Ρ‡Ρ‚ΠΎ Π·Π° Π½Π°ΠΏΡ€Π°Π²Π»Π΅Π½ΠΈΠ΅ Ρ‚ΠΎΠΊΠ° ΠΏΡ€ΠΈΠ½ΠΈΠΌΠ°ΡŽΡ‚ Π½Π°ΠΏΡ€Π°Π²Π»Π΅Π½ΠΈΠ΅ ΠΏΠΎΠ»ΠΎΠΆΠΈΡ‚Π΅Π»ΡŒΠ½Ρ‹Ρ… частиц.
β€ƒΠšΠ°ΠΊ ΠΏΡ€Π°Π²ΠΈΠ»ΠΎ, свободныС заряды двиТутся Π² ΠΏΡ€ΠΎΠ²ΠΎΠ΄Π½ΠΈΠΊΠ΅ хаотичСски равновСроятно Π²ΠΎ всС стороны, поэтому Π² Π½Π΅ΠΏΠΎΠ΄Π²ΠΈΠΆΠ½ΠΎΠΌ ΠΏΡ€ΠΎΠ²ΠΎΠ΄Π½ΠΈΠΊΠ΅ срСднСС Π·Π½Π°Ρ‡Π΅Π½ΠΈΠ΅ Π²Π΅ΠΊΡ‚ΠΎΡ€Π° силы Π›ΠΎΡ€Π΅Π½Ρ†Π° Ρ€Π°Π²Π½ΠΎ Π½ΡƒΠ»ΡŽ. ΠŸΡ€ΠΈ Π΄Π²ΠΈΠΆΠ΅Π½ΠΈΠΈ ΠΏΡ€ΠΎΠ²ΠΎΠ΄Π½ΠΈΠΊΠ° Π½Π° хаотичСскоС Ρ‚Π΅ΠΏΠ»ΠΎΠ²ΠΎΠ΅ Π΄Π²ΠΈΠΆΠ΅Π½ΠΈΠ΅ свободных зарядов накладываСтся Π½Π°ΠΏΡ€Π°Π²Π»Π΅Π½Π½ΠΎΠ΅ Π΄Π²ΠΈΠΆΠ΅Π½ΠΈΠ΅ ΠΏΡ€ΠΎΠ²ΠΎΠ΄Π½ΠΈΠΊΠ° Ρ†Π΅Π»ΠΈΠΊΠΎΠΌ, благодаря Ρ‡Π΅ΠΌΡƒ появляСтся отличная ΠΎΡ‚ нуля Ρ€Π΅Π·ΡƒΠ»ΡŒΡ‚ΠΈΡ€ΡƒΡŽΡ‰Π°Ρ сила Π›ΠΎΡ€Π΅Π½Ρ†Π°, одинаковая для всСх частиц. ИмСнно эта постоянная сила ΠΏΡ€ΠΈΠ²ΠΎΠ΄ΠΈΡ‚ ΠΊ возникновСнию элСктричСского Ρ‚ΠΎΠΊΠ° βˆ’ Π½Π°ΠΏΡ€Π°Π²Π»Π΅Π½Π½ΠΎΠ³ΠΎ двиТСния заряТСнных частиц. Π­Ρ‚ΠΎ Π΄Π°Π΅Ρ‚ вСскиС основания Π½Π΅ ΠΏΡ€ΠΈΠ½ΠΈΠΌΠ°Ρ‚ΡŒ Π²ΠΎ Π²Π½ΠΈΠΌΠ°Π½ΠΈΠ΅ Π±ΡƒΡ€Π½ΠΎΠ΅, Π½ΠΎ хаотичСскоС Ρ‚Π΅ΠΏΠ»ΠΎΠ²ΠΎΠ΅ Π΄Π²ΠΈΠΆΠ΅Π½ΠΈΠ΅.
β€ƒΠŸΠΎΠ΄ дСйствиСм силы Π›ΠΎΡ€Π΅Π½Ρ†Π° свободныС заряды Π½Π°Ρ‡Π½ΡƒΡ‚ ΡΠΌΠ΅Ρ‰Π°Ρ‚ΡŒΡΡ ΠΊ Ρ‚ΠΎΡ€Ρ†Π°ΠΌ Ρ†ΠΈΠ»ΠΈΠ½Π΄Ρ€Π°, Π³Π΄Π΅ Π±ΡƒΠ΄ΡƒΡ‚ ΠΈΠ½Π΄ΡƒΡ†ΠΈΡ€ΠΎΠ²Π°Ρ‚ΡŒΡΡ элСктричСскиС заряды, описываСмыС повСрхностными плотностями Β±Οƒ. Π’ свою ΠΎΡ‡Π΅Ρ€Π΅Π΄ΡŒ, эти заряды Π½Π°Ρ‡Π½ΡƒΡ‚ ΡΠΎΠ·Π΄Π°Π²Π°Ρ‚ΡŒ элСктричСскоС ΠΏΠΎΠ»Π΅, дСйствиС ΠΊΠΎΡ‚ΠΎΡ€ΠΎΠ³ΠΎ Π½Π° заряТСнныС частицы Π±ΡƒΠ΄Π΅Ρ‚ Π½Π°ΠΏΡ€Π°Π²Π»Π΅Π½ΠΎ Π² сторону ΠΏΡ€ΠΎΡ‚ΠΈΠ²ΠΎΠΏΠΎΠ»ΠΎΠΆΠ½ΡƒΡŽ силС Π›ΠΎΡ€Π΅Π½Ρ†Π°. ΠŸΡ€ΠΈ постоянной скорости двиТСния ΠΏΡ€ΠΎΠ²ΠΎΠ΄Π½ΠΈΠΊΠ° установится равновСсиС, ΠΏΡ€ΠΈ ΠΊΠΎΡ‚ΠΎΡ€ΠΎΠΌ Π΄Π²ΠΈΠΆΠ΅Π½ΠΈΠ΅ зарядов прСкратится, Π½ΠΎ Π² ΠΏΡ€ΠΎΠ²ΠΎΠ΄Π½ΠΈΠΊΠ΅ Π±ΡƒΠ΄Π΅Ρ‚ ΡΡƒΡ‰Π΅ΡΡ‚Π²ΠΎΠ²Π°Ρ‚ΡŒ элСктричСскоС ΠΏΠΎΠ»Π΅, созданноС ΠΈΠ½Π΄ΡƒΡ†ΠΈΡ€ΠΎΠ²Π°Π½Π½Ρ‹ΠΌΠΈ зарядами. Π’ ΡƒΡΡ‚Π°Π½ΠΎΠ²ΠΈΠ²ΡˆΠ΅ΠΌΡΡ Ρ€Π΅ΠΆΠΈΠΌΠ΅ сила Π›ΠΎΡ€Π΅Π½Ρ†Π° FΠ› = qvB, Π΄Π΅ΠΉΡΡ‚Π²ΡƒΡŽΡ‰Π°Ρ Π½Π° частицу, Π±ΡƒΠ΄Π΅Ρ‚ ΡƒΡ€Π°Π²Π½ΠΎΠ²Π΅ΡˆΠ΅Π½Π° силой со стороны элСктричСского поля Fэл = qE. ΠŸΡ€ΠΈΡ€Π°Π²Π½ΠΈΠ²Π°Ρ эти силы, ΠΎΠΏΡ€Π΅Π΄Π΅Π»ΠΈΠΌ Π½Π°ΠΏΡ€ΡΠΆΠ΅Π½Π½ΠΎΡΡ‚ΡŒ элСктричСского поля Π² ΠΏΡ€ΠΎΠ²ΠΎΠ΄Π½ΠΈΠΊΠ΅

Π’Π°ΠΊ сила Π›ΠΎΡ€Π΅Π½Ρ†Π° ΠΎΠ΄ΠΈΠ½Π°ΠΊΠΎΠ²Π° Π²ΠΎ всСх Ρ‚ΠΎΡ‡ΠΊΠ°Ρ… ΠΏΡ€ΠΎΠ²ΠΎΠ΄Π½ΠΈΠΊΠ°, Ρ‚ΠΎ ΠΈ элСктричСская сила Ρ‚Π°ΠΊΠΆΠ΅ Π΄ΠΎΠ»ΠΆΠ½Π° Π±Ρ‹Ρ‚ΡŒ постоянна, Ρ‚ΠΎ Π΅ΡΡ‚ΡŒ возникшСС элСктричСскоС ΠΏΠΎΠ»Π΅ являСтся ΠΎΠ΄Π½ΠΎΡ€ΠΎΠ΄Π½Ρ‹ΠΌ. Π­Ρ‚ΠΎ элСктричСскоС ΠΏΠΎΠ»Π΅ ΠΌΠΎΠΆΠ½ΠΎ Ρ‚Π°ΠΊΠΆΠ΅ Ρ…Π°Ρ€Π°ΠΊΡ‚Π΅Ρ€ΠΈΠ·ΠΎΠ²Π°Ρ‚ΡŒ Ρ€Π°Π·Π½ΠΎΡΡ‚ΡŒΡŽ ΠΏΠΎΡ‚Π΅Π½Ρ†ΠΈΠ°Π»ΠΎΠ² ΠΌΠ΅ΠΆΠ΄Ρƒ Ρ‚ΠΎΡ€Ρ†Π°ΠΌΠΈ Ρ†ΠΈΠ»ΠΈΠ½Π΄Ρ€Π°, которая Ρ€Π°Π²Π½Π°

Π³Π΄Π΅ l βˆ’ Π΄Π»ΠΈΠ½Π° ΠΏΡ€ΠΎΠ²ΠΎΠ΄Π½ΠΈΠΊΠ°.
 Била Π›ΠΎΡ€Π΅Π½Ρ†Π°, Π΄Π΅ΠΉΡΡ‚Π²ΡƒΡŽΡ‰Π°Ρ Π½Π° свободныС заряды Π² ΠΏΡ€ΠΎΠ²ΠΎΠ΄Π½ΠΈΠΊΠ΅, ΠΌΠΎΠΆΠ΅Ρ‚ ΡΠ²Π»ΡΡ‚ΡŒΡΡ стороннСй силой, Ρ‚ΠΎ Π΅ΡΡ‚ΡŒ ΠΏΡ€ΠΈΠ²ΠΎΠ΄ΠΈΡ‚ΡŒ ΠΊ возникновСнию элСктричСского Ρ‚ΠΎΠΊΠ° Π² Π·Π°ΠΌΠΊΠ½ΡƒΡ‚ΠΎΠΌ ΠΊΠΎΠ½Ρ‚ΡƒΡ€Π΅, Ссли Π΅Π³ΠΎ ΠΏΠΎΠ΄ΠΊΠ»ΡŽΡ‡ΠΈΡ‚ΡŒ ΠΊ двиТущСмуся ΠΏΡ€ΠΎΠ²ΠΎΠ΄Π½ΠΈΠΊΡƒ.
β€ƒΠŸΡƒΡΡ‚ΡŒ рассматриваСмый ΠΏΡ€ΠΎΠ²ΠΎΠ΄Π½ΠΈΠΊ AC ΠΌΠΎΠΆΠ΅Ρ‚ ΡΠΊΠΎΠ»ΡŒΠ·ΠΈΡ‚ΡŒ ΠΏΠΎ Π΄Π²ΡƒΠΌ ΠΏΠ°Ρ€Π°Π»Π»Π΅Π»ΡŒΠ½Ρ‹ΠΌ шинам (Ρ€Π΅Π»ΡŒΡΠ°ΠΌ), соСдинСнным ΠΌΠ΅ΠΆΠ΄Ρƒ собой (рис. 510).

рис. 510
 Вся систСма ΠΏΠΎΠΌΠ΅Ρ‰Π΅Π½Π° Π² ΠΎΠ΄Π½ΠΎΡ€ΠΎΠ΄Π½ΠΎΠ΅ ΠΌΠ°Π³Π½ΠΈΡ‚Π½ΠΎΠ΅ ΠΏΠΎΠ»Π΅, Π²Π΅ΠΊΡ‚ΠΎΡ€ ΠΈΠ½Π΄ΡƒΠΊΡ†ΠΈΠΈ ΠΊΠΎΡ‚ΠΎΡ€ΠΎΠ³ΠΎ B пСрпСндикулярСн плоскости шин. Для упрощСния Π±ΡƒΠ΄Π΅ΠΌ ΡΡ‡ΠΈΡ‚Π°Ρ‚ΡŒ, Ρ‡Ρ‚ΠΎ сопротивлСния шин ΠΈ двиТущСгося ΠΏΡ€ΠΎΠ²ΠΎΠ΄Π½ΠΈΠΊΠ° (ΠΏΠ΅Ρ€Π΅ΠΌΡ‹Ρ‡ΠΊΠΈ) ΠΏΡ€Π΅Π½Π΅Π±Ρ€Π΅ΠΆΠΈΠΌΠΎ ΠΌΠ°Π»Ρ‹ ΠΏΠΎ ΡΡ€Π°Π²Π½Π΅Π½ΠΈΡŽ с сопротивлСниСм ΡΠΎΠ΅Π΄ΠΈΠ½ΡΡŽΡ‰Π΅Π³ΠΎ рСзистора R.
 Если ΠΊ ΠΏΠΎΠ΄Π²ΠΈΠΆΠ½ΠΎΠΌΡƒ ΠΏΡ€ΠΎΠ²ΠΎΠ΄Π½ΠΈΠΊΡƒ ΠΏΡ€ΠΈΠ»ΠΎΠΆΠΈΡ‚ΡŒ внСшнюю силу F, ΠΊΠ°ΠΊ ΠΏΠΎΠΊΠ°Π·Π°Π½ΠΎ Π½Π° рисункС, Ρ‚ΠΎ ΠΎΠ½ ΠΏΡ€ΠΈΠ΄Π΅Ρ‚ Π² Π΄Π²ΠΈΠΆΠ΅Π½ΠΈΠ΅. Под дСйствиС силы Π›ΠΎΡ€Π΅Π½Ρ†Π° свободныС заряды Π² ΠΏΡ€ΠΎΠ²ΠΎΠ΄Π½ΠΈΠΊΠ΅ ΠΏΡ€ΠΈΠ΄ΡƒΡ‚ Π² Π΄Π²ΠΈΠΆΠ΅Π½ΠΈΠ΅, создавая ΠΈΠ·Π±Ρ‹Ρ‚ΠΎΡ‡Π½Ρ‹Π΅ заряды Π½Π° ΠΊΠΎΠ½Ρ†Π°Ρ…. Π­Ρ‚ΠΈ заряды создадут элСктричСскоС ΠΏΠΎΠ»Π΅ Π²ΠΎ всСм ΠΊΠΎΠ½Ρ‚ΡƒΡ€Π΅, ΠΎΠ±Ρ€Π°Π·ΠΎΠ²Π°Π½Π½ΠΎΠΌ ΠΏΠ΅Ρ€Π΅ΠΌΡ‹Ρ‡ΠΊΠΎΠΉ, шинами ΠΈ ΡΠΎΠ΅Π΄ΠΈΠ½ΡΡŽΡ‰ΠΈΠΌ рСзистором, поэтому Π² ΠΊΠΎΠ½Ρ‚ΡƒΡ€Π΅ Π²ΠΎΠ·Π½ΠΈΠΊΠ½Π΅Ρ‚ элСктричСский Ρ‚ΠΎΠΊ. Π‘ΠΈΠ»Π° Π›ΠΎΡ€Π΅Π½Ρ†Π°, Π΄Π΅ΠΉΡΡ‚Π²ΡƒΡŽΡ‰Π°Ρ Π½Π° заряды двиТущСгося ΠΏΡ€ΠΎΠ²ΠΎΠ΄Π½ΠΈΠΊΠ°, Π±ΡƒΠ΄Π΅Ρ‚ ΠΈΠ³Ρ€Π°Ρ‚ΡŒ Ρ€ΠΎΠ»ΡŒ стороннСй, ΠΏΡ€Π΅ΠΎΠ΄ΠΎΠ»Π΅Π²Π°ΡŽΡ‰Π΅ΠΉ силы, Π΄Π΅ΠΉΡΡ‚Π²ΡƒΡŽΡ‰ΠΈΠ΅ со стороны элСктричСского поля. Π Π°Π±ΠΎΡ‚Π° этой силы ΠΏΠΎ ΠΏΠ΅Ρ€Π΅ΠΌΠ΅Ρ‰Π΅Π½ΠΈΡŽ Π΅Π΄ΠΈΠ½ΠΈΡ‡Π½ΠΎΠ³ΠΎ заряда (Ρ‚ΠΎ Π΅ΡΡ‚ΡŒ Π­Π”Π‘) Ρ€Π°Π²Π½Π° ΠΏΡ€ΠΎΠΈΠ·Π²Π΅Π΄Π΅Π½ΠΈΡŽ силы Π›ΠΎΡ€Π΅Π½Ρ†Π° Π½Π° расстояниС ΠΌΠ΅ΠΆΠ΄Ρƒ шинами

 НС смотря Π½Π° Ρ‚ΠΎ, Ρ‡Ρ‚ΠΎ это Π²Ρ‹Ρ€Π°ΠΆΠ΅Π½ΠΈΠ΅ для Π­Π”Π‘ ΠΏΠΎΠ»Π½ΠΎΡΡ‚ΡŒΡŽ совпадаСт с Ρ„ΠΎΡ€ΠΌΡƒΠ»ΠΎΠΉ (2) для разности ΠΏΠΎΡ‚Π΅Π½Ρ†ΠΈΠ°Π»ΠΎΠ², Π΅Π΅ смысл ΠΏΡ€ΠΈΠ½Ρ†ΠΈΠΏΠΈΠ°Π»ΡŒΠ½ΠΎ ΠΈΠ½ΠΎΠΉ. Π Π°Π·Π½ΠΎΡΡ‚ΡŒ ΠΏΠΎΡ‚Π΅Π½Ρ†ΠΈΠ°Π»ΠΎΠ² βˆ’ Π΅ΡΡ‚ΡŒ возмоТная Ρ€Π°Π±ΠΎΡ‚Π° сил элСктричСского поля, Π² рассматриваСмой Ρ†Π΅ΠΏΠΈ Π½Π°ΠΏΡ€Π°Π²Π»Π΅Π½ΠΈΠ΅ двиТСния заряТСнных частиц ΠΏΡ€ΠΎΡ‚ΠΈΠ²ΠΎΠΏΠΎΠ»ΠΎΠΆΠ½ΠΎ Π½Π°ΠΏΡ€Π°Π²Π»Π΅Π½ΠΈΡŽ силы со стороны элСктричСского поля. Π‘ΠΈΠ»Π° Π›ΠΎΡ€Π΅Π½Ρ†Π° ΡΠΎΠ²Π΅Ρ€ΡˆΠ°Π΅Ρ‚ Ρ€Π°Π±ΠΎΡ‚Ρƒ ΠΏΡ€ΠΎΡ‚ΠΈΠ² сил элСктричСского поля, поэтому ΠΎΠ½Π° ΠΈ являСтся стороннСй. ЭлСктричСскоС ΠΏΠΎΠ»Π΅ ΡΠΎΠ²Π΅Ρ€ΡˆΠ°Π΅Ρ‚ ΠΏΠΎΠ»ΠΎΠΆΠΈΡ‚Π΅Π»ΡŒΠ½ΡƒΡŽ Ρ€Π°Π±ΠΎΡ‚Ρƒ, «проталкивая» заряТСнныС частицы ΠΏΠΎ шинам ΠΈ ΡΠΎΠ΅Π΄ΠΈΠ½ΡΡŽΡ‰Π΅ΠΌΡƒ рСзистору (ΠΊΠΎΡ‚ΠΎΡ€Ρ‹Π΅ Π² Π΄Π°Π½Π½ΠΎΠΌ случаС ΠΎΠ±Ρ€Π°Π·ΡƒΡŽΡ‚ внСшнюю Ρ†Π΅ΠΏΡŒ).
β€ƒΠŸΠΎ Π·Π°ΠΊΠΎΠ½Ρƒ Ома сила возникшСго Π² Ρ†Π΅ΠΏΠΈ элСктричСского Ρ‚ΠΎΠΊΠ° Ρ€Π°Π²Π½Π°

 Вак ΠΊΠ°ΠΊ ΠΏΠΎ ΠΏΡ€ΠΎΠ²ΠΎΠ΄Π½ΠΈΠΊΡƒ ΠΈΠ΄Π΅Ρ‚ элСктричСский Ρ‚ΠΎΠΊ, Ρ‚ΠΎ Π½Π° Π½Π΅Π³ΠΎ со стороны ΠΌΠ°Π³Π½ΠΈΡ‚Π½ΠΎΠ³ΠΎ поля дСйствуСт сила АмпСра, равная

 НаправлСниС этой силы Ρ‚Π°ΠΊΠΆΠ΅ опрСдСляСтся Β«ΠΏΡ€Π°Π²ΠΈΠ»ΠΎΠΌ Π»Π΅Π²ΠΎΠΉ Ρ€ΡƒΠΊΠΈΒ», с ΠΏΠΎΠΌΠΎΡ‰ΡŒΡŽ ΠΊΠΎΡ‚ΠΎΡ€ΠΎΠ³ΠΎ Π»Π΅Π³ΠΊΠΎ ΠΎΠΏΡ€Π΅Π΄Π΅Π»ΠΈΡ‚ΡŒ, Ρ‡Ρ‚ΠΎ эта сила Π½Π°ΠΏΡ€Π°Π²Π»Π΅Π½Π° Π² сторону, ΠΏΡ€ΠΎΡ‚ΠΈΠ²ΠΎΠΏΠΎΠ»ΠΎΠΆΠ½ΡƒΡŽ Π²Π΅ΠΊΡ‚ΠΎΡ€Ρƒ скорости, поэтому Ρ„ΠΎΡ€ΠΌΡƒΠ»Ρƒ (5) ΠΌΠΎΠΆΠ½ΠΎ Π·Π°ΠΏΠΈΡΠ°Ρ‚ΡŒ Π² Π²Π΅ΠΊΡ‚ΠΎΡ€Π½ΠΎΠΌ Π²ΠΈΠ΄Π΅

β€ƒΠŸΠΎ своСму Ρ…Π°Ρ€Π°ΠΊΡ‚Π΅Ρ€Ρƒ эта сила ΠΏΠΎΠ»Π½ΠΎΡΡ‚ΡŒΡŽ совпадаСт с силой вязкого трСния (ΠΏΡ€ΠΎΠΏΠΎΡ€Ρ†ΠΈΠΎΠ½Π°Π»ΡŒΠ½Π° скорости ΠΈ Π½Π°ΠΏΡ€Π°Π²Π»Π΅Π½Π° Π² ΠΏΡ€ΠΎΡ‚ΠΈΠ²ΠΎΠΏΠΎΠ»ΠΎΠΆΠ½ΡƒΡŽ сторону), поэтому Π΅Π΅ часто Π½Π°Π·Ρ‹Π²Π°ΡŽΡ‚ силой ΠΌΠ°Π³Π½ΠΈΡ‚Π½ΠΎΠΉ вязкости.
 Ваким ΠΎΠ±Ρ€Π°Π·ΠΎΠΌ, Π½Π° Π΄Π²ΠΈΠΆΡƒΡ‰ΡƒΡŽΡΡ ΠΏΠ΅Ρ€Π΅ΠΌΡ‹Ρ‡ΠΊΡƒ, ΠΏΠΎΠΌΠΈΠΌΠΎ постоянной внСшнСй силы F, дСйствуСт сила ΠΌΠ°Π³Π½ΠΈΡ‚Π½ΠΎΠΉ вязкости, зависящСй ΠΎΡ‚ скорости. Π£Ρ€Π°Π²Π½Π΅Π½ΠΈΠ΅ Π²Ρ‚ΠΎΡ€ΠΎΠ³ΠΎ Π·Π°ΠΊΠΎΠ½Π° ΠΡŒΡŽΡ‚ΠΎΠ½Π° для ΠΏΠ΅Ρ€Π΅ΠΌΡ‹Ρ‡ΠΊΠΈ ΠΈΠΌΠ΅Π΅Ρ‚ Π²ΠΈΠ΄ (Π² ΠΏΡ€ΠΎΠ΅ΠΊΡ†ΠΈΠΈ Π½Π° Π½Π°ΠΏΡ€Π°Π²Π»Π΅Π½ΠΈΠ΅ Π²Π΅ΠΊΡ‚ΠΎΡ€Π° скорости):

β€ƒΠŸΠΎΠ΄ дСйствиСм этих сил сначала ΠΏΠ΅Ρ€Π΅ΠΌΡ‹Ρ‡ΠΊΠ° Π±ΡƒΠ΄Π΅Ρ‚ Π΄Π²ΠΈΠ³Π°Ρ‚ΡŒΡΡ ускорСно, ΠΏΡ€ΠΈΡ‡Π΅ΠΌ с ΡƒΠ²Π΅Π»ΠΈΡ‡Π΅Π½ΠΈΠ΅ΠΌ скорости ΠΌΠΎΠ΄ΡƒΠ»ΡŒ ускорСния Π±ΡƒΠ΄Π΅Ρ‚ ΡƒΠΌΠ΅Π½ΡŒΡˆΠ°Ρ‚ΡŒΡΡ, Π½Π°ΠΊΠΎΠ½Π΅Ρ†, ΠΏΠ΅Ρ€Π΅ΠΌΡ‹Ρ‡ΠΊΠ° станСт Π΄Π²ΠΈΠ³Π°Ρ‚ΡŒΡΡ с постоянной ΡΠΊΠΎΡ€ΠΎΡΡ‚ΡŒΡŽ, которая называСтся ΡΠΊΠΎΡ€ΠΎΡΡ‚ΡŒΡŽ ΡƒΡΡ‚Π°Π½ΠΎΠ²ΠΈΠ²ΡˆΠ΅Π³ΠΎΡΡ двиТСния v. Π’Π΅Π»ΠΈΡ‡ΠΈΠ½Ρƒ этой скорости ΠΌΠΎΠΆΠ½ΠΎ Π½Π°ΠΉΡ‚ΠΈ ΠΈΠ· условия F = FA, ΠΈΠ· ΠΊΠΎΡ‚ΠΎΡ€ΠΎΠ³ΠΎ слСдуСт

 Рассмотрим Ρ‚Π΅ΠΏΠ΅Ρ€ΡŒ ΠΏΡ€Π΅ΠΎΠ±Ρ€Π°Π·ΠΎΠ²Π°Π½ΠΈΠ΅ энСргии Π² Π΄Π°Π½Π½ΠΎΠΉ систСмС Π² ΡƒΡΡ‚Π°Π½ΠΎΠ²ΠΈΠ²ΡˆΠ΅ΠΌΡΡ Ρ€Π΅ΠΆΠΈΠΌΠ΅ двиТСния. Π—Π° ΠΏΡ€ΠΎΠΌΠ΅ΠΆΡƒΡ‚ΠΎΠΊ Π²Ρ€Π΅ΠΌΠ΅Π½ΠΈ Ξ”t ΠΏΠ΅Ρ€Π΅ΠΌΡ‹Ρ‡ΠΊΠ° смСщаСтся Π½Π° расстояниС Ξ”x = vΞ”t, ΡΠ»Π΅Π΄ΠΎΠ²Π°Ρ‚Π΅Π»ΡŒΠ½ΠΎ, внСшняя сила ΠΏΡ€ΠΈ этом ΡΠΎΠ²Π΅Ρ€ΡˆΠ°Π΅Ρ‚ Ρ€Π°Π±ΠΎΡ‚Ρƒ

 За это ΠΆΠ΅ врСмя Π½Π° рСзисторС выдСлится количСство Ρ‚Π΅ΠΏΠ»ΠΎΡ‚Ρ‹ Ρ€Π°Π²Π½ΠΎΠ΅

β€ƒΠšΠ°ΠΊ ΠΈ слСдовало ΠΎΠΆΠΈΠ΄Π°Ρ‚ΡŒ, количСство Π²Ρ‹Π΄Π΅Π»ΠΈΠ²ΡˆΠ΅ΠΉΡΡ Ρ‚Π΅ΠΏΠ»ΠΎΡ‚Ρ‹ Π² точности Ρ€Π°Π²Π½ΠΎ Ρ€Π°Π±ΠΎΡ‚Π΅ внСшнСй силы. ΠŸΠΎΡΡ‚ΠΎΠΌΡƒ источником энСргии элСктричСского Ρ‚ΠΎΠΊΠ° Π² ΠΊΠΎΠ½Ρ‚ΡƒΡ€Π΅ являСтся устройство, ΠΏΠ΅Ρ€Π΅Π΄Π²ΠΈΠ³Π°ΡŽΡ‰Π΅Π΅ ΠΏΠ΅Ρ€Π΅ΠΌΡ‹Ρ‡ΠΊΡƒ (Ρ‚Π°ΠΊΠΈΠΌ устройством ΠΌΠΎΠΆΠ΅Ρ‚ Π±Ρ‹Ρ‚ΡŒ ΠΈ ваша Ρ€ΡƒΠΊΠ°). Если прСкратится дСйствиС этой силы, Ρ‚ΠΎ ΠΈ Ρ‚ΠΎΠΊ Π² ΠΊΠΎΠ½Ρ‚ΡƒΡ€Π΅ исчСзнСт.
 Задания для ΡΠ°ΠΌΠΎΡΡ‚ΠΎΡΡ‚Π΅Π»ΡŒΠ½ΠΎΠΉ Ρ€Π°Π±ΠΎΡ‚Ρ‹.
 1. ΠžΠ±ΡŠΡΡΠ½ΠΈΡ‚Π΅, ΠΏΠΎΡ‡Π΅ΠΌΡƒ ΠΏΡ€ΠΈ ΠΈΠ½Π΄ΡƒΠΊΡ†ΠΈΠΈ ΠΌΠ°Π³Π½ΠΈΡ‚Π½ΠΎΠ³ΠΎ поля стрСмящСйся ΠΊ Π½ΡƒΠ»ΡŽ, ΡΠΊΠΎΡ€ΠΎΡΡ‚ΡŒ ΠΏΠ΅Ρ€Π΅ΠΌΡ‹Ρ‡ΠΊΠΈ, рассчитанная ΠΏΠΎ Ρ„ΠΎΡ€ΠΌΡƒΠ»Π΅ (8) стрСмится ΠΊ бСсконСчности.
 2. ΠžΠ±ΡŠΡΡΠ½ΠΈΡ‚Π΅, ΠΏΠΎΡ‡Π΅ΠΌΡƒ с ростом сопротивлСния рСзистора ΡΠΊΠΎΡ€ΠΎΡΡ‚ΡŒ ΠΏΠ΅Ρ€Π΅ΠΌΡ‹Ρ‡ΠΊΠΈ возрастаСт.
 3. ΠŸΠΎΠΊΠ°ΠΆΠΈΡ‚Π΅, Ρ‡Ρ‚ΠΎ Π² процСссС Ρ€Π°Π·Π³ΠΎΠ½Π° Ρ€Π°Π±ΠΎΡ‚Π° внСшнСй силы Ρ€Π°Π²Π½Π° суммС измСнСния кинСтичСской энСргии ΠΏΠ΅Ρ€Π΅ΠΌΡ‹Ρ‡ΠΊΠΈ ΠΈ количСства Ρ‚Π΅ΠΏΠ»ΠΎΡ‚Ρ‹, Π²Ρ‹Π΄Π΅Π»ΡΡŽΡ‰Π΅ΠΉΡΡ Π½Π° ΠΏΠ΅Ρ€Π΅ΠΌΡ‹Ρ‡ΠΊΠ΅.

 В Π΄Π°Π½Π½ΠΎΠΌ случаС ΠΌΠ°Π³Π½ΠΈΡ‚Π½ΠΎΠ΅ ΠΏΠΎΠ»Π΅ ΠΈΠ³Ρ€Π°Π΅Ρ‚ Ρ€ΠΎΠ»ΡŒ своСобразного посрСдника, ΡΠΏΠΎΡΠΎΠ±ΡΡ‚Π²ΡƒΡŽΡ‰Π΅Π³ΠΎ ΠΏΡ€Π΅ΠΎΠ±Ρ€Π°Π·ΠΎΠ²Π°Π½ΠΈΡŽ энСргии внСшнСго источника (ΡΠΎΠ·Π΄Π°ΡŽΡ‰Π΅Π³ΠΎ внСшнюю силу) Π² ΡΠ½Π΅Ρ€Π³ΠΈΡŽ элСктричСского Ρ‚ΠΎΠΊΠ°, которая Π·Π°Ρ‚Π΅ΠΌ прСобразуСтся Π² Ρ‚Π΅ΠΏΠ»ΠΎΠ²ΡƒΡŽ ΡΠ½Π΅Ρ€Π³ΠΈΡŽ. Π‘Π°ΠΌΠΎ ΠΆΠ΅ внСшнСС ΠΌΠ°Π³Π½ΠΈΡ‚Π½ΠΎΠ΅ ΠΏΠΎΠ»Π΅ ΠΏΡ€ΠΈ этом Π½Π΅ измСняСтся.
ΠžΠ³ΠΎΠ²ΠΎΡ€ΠΊΠ° ΠΎ внСшнСм ΠΏΠΎΠ»Π΅ Π² Π΄Π°Π½Π½ΠΎΠΌ случаС Π½Π΅ случайно, ΠΈΠ½Π΄ΡƒΡ†ΠΈΡ€ΠΎΠ²Π°Π½Π½Ρ‹ΠΉ Π² ΠΊΠΎΠ½Ρ‚ΡƒΡ€Π΅ элСктричСский Ρ‚ΠΎΠΊ создаСт своС собствСнноС ΠΌΠ°Π³Π½ΠΈΡ‚Π½ΠΎΠ΅ ΠΏΠΎΠ»Π΅1 B/. По ΠΏΡ€Π°Π²ΠΈΠ»Ρƒ Π±ΡƒΡ€Π°Π²Ρ‡ΠΈΠΊΠ° это ΠΏΠΎΠ»Π΅ Π½Π°ΠΏΡ€Π°Π²Π»Π΅Π½ΠΎ ΠΏΡ€ΠΎΡ‚ΠΈΠ²ΠΎΠΏΠΎΠ»ΠΎΠΆΠ½ΠΎ Π²Π½Π΅ΡˆΠ½Π΅ΠΌΡƒ полю B (рис. 511).


рис. 511
 Направим Ρ‚Π΅ΠΏΠ΅Ρ€ΡŒ Π½Π°ΠΏΡ€Π°Π²Π»Π΅Π½ΠΈΠ΅ внСшнСй силы Π½Π° ΠΏΡ€ΠΎΡ‚ΠΈΠ²ΠΎΠΏΠΎΠ»ΠΎΠΆΠ½ΠΎΠ΅. ΠŸΡ€ΠΈ этом измСнятся направлСния двиТСния ΠΏΠ΅Ρ€Π΅ΠΌΡ‹Ρ‡ΠΊΠΈ, силы Π›ΠΎΡ€Π΅Π½Ρ†Π°, элСктричСского Ρ‚ΠΎΠΊΠ° Π² ΠΊΠΎΠ½Ρ‚ΡƒΡ€Π΅ ΠΈ ΠΈΠ½Π΄ΡƒΠΊΡ†ΠΈΠΈ ΠΌΠ°Π³Π½ΠΈΡ‚Π½ΠΎΠ³ΠΎ поля этого Ρ‚ΠΎΠΊΠ° (рис. 512).

рис. 512
 Во Π΅ΡΡ‚ΡŒ Π² этом случаС Π½Π°ΠΏΡ€Π°Π²Π»Π΅Π½ΠΈΠ΅ Π²Π΅ΠΊΡ‚ΠΎΡ€Π° ΠΈΠ½Π΄ΡƒΠΊΡ†ΠΈΠΈ B/ Π±ΡƒΠ΄Π΅Ρ‚ ΡΠΎΠ²ΠΏΠ°Π΄Π°Ρ‚ΡŒ с Π½Π°ΠΏΡ€Π°Π²Π»Π΅Π½ΠΈΠ΅ΠΌ внСшнСго поля B. Π’Π°ΠΊΠΈΠΌ ΠΎΠ±Ρ€Π°Π·ΠΎΠΌ, Π½Π°ΠΏΡ€Π°Π²Π»Π΅Π½ΠΈΠ΅ ΠΈΠ½Π΄ΡƒΡ†ΠΈΡ€ΠΎΠ²Π°Π½Π½ΠΎΠ³ΠΎ поля опрСдСляСтся Π½Π΅ Ρ‚ΠΎΠ»ΡŒΠΊΠΎ Π½Π°ΠΏΡ€Π°Π²Π»Π΅Π½ΠΈΠ΅ΠΌ внСшнСго поля, Π½ΠΎ ΠΈ Π½Π°ΠΏΡ€Π°Π²Π»Π΅Π½ΠΈΠ΅ΠΌ двиТСния ΠΏΠ΅Ρ€Π΅ΠΌΡ‹Ρ‡ΠΊΠΈ.
β€ƒΠŸΠΎΠ΄Ρ‡Π΅Ρ€ΠΊΠ½Π΅ΠΌ, сила АмпСра, ΠΈΠ³Ρ€Π°ΡŽΡ‰Π°Ρ Ρ€ΠΎΠ»ΡŒ силы вязкости, ΠΈ Π² этом (ΠΈ Π²ΠΎ всСх Π΄Ρ€ΡƒΠ³ΠΈΡ…) случаС ΠΏΡ€ΠΎΡ‚ΠΈΠ²ΠΎΠΏΠΎΠ»ΠΎΠΆΠ½Π° скорости двиТСния ΠΏΠ΅Ρ€Π΅ΠΌΡ‹Ρ‡ΠΊΠΈ.
β€ƒΠŸΠΎΠΏΡ‹Ρ‚Π°Π΅ΠΌΡΡ ΡΡ„ΠΎΡ€ΠΌΡƒΠ»ΠΈΡ€ΠΎΠ²Π°Ρ‚ΡŒ ΠΎΠ±Ρ‰Π΅Π΅ ΠΏΡ€Π°Π²ΠΈΠ»ΠΎ, ΠΏΠΎΠ·Π²ΠΎΠ»ΡΡŽΡ‰Π΅Π΅ ΠΎΠΏΡ€Π΅Π΄Π΅Π»ΠΈΡ‚ΡŒ Π½Π°ΠΏΡ€Π°Π²Π»Π΅Π½ΠΈΠ΅ ΠΈΠ½Π΄ΡƒΠΊΡ†ΠΈΠΎΠ½Π½ΠΎΠ³ΠΎ Ρ‚ΠΎΠΊΠ°. На рис. 513 Π΅Ρ‰Π΅ Ρ€Π°Π· ΠΈΠ·ΠΎΠ±Ρ€Π°ΠΆΠ΅Π½Ρ‹ схСмы рассматриваСмых экспСримСнтов, Ссли ΠΏΠΎΡΠΌΠΎΡ‚Ρ€Π΅Ρ‚ΡŒ Π½Π° Π½ΠΈΡ… свСрху.

рис. 513
 НС зависимо ΠΎΡ‚ направлСния двиТСния ΠΏΠ΅Ρ€Π΅ΠΌΡ‹Ρ‡ΠΊΠΈ, Π­Π”Π‘ ΠΈΠ½Π΄ΡƒΠΊΡ†ΠΈΠΈ Π² ΠΊΠΎΠ½Ρ‚ΡƒΡ€Π΅ ΠΏΠΎ ΠΌΠΎΠ΄ΡƒΠ»ΡŽ опрСдСляСтся Ρ„ΠΎΡ€ΠΌΡƒΠ»ΠΎΠΉ (3), ΠΊΠΎΡ‚ΠΎΡ€ΡƒΡŽ ΠΌΡ‹ ΠΏΡ€Π΅ΠΎΠ±Ρ€Π°Π·ΡƒΠ΅ΠΌ ΠΊ Π²ΠΈΠ΄Ρƒ

Π³Π΄Π΅ Ξ”x = vΞ”t βˆ’ расстояниС, Π½Π° ΠΊΠΎΡ‚ΠΎΡ€ΠΎΠ΅ смСщаСтся ΠΏΠ΅Ρ€Π΅ΠΌΡ‹Ρ‡ΠΊΠ° Π·Π° ΠΏΡ€ΠΎΠΌΠ΅ΠΆΡƒΡ‚ΠΎΠΊ Π²Ρ€Π΅ΠΌΠ΅Π½ΠΈ Ξ”t. Π’Ρ‹Ρ€Π°ΠΆΠ΅Π½ΠΈΠ΅, стоящСС Π² числитСлС этого выраТСния Ρ€Π°Π²Π½ΠΎ измСнСнию ΠΌΠ°Π³Π½ΠΈΡ‚Π½ΠΎΠ³ΠΎ ΠΏΠΎΡ‚ΠΎΠΊΠ° Ρ‡Π΅Ρ€Π΅Π· ΠΊΠΎΠ½Ρ‚ΡƒΡ€

ΠΏΡ€ΠΎΠΈΠ·ΠΎΡˆΠ΅Π΄ΡˆΠ΅Π΅ вслСдствиС измСнСния Π΅Π³ΠΎ ΠΏΠ»ΠΎΡ‰Π°Π΄ΠΈ. Π’Π΅ΠΏΠ΅Ρ€ΡŒ ΠΎΠ±Ρ€Π°Ρ‚ΠΈΠΌ Π²Π½ΠΈΠΌΠ°Π½ΠΈΠ΅ Π½Π° Π½Π°ΠΏΡ€Π°Π²Π»Π΅Π½ΠΈΠ΅ этой Π­Π”Π‘.
β€ƒΠšΠΎΠ½Π΅Ρ‡Π½ΠΎ, элСктродвиТущая сила, ΠΊΠ°ΠΊ Ρ€Π°Π±ΠΎΡ‚Π° сторонних сил являСтся скалярной Π²Π΅Π»ΠΈΡ‡ΠΈΠ½ΠΎΠΉ, поэтому Π³ΠΎΠ²ΠΎΡ€ΠΈΡ‚ΡŒ ΠΎ Π΅Π΅ Π½Π°ΠΏΡ€Π°Π²Π»Π΅Π½ΠΈΠΈ Π½Π΅ совсСм ΠΊΠΎΡ€Ρ€Π΅ΠΊΡ‚Π½ΠΎ.
β€ƒΠžΠ΄Π½Π°ΠΊΠΎ Π² Π΄Π°Π½Π½ΠΎΠΌ случаС Ρ€Π΅Ρ‡ΡŒ ΠΈΠ΄Π΅Ρ‚ ΠΎ Ρ€Π°Π±ΠΎΡ‚Π΅ сторонних сил ΠΏΠΎ ΠΊΠΎΠ½Ρ‚ΡƒΡ€Ρƒ, для ΠΊΠΎΡ‚ΠΎΡ€ΠΎΠ³ΠΎ ΠΌΠΎΠΆΠ½ΠΎ ΠΎΠΏΡ€Π΅Π΄Π΅Π»ΠΈΡ‚ΡŒ ΠΏΠΎΠ»ΠΎΠΆΠΈΡ‚Π΅Π»ΡŒΠ½ΠΎΠ΅ Π½Π°ΠΏΡ€Π°Π²Π»Π΅Π½ΠΈΠ΅ ΠΎΠ±Ρ…ΠΎΠ΄Π°. Для этого слСдуСт сначала Π²Ρ‹Π±Ρ€Π°Ρ‚ΡŒ Π½Π°ΠΏΡ€Π°Π²Π»Π΅Π½ΠΈΠ΅ ΠΏΠΎΠ»ΠΎΠΆΠΈΡ‚Π΅Π»ΡŒΠ½ΠΎΠΉ Π½ΠΎΡ€ΠΌΠ°Π»ΠΈ ΠΊ ΠΊΠΎΠ½Ρ‚ΡƒΡ€Ρƒ (ΠΎΡ‡Π΅Π²ΠΈΠ΄Π½ΠΎ, Ρ‡Ρ‚ΠΎ Π²Ρ‹Π±ΠΎΡ€ этого направлСния ΠΏΡ€ΠΎΠΈΠ·Π²ΠΎΠ»Π΅Π½). Как ΠΈ Ρ€Π°Π½Π΅Π΅ ΠΏΡ€ΠΈΠΌΠ΅ΠΌ Π·Π° ΠΏΠΎΠ»ΠΎΠΆΠΈΡ‚Π΅Π»ΡŒΠ½ΠΎΠ΅ Π½Π°ΠΏΡ€Π°Π²Π»Π΅Π½ΠΈΠ΅ Β«ΠΏΡ€ΠΎΡ‚ΠΈΠ² часовой стрСлки», Ссли ΡΠΌΠΎΡ‚Ρ€Π΅Ρ‚ΡŒ с ΠΊΠΎΠ½Ρ†Π° Π²Π΅ΠΊΡ‚ΠΎΡ€Π° ΠΏΠΎΠ»ΠΎΠΆΠΈΡ‚Π΅Π»ΡŒΠ½ΠΎΠΉ Π½ΠΎΡ€ΠΌΠ°Π»ΠΈ, соотвСтствСнно Π½Π°ΠΏΡ€Π°Π²Π»Π΅Π½ΠΈΠ΅ Β«ΠΏΠΎ часовой стрСлкС» Π±ΡƒΠ΄Π΅ΠΌ ΡΡ‡ΠΈΡ‚Π°Ρ‚ΡŒ ΠΎΡ‚Ρ€ΠΈΡ†Π°Ρ‚Π΅Π»ΡŒΠ½Ρ‹ΠΌ (рис. 514).

рис. 514
 В этом смыслС ΠΌΠΎΠΆΠ½ΠΎ Π³ΠΎΠ²ΠΎΡ€ΠΈΡ‚ΡŒ ΠΎ Π·Π½Π°ΠΊΠ΅ Π­Π”Π‘: Ссли ΠΏΡ€ΠΈ ΠΎΠ±Ρ…ΠΎΠ΄Π΅ Π² ΠΏΠΎΠ»ΠΎΠΆΠΈΡ‚Π΅Π»ΡŒΠ½ΠΎΠΌ Π½Π°ΠΏΡ€Π°Π²Π»Π΅Π½ΠΈΠΈ (Ρ‚.Π΅. Β«ΠΏΡ€ΠΎΡ‚ΠΈΠ² часовой стрСлки») сторонниС силы ΡΠΎΠ²Π΅Ρ€ΡˆΠ°ΡŽΡ‚ ΠΏΠΎΠ»ΠΎΠΆΠΈΡ‚Π΅Π»ΡŒΠ½ΡƒΡŽ Ρ€Π°Π±ΠΎΡ‚Ρƒ, Ρ‚ΠΎ ΠΈ Π²Π΅Π»ΠΈΡ‡ΠΈΠ½Ρƒ Π­Π”Π‘ Π±ΡƒΠ΄Π΅ΠΌ ΡΡ‡ΠΈΡ‚Π°Ρ‚ΡŒ ΠΏΠΎΠ»ΠΎΠΆΠΈΡ‚Π΅Π»ΡŒΠ½ΠΎΠΉ ΠΈ Π½Π°ΠΎΠ±ΠΎΡ€ΠΎΡ‚.
 В Π΄Π°Π½Π½ΠΎΠΌ случаС ΠΏΠΎΠ»ΠΎΠΆΠΈΡ‚Π΅Π»ΡŒΠ½ΠΎΠ΅ Π½Π°ΠΏΡ€Π°Π²Π»Π΅Π½ΠΈΠ΅ Π½ΠΎΡ€ΠΌΠ°Π»ΠΈ совмСстим с Π½Π°ΠΏΡ€Π°Π²Π»Π΅Π½ΠΈΠ΅ΠΌ Π²Π΅ΠΊΡ‚ΠΎΡ€Π° ΠΈΠ½Π΄ΡƒΠΊΡ†ΠΈΠΈ внСшнСго поля. ΠžΡ‡Π΅Π²ΠΈΠ΄Π½ΠΎ, Ρ‡Ρ‚ΠΎ Π½Π°ΠΏΡ€Π°Π²Π»Π΅Π½ΠΈΠ΅ ΠΈΠ½Π΄ΡƒΠΊΡ†ΠΈΠΎΠ½Π½ΠΎΠ³ΠΎ Ρ‚ΠΎΠΊΠ° совпадаСт с Π½Π°ΠΏΡ€Π°Π²Π»Π΅Π½ΠΈΠ΅ΠΌ Π­Π”Π‘.
 Богласно принятому ΠΎΠΏΡ€Π΅Π΄Π΅Π»Π΅Π½ΠΈΡŽ Π² случаС Π°) индуцируСмая Π­Π”Π‘ ΠΈ Ρ‚ΠΎΠΊ Π² ΠΊΠΎΠ½Ρ‚ΡƒΡ€Π΅ ΠΎΡ‚Ρ€ΠΈΡ†Π°Ρ‚Π΅Π»ΡŒΠ½Ρ‹, Π² случаС Π±) βˆ’ ΠΏΠΎΠ»ΠΎΠΆΠΈΡ‚Π΅Π»ΡŒΠ½Ρ‹. МоТно ΠΎΠ±ΠΎΠ±Ρ‰ΠΈΡ‚ΡŒ: Π·Π½Π°ΠΊ Π­Π”Π‘ ΠΏΡ€ΠΎΡ‚ΠΈΠ²ΠΎΠΏΠΎΠ»ΠΎΠΆΠ΅Π½ Π·Π½Π°ΠΊΡƒ измСнСния ΠΌΠ°Π³Π½ΠΈΡ‚Π½ΠΎΠ³ΠΎ ΠΏΠΎΡ‚ΠΎΠΊΠ° Ρ‡Π΅Ρ€Π΅Π· ΠΊΠΎΠ½Ρ‚ΡƒΡ€.
 Ваким ΠΎΠ±Ρ€Π°Π·ΠΎΠΌ, Π­Π”Π‘ ΠΈΠ½Π΄ΡƒΠΊΡ†ΠΈΠΈ Π² ΠΊΠΎΠ½Ρ‚ΡƒΡ€Π΅ Ρ€Π°Π²Π½Π° измСнСнию ΠΌΠ°Π³Π½ΠΈΡ‚Π½ΠΎΠ³ΠΎ ΠΏΠΎΡ‚ΠΎΠΊΠ° Ρ‡Π΅Ρ€Π΅Π· ΠΊΠΎΠ½Ρ‚ΡƒΡ€, взятому с ΠΏΡ€ΠΎΡ‚ΠΈΠ²ΠΎΠΏΠΎΠ»ΠΎΠΆΠ½Ρ‹ΠΌ Π·Π½Π°ΠΊΠΎΠΌ:

β€ƒΠŸΠΎΠ»ΡƒΡ‡Π΅Π½Π½ΠΎΠΌΡƒ ΠΏΡ€Π°Π²ΠΈΠ»Ρƒ ΠΌΠΎΠΆΠ½ΠΎ Π΄Π°Ρ‚ΡŒ нСсколько ΠΈΠ½ΡƒΡŽ ΠΈΠ½Ρ‚Π΅Ρ€ΠΏΡ€Π΅Ρ‚Π°Ρ†ΠΈΡŽ. ΠžΠ±Ρ€Π°Ρ‚ΠΈΠΌ Π²Π½ΠΈΠΌΠ°Π½ΠΈΠ΅ Π½Π° Π½Π°ΠΏΡ€Π°Π²Π»Π΅Π½ΠΈΠ΅ ΠΌΠ°Π³Π½ΠΈΡ‚Π½ΠΎΠ³ΠΎ поля, созданного ΠΈΠ½Π΄ΡƒΠΊΡ†ΠΈΠΎΠ½Π½Ρ‹ΠΌ Ρ‚ΠΎΠΊΠΎΠΌ: ΠΏΡ€ΠΈ ΡƒΠ²Π΅Π»ΠΈΡ‡Π΅Π½ΠΈΠΈ ΠΌΠ°Π³Π½ΠΈΡ‚Π½ΠΎΠ³ΠΎ ΠΏΠΎΡ‚ΠΎΠΊΠ° Ρ‡Π΅Ρ€Π΅Π· ΠΊΠΎΠ½Ρ‚ΡƒΡ€, это ΠΏΠΎΠ»Π΅ ΠΏΡ€ΠΎΡ‚ΠΈΠ²ΠΎΠΏΠΎΠ»ΠΎΠΆΠ½ΠΎ ΠΈΠ½Π΄ΡƒΠΊΡ†ΠΈΠΈ внСшнСго поля, ΠΏΡ€ΠΈ ΡƒΠΌΠ΅Π½ΡŒΡˆΠ΅Π½ΠΈΠΈ ΠΌΠ°Π³Π½ΠΈΡ‚Π½ΠΎΠ³ΠΎ ΠΏΠΎΡ‚ΠΎΠΊΠ°, ΠΏΠΎΠ»Π΅ ΠΈΠ½Π΄ΡƒΠΊΡ†ΠΈΠΎΠ½Π½ΠΎΠ³ΠΎ Ρ‚ΠΎΠΊΠ° Π½Π°ΠΏΡ€Π°Π²Π»Π΅Π½ΠΎ Ρ‚Π°ΠΊ ΠΆΠ΅, ΠΊΠ°ΠΊ внСшнСС ΠΏΠΎΠ»Π΅. Π’ΠΎ Π΅ΡΡ‚ΡŒ, ΠΏΠΎΠ»Π΅ ΠΈΠ½Π΄ΡƒΠΊΡ†ΠΈΠΎΠ½Π½ΠΎΠ³ΠΎ Ρ‚ΠΎΠΊΠ° Π² ΠΊΠΎΠ½Ρ‚ΡƒΡ€Π΅ прСпятствуСт измСнСнию ΠΌΠ°Π³Π½ΠΈΡ‚Π½ΠΎΠ³ΠΎ ΠΏΠΎΡ‚ΠΎΠΊΠ° Ρ‡Π΅Ρ€Π΅Π· этот ΠΊΠΎΠ½Ρ‚ΡƒΡ€. Π­Ρ‚ΠΎ ΠΏΡ€Π°Π²ΠΈΠ»ΠΎ являСтся ΡƒΠ½ΠΈΠ²Π΅Ρ€ΡΠ°Π»ΡŒΠ½Ρ‹ΠΌ для Π΄Π°Π½Π½ΠΎΠ³ΠΎ явлСния ΠΈ носит Π½Π°Π·Π²Π°Π½ΠΈΠ΅ ΠΏΡ€Π°Π²ΠΈΠ»ΠΎ Π›Π΅Π½Ρ†Π°2.
 Это ΠΏΡ€Π°Π²ΠΈΠ»ΠΎ тСсно связано с Π·Π°ΠΊΠΎΠ½ΠΎΠΌ сохранСния энСргии. Π”Π΅ΠΉΡΡ‚Π²ΠΈΡ‚Π΅Π»ΡŒΠ½ΠΎ, ΠΏΡ€Π΅Π΄ΠΏΠΎΠ»ΠΎΠΆΠΈΠΌ ΠΏΡ€ΠΎΡ‚ΠΈΠ²ΠΎΠΏΠΎΠ»ΠΎΠΆΠ½ΠΎΠ΅: ΠΏΡƒΡΡ‚ΡŒ Π½Π°ΠΏΡ€Π°Π²Π»Π΅Π½ΠΈΠ΅ ΠΈΠ½Π΄ΡƒΠΊΡ†ΠΈΠΈ ΠΌΠ°Π³Π½ΠΈΡ‚Π½ΠΎΠ³ΠΎ поля, созданного Ρ‚ΠΎΠΊΠΎΠΌ Π² ΠΊΠΎΠ½Ρ‚ΡƒΡ€Π΅ усиливаСт ΠΈΠ·ΠΌΠ΅Π½Π΅Π½ΠΈΠ΅ ΠΌΠ°Π³Π½ΠΈΡ‚Π½ΠΎΠ³ΠΎ ΠΏΠΎΡ‚ΠΎΠΊΠ° Ρ‡Π΅Ρ€Π΅Π· ΠΊΠΎΠ½Ρ‚ΡƒΡ€. Π’ этом случаС ΠΌΡ‹ ΠΏΠΎΠ»ΡƒΡ‡Π°Π΅ΠΌ Β«ΡΠ°ΠΌΠΎΡ€Π°Π·Π³ΠΎΠ½ΡΡŽΡ‰ΡƒΡŽΡΡΒ» систСму: Ссли ΠΌΠ°Π³Π½ΠΈΡ‚Π½Ρ‹ΠΉ ΠΏΠΎΡ‚ΠΎΠΊ Ρ‡Π΅Ρ€Π΅Π· ΠΊΠΎΠ½Ρ‚ΡƒΡ€ случайно увСличился, Ρ‚ΠΎ это ΠΏΡ€ΠΈΠ²Π΅Π΄Π΅Ρ‚ ΠΊ появлСнию элСктричСского Ρ‚ΠΎΠΊΠ°, ΠΊΠΎΡ‚ΠΎΡ€Ρ‹Π΅ Π΅Ρ‰Π΅ большС ΡƒΠ²Π΅Π»ΠΈΡ‡ΠΈΡ‚ ΠΏΠΎΡ‚ΠΎΠΊ Ρ‡Π΅Ρ€Π΅Π· ΠΊΠΎΠ½Ρ‚ΡƒΡ€, Ρ‡Ρ‚ΠΎ ΠΏΡ€ΠΈΠ²Π΅Π΄Π΅Ρ‚ ΠΊ Π΅Ρ‰Π΅ Π±ΠΎΠ»ΡŒΡˆΠ΅ΠΌΡƒ Π²ΠΎΠ·Ρ€Π°ΡΡ‚Π°Π½ΠΈΡŽ Ρ‚ΠΎΠΊΠ° ΠΈ Ρ‚.Π΄. Π’Π°ΠΊΠΈΠΌ ΠΎΠ±Ρ€Π°Π·ΠΎΠΌ, получаСтся, Ρ‡Ρ‚ΠΎ Π±Π΅Π· внСшнСго источника сила Ρ‚ΠΎΠΊ Π² ΠΊΠΎΠ½Ρ‚ΡƒΡ€Π΅ (ΠΈ Π΅Π³ΠΎ энСргия) Π½Π΅ΠΎΠ³Ρ€Π°Π½ΠΈΡ‡Π΅Π½Π½ΠΎ возрастаСт, Ρ‡Ρ‚ΠΎ ΠΈ ΠΏΡ€ΠΎΡ‚ΠΈΠ²ΠΎΡ€Π΅Ρ‡ΠΈΡ‚ Π·Π°ΠΊΠΎΠ½Ρƒ сохранСния энСргии.
β€ƒΠžΠ±Ρ€Π°Ρ‚ΠΈΡ‚Π΅ Π²Π½ΠΈΠΌΠ°Π½ΠΈΠ΅, Ρ‡Ρ‚ΠΎ Π² Π΄Π°Π½Π½ΠΎΠΌ рассуТдСнии ΠΌΡ‹ ΠΏΡ€ΠΈΠ½ΠΈΠΌΠ°Π΅ΠΌ Π²ΠΎ Π²Π½ΠΈΠΌΠ°Π½ΠΈΠ΅ ΠΌΠ°Π³Π½ΠΈΡ‚Π½Ρ‹ΠΉ ΠΏΠΎΡ‚ΠΎΠΊ Π½Π΅ Ρ‚ΠΎΠ»ΡŒΠΊΠΎ внСшнСго поля, Π½ΠΎ ΠΈ поля, создаваСмого ΠΈΠ½Π΄ΡƒΡ†ΠΈΡ€ΠΎΠ²Π°Π½Π½Ρ‹ΠΌ Ρ‚ΠΎΠΊΠΎΠΌ. Π­Ρ‚ΠΎ ΠΏΠΎΠ»Π΅ Π΄Π΅ΠΉΡΡ‚Π²ΠΈΡ‚Π΅Π»ΡŒΠ½ΠΎ Π½Π°Π΄ΠΎ ΡƒΡ‡ΠΈΡ‚Ρ‹Π²Π°Ρ‚ΡŒ: сила Π›ΠΎΡ€Π΅Π½Ρ†Π°, Π΄Π΅ΠΉΡΡ‚Π²ΡƒΡŽΡ‰Π°Ρ Π½Π° заряТСнныС частицы, опрСдСляСтся ΠΏΠΎΠ»Π½Ρ‹ΠΌ ΠΌΠ°Π³Π½ΠΈΡ‚Π½Ρ‹ΠΌ ΠΏΠΎΠ»Π΅ΠΌ Π² мСстС нахоТдСния заряда, нСзависимо ΠΎΡ‚ происхоТдСния этого поля. Π’Π°ΠΊΠΈΠΌ ΠΎΠ±Ρ€Π°Π·ΠΎΠΌ, посрСдством ΠΌΠ°Π³Π½ΠΈΡ‚Π½ΠΎΠ³ΠΎ поля элСктричСский Ρ‚ΠΎΠΊ способСн Π²ΠΎΠ·Π΄Π΅ΠΉΡΡ‚Π²ΠΎΠ²Π°Ρ‚ΡŒ сам Π½Π° сСбя βˆ’ ΠΈΠ·ΠΌΠ΅Π½ΡΡŽΡ‰ΠΈΠΉΡΡ Ρ‚ΠΎΠΊ создаСт ΠΈΠ·ΠΌΠ΅Π½ΡΡŽΡ‰Π΅Π΅ΡΡ ΠΌΠ°Π³Π½ΠΈΡ‚Π½ΠΎΠ΅ ΠΏΠΎΠ»Π΅, ΠΊΠΎΡ‚ΠΎΡ€ΠΎΠ΅ влияСт Π½Π° элСктричСский Ρ‚ΠΎΠΊ. Π­Ρ‚ΠΎ явлСниС называСтся самоиндукциСй, Π±ΠΎΠ»Π΅Π΅ ΠΏΠΎΠ΄Ρ€ΠΎΠ±Π½ΠΎ ΠΌΡ‹ познакомимся с Π½ΠΈΠΌ ΠΏΠΎΠ·Π΄Π½Π΅Π΅. Π—Π΄Π΅ΡΡŒ ΠΆΠ΅ ΠΎΡ‚ΠΌΠ΅Ρ‚ΠΈΠΌ, Ρ‡Ρ‚ΠΎ Π²ΠΎ ΠΌΠ½ΠΎΠ³ΠΈΡ… случаях этим явлСниСм ΠΌΠΎΠΆΠ½ΠΎ ΠΏΡ€Π΅Π½Π΅Π±Ρ€Π΅Ρ‡ΡŒ, Ρ‚Π°ΠΊ ΠΊΠ°ΠΊ ΠΎΠ±Ρ‹Ρ‡Π½ΠΎ ΠΈΠ½Π΄ΡƒΡ†ΠΈΡ€ΠΎΠ²Π°Π½Π½Ρ‹Π΅ поля достаточно слабы.
β€ƒΠœΠΎΠΆΠ½ΠΎ Ρ‚Π°ΠΊΠΆΠ΅ ΠΏΠΎΠΊΠ°Π·Π°Ρ‚ΡŒ, Ρ‡Ρ‚ΠΎ с ΠΏΡ€Π°Π²ΠΈΠ»ΠΎΠΌ Π›Π΅Π½Ρ†Π° связано ΠΈ Π½Π°ΠΏΡ€Π°Π²Π»Π΅Π½ΠΈΠ΅ силы ΠΌΠ°Π³Π½ΠΈΡ‚Π½ΠΎΠΉ вязкости, которая всСгда ΠΏΡ€ΠΎΡ‚ΠΈΠ²ΠΎΠΏΠΎΠ»ΠΎΠΆΠ½Π° скорости двиТСния ΠΏΡ€ΠΎΠ²ΠΎΠ΄Π½ΠΈΠΊΠ° Π² ΠΌΠ°Π³Π½ΠΈΡ‚Π½ΠΎΠΌ ΠΏΠΎΠ»Π΅.
 БамоС ΡˆΠΈΡ€ΠΎΠΊΠΎΠ΅ ΠΎΠ±ΠΎΠ±Ρ‰Π΅Π½ΠΈΠ΅ ΠΏΡ€Π°Π²ΠΈΠ»Π° Π›Π΅Π½Ρ†Π° Β«Π½Π° всС случаи ΠΆΠΈΠ·Π½ΠΈΒ» Π·Π²ΡƒΡ‡ΠΈΡ‚ Ρ‚Π°ΠΊ: слСдствиС стрСмится ΡƒΠΌΠ΅Π½ΡŒΡˆΠΈΡ‚ΡŒ ΠΏΡ€ΠΈΡ‡ΠΈΠ½Ρƒ. ΠŸΠΎΠΏΡ€ΠΎΠ±ΡƒΠΉΡ‚Π΅ ΡΠ°ΠΌΠΎΡΡ‚ΠΎΡΡ‚Π΅Π»ΡŒΠ½ΠΎ ΠΏΡ€ΠΈΠ΄ΡƒΠΌΠ°Ρ‚ΡŒ ΠΏΡ€ΠΈΠΌΠ΅Ρ€Ρ‹ ΠΈΠ· Ρ€Π°Π·Π»ΠΈΡ‡Π½Ρ‹Ρ… Ρ€Π°Π·Π΄Π΅Π»ΠΎΠ² Π½Π°ΡƒΠΊ, ΠΊΠΎΠ³Π΄Π° это ΠΏΡ€Π°Π²ΠΈΠ»ΠΎ справСдливо. Π‘Π»ΠΎΠΆΠ½Π΅Π΅ (хотя ΠΈ Π²ΠΎΠ·ΠΌΠΎΠΆΠ½ΠΎ) ΠΏΡ€ΠΈΠ΄ΡƒΠΌΠ°Ρ‚ΡŒ ΠΏΡ€ΠΈΠΌΠ΅Ρ€Ρ‹, ΠΊΠΎΠ³Π΄Π° это ΠΏΡ€Π°Π²ΠΈΠ»ΠΎ Π½Π΅ ΠΏΡ€ΠΈΠΌΠ΅Π½ΠΈΠΌΠΎ.
Рассмотрим Π΅Ρ‰Π΅ ΠΎΠ΄ΠΈΠ½ ΠΏΡ€ΠΈΠΌΠ΅Ρ€ возникновСния Π­Π”Π‘ Π² проводящСм ΠΊΠΎΠ½Ρ‚ΡƒΡ€Π΅, двиТущСмся Π² ΠΌΠ°Π³Π½ΠΈΡ‚Π½ΠΎΠΌ ΠΏΠΎΠ»Π΅. ΠŸΡƒΡΡ‚ΡŒ ΠΏΠΎΠ»Π΅ создаСтся цилиндричСским постоянным ΠΌΠ°Π³Π½ΠΈΡ‚ΠΎΠΌ, Π° ΠΊΡ€ΡƒΠ³ΠΎΠ²ΠΎΠΉ ΠΊΠΎΠ½Ρ‚ΡƒΡ€ L двиТСтся со ΡΠΊΠΎΡ€ΠΎΡΡ‚ΡŒΡŽ v вдоль оси этого ΠΌΠ°Π³Π½ΠΈΡ‚Π°, Ρ‚Π°ΠΊ, Ρ‡Ρ‚ΠΎ ΠΏΠ»ΠΎΡΠΊΠΎΡΡ‚ΡŒ ΠΊΠΎΠ½Ρ‚ΡƒΡ€Π° остаСтся всС врСмя пСрпСндикулярной оси ΠΌΠ°Π³Π½ΠΈΡ‚Π° (рис. 515).

рис. 515
 В этом случаС ΠΌΠ°Π³Π½ΠΈΡ‚Π½ΠΎΠ΅ ΠΏΠΎΠ»Π΅ Π½Π΅ являСтся ΠΎΠ΄Π½ΠΎΡ€ΠΎΠ΄Π½Ρ‹ΠΌ, Π½ΠΎ ΠΎΠ±Π»Π°Π΄Π°Π΅Ρ‚ осСвой симмСтриСй. ΠŸΡ€ΠΈ Π΄Π²ΠΈΠΆΠ΅Π½ΠΈΠΈ ΠΏΡ€ΠΎΠ²ΠΎΠ΄Π½ΠΈΠΊΠ° Π² этом ΠΏΠΎΠ»Π΅, Π½Π° заряТСнныС частицы дСйствуСт сила Π›ΠΎΡ€Π΅Π½Ρ†Π°, направлСнная вдоль ΠΏΡ€ΠΎΠ²ΠΎΠ΄Π½ΠΈΠΊΠ°, постоянна ΠΏΠΎ ΠΌΠΎΠ΄ΡƒΠ»ΡŽ Π½Π° всСм ΠΊΠΎΠ½Ρ‚ΡƒΡ€Π΅. Π’ этом случаС сила Π›ΠΎΡ€Π΅Π½Ρ†Π° ΠΎΠΏΡΡ‚ΡŒ выступаСт Π² качСствС стороннСй силы, приводящСй ΠΊ возникновСнию элСктричСского Ρ‚ΠΎΠΊΠ° Π² ΠΊΠΎΠ½Ρ‚ΡƒΡ€Π΅. Π Π°Π±ΠΎΡ‚Π° этой силы ΠΏΠΎ ΠΏΠ΅Ρ€Π΅ΠΌΠ΅Ρ‰Π΅Π½ΠΈΡŽ заряда ΠΏΠΎ Π·Π°ΠΌΠΊΠ½ΡƒΡ‚ΠΎΠΌΡƒ ΠΊΠΎΠ½Ρ‚ΡƒΡ€Ρƒ ΠΎΡ‚Π»ΠΈΡ‡Π½Π° ΠΎΡ‚ нуля, поэтому эта сила Π½Π΅ являСтся ΠΏΠΎΡ‚Π΅Π½Ρ†ΠΈΠ°Π»ΡŒΠ½ΠΎΠΉ. Вычислим Π­Π”Π‘ ΠΈΠ½Π΄ΡƒΠΊΡ†ΠΈΠΈ, Π²ΠΎΠ·Π½ΠΈΠΊΠ°ΡŽΡ‰Π΅ΠΉ Π² ΠΊΠΎΠ½Ρ‚ΡƒΡ€Π΅. На Π·Π°Ρ€ΡΠΆΠ΅Π½Π½ΡƒΡŽ частицу дСйствуСт сила, равная

Π³Π΄Π΅ Br βˆ’ ΠΊΠΎΠΌΠΏΠΎΠ½Π΅Π½Ρ‚Π° Π²Π΅ΠΊΡ‚ΠΎΡ€Π° ΠΈΠ½Π΄ΡƒΠΊΡ†ΠΈΠΈ, пСрпСндикулярная Π²Π΅ΠΊΡ‚ΠΎΡ€Ρƒ скорости ΠΏΡ€ΠΎΠ²ΠΎΠ΄Π½ΠΈΠΊΠ°, Π² Π΄Π°Π½Π½ΠΎΠΌ случаС ΠΎΠ½Π° Π½Π°ΠΏΡ€Π°Π²Π»Π΅Π½Π° Ρ€Π°Π΄ΠΈΠ°Π»ΡŒΠ½ΠΎ. Π’Π°ΠΊ ΠΊΠ°ΠΊ эта сила Π½Π° всСм ΠΊΠΎΠ½Ρ‚ΡƒΡ€Π΅ Π½Π°ΠΏΡ€Π°Π²Π»Π΅Π½Π° ΠΏΠΎ ΠΊΠ°ΡΠ°Ρ‚Π΅Π»ΡŒΠ½ΠΎΠΉ ΠΊ ΠΊΠΎΠ½Ρ‚ΡƒΡ€Ρƒ ΠΈ постоянна ΠΏΠΎ ΠΌΠΎΠ΄ΡƒΠ»ΡŽ, Ρ‚ΠΎ Π΅Π΅ Ρ€Π°Π±ΠΎΡ‚Π° ΠΏΠΎ ΠΏΠ΅Ρ€Π΅ΠΌΠ΅Ρ‰Π΅Π½ΠΈΡŽ Π΅Π΄ΠΈΠ½ΠΈΡ‡Π½ΠΎΠ³ΠΎ заряда, Ρ‚ΠΎ Π΅ΡΡ‚ΡŒ Π­Π”Π‘, Ρ€Π°Π²Π½Π°

Π³Π΄Π΅ L βˆ’ Π΄Π»ΠΈΠ½Π° ΠΊΠΎΠ½Ρ‚ΡƒΡ€Π°. Π§Ρ‚ΠΎΠ±Ρ‹ Π½Π°ΠΉΡ‚ΠΈ Π²Ρ‹Ρ€Π°ΠΆΠ΅Π½ΠΈΠ΅ для Ρ€Π°Π΄ΠΈΠ°Π»ΡŒΠ½ΠΎΠΉ ΡΠΎΡΡ‚Π°Π²Π»ΡΡŽΡ‰Π΅ΠΉ Π²Π΅ΠΊΡ‚ΠΎΡ€Π° ΠΈΠ½Π΄ΡƒΠΊΡ†ΠΈΠΈ Π²ΠΎΡΠΏΠΎΠ»ΡŒΠ·ΡƒΠ΅ΠΌΡΡ Ρ‚Π΅ΠΎΡ€Π΅ΠΌΠΎΠΉ ΠΎ ΠΌΠ°Π³Π½ΠΈΡ‚Π½ΠΎΠΌ ΠΏΠΎΡ‚ΠΎΠΊΠ΅. Π’ качСствС Π·Π°ΠΌΠΊΠ½ΡƒΡ‚ΠΎΠΉ повСрхности Π²Ρ‹Π±Π΅Ρ€Π΅ΠΌ Ρ‚ΠΎΠ½ΠΊΠΈΠΉ Ρ†ΠΈΠ»ΠΈΠ½Π΄Ρ€ Ρ‚ΠΎΠ»Ρ‰ΠΈΠ½ΠΎΠΉ Ξ”z = vΞ”t, ось ΠΊΠΎΡ‚ΠΎΡ€ΠΎΠ³ΠΎ совпадаСт с осью ΠΌΠ°Π³Π½ΠΈΡ‚Π°, Π° радиус Ρ€Π°Π²Π΅Π½ радиусу ΠΊΠΎΠ½Ρ‚ΡƒΡ€Π° (рис. 516).

рис. 516
β€ƒΠœΠ°Π³Π½ΠΈΡ‚Π½Ρ‹ΠΉ ΠΏΠΎΡ‚ΠΎΠΊ Ρ‡Π΅Ρ€Π΅Π· эту ΠΏΠΎΠ²Π΅Ρ€Ρ…Π½ΠΎΡΡ‚ΡŒ прСдставим Π² Π²ΠΈΠ΄Π΅ суммы ΠΏΠΎΡ‚ΠΎΠΊΠΎΠ² Ρ‡Π΅Ρ€Π΅Π· Π½ΠΈΠΆΠ½Π΅Π΅ основаниС Π€ΠΎ, Ρ‡Π΅Ρ€Π΅Π· Π²Π΅Ρ€Ρ…Π½Π΅Π΅ основаниС Π€1 ΠΈ Ρ‡Π΅Ρ€Π΅Π· Π±ΠΎΠΊΠΎΠ²ΡƒΡŽ ΠΏΠΎΠ²Π΅Ρ€Ρ…Π½ΠΎΡΡ‚ΡŒ

 Бумма этих ΠΏΠΎΡ‚ΠΎΠΊΠΎΠ² Ρ€Π°Π²Π½Π° Π½ΡƒΠ»ΡŽ

β€ƒΠ’Π΅ΠΏΠ΅Ρ€ΡŒ соотнСсСм эти повСрхности с рассматриваСмым ΠΊΠΎΠ½Ρ‚ΡƒΡ€ΠΎΠΌ.
 Боковая ΠΏΠΎΠ²Π΅Ρ€Ρ…Π½ΠΎΡΡ‚ΡŒ Ρ†ΠΈΠ»ΠΈΠ½Π΄Ρ€Π° Π΅ΡΡ‚ΡŒ ΠΏΠΎΠ²Π΅Ρ€Ρ…Π½ΠΎΡΡ‚ΡŒ, ΠΊΠΎΡ‚ΠΎΡ€ΡƒΡŽ Π·Π°ΠΌΠ΅Ρ‚Π°Π΅Ρ‚ рассматриваСмый ΠΊΠΎΠ½Ρ‚ΡƒΡ€, поэтому ΠΌΡ‹ связали высоту Ρ†ΠΈΠ»ΠΈΠ½Π΄Ρ€Π° со ΡΠΊΠΎΡ€ΠΎΡΡ‚ΡŒΡŽ двиТСния ΠΊΠΎΠ½Ρ‚ΡƒΡ€Π°. НиТнСС основаниС опираСтся Π½Π° ΠΏΠΎΠ»ΠΎΠΆΠ΅Π½ΠΈΠ΅ ΠΊΠΎΠ½Ρ‚ΡƒΡ€Π° Π² Π½Π΅ΠΊΠΎΡ‚ΠΎΡ€Ρ‹ΠΉ ΠΌΠΎΠΌΠ΅Π½Ρ‚ Π²Ρ€Π΅ΠΌΠ΅Π½ΠΈ t. По договорСнности ΠΏΠΎΠ»ΠΎΠΆΠΈΡ‚Π΅Π»ΡŒΠ½ΠΎΠΉ Π½ΠΎΡ€ΠΌΠ°Π»ΡŒΡŽ для Π·Π°ΠΌΠΊΠ½ΡƒΡ‚ΠΎΠΉ повСрхности считаСтся внСшняя Π½ΠΎΡ€ΠΌΠ°Π»ΡŒ (ΠΎΠ½ΠΈ ΠΈΠ·ΠΎΠ±Ρ€Π°ΠΆΠ΅Π½Π° Π½Π° рисункС ΠΊΠ°ΠΊ no ΠΈ n1). ΠŸΡ€ΠΈ описании ΠΌΠ°Π³Π½ΠΈΡ‚Π½ΠΎΠ³ΠΎ ΠΏΠΎΡ‚ΠΎΠΊΠ° Ρ‡Π΅Ρ€Π΅Π· ΠΊΠΎΠ½Ρ‚ΡƒΡ€3 ΠΌΡ‹ Π΄ΠΎΠ³ΠΎΠ²ΠΎΡ€ΠΈΠ»ΠΈΡΡŒ ΡΡ‡ΠΈΡ‚Π°Ρ‚ΡŒ ΠΏΠΎΠ»ΠΎΠΆΠΈΡ‚Π΅Π»ΡŒΠ½Ρ‹ΠΌ Π½Π°ΠΏΡ€Π°Π²Π»Π΅Π½ΠΈΠ΅ΠΌ Π½ΠΎΡ€ΠΌΠ°Π»ΠΈ, Π½Π°ΠΏΡ€Π°Π²Π»Π΅Π½ΠΈΠ΅ Β«ΠΏΠΎ полю». Π’ΠΎ Π΅ΡΡ‚ΡŒ ΠΏΠΎΡ‚ΠΎΠΊ Ρ‡Π΅Ρ€Π΅Π· ΠΊΠΎΠ½Ρ‚ΡƒΡ€ ΠΏΡ€ΠΎΡ‚ΠΈΠ²ΠΎΠΏΠΎΠ»ΠΎΠΆΠ΅Π½ ΠΏΠΎΡ‚ΠΎΠΊΡƒ Ρ‡Π΅Ρ€Π΅Π· Ρ‡Π°ΡΡ‚ΡŒ Π·Π°ΠΌΠΊΠ½ΡƒΡ‚ΠΎΠΉ повСрхности. ΠŸΠΎΡΡ‚ΠΎΠΌΡƒ Π² Π΄Π°Π½Π½ΠΎΠΌ случаС Π€ΠΎ = βˆ’Π€(t), Π³Π΄Π΅ Π€(t) βˆ’ ΠΏΠΎΡ‚ΠΎΠΊ Ρ‡Π΅Ρ€Π΅Π· ΠΊΠΎΠ½Ρ‚ΡƒΡ€, Π² ΠΌΠΎΠΌΠ΅Π½Ρ‚ Π²Ρ€Π΅ΠΌΠ΅Π½ΠΈ t. ΠŸΠΎΡ‚ΠΎΠΊ Ρ‡Π΅Ρ€Π΅Π· Π²Π΅Ρ€Ρ…Π½Π΅Π΅ основаниС Π΅ΡΡ‚ΡŒ ΠΏΠΎΡ‚ΠΎΠΊ Ρ‡Π΅Ρ€Π΅Π· ΠΊΠΎΠ½Ρ‚ΡƒΡ€ Π² ΠΌΠΎΠΌΠ΅Π½Ρ‚ Π²Ρ€Π΅ΠΌΠ΅Π½ΠΈ t + Ξ”t

 ЕщС ΠΎΠ΄ΠΈΠ½ Π°Ρ€Π³ΡƒΠΌΠ΅Π½Ρ‚ Π² ΠΏΠΎΠ»ΡŒΠ·Ρƒ измСнСния Π·Π½Π°ΠΊΠ° Π² ΠΏΠΎΡ‚ΠΎΠΊΠ΅ Ρ‡Π΅Ρ€Π΅Π· Π½ΠΈΠΆΠ½Π΅Π΅ основаниС βˆ’ Ссли ΠΌΡ‹ рассчитываСм ΠΈΠ·ΠΌΠ΅Π½Π΅Π½ΠΈΠ΅ ΠΏΠΎΡ‚ΠΎΠΊΠ°, Ρ‚ΠΎ Π΄ΠΎΠ»ΠΆΠ½Ρ‹ ΠΆΠ΅ ΠΌΡ‹ Π½Π°ΠΏΡ€Π°Π²Π»Π΅Π½ΠΈΠ΅ Π½ΠΎΡ€ΠΌΠ°Π»ΠΈ ΡΠΎΡ…Ρ€Π°Π½ΡΡ‚ΡŒ Π½Π΅ΠΈΠ·ΠΌΠ΅Π½Π½Ρ‹ΠΌ.
β€ƒΠ’Π΅ΠΏΠ΅Ρ€ΡŒ ΡΠΎΠΎΡ‚Π½ΠΎΡˆΠ΅Π½ΠΈΠ΅ (16) ΠΏΠ΅Ρ€Π΅ΠΏΠΈΡˆΠ΅ΠΌ Π² Π²ΠΈΠ΄Π΅

β€ƒΠ˜Π· ΠΊΠΎΡ‚ΠΎΡ€ΠΎΠ³ΠΎ Π²Ρ‹Ρ€Π°Π·ΠΈΠΌ Π­Π”Π‘ ΠΈΠ½Π΄ΡƒΠΊΡ†ΠΈΠΈ Π² ΠΊΠΎΠ½Ρ‚ΡƒΡ€Π΅ (опрСдСляСмой Ρ„ΠΎΡ€ΠΌΡƒΠ»ΠΎΠΉ (15))

β€ƒΠœΡ‹ ΠΏΠΎΠ»ΡƒΡ‡ΠΈΠ»ΠΈ Ρ‚Ρƒ ΠΆΠ΅ Ρ„ΠΎΡ€ΠΌΡƒΠ»Ρƒ для Π­Π”Π‘ ΠΈΠ½Π΄ΡƒΠΊΡ†ΠΈΠΈ Π² ΠΊΠΎΠ½Ρ‚ΡƒΡ€Π΅, Ρ‡Ρ‚ΠΎ ΠΈ Π² ΠΏΡ€Π΅Π΄Ρ‹Π΄ΡƒΡ‰Π΅ΠΌ ΠΏΡ€ΠΈΠΌΠ΅Ρ€Π΅.
Π’ рассмотрСнном ΠΏΡ€ΠΈΠΌΠ΅Ρ€Π΅ ΠΌΠ°Π³Π½ΠΈΡ‚Π½Ρ‹ΠΉ ΠΏΠΎΡ‚ΠΎΠΊ Ρ‡Π΅Ρ€Π΅Π· ΠΊΠΎΠ½Ρ‚ΡƒΡ€ ΡƒΠΌΠ΅Π½ΡŒΡˆΠ°Π΅Ρ‚ΡΡ, Ρ‚Π°ΠΊ ΠΊΠ°ΠΊ ΠΏΡ€ΠΈ ΡƒΠ²Π΅Π»ΠΈΡ‡Π΅Π½ΠΈΠΈ расстояния ΠΎΡ‚ ΠΌΠ°Π³Π½ΠΈΡ‚Π° индукция поля ΡƒΠΌΠ΅Π½ΡŒΡˆΠ°Π΅Ρ‚ΡΡ. ΠŸΠΎΡΡ‚ΠΎΠΌΡƒ Π² соотвСтствии с ΠΏΠΎΠ»ΡƒΡ‡Π΅Π½Π½ΠΎΠΉ Ρ„ΠΎΡ€ΠΌΡƒΠ»ΠΎΠΉ ΠΈ ΠΏΡ€Π°Π²ΠΈΠ»ΠΎΠΌ Π›Π΅Π½Ρ†Π° Π­Π”Π‘ ΠΈΠ½Π΄ΡƒΠΊΡ†ΠΈΠΈ Π² ΠΊΠΎΠ½Ρ‚ΡƒΡ€Π΅ ΠΏΠΎΠ»ΠΎΠΆΠΈΡ‚Π΅Π»ΡŒΠ½Π°, ΠΊΡ€ΠΎΠΌΠ΅ Ρ‚ΠΎΠ³ΠΎ, ΠΈΠ½Π΄ΡƒΠΊΡ†ΠΈΠΎΠ½Π½Ρ‹ΠΉ Ρ‚ΠΎΠΊ создаСт ΠΌΠ°Π³Π½ΠΈΡ‚Π½ΠΎΠ΅ ΠΏΠΎΠ»Π΅, Π½Π°ΠΏΡ€Π°Π²Π»Π΅Π½Π½ΠΎΠ΅ Ρ‚Π°ΠΊ ΠΆΠ΅, ΠΊΠ°ΠΊ ΠΈ ΠΏΠΎΠ»Π΅ постоянного ΠΌΠ°Π³Π½ΠΈΡ‚Π°.
β€ƒΠžΠ±Ρ€Π°Ρ‚ΠΈΡ‚Π΅ Π²Π½ΠΈΠΌΠ°Π½ΠΈΠ΅, Ρ‡Ρ‚ΠΎ Π² ΠΏΡ€ΠΈΠ²Π΅Π΄Π΅Π½Π½ΠΎΠΌ Π²Ρ‹Π²ΠΎΠ΄Π΅ ΠΌΡ‹ Π½Π΅ Π΄Π΅Π»Π°Π»ΠΈ Π½ΠΈΠΊΠ°ΠΊΠΈΡ… ΠΏΡ€Π΅Π΄ΠΏΠΎΠ»ΠΎΠΆΠ΅Π½ΠΈΠΉ ΠΎ зависимости Π²Π΅ΠΊΡ‚ΠΎΡ€Π° ΠΈΠ½Π΄ΡƒΠΊΡ†ΠΈΠΈ поля ΠΎΡ‚ ΠΊΠΎΠΎΡ€Π΄ΠΈΠ½Π°Ρ‚. ЕдинствСнноС ΠΏΡ€Π΅Π΄ΠΏΠΎΠ»ΠΎΠΆΠ΅Π½ΠΈΠ΅ Π·Π°ΠΊΠ»ΡŽΡ‡Π°Π»ΠΎΡΡŒ ΠΎΠ± осСвой симмСтрии поля. Однако ΠΈ Π΅Π³ΠΎ ΠΌΠΎΠΆΠ½ΠΎ ΡΠ½ΡΡ‚ΡŒ, для этого ΠΏΡ€ΠΈ вычислСнии Π­Π”Π‘ ΠΏΠΎ ΠΊΠΎΠ½Ρ‚ΡƒΡ€Ρƒ просто Π½Π΅ΠΎΠ±Ρ…ΠΎΠ΄ΠΈΠΌΠΎ Ρ€Π°Π·Π±ΠΈΡ‚ΡŒ послСдний Π½Π° ΠΌΠ°Π»Ρ‹Π΅ участки, Π° Π·Π°Ρ‚Π΅ΠΌ ΠΏΡ€ΠΎΡΡƒΠΌΠΌΠΈΡ€ΠΎΠ²Π°Ρ‚ΡŒ Ρ€Π°Π±ΠΎΡ‚Ρƒ силы Π›ΠΎΡ€Π΅Π½Ρ†Π° ΠΏΠΎ всСм участкам.
 Задания для ΡΠ°ΠΌΠΎΡΡ‚ΠΎΡΡ‚Π΅Π»ΡŒΠ½ΠΎΠΉ Ρ€Π°Π±ΠΎΡ‚Ρ‹.
 1. РассмотритС Π½Π°ΠΏΡ€Π°Π²Π»Π΅Π½ΠΈΠ΅ поля, созданного ΠΈΠ½Π΄ΡƒΡ†ΠΈΡ€ΠΎΠ²Π°Π½Π½Ρ‹ΠΌ Ρ‚ΠΎΠΊΠΎΠΌ Π² схСмС Π½Π° рис. 515, ΠΏΠΎΠΊΠ°ΠΆΠΈΡ‚Π΅, Ρ‡Ρ‚ΠΎ ΠΏΡ€Π°Π²ΠΈΠ»ΠΎ Π›Π΅Π½Ρ†Π° выполняСтся.
 2. ΠŸΠΎΠΊΠ°ΠΆΠΈΡ‚Π΅, Ρ‡Ρ‚ΠΎ Π² схСмС, ΠΏΠΎΠΊΠ°Π·Π°Π½Π½ΠΎΠΉ Π½Π° рис. 515, сила АмпСра, Π΄Π΅ΠΉΡΡ‚Π²ΡƒΡŽΡ‰Π°Ρ Π½Π° ΠΊΠΎΠ½Ρ‚ΡƒΡ€ с ΠΈΠ½Π΄ΡƒΡ†ΠΈΡ€ΠΎΠ²Π°Π½Π½Ρ‹ΠΌ Ρ‚ΠΎΠΊΠΎΠΌ, Π½Π°ΠΏΡ€Π°Π²Π»Π΅Π½Π° Π² сторону ΠΏΡ€ΠΎΡ‚ΠΈΠ²ΠΎΠΏΠΎΠ»ΠΎΠΆΠ½ΡƒΡŽ Π΅Π³ΠΎ скорости.
 3. ΠŸΡƒΡΡ‚ΡŒ ΠΏΡ€ΠΎΠΈΠ·Π²ΠΎΠ»ΡŒΠ½Ρ‹ΠΉ ΠΊΠΎΠ½Ρ‚ΡƒΡ€ Π·Π° ΠΌΠ°Π»Ρ‹ΠΉ ΠΏΡ€ΠΎΠΌΠ΅ΠΆΡƒΡ‚ΠΎΠΊ Π²Ρ€Π΅ΠΌΠ΅Π½ΠΈ смСстился ΠΈΠ· полоТСния 1 Π² ΠΏΠΎΠ»ΠΎΠΆΠ΅Π½ΠΈΠ΅ 2 Π² ΠΏΡ€ΠΎΠΈΠ·Π²ΠΎΠ»ΡŒΠ½ΠΎΠΌ постоянном ΠΌΠ°Π³Π½ΠΈΡ‚Π½ΠΎΠΌ ΠΏΠΎΠ»Π΅. Π˜ΡΠΏΠΎΠ»ΡŒΠ·ΡƒΡ Π²Ρ‹Ρ€Π°ΠΆΠ΅Π½ΠΈΠ΅ для силы Π›ΠΎΡ€Π΅Π½Ρ†Π° ΠΈ Ρ‚Π΅ΠΎΡ€Π΅ΠΌΡƒ ΠΎ ΠΌΠ°Π³Π½ΠΈΡ‚Π½ΠΎΠΌ ΠΏΠΎΡ‚ΠΎΠΊΠ΅, Π΄ΠΎΠΊΠ°ΠΆΠΈΡ‚Π΅ Π² ΠΎΠ±Ρ‰Π΅ΠΌ случаС Ρ„ΠΎΡ€ΠΌΡƒΠ»Ρƒ (18) для Π­Π”Π‘ ΠΈΠ½Π΄ΡƒΠΊΡ†ΠΈΠΈ Π² ΠΊΠΎΠ½Ρ‚ΡƒΡ€Π΅.

рис. 517
1Π’ΠΎΠΊ, Π²ΠΎΠ·Π½ΠΈΠΊΠ°ΡŽΡ‰ΠΈΠΉ Π² ΠΊΠΎΠ½Ρ‚ΡƒΡ€Π΅, называСтся ΠΈΠ½Π΄ΡƒΡ†ΠΈΡ€ΠΎΠ²Π°Π½Π½Ρ‹ΠΌ (Π½Π°Π²Π΅Π΄Π΅Π½Π½Ρ‹ΠΌ), поэтому ΠΈ создаваСмоС ΠΈΠΌ ΠΌΠ°Π³Π½ΠΈΡ‚Π½ΠΎΠ΅ ΠΏΠΎΠ»Π΅ ΠΌΠΎΠΆΠ½ΠΎ Π½Π°Π·Π²Π°Ρ‚ΡŒ ΠΈΠ½Π΄ΡƒΡ†ΠΈΡ€ΠΎΠ²Π°Π½Π½Ρ‹ΠΌ. Но ΠΊΠ°ΠΊ Ρ‚ΠΎΠ³Π΄Π° Π½Π°Π·Π²Π°Ρ‚ΡŒ Π΅Π³ΠΎ характСристику βˆ’ «индукция поля, ΠΈΠ½Π΄ΡƒΡ†ΠΈΡ€ΠΎΠ²Π°Π½Π½ΠΎΠ³ΠΎ благодаря явлСнию элСктромагнитной ΠΈΠ½Π΄ΡƒΠΊΡ†ΠΈΠΈΒ»?
2Названо Π² Ρ‡Π΅ΡΡ‚ΡŒ русского ΡƒΡ‡Π΅Π½ΠΎΠ³ΠΎ Эмиля Π›Π΅Π½Ρ†Π°, ΡƒΡΡ‚Π°Π½ΠΎΠ²ΠΈΠ²ΡˆΠ΅Π³ΠΎ ΠΈ ΠΎΠ±ΠΎΠ±Ρ‰ΠΈΠ²ΡˆΠ΅Π³ΠΎ Π΅Π³ΠΎ.
3Напомним βˆ’ Ρ‚ΠΎΡ‡Π½Π΅Π΅ Π½Π°Π΄ΠΎ Π³ΠΎΠ²ΠΎΡ€ΠΈΡ‚ΡŒ ΠΎ ΠΏΠΎΡ‚ΠΎΠΊΠ΅ Ρ‡Π΅Ρ€Π΅Π· Π»ΡŽΠ±ΡƒΡŽ ΠΏΠΎΠ²Π΅Ρ€Ρ…Π½ΠΎΡΡ‚ΡŒ Π½Π°Ρ‚ΡΠ½ΡƒΡ‚ΡƒΡŽ (ΠΎΠΏΠΈΡ€Π°ΡŽΡ‰ΡƒΡŽΡΡ) Π½Π° ΠΊΠΎΠ½Ρ‚ΡƒΡ€.

fizportal.ru

КСйсЀ1-2013 / Π­ΠΊΠ·Π°ΠΌΠ΅Π½ Ρ„ΠΈΠ·ΠΈΠΊΠ°1 / 3

3.5. Π―Π²Π»Π΅Π½ΠΈΠ΅ элСктромагнитной ΠΈΠ½Π΄ΡƒΠΊΡ†ΠΈΠΈ

  • Π’ Ρ‡Π΅ΠΌ Π·Π°ΠΊΠ»ΡŽΡ‡Π°Π΅Ρ‚ΡΡ явлСниС элСктромагнитной ΠΈΠ½Π΄ΡƒΠΊΡ†ΠΈΠΈ?

  • Π—Π°ΠΊΠΎΠ½ ЀарадСя для элСктромагнитной ΠΈΠ½Π΄ΡƒΠΊΡ†ΠΈΠΈ.

  • ΠŸΡ€Π°Π²ΠΈΠ»ΠΎ Π›Π΅Π½Ρ†Π°.

  • Π§Ρ‚ΠΎ Ρ‚Π°ΠΊΠΎΠ΅ ΠΈΠ½Π΄ΡƒΠΊΡ‚ΠΈΠ²Π½ΠΎΡΡ‚ΡŒ?

  • Π—Π°ΠΏΠΈΡΠ°Ρ‚ΡŒ Π²Ρ‹Ρ€Π°ΠΆΠ΅Π½ΠΈΠ΅ для Π­Π”Π‘. самоиндукции.

  • Π§Ρ‚ΠΎ Ρ‚Π°ΠΊΠΎΠ΅ явлСниС самоиндукции?

  • Π§Ρ‚ΠΎ Ρ‚Π°ΠΊΠΎΠ΅ явлСниС Π²Π·Π°ΠΈΠΌΠ½ΠΎΠΉ ΠΈΠ½Π΄ΡƒΠΊΡ†ΠΈΠΈ?

  • ΠžΡ‚ Ρ‡Π΅Π³ΠΎ зависит ΠΈΠ½Π΄ΡƒΠΊΡ‚ΠΈΠ²Π½ΠΎΡΡ‚ΡŒ ΠΏΡ€ΠΎΠ²ΠΎΠ΄Π½ΠΈΠΊΠ° Π² Π²Π°ΠΊΡƒΡƒΠΌΠ΅?

  • Π—Π°ΠΏΠΈΡΠ°Ρ‚ΡŒ Π²Ρ‹Ρ€Π°ΠΆΠ΅Π½ΠΈΠ΅ для энСргии ΠΌΠ°Π³Π½ΠΈΡ‚Π½ΠΎΠ³ΠΎ поля ΠΊΠ°Ρ‚ΡƒΡˆΠΊΠΈ с ΠΈΠ½Π΄ΡƒΠΊΡ‚ΠΈΠ²Π½ΠΎΡΡ‚ΡŒΡŽ L.

  • На Ρ‡Π΅ΠΌ основан ΠΏΡ€ΠΈΠ½Ρ†ΠΈΠΏ Ρ€Π°Π±ΠΎΡ‚Ρ‹ трансформаторов?

  • Π—Π°ΠΏΠΈΡΠ°Ρ‚ΡŒ Π²Ρ‹Ρ€Π°ΠΆΠ΅Π½ΠΈΠ΅ для объСмной плотности энСргии ΠΌΠ°Π³Π½ΠΈΡ‚Π½ΠΎΠ³ΠΎ поля.

  • Π—Π°ΠΏΠΈΡΠ°Ρ‚ΡŒ Π²Ρ‹Ρ€Π°ΠΆΠ΅Π½ΠΈΠ΅ для индуктивности солСноида.

  • Π§Π΅Ρ€Π΅Π· ΠΊΠ°Ρ‚ΡƒΡˆΠΊΡƒ, ΠΈΠ½Π΄ΡƒΠΊΡ‚ΠΈΠ²Π½ΠΎΡΡ‚ΡŒ ΠΊΠΎΡ‚ΠΎΡ€ΠΎΠΉ L=10-3 Π“Π½, ΠΏΡ€ΠΎΡ…ΠΎΠ΄ΠΈΡ‚ Ρ‚ΠΎΠΊ I=1 А. Π§Π΅ΠΌΡƒ Ρ€Π°Π²Π΅Π½ ΠΌΠ°Π³Π½ΠΈΡ‚Π½Ρ‹ΠΉ ΠΏΠΎΡ‚ΠΎΠΊ сквозь ΠΊΠ°Ρ‚ΡƒΡˆΠΊΡƒ?

  • Π’ ΠΊΠ°Ρ‚ΡƒΡˆΠΊΠ΅ ΠΈΠ½Π΄ΡƒΠΊΡ‚ΠΈΠ²Π½ΠΎΡΡ‚ΡŒΡŽ 0,7 Π“Π½ сила Ρ‚ΠΎΠΊΠ° Ρ€Π°Π²Π½Π° 10 А. Π§Π΅ΠΌΡƒ Ρ€Π°Π²Π½Π° энСргия ΠΌΠ°Π³Π½ΠΈΡ‚Π½ΠΎΠ³ΠΎ поля ΠΊΠ°Ρ‚ΡƒΡˆΠΊΠΈ?

  • Π§Π΅ΠΌΡƒ Ρ€Π°Π²Π½Π° Π­Π”Π‘ самоиндукции Π² ΠΊΠ°Ρ‚ΡƒΡˆΠΊΠ΅ с ΠΈΠ½Π΄ΡƒΠΊΡ‚ΠΈΠ²Π½ΠΎΡΡ‚ΡŒΡŽ 2 Π“Π½, Ссли сила Ρ‚ΠΎΠΊΠ° Π² Π½Π΅ΠΉ Π·Π° 0,1 с Ρ€Π°Π²Π½ΠΎΠΌΠ΅Ρ€Π½ΠΎ ΡƒΠΌΠ΅Π½ΡŒΡˆΠΈΠ»Π°ΡΡŒ ΠΎΡ‚ 5 А Π΄ΠΎ 3 А?

  • Π’ ΠΎΠ΄Π½ΠΎΡ€ΠΎΠ΄Π½ΠΎΠΌ ΠΌΠ°Π³Π½ΠΈΡ‚Π½ΠΎΠΌ ΠΏΠΎΠ»Π΅ с ΠΈΠ½Π΄ΡƒΠΊΡ†ΠΈΠ΅ΠΉ 0,2 Π’Π» двиТСтся прямолинСйно ΠΏΡ€ΠΎΠ²ΠΎΠ΄Π½ΠΈΠΊ Π΄Π»ΠΈΠ½ΠΎΠΉ 0.5ΠΌ. ΠžΠΏΡ€Π΅Π΄Π΅Π»ΠΈΡ‚ΡŒ Π­Π”Π‘ ΠΈΠ½Π΄ΡƒΠΊΡ†ΠΈΠΈ Π½Π° ΠΊΠΎΠ½Ρ†Π°Ρ… ΠΏΡ€ΠΎΠ²ΠΎΠ΄Π½ΠΈΠΊΠ°, Ссли Π΅Π³ΠΎ ΡΠΊΠΎΡ€ΠΎΡΡ‚ΡŒ Ρ€Π°Π²Π½Π° 10 ΠΌ/с.

  • Бколько Π²ΠΈΡ‚ΠΊΠΎΠ² Π΄ΠΎΠ»ΠΆΠ½Π° ΡΠΎΠ΄Π΅Ρ€ΠΆΠ°Ρ‚ΡŒ ΠΊΠ°Ρ‚ΡƒΡˆΠΊΠ° с ΠΏΠ»ΠΎΡ‰Π°Π΄ΡŒΡŽ ΠΏΠΎΠΏΠ΅Ρ€Π΅Ρ‡Π½ΠΎΠ³ΠΎ сСчСния 50 см2, Ρ‡Ρ‚ΠΎΠ±Ρ‹ ΠΏΡ€ΠΈ ΠΈΠ·ΠΌΠ΅Π½Π΅Π½ΠΈΠΈ ΠΌΠ°Π³Π½ΠΈΡ‚Π½ΠΎΠΉ ΠΈΠ½Π΄ΡƒΠΊΡ†ΠΈΠΈ ΠΎΡ‚ 0,2 Π’Π» Π΄ΠΎ 0,3 Π’Π» Π² Ρ‚Π΅Ρ‡Π΅Π½ΠΈΠΈ 4 мс, Π² Π½Π΅ΠΉ Π²ΠΎΠ·Π±ΡƒΠΆΠ΄Π°Π»Π°ΡΡŒ Π­Π”Π‘ 10 Π’?

  • Найти ΡΠΊΠΎΡ€ΠΎΡΡ‚ΡŒ измСнСния ΠΌΠ°Π³Π½ΠΈΡ‚Π½ΠΎΠ³ΠΎ ΠΏΠΎΡ‚ΠΎΠΊΠ° Π² солСноидС ΠΈΠ· 2000 Π²ΠΈΡ‚ΠΊΠΎΠ² ΠΏΡ€ΠΈ Π²ΠΎΠ·Π±ΡƒΠΆΠ΄Π΅Π½ΠΈΠΈ Π² Π½Π΅ΠΌ эдс ΠΈΠ½Π΄ΡƒΠΊΡ†ΠΈΠΈ 120 Π’.

  • Π’ Π²ΠΈΡ‚ΠΊΠ΅ сопротивлСниСм 2 Ом ΡΠΊΠΎΡ€ΠΎΡΡ‚ΡŒ измСнСния ΠΌΠ°Π³Π½ΠΈΡ‚Π½ΠΎΠ³ΠΎ ΠΏΠΎΡ‚ΠΎΠΊΠ° 10 ΠΌΠ’Π±/с. Найти силу ΠΈΠ½Π΄ΡƒΠΊΡ†ΠΈΠΎΠ½Π½ΠΎΠ³ΠΎ Ρ‚ΠΎΠΊΠ°.

  • ΠšΠ°Ρ‚ΡƒΡˆΠΊΠ° Π΄ΠΈΠ°ΠΌΠ΅Ρ‚Ρ€ΠΎΠΌ 10 см, ΠΈΠΌΠ΅ΡŽΡ‰Π°Ρ 500 Π²ΠΈΡ‚ΠΊΠΎΠ², находится Π² ΠΌΠ°Π³Π½ΠΈΡ‚Π½ΠΎΠΌ ΠΏΠΎΠ»Π΅. Π§Π΅ΠΌΡƒ Π±ΡƒΠ΄Π΅Ρ‚ Ρ€Π°Π²Π½ΠΎ срСднСС Π·Π½Π°Ρ‡Π΅Π½ΠΈΠ΅ эдс ΠΈΠ½Π΄ΡƒΠΊΡ†ΠΈΠΈ Π² этой ΠΊΠ°Ρ‚ΡƒΡˆΠΊΠ΅, Ссли индукция ΠΌΠ°Π³Π½ΠΈΡ‚Π½ΠΎΠ³ΠΎ поля увСличиваСтся Π² Ρ‚Π΅Ρ‡Π΅Π½ΠΈΠ΅ 0,1с с 0 Π΄ΠΎ 2 Π’Π»?

  • ΠšΡ€ΡƒΠ³ΠΎΠ²ΠΎΠΉ ΠΏΡ€ΠΎΠ²ΠΎΠ»ΠΎΡ‡Π½Ρ‹ΠΉ ΠΊΠΎΠ½Ρ‚ΡƒΡ€ ΠΏΠ»ΠΎΡ‰Π°Π΄ΡŒΡŽ 100 см2 находится Π² ΠΎΠ΄Π½ΠΎΡ€ΠΎΠ΄Π½ΠΎΠΌ ΠΌΠ°Π³Π½ΠΈΡ‚Π½ΠΎΠΌ ΠΏΠΎΠ»Π΅, индукция ΠΊΠΎΡ‚ΠΎΡ€ΠΎΠ³ΠΎ 2 Π’Π». ΠŸΠ»ΠΎΡΠΊΠΎΡΡ‚ΡŒ Π²ΠΈΡ‚ΠΊΠ° пСрпСндикулярна Π½Π°ΠΏΡ€Π°Π²Π»Π΅Π½ΠΈΡŽ ΠΌΠ°Π³Π½ΠΈΡ‚Π½ΠΎΠ³ΠΎ поля. Π§Π΅ΠΌΡƒ Π±ΡƒΠ΄Π΅Ρ‚ Ρ€Π°Π²Π½ΠΎ срСднСС Π·Π½Π°Ρ‡Π΅Π½ΠΈΠ΅ эдс ΠΈΠ½Π΄ΡƒΠΊΡ†ΠΈΠΈ, Π²ΠΎΠ·Π½ΠΈΠΊΠ°ΡŽΡ‰Π΅ΠΉ Π² Π²ΠΈΡ‚ΠΊΠ΅ ΠΏΡ€ΠΈ Π²Ρ‹ΠΊΠ»ΡŽΡ‡Π΅Π½ΠΈΠΈ поля Π² Ρ‚Π΅Ρ‡Π΅Π½ΠΈΠ΅ 0,01с?

  • Π’ ΠΎΠ΄Π½ΠΎΡ€ΠΎΠ΄Π½ΠΎΠΌ ΠΌΠ°Π³Π½ΠΈΡ‚Π½ΠΎΠΌ ΠΏΠΎΠ»Π΅, индукция ΠΊΠΎΡ‚ΠΎΡ€ΠΎΠ³ΠΎ 0,1 Π’Π» двиТСтся ΠΏΡ€ΠΎΠ²ΠΎΠ΄Π½ΠΈΠΊ Π΄Π»ΠΈΠ½ΠΎΠΉ 10 см. Π‘ΠΊΠΎΡ€ΠΎΡΡ‚ΡŒ двиТСния ΠΏΡ€ΠΎΠ²ΠΎΠ΄Π½ΠΈΠΊΠ° 5 ΠΌ/с ΠΈ Π½Π°ΠΏΡ€Π°Π²Π»Π΅Π½Π° ΠΎΠ½Π° пСрпСндикулярно ΠΌΠ°Π³Π½ΠΈΡ‚Π½ΠΎΠΌΡƒ полю. Π§Π΅ΠΌΡƒ Ρ€Π°Π²Π½Π° индуцированная Π² ΠΏΡ€ΠΎΠ²ΠΎΠ΄Π½ΠΈΠΊΠ΅ эдс?

  • ΠšΠ²Π°Π΄Ρ€Π°Ρ‚Π½Ρ‹ΠΉ Π²ΠΈΡ‚ΠΎΠΊ ΠΏΠ»ΠΎΡ‰Π°Π΄ΡŒΡŽ 100 см2 ΠΏΠ΅Ρ€Π²ΠΎΠ½Π°Ρ‡Π°Π»ΡŒΠ½ΠΎ располагался ΠΏΠ°Ρ€Π°Π»Π»Π΅Π»ΡŒΠ½ΠΎ линиям ΠΌΠ°Π³Π½ΠΈΡ‚Π½ΠΎΠΉ ΠΈΠ½Π΄ΡƒΠΊΡ†ΠΈΠΈ Π²Π΅Π»ΠΈΡ‡ΠΈΠ½ΠΎΠΉ 0,1 Π’Π», Π° Π·Π°Ρ‚Π΅ΠΌ Π·Π° 0,1 с повСрнулся Π½Π° 90ΒΊ. ΠžΠΏΡ€Π΅Π΄Π΅Π»ΠΈΡ‚ΡŒ Π­Π”Π‘ ΠΈΠ½Π΄ΡƒΠΊΡ†ΠΈΠΈ Π² этом Π²ΠΈΡ‚ΠΊΠ΅.

  • Π§Π΅Ρ€Π΅Π· ΠΊΠ°Ρ‚ΡƒΡˆΠΊΡƒ, ΠΈΠ½Π΄ΡƒΠΊΡ‚ΠΈΠ²Π½ΠΎΡΡ‚ΡŒ ΠΊΠΎΡ‚ΠΎΡ€ΠΎΠΉ L = 0,021 Π“Π½ Ρ‚Π΅Ρ‡Ρ‘Ρ‚ Ρ‚ΠΎΠΊ, ΠΈΠ·ΠΌΠ΅Π½ΡΡŽΡ‰ΠΈΠΉΡΡ со Π²Ρ€Π΅ΠΌΠ΅Π½Π΅ΠΌ ΠΏΠΎ Π·Π°ΠΊΠΎΠ½Ρƒ I = 58Β·sin(314Β·t). Найти ΠΌΠΎΠ΄ΡƒΠ»ΡŒ Π­Π”Π‘ самоиндукции.

  • Врансформатор с ΠšΠŸΠ” 80% Π΄Π°Π΅Ρ‚ Π½Π° Π²Ρ‹Ρ…ΠΎΠ΄Π΅ 10 Π’ ΠΈ 4 А. Какова Π΅Π³ΠΎ потрСбляСмая ΠΌΠΎΡ‰Π½ΠΎΡΡ‚ΡŒ?

  • КакоС количСство Π²ΠΈΡ‚ΠΊΠΎΠ² Π΄ΠΎΠ»ΠΆΠ½Π° ΡΠΎΠ΄Π΅Ρ€ΠΆΠ°Ρ‚ΡŒ ΠΊΠ°Ρ‚ΡƒΡˆΠΊΠ°, Ρ‡Ρ‚ΠΎΠ±Ρ‹ ΠΏΡ€ΠΈ ΠΈΠ·ΠΌΠ΅Π½Π΅Π½ΠΈΠΈ ΠΌΠ°Π³Π½ΠΈΡ‚Π½ΠΎΠΉ ΠΈΠ½Π΄ΡƒΠΊΡ†ΠΈΠΈ ΠΎΡ‚ 0,2 Π΄ΠΎ 0,6 Π’Π» Π² Ρ‚Π΅Ρ‡Π΅Π½ΠΈΠ΅ 4 мс Π² ΠΊΠ°Ρ‚ΡƒΡˆΠΊΠ΅ с ΠΏΠ»ΠΎΡ‰Π°Π΄ΡŒΡŽ ΠΏΠΎΠΏΠ΅Ρ€Π΅Ρ‡Π½ΠΎΠ³ΠΎ сСчСния 50 см2 Π²ΠΎΠ·Π±ΡƒΠΆΠ΄Π°Π»ΠΎΡΡŒ Π­Π”Π‘ ΠΈΠ½Π΄ΡƒΠΊΡ†ΠΈΠΈ 5Π’?

  • Из ΠΏΡ€ΠΎΠ²ΠΎΠ΄Π° ΠΈΠ·Π³ΠΎΡ‚ΠΎΠ²Π»Π΅Π½Π° ΠΊΠ°Ρ‚ΡƒΡˆΠΊΠ° Π΄Π»ΠΈΠ½ΠΎΠΉ 6,28 см радиусом 1 см. Она содСрТит 200 Π²ΠΈΡ‚ΠΊΠΎΠ² ΠΈ ΠΏΠΎ Π½Π΅ΠΉ ΠΏΡ€ΠΎΡ…ΠΎΠ΄ΠΈΡ‚ Ρ‚ΠΎΠΊ 1 А. Π§Π΅ΠΌΡƒ Ρ€Π°Π²Π΅Π½ ΠΌΠ°Π³Π½ΠΈΡ‚Π½Ρ‹ΠΉ ΠΏΠΎΡ‚ΠΎΠΊ Π²Π½ΡƒΡ‚Ρ€ΠΈ ΠΊΠ°Ρ‚ΡƒΡˆΠΊΠΈ?

  • ΠžΠΏΡ€Π΅Π΄Π΅Π»ΠΈΡ‚ΡŒ ΠΈΠ½Π΄ΡƒΠΊΡ‚ΠΈΠ²Π½ΠΎΡΡ‚ΡŒ солСноида Π΄Π»ΠΈΠ½ΠΎΠΉ 20 см, ΠΈΠΌΠ΅ΡŽΡ‰Π΅Π³ΠΎ 500 Π²ΠΈΡ‚ΠΊΠΎΠ² ΠΈ ΠΏΠ»ΠΎΡ‰Π°Π΄ΡŒ ΠΏΠΎΠΏΠ΅Ρ€Π΅Ρ‡Π½ΠΎΠ³ΠΎ сСчСния 10 см2. ΠœΠ°Π³Π½ΠΈΡ‚Π½Π°Ρ ΠΏΡ€ΠΎΠ½ΠΈΡ†Π°Π΅ΠΌΠΎΡΡ‚ΡŒ Ρ€Π°Π²Π½Π° 1, Π° m0=4pΒ·10–7 Π“Π½/ΠΌ.

  • ΠžΠΏΡ€Π΅Π΄Π΅Π»ΠΈΡ‚ΡŒ Π°Π±ΡΠΎΠ»ΡŽΡ‚Π½ΠΎΠ΅ Π·Π½Π°Ρ‡Π΅Π½ΠΈΠ΅ скорости измСнСния Ρ‚ΠΎΠΊΠ° Π² ΠΊΠ°Ρ‚ΡƒΡˆΠΊΠ΅ ΠΈΠ½Π΄ΡƒΠΊΡ‚ΠΈΠ²Π½ΠΎΡΡ‚ΡŒΡŽ 0,1 Π“Π½, Ссли Π² Π½Π΅ΠΉ Π²ΠΎΠ·Π½ΠΈΠΊΠ»Π° Π­Π”Π‘ самоиндукция 80 Π’

  • Π—Π° врСмя 10 с ΠΏΠ»ΠΎΡ‰Π°Π΄ΡŒ ΠΊΠΎΠ½Ρ‚ΡƒΡ€Π°, находящСгося Π² ΠΎΠ΄Π½ΠΎΡ€ΠΎΠ΄Π½ΠΎΠΌ ΠΌΠ°Π³Π½ΠΈΡ‚Π½ΠΎΠΌ ΠΏΠΎΠ»Π΅ с ΠΈΠ½Π΄ΡƒΠΊΡ†ΠΈΠ΅ΠΉ 0,01 Π’Π», Ρ€Π°Π²Π½ΠΎΠΌΠ΅Ρ€Π½ΠΎ ΡƒΠΌΠ΅Π½ΡŒΡˆΠΈΠ»Π°ΡΡŒ Π½Π° 10 см2. ΠšΠΎΠ½Ρ‚ΡƒΡ€ располоТСн пСрпСндикулярно Π²Π΅ΠΊΡ‚ΠΎΡ€Ρƒ B. ΠžΠΏΡ€Π΅Π΄Π΅Π»ΠΈΡ‚ΡŒ Π­Π”Π‘ ΠΈΠ½Π΄ΡƒΠΊΡ†ΠΈΠΈ.

  • Из ΠΏΡ€ΠΎΠ²ΠΎΠ΄Π° Π΄Π»ΠΈΠ½ΠΎΠΉ L=2 ΠΌ сдСлан ΠΊΠ²Π°Π΄Ρ€Π°Ρ‚, ΠΊΠΎΡ‚ΠΎΡ€Ρ‹ΠΉ располоТСн Π³ΠΎΡ€ΠΈΠ·ΠΎΠ½Ρ‚Π°Π»ΡŒΠ½ΠΎ. Π˜Π½Π΄ΡƒΠΊΡ†ΠΈΡ ΠΌΠ°Π³Π½ΠΈΡ‚Π½ΠΎΠ³ΠΎ поля пСрпСндикулярна ΠΊΠΎΠ½Ρ‚ΡƒΡ€Ρƒ ΠΈ Ρ€Π°Π²Π½Π° Π’= 5Β· 10-2 Π’Π». Какой заряд ΠΏΡ€ΠΎΠΉΠ΄Π΅Ρ‚ ΠΏΠΎ ΠΏΡ€ΠΎΠ²ΠΎΠ΄Ρƒ, Ссли Π΅Π³ΠΎ ΠΏΠΎΡ‚ΡΠ½ΡƒΡ‚ΡŒ Π·Π° Π΄Π²Π΅ диагонально ΠΏΡ€ΠΎΡ‚ΠΈΠ²ΠΎΠΏΠΎΠ»ΠΎΠΆΠ½Ρ‹Π΅ Π²Π΅Ρ€ΡˆΠΈΠ½Ρ‹ Ρ‚Π°ΠΊ, Ρ‡Ρ‚ΠΎΠ±Ρ‹ ΠΎΠ½ слоТился? Π‘ΠΎΠΏΡ€ΠΎΡ‚ΠΈΠ²Π»Π΅Π½ΠΈΠ΅ ΠΏΡ€ΠΎΠ²ΠΎΠ΄Π° R=0,1Ом.

  • ΠšΠΎΠ½Ρ‚ΡƒΡ€ ΠΏΠ»ΠΎΡ‰Π°Π΄ΡŒΡŽ S ΠΏΠΎΠΌΠ΅Ρ‰Π΅Π½ Π² ΠΎΠ΄Π½ΠΎΡ€ΠΎΠ΄Π½ΠΎΠ΅ ΠΌΠ°Π³Π½ΠΈΡ‚Π½ΠΎΠ΅ ΠΏΠΎΠ»Π΅ ΠΈΠ½Π΄ΡƒΠΊΡ†ΠΈΠ΅ΠΉ Π’. ΠŸΠ»ΠΎΡΠΊΠΎΡΡ‚ΡŒ ΠΊΠΎΠ½Ρ‚ΡƒΡ€Π° пСрпСндикулярна Π½Π°ΠΏΡ€Π°Π²Π»Π΅Π½ΠΈΡŽ ΠΌΠ°Π³Π½ΠΈΡ‚Π½ΠΎΠ³ΠΎ поля, сопротивлСниС ΠΊΠΎΠ½Ρ‚ΡƒΡ€Π° 1 Ом. Какой заряд ΠΏΡ€ΠΎΡ‚Π΅Ρ‡Π΅Ρ‚ ΠΏΠΎ ΠΊΠΎΠ½Ρ‚ΡƒΡ€Ρƒ ΠΏΡ€ΠΈ ΠΏΠΎΠ²ΠΎΡ€ΠΎΡ‚Π΅ Π΅Π³ΠΎ Π½Π° 90ΒΊ?

  • Π“ΠΎΡ€ΠΈΠ·ΠΎΠ½Ρ‚Π°Π»ΡŒΠ½Ρ‹ΠΉ мСталличСский ΡΡ‚Π΅Ρ€ΠΆΠ΅Π½ΡŒ Π΄Π»ΠΈΠ½ΠΎΠΉ 50 см вращаСтся Π²ΠΎΠΊΡ€ΡƒΠ³ Π²Π΅Ρ€Ρ‚ΠΈΠΊΠ°Π»ΡŒΠ½ΠΎΠΉ оси, проходящСй Ρ‡Π΅Ρ€Π΅Π· ΠΎΠ΄ΠΈΠ½ ΠΈΠ· Π΅Π³ΠΎ ΠΊΠΎΠ½Ρ†ΠΎΠ² с частотой 2 ΠΎΠ±/с. Π˜Π½Π΄ΡƒΠΊΡ†ΠΈΡ ΠΌΠ°Π³Π½ΠΈΡ‚Π½ΠΎΠ³ΠΎ поля 5Β·10-2 Π’Π». Π‘ΠΈΠ»ΠΎΠ²Ρ‹Π΅ Π»ΠΈΠ½ΠΈΠΈ ΠΌΠ°Π³Π½ΠΈΡ‚Π½ΠΎΠ³ΠΎ поля ΠΏΠ°Ρ€Π°Π»Π»Π΅Π»ΡŒΠ½Ρ‹ оси вращСния. Найти Ρ€Π°Π·Π½ΠΎΡΡ‚ΡŒ ΠΏΠΎΡ‚Π΅Π½Ρ†ΠΈΠ°Π»ΠΎΠ² ΠΌΠ΅ΠΆΠ΄Ρƒ ΠΊΠΎΠ½Ρ†Π°ΠΌΠΈ стСрТня.

  • Π’ ΠΎΠ΄Π½ΠΎΡ€ΠΎΠ΄Π½ΠΎΠΌ ΠΌΠ°Π³Π½ΠΈΡ‚Π½ΠΎΠΌ ΠΏΠΎΠ»Π΅ с ΠΈΠ½Π΄ΡƒΠΊΡ†ΠΈΠ΅ΠΉ Π’ Ρ€Π°Π²Π½ΠΎΠΌΠ΅Ρ€Π½ΠΎ с ΡƒΠ³Π»ΠΎΠ²ΠΎΠΉ ΡΠΊΠΎΡ€ΠΎΡΡ‚ΡŒΡŽ w вращаСтся Ρ€Π°ΠΌΠΊΠ°, содСрТащая N Π²ΠΈΡ‚ΠΊΠΎΠ² ΠΏΠ»ΠΎΡ‰Π°Π΄ΡŒΡŽ S. Ось вращСния Π»Π΅ΠΆΠΈΡ‚ Π² плоскости Ρ€Π°ΠΌΠΊΠΈ ΠΈ пСрпСндикулярна линиям ΠΈΠ½Π΄ΡƒΠΊΡ†ΠΈΠΈ. Π§Π΅ΠΌΡƒ Ρ€Π°Π²Π½Π° максимальная Π­Π”Π‘ ΠΈΠ½Π΄ΡƒΠΊΡ†ΠΈΠΈ, Π²ΠΎΠ·Π½ΠΈΠΊΠ°ΡŽΡ‰Π°Ρ Π² Ρ€Π°ΠΌΠΊΠ΅?

  • Π‘Ρ‚Π΅Ρ€ΠΆΠ΅Π½ΡŒ Π΄Π»ΠΈΠ½ΠΎΠΉ l вращаСтся Π² ΠΎΠ΄Π½ΠΎΡ€ΠΎΠ΄Π½ΠΎΠΌ ΠΌΠ°Π³Π½ΠΈΡ‚Π½ΠΎΠΌ ΠΏΠΎΠ»Π΅ с ΡƒΠ³Π»ΠΎΠ²ΠΎΠΉ ΡΠΊΠΎΡ€ΠΎΡΡ‚ΡŒΡŽ w Π²ΠΎΠΊΡ€ΡƒΠ³ оси пСрпСндикулярной ΡΡ‚Π΅Ρ€ΠΆΠ½ΡŽ ΠΈ ΠΏΡ€ΠΎΡ…ΠΎΠ΄ΡΡ‰ΡƒΡŽ Ρ‡Π΅Ρ€Π΅Π· ΠΎΠ΄ΠΈΠ½ ΠΈΠ· Π΅Π³ΠΎ ΠΊΠΎΠ½Ρ†ΠΎΠ². ΠžΠΏΡ€Π΅Π΄Π΅Π»ΠΈΡ‚ΡŒ Ρ€Π°Π·Π½ΠΎΡΡ‚ΡŒ ΠΏΠΎΡ‚Π΅Π½Ρ†ΠΈΠ°Π»ΠΎΠ² Π½Π° ΠΊΠΎΠ½Ρ†Π°Ρ… стСрТня, Ссли Π²Π΅ΠΊΡ‚ΠΎΡ€ ΠΌΠ°Π³Π½ΠΈΡ‚Π½ΠΎΠΉ ΠΈΠ½Π΄ΡƒΠΊΡ†ΠΈΠΈ Π’ ΠΏΠ°Ρ€Π°Π»Π»Π΅Π»Π΅Π½ плоскости, Π² ΠΊΠΎΡ‚ΠΎΡ€ΠΎΠΉ вращаСтся ΡΡ‚Π΅Ρ€ΠΆΠ΅Π½ΡŒ.

  • studfiles.net

    ЭлСктричСство ΠΈ ΠΌΠ°Π³Π½Π΅Ρ‚ΠΈΠ·ΠΌ

    Π˜Π½Π΄ΡƒΠΊΡ†ΠΈΠΎΠ½Π½Ρ‹Π΅ Ρ‚ΠΎΠΊΠΈ Π²ΠΎΠ·Π½ΠΈΠΊΠ°ΡŽΡ‚ Π½Π΅ Ρ‚ΠΎΠ»ΡŒΠΊΠΎ Π² ΠΏΡ€ΠΎΠ²ΠΎΠ»ΠΎΡ‡Π½Ρ‹Ρ… Π²ΠΈΡ‚ΠΊΠ°Ρ…, Π½ΠΎ ΠΈ Π² Ρ‚ΠΎΠ»Ρ‰Π΅ массивных ΠΏΡ€ΠΎΠ²ΠΎΠ΄Π½ΠΈΠΊΠΎΠ². Π’ этом случаС ΠΈΡ… Π½Π°Π·Ρ‹Π²Π°ΡŽΡ‚ Π²ΠΈΡ…Ρ€Π΅Π²Ρ‹ΠΌΠΈ Ρ‚ΠΎΠΊΠ°ΠΌΠΈ ΠΈΠ»ΠΈ Ρ‚ΠΎΠΊΠ°ΠΌΠΈ Π€ΡƒΠΊΠΎ. Π˜Π·β€“Π·Π° ΠΌΠ°Π»ΠΎΠ³ΠΎ сопротивлСния ΠΏΡ€ΠΎΠ²ΠΎΠ΄Π½ΠΈΠΊΠΎΠ² ΠΎΠ½ΠΈ ΠΌΠΎΠ³ΡƒΡ‚ Π΄ΠΎΡΡ‚ΠΈΠ³Π°Ρ‚ΡŒ большой силы. По ΠΏΡ€Π°Π²ΠΈΠ»Ρƒ Π›Π΅Π½Ρ†Π° Π²ΠΈΡ…Ρ€Π΅Π²Ρ‹Π΅ Ρ‚ΠΎΠΊΠΈ Ρ‚Π°ΠΊΠΆΠ΅ Π΄Π΅ΠΉΡΡ‚Π²ΡƒΡŽΡ‚ ΠΏΡ€ΠΎΡ‚ΠΈΠ² ΠΏΡ€ΠΈΡ‡ΠΈΠ½Ρ‹, ΠΈΡ… Π²Ρ‹Π·Ρ‹Π²Π°ΡŽΡ‰Π΅ΠΉ. На этом основана идСя элСктромагнитных Π΄Π΅ΠΌΠΏΡ„Π΅Ρ€ΠΎΠ², ΡƒΡΠΏΠΎΠΊΠ°ΠΈΠ²Π°ΡŽΡ‰ΠΈΡ… ΠΊΠΎΠ»Π΅Π±Π»ΡŽΡ‰ΠΈΠ΅ΡΡ части ΠΏΡ€ΠΈΠ±ΠΎΡ€ΠΎΠ² (стрСлки Π³Π°Π»ΡŒΠ²Π°Π½ΠΎΠΌΠ΅Ρ‚Ρ€ΠΎΠ² ΠΈ Ρ‚.Β ΠΏ.). На ΠΏΠΎΠ΄Π²ΠΈΠΆΠ½ΠΎΠΉ части ΠΏΡ€ΠΈΠ±ΠΎΡ€Π° укрСпляСтся мСталличСская полоска, находящаяся Π² ΠΏΠΎΠ»Π΅ сильного ΠΌΠ°Π³Π½ΠΈΡ‚Π°. ΠŸΡ€ΠΈ Π΄Π²ΠΈΠΆΠ΅Π½ΠΈΠΈ систСмы Ρ‚ΠΎΠΊΠΈ Π–.Β Π€ΡƒΠΊΠΎ (рис. 8.23) тормозят Π΅Π΅, Π½ΠΎ ΠΎΠ½ΠΈ ΠΎΡ‚ΡΡƒΡ‚ΡΡ‚Π²ΡƒΡŽΡ‚ ΠΏΡ€ΠΈ покоящСйся стрСлкС ΠΈ Π½Π΅ ΠΏΡ€Π΅ΠΏΡΡ‚ΡΡ‚Π²ΡƒΡŽΡ‚ Π΅Ρ‘ остановкС Π² Π½ΡƒΠΆΠ½ΠΎΠΌ мСстС, согласно Π·Π½Π°Ρ‡Π΅Π½ΠΈΡŽ измСряСмой Π²Π΅Π»ΠΈΡ‡ΠΈΠ½Ρ‹ (Π² ΠΎΡ‚Π»ΠΈΡ‡ΠΈΠ΅ ΠΎΡ‚ сил трСния).

    Рис. 8.23. Π›Π΅ΠΎΠ½ Π€ΡƒΠΊΠΎ (1819–1868) β€” французский Ρ„ΠΈΠ·ΠΈΠΊ ΠΈ астроном 

    Π˜Ρ‚ΠΎΠ³ΠΎΠΌ ΠΏΡ€ΠΎΠ²Π΅Π΄Π΅Π½Π½Ρ‹Ρ… рассуТдСний ΠΌΠΎΠΆΠ΅Ρ‚ Π±Ρ‹Ρ‚ΡŒ такая Ρ„ΠΎΡ€ΠΌΡƒΠ»ΠΈΡ€ΠΎΠ²ΠΊΠ° ΠΏΡ€Π°Π²ΠΈΠ»Π° Π›Π΅Π½Ρ†Π°: ΠΈΠ½Π΄ΡƒΠΊΡ†ΠΈΠΎΠ½Π½Ρ‹ΠΉ Ρ‚ΠΎΠΊ всСгда Π½Π°ΠΏΡ€Π°Π²Π»Π΅Π½ Ρ‚Π°ΠΊ, Ρ‡Ρ‚ΠΎΠ±Ρ‹ ΠΏΡ€Π΅ΠΏΡΡ‚ΡΡ‚Π²ΠΎΠ²Π°Ρ‚ΡŒ Ρ‚ΠΎΠΉ ΠΏΡ€ΠΈΡ‡ΠΈΠ½Π΅, которая Π΅Π³ΠΎ ΠΏΠΎΡ€ΠΎΠ΄ΠΈΠ»Π°. Π’Π½Π΅ зависимости ΠΎΡ‚ Ρ‚ΠΎΠ³ΠΎ, Ρ‡Ρ‚ΠΎ это Π·Π° ΠΏΡ€ΠΈΡ‡ΠΈΠ½Π°.

    НапримСр, Ссли ΠΏΡ€ΠΎΠ²ΠΎΠ»ΠΎΡ‡Π½ΠΎΠ΅ ΠΊΠΎΠ»ΡŒΡ†ΠΎ ΠΏΠ°Π΄Π°Π΅Ρ‚ Π² Π½Π΅ΠΎΠ΄Π½ΠΎΡ€ΠΎΠ΄Π½ΠΎΠΌ ΠΌΠ°Π³Π½ΠΈΡ‚Π½ΠΎΠΌ ΠΏΠΎΠ»Π΅ ΠΏΠΎΠ΄ дСйствиСм силы тяТСсти, Ρ‚ΠΎ Π² Π½Π΅ΠΌ Ρ‚Π΅Ρ‡Π΅Ρ‚ ΠΈΠ½Π΄ΡƒΠΊΡ†ΠΈΠΎΠ½Π½Ρ‹ΠΉ Ρ‚ΠΎΠΊ. БоотвСтствСнно Π½Π° ΠΊΠΎΠ»ΡŒΡ†ΠΎ дСйствуСт сила АмпСра. НичСго Π½Π΅ вычисляя, ΠΌΠΎΠΆΠ½ΠΎ Π±Ρ‹Ρ‚ΡŒ ΡƒΠ²Π΅Ρ€Π΅Π½Π½Ρ‹ΠΌ Π² Ρ‚ΠΎΠΌ, Ρ‡Ρ‚ΠΎ эта сила АмпСра Π±ΡƒΠ΄Π΅Ρ‚ Π½Π°ΠΏΡ€Π°Π²Π»Π΅Π½Π° Π²Π²Π΅Ρ€Ρ…, Ρ‡Ρ‚ΠΎΠ±Ρ‹ β€” согласно ΠΏΡ€Π°Π²ΠΈΠ»Ρƒ Π›Π΅Π½Ρ†Π° β€” ΠΌΠ΅ΡˆΠ°Ρ‚ΡŒ силС тяТСсти, которая являСтся ΠΏΡ€ΠΈΡ‡ΠΈΠ½ΠΎΠΉ падСния ΠΊΠΎΠ»ΡŒΡ†Π°, Ρ‡Ρ‚ΠΎ Π²Π»Π΅Ρ‡Π΅Ρ‚ Π·Π° собой ΠΈΠ·ΠΌΠ΅Π½Π΅Π½ΠΈΠ΅ ΠΌΠ°Π³Π½ΠΈΡ‚Π½ΠΎΠ³ΠΎ ΠΏΠΎΡ‚ΠΎΠΊΠ°, Π° это ΠΏΡ€ΠΈΠ²ΠΎΠ΄ΠΈΡ‚ ΠΊ появлСнию ΠΈΠ½Π΄ΡƒΠΊΡ†ΠΈΠΎΠ½Π½ΠΎΠ³ΠΎ Ρ‚ΠΎΠΊΠ°, Π½Π° ΠΊΠΎΡ‚ΠΎΡ€Ρ‹ΠΉ дСйствуСт сила АмпСра, тормозящая падСниС…

    НиТС Ρ€Π°ΡΡΠΌΠ°Ρ‚Ρ€ΠΈΠ²Π°ΡŽΡ‚ΡΡ ΠΎΠΏΡ‹Ρ‚Ρ‹, Π² ΠΊΠΎΡ‚ΠΎΡ€Ρ‹Ρ… ΠΈΠ·ΡƒΡ‡Π°ΡŽΡ‚ΡΡ свойства Ρ‚ΠΎΠΊΠΎΠ² Π€ΡƒΠΊΠΎ.Β 

    На рис. 8.24 ΠΏΠΎΠΊΠ°Π·Π°Π½ ΠΎΠΏΡ‹Ρ‚, Π΄Π΅ΠΌΠΎΠ½ΡΡ‚Ρ€ΠΈΡ€ΡƒΡŽΡ‰ΠΈΠΉ ΠΏΠ°Π΄Π΅Π½ΠΈΠ΅ Ρ‚Π΅Π» Π² Π½Π΅ΠΎΠ΄Π½ΠΎΡ€ΠΎΠ΄Π½ΠΎΠΌ ΠΌΠ°Π³Π½ΠΈΡ‚Π½ΠΎΠΌ ΠΏΠΎΠ»Π΅. НСоднородноС ΠΌΠ°Π³Π½ΠΈΡ‚Π½ΠΎΠ΅ ΠΏΠΎΠ»Π΅ Ρ‚ΠΎΡ€ΠΌΠΎΠ·ΠΈΡ‚ Π΄Π²ΠΈΠΆΠ΅Π½ΠΈΠ΅ проводящих ΠΏΡ€Π΅Π΄ΠΌΠ΅Ρ‚ΠΎΠ² ΠΈΠ·-Π·Π° Ρ‚ΠΎΠΊΠΎΠ² Π€ΡƒΠΊΠΎ, Π²ΠΎΠ·Π½ΠΈΠΊΠ°ΡŽΡ‰ΠΈΡ… Π² ΠΏΡ€ΠΎΠ²ΠΎΠ΄Π½ΠΈΠΊΠ°Ρ… ΠΏΡ€ΠΈ ΠΈΠ·ΠΌΠ΅Π½Π΅Π½ΠΈΠΈ ΠΌΠ°Π³Π½ΠΈΡ‚Π½ΠΎΠ³ΠΎ ΠΏΠΎΡ‚ΠΎΠΊΠ° Ρ‡Π΅Ρ€Π΅Π· Π½ΠΈΡ…. ДСмонстрируСтся бСспрСпятствСнноС ΠΏΠ°Π΄Π΅Π½ΠΈΠ΅ диэлСктричСского дСрСвянного диска ΠΌΠ΅ΠΆΠ΄Ρƒ полюсами сильного элСктромагнита ΠΈ ΠΌΠ΅Π΄Π»Π΅Π½Π½ΠΎΠ΅ ΠΏΠ°Π΄Π΅Π½ΠΈΠ΅ ΠΌΠ΅Π΄Π½ΠΎΠ³ΠΎ ΠΈ алюминиСвого дисков Π² ΠΌΠ°Π³Π½ΠΈΡ‚Π½ΠΎΠΌ ΠΏΠΎΠ»Π΅, Π½Π°ΠΏΠΎΠΌΠΈΠ½Π°ΡŽΡ‰Π΅Π΅ Π΄Π²ΠΈΠΆΠ΅Π½ΠΈΠ΅ Ρ‚Π΅Π» Π² срСдС с большой Π²ΡΠ·ΠΊΠΎΡΡ‚ΡŒΡŽ.

    Рис. 8.24. ПадСниС Ρ‚Π΅Π» Π² Π½Π΅ΠΎΠ΄Π½ΠΎΡ€ΠΎΠ΄Π½ΠΎΠΌ ΠΌΠ°Π³Π½ΠΈΡ‚Π½ΠΎΠΌ ΠΏΠΎΠ»Π΅Β 

    Π’ΠΈΠ΄Π΅ΠΎ 8.9. Π­Π»Π΅ΠΊΡ‚Ρ€ΠΎΠΌΠ°Π³Π½ΠΈΡ‚Π½ΠΎΠ΅ Ρ‚ΠΎΡ€ΠΌΠΎΠΆΠ΅Π½ΠΈΠ΅: ΠΏΠ°Π΄Π΅Π½ΠΈΠ΅ ΠΌΠ΅Π΄Π½Ρ‹Ρ… ΠΈ Π°Π»ΡŽΠΌΠΈΠ½ΠΈΠ΅Π²Ρ‹Ρ… дисков  (Β«ΠΌΠΎΠ½Π΅Ρ‚Β») Π² ΠΌΠ°Π³Π½ΠΈΡ‚Π½ΠΎΠΌ ΠΏΠΎΠ»Π΅.

    ΠŸΡ€ΠΈ ΠΏΠ°Π΄Π΅Π½ΠΈΠΈ сильного постоянного ΠΌΠ°Π³Π½ΠΈΡ‚Π° Π²Π½ΡƒΡ‚Ρ€ΠΈ Π²Π΅Ρ€Ρ‚ΠΈΠΊΠ°Π»ΡŒΠ½ΠΎΠΉ проводящСй Ρ‚Ρ€ΡƒΠ±ΠΊΠΈ Π² Π΅Π΅ стСнках Π²ΠΎΠ·Π½ΠΈΠΊΠ°ΡŽΡ‚ Ρ‚ΠΎΠΊΠΈ Π€ΡƒΠΊΠΎ, тормозящиС это ΠΏΠ°Π΄Π΅Π½ΠΈΠ΅. Π’ ΠΎΠΏΡ‹Ρ‚Π΅ (рис. 8.25) дСмонстрируСтся свободноС ΠΏΠ°Π΄Π΅Π½ΠΈΠ΅ Π½Π΅ΠΌΠ°Π³Π½ΠΈΡ‚Π½ΠΎΠ³ΠΎ алюминиСвого Ρ†ΠΈΠ»ΠΈΠ½Π΄Ρ€Π° Π² Ρ€Π°Π·Π½Ρ‹Ρ… Ρ‚Ρ€ΡƒΠ±ΠΊΠ°Ρ…, Π° Ρ‚Π°ΠΊΠΆΠ΅ малСнького ΠΌΠ°Π³Π½ΠΈΡ‚Π° Π² стСклянной Ρ‚Ρ€ΡƒΠ±ΠΊΠ΅. Π—Π°Ρ‚Π΅ΠΌ ΠΏΠΎΠΊΠ°Π·Ρ‹Π²Π°ΡŽΡ‚ Π·Π°ΠΌΠ΅Π΄Π»Π΅Π½ΠΈΠ΅ падСния этого ΠΌΠ°Π³Π½ΠΈΡ‚Π° Π² алюминиСвой Ρ‚Ρ€ΡƒΠ±ΠΊΠ΅ ΠΈ Π΅Π³ΠΎ ΠΎΡ‡Π΅Π½ΡŒ ΠΌΠ΅Π΄Π»Π΅Π½Π½ΠΎΠ΅ ΠΏΠ°Π΄Π΅Π½ΠΈΠ΅ Π² толстостСнной ΠΌΠ΅Π΄Π½ΠΎΠΉ Ρ‚Ρ€ΡƒΠ±ΠΊΠ΅.

    Β 

    Рис. 8.25. ПадСниС ΠΌΠ°Π³Π½ΠΈΡ‚Π° Π² Ρ‚Ρ€ΡƒΠ±ΠΊΠ°Ρ…Β 

    На рис. 8.26 ΠΏΠΎΠΊΠ°Π·Π°Π½ΠΎ Π΄Π΅ΠΌΠΏΡ„ΠΈΡ€ΠΎΠ²Π°Π½ΠΈΠ΅ ΠΊΠΎΠ»Π΅Π±Π°Π½ΠΈΠΉ маятника. Волстая сплошная мСдная пластина, прикрСплСнная Π½Π° ΠΊΠΎΠ½Ρ†Π΅ физичСского маятника, двиТСтся ΠΏΡ€ΠΈ Π΅Π³ΠΎ колСбаниях ΠΌΠ΅ΠΆΠ΄Ρƒ полюсами сильного элСктромагнита. Π‘Π»Π°Π±ΠΎ Π·Π°Ρ‚ΡƒΡ…Π°ΡŽΡ‰ΠΈΠ΅ колСбания маятника послС Π²ΠΊΠ»ΡŽΡ‡Π΅Π½ΠΈΡ ΠΌΠ°Π³Π½ΠΈΡ‚Π½ΠΎΠ³ΠΎ поля Π½Π°Ρ‡ΠΈΠ½Π°ΡŽΡ‚ быстро Π·Π°Ρ‚ΡƒΡ…Π°Ρ‚ΡŒ, ΠΏΡ€Π΅Π²Ρ€Π°Ρ‰Π°ΡΡΡŒ практичСски Π² апСриодичСскиС колСбания. Если Π½Π° ΠΊΠΎΠ½Ρ†Π΅ маятника Π·Π°ΠΊΡ€Π΅ΠΏΠΈΡ‚ΡŒ ΠΌΠ΅Π΄Π½ΡƒΡŽ пластинку, Ρ€Π°Π·Ρ€Π΅Π·Π°Π½Π½ΡƒΡŽ Π² Π²ΠΈΠ΄Π΅ Π³Ρ€Π΅Π±Π΅Π½ΠΊΠΈ, Ρ‚ΠΎ сильноС Π·Π°Ρ‚ΡƒΡ…Π°Π½ΠΈΠ΅ ΠΊΠΎΠ»Π΅Π±Π°Π½ΠΈΠΉ маятника исчСзаСт, ΠΏΠΎΡΠΊΠΎΠ»ΡŒΠΊΡƒ Ρ‚ΠΎΠΊΠΈ Π€ΡƒΠΊΠΎ ΡƒΠΆΠ΅ Π½Π΅ ΠΌΠΎΠ³ΡƒΡ‚ Π·Π°ΠΌΡ‹ΠΊΠ°Ρ‚ΡŒΡΡ Π² объСмС ΠΏΡ€ΠΎΠ²ΠΎΠ΄Π½ΠΈΠΊΠ°.Β 

    Β 

    Рис. 8.26. Π”Π΅ΠΌΠΏΡ„ΠΈΡ€ΠΎΠ²Π°Π½ΠΈΠ΅ ΠΊΠΎΠ»Π΅Π±Π°Π½ΠΈΠΉ маятника 

    Π’ΠΈΠ΄Π΅ΠΎ 8.10. Π­Π»Π΅ΠΊΡ‚Ρ€ΠΎΠΌΠ°Π³Π½ΠΈΡ‚Π½ΠΎΠ΅ Ρ‚ΠΎΡ€ΠΌΠΎΠΆΠ΅Π½ΠΈΠ΅: маятник.

    Π’ ΠΎΠΏΡ‹Ρ‚Π΅ Π½Π° рис. 8.27 ΠΏΠΎΠΊΠ°Π·Π°Π½Π° лСвитация сплошного проводящСго ΠΊΠΎΠ»ΡŒΡ†Π°. Π’ΠΎΠΊΠΈ Π€ΡƒΠΊΠΎ ΠΌΠΎΠ³ΡƒΡ‚ Π²ΠΎΠ·Π½ΠΈΠΊΠ°Ρ‚ΡŒ Π½Π΅ Ρ‚ΠΎΠ»ΡŒΠΊΠΎ Π² ΠΏΡ€ΠΎΠ²ΠΎΠ΄Π½ΠΈΠΊΠ°Ρ… ΠΏΡ€ΠΈ ΠΈΡ… ΠΏΠ΅Ρ€Π΅ΠΌΠ΅Ρ‰Π΅Π½ΠΈΠΈ Π² Π½Π΅ΠΎΠ΄Π½ΠΎΡ€ΠΎΠ΄Π½ΠΎΠΌ ΠΌΠ°Π³Π½ΠΈΡ‚Π½ΠΎΠΌ ΠΏΠΎΠ»Π΅, Π½ΠΎ ΠΈ ΠΏΡ€ΠΈ быстром ΠΈΠ·ΠΌΠ΅Π½Π΅Π½ΠΈΠΈ этого поля. сплошноС ΠΊΠΎΠ»ΡŒΡ†ΠΎ ΠΈΠ· алюминия, Π½Π°Π΄Π΅Ρ‚ΠΎΠ΅ Π½Π° Π²Π΅Ρ€Ρ‚ΠΈΠΊΠ°Π»ΡŒΠ½Ρ‹ΠΉ сСрдСчник элСктромагнита, ΠΏΠΈΡ‚Π°Π΅ΠΌΠΎΠ³ΠΎ ΠΏΠ΅Ρ€Π΅ΠΌΠ΅Π½Π½Ρ‹ΠΌ Ρ‚ΠΎΠΊΠΎΠΌ частотой 50 Π“Ρ†, висит Π² Π²ΠΎΠ·Π΄ΡƒΡ…Π΅. Π² Ρ‚ΠΎ врСмя ΠΊΠ°ΠΊ Ρ‚Π°ΠΊΠΎΠ΅ ΠΆΠ΅, Π½ΠΎ Ρ€Π°Π·Ρ€Π΅Π·Π°Π½Π½ΠΎΠ΅ ΠΊΠΎΠ»ΡŒΡ†ΠΎ свободно ΠΏΠ°Π΄Π°Π΅Ρ‚ Π½Π° ΠΎΠ±ΠΌΠΎΡ‚ΠΊΡƒ.Β 

    Β 

    Рис. 8.27. ЛСвитация сплошного проводящСго ΠΊΠΎΠ»ΡŒΡ†Π°Β 

    На рис. 8.28 ΠΏΠΎΠΊΠ°Π·Π°Π½ΠΎ взаимодСйствиС ΠΏΡ€ΠΎΠ²ΠΎΠ΄Π½ΠΈΠΊΠ° ΠΈ элСктромагнита. Волстый ΠΌΠ΅Π΄Π½Ρ‹ΠΉ диск ΡƒΠΊΡ€Π΅ΠΏΠ»Π΅Π½ Π² ΠΏΠΎΠ΄ΡˆΠΈΠΏΠ½ΠΈΠΊΠ°Ρ… Π½Π° оси с Ρ€ΡƒΡ‡ΠΊΠΎΠΉ. Π’Π±Π»ΠΈΠ·ΠΈ Π½Π΅Π³ΠΎ Π½Π° Ρ‚Π°ΠΊΠΎΠΉ ΠΆΠ΅ оси Π·Π°ΠΊΡ€Π΅ΠΏΠ»Π΅Π½ элСктромагнит. Если Π²Ρ€Π°Ρ‰Π°Ρ‚ΡŒ Π·Π° Ρ€ΡƒΡ‡ΠΊΡƒ Π²ΠΊΠ»ΡŽΡ‡Π΅Π½Π½Ρ‹ΠΉ элСктромагнит, Ρ‚ΠΎ диск Π½Π°Ρ‡ΠΈΠ½Π°Π΅Ρ‚ Π²Ρ€Π°Ρ‰Π°Ρ‚ΡŒΡΡ Π² Ρ‚Ρƒ ΠΆΠ΅ сторону. Если ΠΆΠ΅, Π½Π°ΠΎΠ±ΠΎΡ€ΠΎΡ‚, Π²Ρ€Π°Ρ‰Π°Ρ‚ΡŒ Π·Π° Ρ€ΡƒΡ‡ΠΊΡƒ диск Π²Π±Π»ΠΈΠ·ΠΈ элСктромагнита, Ρ‚ΠΎ послСдний Ρ‚Π°ΠΊΠΆΠ΅ Π½Π°Ρ‡ΠΈΠ½Π°Π΅Ρ‚ Π²Ρ€Π°Ρ‰Π°Ρ‚ΡŒΡΡ. Π‘ΠΈΠ»Ρ‹ взаимодСйствия диска ΠΈ элСктромагнита, ΠΏΠΎΡ…ΠΎΠΆΠΈΠ΅ ΠΏΠΎ Ρ…Π°Ρ€Π°ΠΊΡ‚Π΅Ρ€Ρƒ Π½Π° силы вязкого трСния, обусловлСны Π²ΠΎΠ·Π½ΠΈΠΊΠ½ΠΎΠ²Π΅Π½ΠΈΠ΅ΠΌ Ρ‚ΠΎΠΊΠΎΠ² Π€ΡƒΠΊΠΎ Π² дискС.

    online.mephi.ru

    Π—Π°Π΄Π°Ρ‡ΠΈ для ΡΠ°ΠΌΠΎΡΡ‚ΠΎΡΡ‚Π΅Π»ΡŒΠ½ΠΎΠ³ΠΎ Ρ€Π΅ΡˆΠ΅Π½ΠΈΡ

  • B ΠΎΠ΄Π½ΠΎΡ€ΠΎΠ΄Π½ΠΎΠΌ ΠΌΠ°Π³Π½ΠΈΡ‚Π½ΠΎΠΌ ΠΏΠΎΠ»Π΅, индукция ΠΊΠΎΡ‚ΠΎΡ€ΠΎΠ³ΠΎ 0,1 Π’Π», двиТСтся ΠΏΡ€ΠΎΠ²ΠΎΠ΄Π½ΠΈΠΊ Π΄Π»ΠΈΠ½ΠΎΠΉ 10 см. Π‘ΠΊΠΎΡ€ΠΎΡΡ‚ΡŒ двиТСния ΠΏΡ€ΠΎΠ²ΠΎΠ΄Π½ΠΈΠΊΠ° 15 ΠΌ/с ΠΈ Π½Π°ΠΏΡ€Π°Π²Π»Π΅Π½Π° пСрпСндикулярно ΠΌΠ°Π³Π½ΠΈΡ‚Π½ΠΎΠΌΡƒ полю. Π§Π΅ΠΌΡƒ Ρ€Π°Π²Π½Π° индуцированная Π² ΠΏΡ€ΠΎΠ²ΠΎΠ΄Π½ΠΈΠΊΠ΅ Π­Π”Π‘?

  • Π‘ΠΊΠΎΡ€ΠΎΡΡ‚ΡŒ самолСта 950 ΠΊΠΌ/Ρ‡. Найти Π­Π”Π‘ ΠΈΠ½Π΄ΡƒΠΊΡ†ΠΈΠΈ, Π²ΠΎΠ·Π½ΠΈΠΊΠ°ΡŽΡ‰ΡƒΡŽ Π½Π° ΠΊΠΎΠ½Ρ†Π°Ρ… ΠΊΡ€Ρ‹Π»ΡŒΠ΅Π² самолСта, Ссли Π²Π΅Ρ€Ρ‚ΠΈΠΊΠ°Π»ΡŒΠ½Π°Ρ ΡΠΎΡΡ‚Π°Π²Π»ΡΡŽΡ‰Π°Ρ напряТСнности ΠΌΠ°Π³Π½ΠΈΡ‚Π½ΠΎΠ³ΠΎ поля Π—Π΅ΠΌΠ»ΠΈ Ρ€Π°Π²Π½Π° 40 А/ΠΌ ΠΈ Ρ€Π°Π·ΠΌΠ°Ρ… ΠΊΡ€Ρ‹Π»ΡŒΠ΅Π² самолСта 12,5 ΠΌ.

  • Π‘ ΠΊΠ°ΠΊΠΎΠΉ ΡΠΊΠΎΡ€ΠΎΡΡ‚ΡŒΡŽ Π΄ΠΎΠ»ΠΆΠ΅Π½ Π΄Π²ΠΈΠ³Π°Ρ‚ΡŒΡΡ ΠΏΡ€ΠΎΠ²ΠΎΠ΄Π½ΠΈΠΊ Π΄Π»ΠΈΠ½ΠΎΠΉ 10 см пСрпСн­дикулярно ΠΊ линиям ΠΈΠ½Π΄ΡƒΠΊΡ†ΠΈΠΈ ΠΎΠ΄Π½ΠΎΡ€ΠΎΠ΄Π½ΠΎΠ³ΠΎ ΠΌΠ°Π³Π½ΠΈΡ‚Π½ΠΎΠ³ΠΎ поля, Ρ‡Ρ‚ΠΎΠ±Ρ‹ ΠΌΠ΅ΠΆΠ΄Ρƒ ΠΊΠΎΠ½Ρ†Π°ΠΌΠΈ ΠΏΡ€ΠΎΠ²ΠΎΠ΄Π½ΠΈΠΊΠ° Π²ΠΎΠ·Π½ΠΈΠΊΠ»Π° Ρ€Π°Π·Π½ΠΎΡΡ‚ΡŒ ΠΏΠΎΡ‚Π΅Π½Ρ†ΠΈΠ°Π»ΠΎΠ² 0,01 Π’? Π›ΠΈΠ½ΠΈΠΈ ΠΈΠ½Π΄ΡƒΠΊΡ†ΠΈΠΈ пСрпСндикулярны ΠΏΡ€ΠΎΠ²ΠΎΠ΄Π½ΠΈΠΊΡƒ, индукция Π’ = 0,2 Π’Π».

  • Найти Π­Π”Π‘ ΠΈΠ½Π΄ΡƒΠΊΡ†ΠΈΠΈ Π² ΠΏΡ€ΠΎΠ²ΠΎΠ΄Π½ΠΈΠΊΠ΅ с Π΄Π»ΠΈΠ½ΠΎΠΉ Π°ΠΊΡ‚ΠΈΠ²Π½ΠΎΠΉ части 0,25 ΠΌ, ΠΏΠ΅Ρ€Π΅ΠΌΠ΅Ρ‰Π°ΡŽΡ‰Π΅ΠΌΡΡ Π² ΠΎΠ΄Π½ΠΎΡ€ΠΎΠ΄Π½ΠΎΠΌ ΠΌΠ°Π³Π½ΠΈΡ‚Π½ΠΎΠΌ ΠΏΠΎΠ»Π΅ с ΠΈΠ½Π΄ΡƒΠΊΡ†ΠΈΠ΅ΠΉ 8 ΠΌΠ’Π» со ΡΠΊΠΎΡ€ΠΎΡΡ‚ΡŒΡŽ 5 ΠΌ/с ΠΏΠΎΠ΄ ΡƒΠ³Π»ΠΎΠΌ 30Β° ΠΊ Π²Π΅ΠΊΡ‚ΠΎΡ€Ρƒ ΠΌΠ°Π³Π½ΠΈΡ‚Π½ΠΎΠΉ ΠΈΠ½Π΄ΡƒΠΊΡ†ΠΈΠΈ.

  • Π‘ ΠΊΠ°ΠΊΠΎΠΉ ΡΠΊΠΎΡ€ΠΎΡΡ‚ΡŒΡŽ Π½Π°Π΄ΠΎ ΠΏΠ΅Ρ€Π΅ΠΌΠ΅Ρ‰Π°Ρ‚ΡŒ ΠΏΡ€ΠΎΠ²ΠΎΠ΄Π½ΠΈΠΊ, Π΄Π»ΠΈΠ½Π° Π°ΠΊΡ‚ΠΈΠ²Π½ΠΎΠΉ части ΠΊΠΎΡ‚ΠΎΡ€ΠΎΠ³ΠΎ 1 ΠΌ, ΠΏΠΎΠ΄ ΡƒΠ³Π»ΠΎΠΌ 60Β° ΠΊ линиям ΠΈΠ½Π΄ΡƒΠΊΡ†ΠΈΠΈ ΠΌΠ°Π³Π½ΠΈΡ‚Π½ΠΎΠ³ΠΎ поля, Ρ‡Ρ‚ΠΎΠ±Ρ‹ Π² ΠΏΡ€ΠΎΠ²ΠΎΠ΄Π½ΠΈΠΊΠ΅ Π²ΠΎΠ·Π±ΡƒΠΆΠ΄Π°Π»Π°ΡΡŒ Π­Π”Π‘ ΠΈΠ½Π΄ΡƒΠΊΡ†ΠΈΠΈ 1 Π’? Π˜Π½Π΄ΡƒΠΊΡ†ΠΈΡ ΠΌΠ°Π³Π½ΠΈΡ‚Π½ΠΎΠ³ΠΎ поля Ρ€Π°Π²Π½Π° 0,2 Π’Π».

  • Найти Ρ€Π°Π·Π½ΠΎΡΡ‚ΡŒ ΠΏΠΎΡ‚Π΅Π½Ρ†ΠΈΠ°Π»ΠΎΠ², Π²ΠΎΠ·Π½ΠΈΠΊΠ°ΡŽΡ‰ΡƒΡŽ Π½Π° ΠΊΠΎΠ½Ρ†Π°Ρ… ΠΊΡ€Ρ‹Π»Π° самолСта ΠΏΡ€ΠΈ Π³ΠΎΡ€ΠΈΠ·ΠΎΠ½Ρ‚Π°Π»ΡŒΠ½ΠΎΠΌ ΠΏΠΎΠ»Π΅Ρ‚Π΅ со ΡΠΊΠΎΡ€ΠΎΡΡ‚ΡŒΡŽ 900 ΠΊΠΌ/Ρ‡, Ссли Ρ€Π°Π·ΠΌΠ°Ρ… ΠΊΡ€Ρ‹Π»Π° самолСта 40 ΠΌ. Π’Π΅Ρ€Ρ‚ΠΈΠΊΠ°Π»ΡŒΠ½Π°Ρ ΡΠΎΡΡ‚Π°Π²Π»ΡΡŽΡ‰Π°Ρ ΠΈΠ½Π΄ΡƒΠΊΡ†ΠΈΠΈ ΠΌΠ°Π³Π½ΠΈΡ‚Π½ΠΎΠ³ΠΎ поля Π—Π΅ΠΌΠ»ΠΈ Π’ = 5Β·105Π’Π».

  • Π‘Π°ΠΌΠΎΠ»Π΅Ρ‚ Π»Π΅Ρ‚ΠΈΡ‚ Π³ΠΎΡ€ΠΈΠ·ΠΎΠ½Ρ‚Π°Π»ΡŒΠ½ΠΎ со ΡΠΊΠΎΡ€ΠΎΡΡ‚ΡŒΡŽ 900 ΠΊΠΌ/Ρ‡. ΠŸΡ€ΠΈ этом Π½Π° ΠΊΠΎΠ½Ρ†Π°Ρ… ΠΊΡ€Ρ‹Π»ΡŒΠ΅Π² самолСта Π²ΠΎΠ·Π½ΠΈΠΊΠ°Π΅Ρ‚ Ρ€Π°Π·Π½ΠΎΡΡ‚ΡŒ ΠΏΠΎΡ‚Π΅Π½Ρ†ΠΈΠ°Π»ΠΎΠ² 0,6 Π’.. Π’Π΅Ρ€Ρ‚ΠΈΠΊΠ°Π»ΡŒΒ­Π½Π°Ρ ΡΠΎΡΡ‚Π°Π²Π»ΡΡŽΡ‰Π°Ρ ΠΈΠ½Π΄ΡƒΠΊΡ†ΠΈΠΈ ΠΌΠ°Π³Π½ΠΈΡ‚Π½ΠΎΠ³ΠΎ поля Π—Π΅ΠΌΠ»ΠΈ Ρ€Π°Π²Π½Π° 5Β·105 Π’Π». Какой Ρ€Π°Π·ΠΌΠ°Ρ… ΠΈΠΌΠ΅ΡŽΡ‚ ΠΊΡ€Ρ‹Π»ΡŒΡ самолСта?

  • ΠŸΡ€ΡΠΌΠΎΠΉ ΠΏΡ€ΠΎΠ²ΠΎΠ΄Π½ΠΈΠΊ Π΄Π»ΠΈΠ½ΠΎΠΉ 40 см двиТСтся Π² ΠΎΠ΄Π½ΠΎΡ€ΠΎΠ΄Π½ΠΎΠΌ ΠΌΠ°Π³Π½ΠΈΡ‚Π½ΠΎΠΌ ΠΏΠΎΠ»Π΅ со ΡΠΊΠΎΡ€ΠΎΡΡ‚ΡŒΡŽ 5 ΠΌ/с пСрпСндикулярно ΠΊ линиям ΠΈΠ½Π΄ΡƒΠΊΡ†ΠΈΠΈ. Π Π°Π·Π½ΠΎΡΡ‚ΡŒ ΠΏΠΎΡ‚Π΅Π½Ρ†ΠΈΠ°Π»ΠΎΠ² ΠΌΠ΅ΠΆΠ΄Ρƒ ΠΊΠΎΠ½Ρ†Π°ΠΌΠΈ ΠΏΡ€ΠΎΠ²ΠΎΠ΄Π½ΠΈΠΊΠ° 0,6 Π’. Π’Ρ‹Ρ‡ΠΈΡΠ»ΠΈΡ‚ΡŒ ΠΈΠ½Π΄ΡƒΠΊΡ†ΠΈΡŽ ΠΌΠ°Π³Π½ΠΈΡ‚Π½ΠΎΠ³ΠΎ поля.

  • Π‘ ΠΊΠ°ΠΊΠΎΠΉ ΡΠΊΠΎΡ€ΠΎΡΡ‚ΡŒΡŽ Π΄ΠΎΠ»ΠΆΠ΅Π½ Π΄Π²ΠΈΠ³Π°Ρ‚ΡŒΡΡ ΠΏΡ€ΠΎΠ²ΠΎΠ΄Π½ΠΈΠΊ Π΄Π»ΠΈΠ½ΠΎΠΉ 10 см пСрпСндикулярно силовым линиям ΠΎΠ΄Π½ΠΎΡ€ΠΎΠ΄Π½ΠΎΠ³ΠΎ ΠΌΠ°Π³Π½ΠΈΡ‚Π½ΠΎΠ³ΠΎ поля Π½Π°ΠΏΡ€ΡΠΆΠ΅Π½Π½ΠΎΡΡ‚ΡŒΡŽ 1000 А/ΠΌ, Ρ‡Ρ‚ΠΎΠ±Ρ‹ ΠΌΠ΅ΠΆΠ΄Ρƒ ΠΊΠΎΠ½Ρ†Π°ΠΌΠΈ ΠΏΡ€ΠΎΠ²ΠΎΠ΄Π½ΠΈΠΊΠ° Π²ΠΎΠ·Π½ΠΈΠΊΠ»Π° Ρ€Π°Π·Π½ΠΎΡΡ‚ΡŒ ΠΏΠΎΡ‚Π΅Π½Ρ†ΠΈΠ°Π»ΠΎΠ² 0,001 Π’? НаправлСниС скорости ΠΏΡ€ΠΎΠ²ΠΎΠ΄Π½ΠΈΠΊΠ° с Π½Π°ΠΏΡ€Π°Π²Π»Π΅Π½ΠΈΠ΅ΠΌ самого ΠΏΡ€ΠΎΠ²ΠΎΠ΄Π½ΠΈΠΊΠ° составляСт 45Β°.

  • ΠžΠΏΡ€Π΅Π΄Π΅Π»ΠΈΡ‚ΡŒ Ρ€Π°Π·Π½ΠΎΡΡ‚ΡŒ ΠΏΠΎΡ‚Π΅Π½Ρ†ΠΈΠ°Π»ΠΎΠ², Π²ΠΎΠ·Π½ΠΈΠΊΠ°ΡŽΡ‰ΡƒΡŽ Π½Π° ΠΊΠΎΠ½Ρ†Π°Ρ… Π²Π΅Ρ€Ρ‚ΠΈΠΊΠ°Π»ΡŒΠ½ΠΎΠΉ Π°Π²Ρ‚ΠΎΠΌΠΎΠ±ΠΈΠ»ΡŒΠ½ΠΎΠΉ Π°Π½Ρ‚Π΅Π½Π½Ρ‹ Π΄Π»ΠΈΠ½ΠΎΠΉ 1,2 ΠΌ ΠΏΡ€ΠΈ Π΄Π²ΠΈΠΆΠ΅Π½ΠΈΠΈ автомобиля с востока Π½Π° Π·Π°ΠΏΠ°Π΄ Π² ΠΌΠ°Π³Π½ΠΈΡ‚Π½ΠΎΠΌ ΠΏΠΎΠ»Π΅ Π—Π΅ΠΌΠ»ΠΈ со ΡΠΊΠΎΡ€ΠΎΡΡ‚ΡŒΡŽ 72 ΠΊΠΌ/Ρ‡. Π“ΠΎΡ€ΠΈΠ·ΠΎΠ½Ρ‚Π°Π»ΡŒΠ½Π°Ρ ΡΠΎΡΡ‚Π°Π²Π»ΡΡŽΡ‰Π°Ρ напряТСнности Π·Π΅ΠΌΠ½ΠΎΠ³ΠΎ ΠΌΠ°Π³Π½ΠΈΡ‚Π½ΠΎΠ³ΠΎ поля Ρ€Π°Π²Π½Π° 16 А/ΠΌ.

  • Π–Π΅Π»Π΅Π·Π½ΠΎΠ΄ΠΎΡ€ΠΎΠΆΠ½Ρ‹Π΅ Ρ€Π΅Π»ΡŒΡΡ‹ ΠΈΠ·ΠΎΠ»ΠΈΡ€ΠΎΠ²Π°Π½Ρ‹ Π΄Ρ€ΡƒΠ³ ΠΎΡ‚ Π΄Ρ€ΡƒΠ³Π° ΠΈ ΠΎΡ‚ Π—Π΅ΠΌΠ»ΠΈ ΠΈ соСдинСны Ρ‡Π΅Ρ€Π΅Π· ΠΌΠΈΠ»Π»ΠΈΠ²ΠΎΠ»ΡŒΡ‚ΠΌΠ΅Ρ‚Ρ€. Каково ΠΏΠΎΠΊΠ°Π·Π°Π½ΠΈΠ΅ ΠΏΡ€ΠΈΠ±ΠΎΡ€Π°, Ссли ΠΏΠΎ Ρ€Π΅Π»ΡŒΡΠ°ΠΌ ΠΏΡ€ΠΎΡ…ΠΎΠ΄ΠΈΡ‚ ΠΏΠΎΠ΅Π·Π΄ со ΡΠΊΠΎΡ€ΠΎΡΡ‚ΡŒΡŽ 20 ΠΌ/с? Π’Π΅Ρ€Ρ‚ΠΈΠΊΠ°Π»ΡŒΠ½ΡƒΡŽ ΡΠΎΡΡ‚Π°Π²Π»ΡΡŽΡ‰ΡƒΡŽ напряТСнности ΠΌΠ°Π³Π½ΠΈΡ‚Π½ΠΎΠ³ΠΎ поля Π—Π΅ΠΌΠ»ΠΈ ΠΏΡ€ΠΈΠ½ΡΡ‚ΡŒ Ρ€Π°Π²Π½ΠΎΠΉ 40 А/ΠΌ, Π° расстояниС ΠΌΠ΅ΠΆΠ΄Ρƒ Ρ€Π΅Π»ΡŒΡΠ°ΠΌΠΈ 1,54 ΠΌ.

  • Π’ ΠΎΠ΄Π½ΠΎΡ€ΠΎΠ΄Π½ΠΎΠΌ ΠΌΠ°Π³Π½ΠΈΡ‚Π½ΠΎΠΌ ΠΏΠΎΠ»Π΅ с ΠΈΠ½Π΄ΡƒΠΊΡ†ΠΈΠ΅ΠΉ 1 Π’Π» находится прямой ΠΏΡ€ΠΎΠ²ΠΎΠ΄Π½ΠΈΠΊ Π΄Π»ΠΈΠ½ΠΎΠΉ 20 см. ΠšΠΎΠ½Ρ†Ρ‹ ΠΏΡ€ΠΎΠ²ΠΎΠ΄Π½ΠΈΠΊΠ° Π·Π°ΠΌΠΊΠ½ΡƒΡ‚Ρ‹ ΠΏΡ€ΠΎΠ²ΠΎΠ΄ΠΎΠΌ, находящСмся Π²Π½Π΅ поля. Π‘ΠΎΠΏΡ€ΠΎΡ‚ΠΈΠ²Π»Π΅Π½ΠΈΠ΅ всСй Ρ†Π΅ΠΏΠΈ 0,1 Ом. Найти силу, ΠΊΠΎΡ‚ΠΎΡ€ΡƒΡŽ Π½ΡƒΠΆΠ½ΠΎ ΠΏΡ€ΠΈΠ»ΠΎΠΆΠΈΡ‚ΡŒ ΠΊ ΠΏΡ€ΠΎΠ²ΠΎΠ΄Π½ΠΈΠΊΡƒ, Ρ‡Ρ‚ΠΎΠ±Ρ‹ ΠΏΠ΅Ρ€Π΅ΠΌΠ΅Ρ‰Π°Ρ‚ΡŒ Π΅Π³ΠΎ пСрпСнди­кулярно линиям ΠΈΠ½Π΄ΡƒΠΊΡ†ΠΈΠΈ со ΡΠΊΠΎΡ€ΠΎΡΡ‚ΡŒΡŽ 2,5 ΠΌ/с.

  • Π”Π²Π° мСталличСских стСрТня располоТСны Π²Π΅Ρ€Ρ‚ΠΈΠΊΠ°Π»ΡŒΠ½ΠΎ ΠΈ Π·Π°ΠΌΠΊΠ½ΡƒΡ‚Ρ‹ Π²Π²Π΅Ρ€Ρ…Ρƒ ΠΏΡ€ΠΎΠ²ΠΎΠ΄Π½ΠΈΠΊΠΎΠΌ. По этим стСрТням Π±Π΅Π· трСния ΠΈ Π½Π°Ρ€ΡƒΡˆΠ΅Π½ΠΈΡ ΠΊΠΎΠ½Β­Ρ‚Π°ΠΊΡ‚Π° ΡΠΊΠΎΠ»ΡŒΠ·ΠΈΡ‚ ΠΏΠ΅Ρ€Π΅ΠΌΡ‹Ρ‡ΠΊΠ° Π΄Π»ΠΈΠ½ΠΎΠΉ 0,5 см ΠΈ массой 1 Π³. Вся систСма нахо­дится Π² ΠΎΠ΄Π½ΠΎΡ€ΠΎΠ΄Π½ΠΎΠΌ ΠΌΠ°Π³Π½ΠΈΡ‚Π½ΠΎΠΌ ΠΏΠΎΠ»Π΅ с ΠΈΠ½Π΄ΡƒΠΊΡ†ΠΈΠ΅ΠΉ 10-2 Π’Π», пСрпСндику­лярной плоскости Ρ€Π°ΠΌΠΊΠΈ. Π£ΡΡ‚Π°Π½ΠΎΠ²ΠΈΠ²ΡˆΠ°ΡΡΡ ΡΠΊΠΎΡ€ΠΎΡΡ‚ΡŒ 1 ΠΌ/с. Найти сопро­тивлСниС ΠΏΠ΅Ρ€Π΅ΠΌΡ‹Ρ‡ΠΊΠΈ. Π‘ΠΎΠΏΡ€ΠΎΡ‚ΠΈΠ²Π»Π΅Π½ΠΈΠ΅ΠΌ стСрТня ΠΈ ΠΏΡ€ΠΎΠ²ΠΎΠ΄Π° ΠΌΠΎΠΆΠ½ΠΎ ΠΏΡ€Π΅Β­Π½Π΅Π±Ρ€Π΅Ρ‡ΡŒ.

  • Π’ ΠΌΠ°Π³Π½ΠΈΡ‚Π½ΠΎΠΌ ΠΏΠΎΠ»Π΅ с ΠΈΠ½Π΄ΡƒΠΊΡ†ΠΈΠ΅ΠΉ 0,05 Π’Π» с постоянной ΡƒΠ³Π»ΠΎΠ²ΠΎΠΉ ΡΠΊΠΎΡ€ΠΎΡΡ‚ΡŒΡŽ 20 Ρ€Π°Π΄/с вращаСтся ΡΡ‚Π΅Ρ€ΠΆΠ΅Π½ΡŒ Π΄Π»ΠΈΠ½ΠΎΠΉ 1 ΠΌ. Ось вращСния ΠΏΡ€ΠΎΡ…ΠΎΠ΄ΠΈΡ‚ Ρ‡Π΅Ρ€Π΅Π· ΠΊΠΎΠ½Π΅Ρ† стСрТня ΠΈ ΠΏΠ°Ρ€Π°Π»Π»Π΅Π»ΡŒΠ½Π° силовым линиям ΠΌΠ°Π³Π½ΠΈΡ‚Π½ΠΎΠ³ΠΎ поля. Найти Π­Π”Π‘ ΠΈΠ½Π΄ΡƒΠΊΡ†ΠΈΠΈ, Π²ΠΎΠ·Π½ΠΈΠΊΠ°ΡŽΡ‰ΡƒΡŽ Π½Π° ΠΊΠΎΠ½Ρ†Π°Ρ… стСрТня.

  • Π‘ ΠΊΠ°ΠΊΠΎΠΉ ΡƒΠ³Π»ΠΎΠ²ΠΎΠΉ ΡΠΊΠΎΡ€ΠΎΡΡ‚ΡŒΡŽ Π½Π°Π΄ΠΎ Π²Ρ€Π°Ρ‰Π°Ρ‚ΡŒ прямой ΠΏΡ€ΠΎΠ²ΠΎΠ΄Π½ΠΈΠΊ Π΄Π»ΠΈΠ½Π½ΠΎΠΉ 20 см Π²ΠΎΠΊΡ€ΡƒΠ³ ΠΎΠ΄Π½ΠΎΠ³ΠΎ ΠΈΠ· Π΅Π³ΠΎ ΠΊΠΎΠ½Ρ†ΠΎΠ² Π² плоскости, пСрпСндикулярной ΠΊ линиям ΠΈΠ½Π΄ΡƒΠΊΡ†ΠΈΠΈ ΠΎΠ΄Π½ΠΎΡ€ΠΎΠ΄Π½ΠΎΠ³ΠΎ ΠΌΠ°Π³Π½ΠΈΡ‚Π½ΠΎΠ³ΠΎ поля, Ρ‡Ρ‚ΠΎΠ±Ρ‹ Π² ΠΏΡ€ΠΎΠ²ΠΎΠ΄Π½ΠΈΠΊΠ΅ ΠΈΠ½Β­Π΄ΡƒΡ†ΠΈΡ€ΠΎΠ²Π°Π»Π°ΡΡŒ Π­Π”Π‘, равная 0,3 Π’? ΠœΠ°Π³Π½ΠΈΡ‚Π½Π°Ρ индукция поля 0,2 Π’Π».

  • Π“ΠΎΡ€ΠΈΠ·ΠΎΠ½Ρ‚Π°Π»ΡŒΠ½Ρ‹ΠΉ ΡΡ‚Π΅Ρ€ΠΆΠ΅Π½ΡŒ Π΄Π»ΠΈΠ½ΠΎΠΉ 1 ΠΌ вращаСтся Π²ΠΎΠΊΡ€ΡƒΠ³ Π²Π΅Ρ€Ρ‚ΠΈΠΊΠ°Π»ΡŒΠ½ΠΎΠΉ оси, проходящСй Ρ‡Π΅Ρ€Π΅Π· ΠΎΠ΄Π½ΠΈ ΠΈΠ· Π΅Π³ΠΎ ΠΊΠΎΠ½Ρ†ΠΎΠ². Ось вращСния ΠΏΠ°Ρ€Π°Π»Π»Π΅Π»ΡŒΠ½Π° ΠΌΠ°Π³Π½ΠΈΡ‚Π½ΠΎΠΌΡƒ полю, индукция ΠΊΠΎΡ‚ΠΎΡ€ΠΎΠ³ΠΎ 50 ΠΌΠΊΠ’Π». ΠŸΡ€ΠΈ ΠΊΠ°ΠΊΠΎΠΉ частотС вращСния стСрТня Ρ€Π°Π·Π½ΠΎΡΡ‚ΡŒ ΠΏΠΎΡ‚Π΅Π½Ρ†ΠΈΠ°Π»ΠΎΠ² Π½Π° ΠΊΠΎΠ½Ρ†Π°Ρ… этого стСрТня составит 1 ΠΌΠ’?

  • Π“ΠΎΡ€ΠΈΠ·ΠΎΠ½Ρ‚Π°Π»ΡŒΠ½Ρ‹ΠΉ ΡΡ‚Π΅Ρ€ΠΆΠ΅Π½ΡŒ Π΄Π»ΠΈΠ½ΠΎΠΉ 0,5 ΠΌ вращаСтся с частотой 5 ΠΎΠ±/с Π²ΠΎΒ­ΠΊΡ€ΡƒΠ³ Π²Π΅Ρ€Ρ‚ΠΈΠΊΠ°Π»ΡŒΠ½ΠΎΠΉ оси, проходящСй Ρ‡Π΅Ρ€Π΅Π· ΠΎΠ΄ΠΈΠ½ ΠΈΠ· Π΅Π³ΠΎ ΠΊΠΎΠ½Ρ†ΠΎΠ². Ось вращСния ΠΏΠ°Ρ€Π°Π»Π»Π΅Π»ΡŒΠ½Π° Π²Π΅ΠΊΡ‚ΠΎΡ€Ρƒ ΠΈΠ½Π΄ΡƒΠΊΡ†ΠΈΠΈ ΠΎΠ΄Π½ΠΎΡ€ΠΎΠ΄Π½ΠΎΠ³ΠΎ ΠΌΠ°Π³Π½ΠΈΡ‚Π½ΠΎΠ³ΠΎ поля. Какова Π²Π΅Π»ΠΈΡ‡ΠΈΠ½Π° ΠΌΠ°Π³Π½ΠΈΡ‚Π½ΠΎΠΉ ΠΈΠ½Π΄ΡƒΠΊΡ†ΠΈΠΈ, Ссли Ρ€Π°Π·Π½ΠΎΡΡ‚ΡŒ ΠΏΠΎΡ‚Π΅Π½Ρ†ΠΈΠ°Π»ΠΎΠ² Π½Π° ΠΊΠΎΠ½Ρ†Π°Ρ… стСрТня составляСт 5 ΠΌΠ’?

  • Π’ ΠΌΠ°Π³Π½ΠΈΡ‚Π½ΠΎΠΌ ΠΏΠΎΠ»Π΅ с постоянной ΡƒΠ³Π»ΠΎΠ²ΠΎΠΉ ΡΠΊΠΎΡ€ΠΎΡΡ‚ΡŒΡŽ 30 Ρ€Π°Π΄/с вращаСтся ΡΡ‚Π΅Ρ€ΠΆΠ΅Π½ΡŒ Π΄Π»ΠΈΠ½ΠΎΠΉ 25 см. Ось вращСния ΠΏΡ€ΠΎΡ…ΠΎΠ΄ΠΈΡ‚ Ρ‡Π΅Ρ€Π΅Π· ΠΊΠΎΠ½Π΅Ρ† стСрТня ΠΈ ΠΏΠ°Ρ€Π°Π»Π»Π΅Π»ΡŒΠ½Π° силовым линиям ΠΌΠ°Π³Π½ΠΈΡ‚Π½ΠΎΠ³ΠΎ поля. Какова Π½Π°ΠΏΡ€ΡΠΆΠ΅Π½Β­Π½ΠΎΡΡ‚ΡŒ ΠΌΠ°Π³Π½ΠΈΡ‚Π½ΠΎΠ³ΠΎ поля, Ссли Π­Π”Π‘ ΠΈΠ½Π΄ΡƒΠΊΡ†ΠΈΠΈ, Π²ΠΎΠ·Π½ΠΈΠΊΠ°ΡŽΡ‰Π°Ρ Π½Π° ΠΊΠΎΠ½Ρ†Π°Ρ… стСрТня, Ρ€Π°Π²Π½Π° 1 ΠΌΠ’?

  • Π’ ΠΎΠ΄Π½ΠΎΡ€ΠΎΠ΄Π½ΠΎΠΌ ΠΌΠ°Π³Π½ΠΈΡ‚Π½ΠΎΠΌ ΠΏΠΎΠ»Π΅ с ΠΈΠ½Π΄ΡƒΠΊΡ†ΠΈΠ΅ΠΉ 0,5 Π’Π» Ρ€Π°Π²Π½ΠΎΠΌΠ΅Ρ€Π½ΠΎ двиТСтся ΠΏΡ€ΠΎΠ²ΠΎΠ΄Π½ΠΈΠΊ Π΄Π»ΠΈΠ½ΠΎΠΉ 10 см. По ΠΏΡ€ΠΎΠ²ΠΎΠ΄Π½ΠΈΠΊΡƒ Ρ‚Π΅Ρ‡Π΅Ρ‚ Ρ‚ΠΎΠΊ силой 2 А. Π‘ΠΊΠΎΡ€ΠΎΡΡ‚ΡŒ двиТСния ΠΏΡ€ΠΎΠ²ΠΎΠ΄Π½ΠΈΠΊΠ° 20 см/с ΠΈ Π½Π°ΠΏΡ€Π°Π²Π»Π΅Π½Π° пСрпСндикулярно ΠΈΠ½Β­Π΄ΡƒΠΊΡ†ΠΈΠΈ ΠΌΠ°Π³Π½ΠΈΡ‚Π½ΠΎΠ³ΠΎ поля. Найти Ρ€Π°Π±ΠΎΡ‚Ρƒ пСрСмСщСния ΠΏΡ€ΠΎΠ²ΠΎΠ΄Π½ΠΈΠΊΠ° Π·Π° врСмя 10 с ΠΈ ΠΌΠΎΡ‰Π½ΠΎΡΡ‚ΡŒ, Π·Π°Ρ‚Ρ€Π°Ρ‡Π΅Π½Π½ΡƒΡŽ Π½Π° это ΠΏΠ΅Ρ€Π΅ΠΌΠ΅Ρ‰Π΅Π½ΠΈΠ΅.

  • ΠŸΡ€ΡΠΌΠΎΠΉ ΠΏΡ€ΠΎΠ²ΠΎΠ΄Π½ΠΈΠΊ Π΄Π»ΠΈΠ½ΠΎΠΉ 10 см ΠΏΠΎΠΌΠ΅Ρ‰Π΅Π½ Π² ΠΎΠ΄Π½ΠΎΡ€ΠΎΠ΄Π½ΠΎΠ΅ ΠΌΠ°Π³Π½ΠΈΡ‚Π½ΠΎΠ΅ ΠΏΠΎΠ»Π΅ с ΠΈΠ½Π΄ΡƒΠΊΡ†ΠΈΠ΅ΠΉ 1 Π’Π». ΠšΠΎΠ½Ρ†Ρ‹ ΠΏΡ€ΠΎΠ²ΠΎΠ΄Π½ΠΈΠΊΠ° Π·Π°ΠΌΠΊΠ½ΡƒΡ‚Ρ‹ Π³ΠΈΠ±ΠΊΠΈΠΌ ΠΏΡ€ΠΎΠ²ΠΎΠ΄ΠΎΠΌ, находящимся Π²Π½Π΅ поля. Π‘ΠΎΠΏΡ€ΠΎΡ‚ΠΈΠ²Π»Π΅Π½ΠΈΠ΅ всСй Ρ†Π΅ΠΏΠΈ 0,4 Ом. Какая ΠΌΠΎΡ‰Π½ΠΎΡΡ‚ΡŒ трСбуСтся для Ρ‚ΠΎΠ³ΠΎ, Ρ‡Ρ‚ΠΎΠ±Ρ‹ Π΄Π²ΠΈΠ³Π°Ρ‚ΡŒ ΠΏΡ€ΠΎΠ²ΠΎΠ΄Π½ΠΈΠΊ пСрпСндикулярно линиям ΠΈΠ½Π΄ΡƒΠΊΡ†ΠΈΠΈ ΠΌΠ°Π³Π½ΠΈΡ‚Π½ΠΎΠ³ΠΎ поля со ΡΠΊΠΎΡ€ΠΎΡΡ‚ΡŒΡŽ 20 см/с?

  • ΠŸΡ€ΡΠΌΠΎΠ»ΠΈΠ½Π΅ΠΉΠ½Ρ‹ΠΉ ΠΏΡ€ΠΎΠ²ΠΎΠ΄Π½ΠΈΠΊ Π΄Π»ΠΈΠ½ΠΎΠΉ 8 ΠΌ двиТСтся ΠΏΠΎΠ΄ ΡƒΠ³Π»ΠΎΠΌ 30Β° ΠΊ Π³ΠΎΡ€ΠΈΠ·ΠΎΠ½Ρ‚Ρƒ Π² Π²Π΅Ρ€Ρ‚ΠΈΠΊΠ°Π»ΡŒΠ½ΠΎΠΌ ΠΌΠ°Π³Π½ΠΈΡ‚Π½ΠΎΠΌ ΠΏΠΎΠ»Π΅ со ΡΠΊΠΎΡ€ΠΎΡΡ‚ΡŒΡŽ 200 ΠΌ/с. Π£Π³ΠΎΠ» ΠΌΠ΅ΠΆΠ΄Ρƒ ΠΏΡ€ΠΎΠ²ΠΎΠ΄Π½ΠΈΠΊΠΎΠΌ ΠΈ линиями ΠΌΠ°Π³Π½ΠΈΡ‚Π½ΠΎΠΉ ΠΈΠ½Π΄ΡƒΠΊΡ†ΠΈΠΈ Ρ€Π°Π²Π΅Π½ 90Β°. ΠŸΡ€ΠΈ этом Π² ΠΏΡ€ΠΎΠ²ΠΎΠ΄Π½ΠΈΠΊΠ΅ возбуТдаСтся Π­Π”Π‘ ΠΈΠ½Π΄ΡƒΠΊΡ†ΠΈΠΈ 20 Π’. ΠžΠΏΡ€Π΅Π΄Π΅Π»ΠΈΡ‚ΡŒ ΠΈΠ½Π΄ΡƒΠΊΡ†ΠΈΡŽ ΠΌΠ°Π³Π½ΠΈΡ‚Π½ΠΎΠ³ΠΎ поля, Π° Ρ‚Π°ΠΊ ΠΆΠ΅ Ρ€Π°Π±ΠΎΡ‚Ρƒ, ΠΊΠΎΡ‚ΠΎΡ€ΡƒΡŽ ΡΠΎΠ²Π΅Ρ€ΡˆΠ°ΡŽΡ‚ силы ΠΌΠ°Π³Π½ΠΈΡ‚Π½ΠΎΠ³ΠΎ поля Π·Π° 1 ΠΌΠΈΠ½ΡƒΡ‚Ρƒ, Ссли сопротивлСниС двиТущСгося ΠΏΡ€ΠΎΠ²ΠΎΠ΄Π½ΠΈΠΊΠ° 10 Ом ΠΈ этот ΠΏΡ€ΠΎΠ²ΠΎΠ΄Π½ΠΈΠΊ Π·Π°ΠΌΠΊΠ½ΡƒΡ‚ Π΄Ρ€ΡƒΠ³ΠΈΠΌ ΠΏΡ€ΠΎΠ²ΠΎΠ΄Π½ΠΈΠΊΠΎΠΌ, сопротивлСниСм ΠΊΠΎΡ‚ΠΎΡ€ΠΎΠ³ΠΎ ΠΌΠΎΠΆΠ½ΠΎ ΠΏΡ€Π΅Π½Π΅Π±Ρ€Π΅Ρ‡ΡŒ.

  • Π’ ΠΌΠ°Π³Π½ΠΈΡ‚Π½ΠΎΠΌ ΠΏΠΎΠ»Π΅ с ΠΈΠ½Π΄ΡƒΠΊΡ†ΠΈΠ΅ΠΉ 1 Π’Π» двиТСтся Ρ€Π°Π²Π½ΠΎΠΌΠ΅Ρ€Π½ΠΎ ΠΏΡ€ΠΎΠ²ΠΎΠ΄Π½ΠΈΠΊ Π΄Π»ΠΈΠ½ΠΎΠΉ 0,5 ΠΌ. Π’ΠΎΠΊ Π² ΠΏΡ€ΠΎΠ²ΠΎΠ΄Π½ΠΈΠΊΠ΅ составляСт 1 А. ΠœΠΎΡ‰Π½ΠΎΡΡ‚ΡŒ, затрачи­ваСмая Π½Π° ΠΏΠ΅Ρ€Π΅ΠΌΠ΅Ρ‰Π΅Π½ΠΈΠ΅ ΠΏΡ€ΠΎΠ²ΠΎΠ΄Π½ΠΈΠΊΠ° Ρ€Π°Π²Π½Π° 0,02 Π’Ρ‚. Π§Π΅ΠΌΡƒ Ρ€Π°Π²Π½Π° ΡΠΊΠΎΡ€ΠΎΡΡ‚ΡŒ двиТСния ΠΏΡ€ΠΎΠ²ΠΎΠ΄Π½ΠΈΠΊΠ°, Ссли извСстно, Ρ‡Ρ‚ΠΎ ΠΎΠ½Π° Π½Π°ΠΏΡ€Π°Π²Π»Π΅Π½Π° пСрпСндикулярно линиям ΠΌΠ°Π³Π½ΠΈΡ‚Π½ΠΎΠΉ ΠΈΠ½Π΄ΡƒΠΊΡ†ΠΈΠΈ?

  • ΠŸΡ€ΡΠΌΠΎΠΉ ΠΏΡ€ΠΎΠ²ΠΎΠ΄Π½ΠΈΠΊ находится Π² ΠΎΠ΄Π½ΠΎΡ€ΠΎΠ΄Π½ΠΎΠΌ ΠΌΠ°Π³Π½ΠΈΡ‚Π½ΠΎΠΌ ΠΏΠΎΠΏΠ΅ с Π½Π°Β­ΠΏΡ€ΡΠΆΠ΅Π½Π½ΠΎΡΡ‚ΡŒΡŽ 8Β·106 А/ΠΌ. ΠšΠΎΠ½Ρ†Ρ‹ ΠΏΡ€ΠΎΠ²ΠΎΠ΄Π½ΠΈΠΊΠ° Π·Π°ΠΌΠΊΠ½ΡƒΡ‚Ρ‹ Π³ΠΈΠ±ΠΊΠΈΠΌ ΠΏΡ€ΠΎΠ²ΠΎΠ·ΠΎΠΌ, находящимся Π²Π½Π΅ поля. Π‘ΠΎΠΏΡ€ΠΎΡ‚ΠΈΠ²Π»Π΅Π½ΠΈΠ΅ всСй Ρ†Π΅ΠΏΠΈ 1 Ом. Какова Π΄Π»ΠΈΠ½Π° ΠΏΡ€ΠΎΠ²ΠΎΠ΄Π½ΠΈΠΊΠ°, Ссли ΠΌΠΎΡ‰Π½ΠΎΡΡ‚ΡŒ, затрачиваСмая Π½Π° Π΅Π³ΠΎ ΠΏΠ΅Ρ€Π΅Π΄Π²ΠΈΠΆΠ΅Π½ΠΈΠ΅, составляСт 10 ΠΌΠ’Ρ‚, Π‘ΠΊΠΎΡ€ΠΎΡΡ‚ΡŒ Ρ€Π°Π²Π½Π° 10 см/с ΠΈ пСрпСндикулярна линиям поля.

  • ΠŸΡ€ΡΠΌΠΎΠΉ ΠΏΡ€ΠΎΠ²ΠΎΠ΄Π½ΠΈΠΊ Π΄Π»ΠΈΠ½ΠΎΠΉ 1 ΠΌ ΠΏΠΎΠΌΠ΅Ρ‰Π΅Π½ Π² ΠΎΠ΄Π½ΠΎΡ€ΠΎΠ΄Π½ΠΎΠ΅ ΠΌΠ°Π³Π½ΠΈΡ‚Π½ΠΎΠ΅ ΠΏΠΎΠ»Π΅ с ΠΈΠ½Π΄ΡƒΠΊΡ†ΠΈΠ΅ΠΉ 0,5 Π’Π». ΠšΠΎΠ½Ρ†Ρ‹ ΠΏΡ€ΠΎΠ²ΠΎΠ΄Π½ΠΈΠΊΠ° Π·Π°ΠΌΠΊΠ½ΡƒΡ‚Ρ‹ Π³ΠΈΠ±ΠΊΠΈΠΌ ΠΏΡ€ΠΎΠ²ΠΎΠ΄ΠΎΠΌ, находящимся Π²Π½Π΅ поля. Каково сопротивлСниС Ρ†Π΅ΠΏΠΈ, Ссли ΠΌΠΎΡ‰Π½ΠΎΡΡ‚ΡŒ, затрачиваСмая Π½Π° ΠΏΠ΅Ρ€Π΅ΠΌΠ΅Ρ‰Π΅Π½ΠΈΠ΅ ΠΏΡ€ΠΎΠ²ΠΎΠ΄Π½ΠΈΠΊΠ° пСрпСндикулярно линиям поля со ΡΠΊΠΎΡ€ΠΎΡΡ‚ΡŒΡŽ 20 см/с, Ρ€Π°Π²Π½Π° 0,5 Π’Ρ‚?

  • Π’ ΠΎΠ΄Π½ΠΎΡ€ΠΎΠ΄Π½ΠΎΠΌ ΠΌΠ°Π³Π½ΠΈΡ‚Π½ΠΎΠΌ ΠΏΠΎΠ»Π΅ с ΠΈΠ½Π΄ΡƒΠΊΡ†ΠΈΠ΅ΠΉ 2 Π’Π» двиТСтся ΠΏΡ€ΠΎΠ²ΠΎΠ΄Π½ΠΈΠΊ Π΄Π»ΠΈΠ½ΠΎΠΉ 20 см. По ΠΏΡ€ΠΎΠ²ΠΎΠ΄Π½ΠΈΠΊΡƒ Ρ‚Π΅Ρ‡Π΅Ρ‚ Ρ‚ΠΎΠΊ. Π‘ΠΊΠΎΡ€ΠΎΡΡ‚ΡŒ двиТСния ΠΏΡ€ΠΎΠ²ΠΎΠ΄Π½ΠΈΠΊΠ° 10 см/с ΠΈ Π½Π°ΠΏΡ€Π°Π²Π»Π΅Π½Π° пСрпСндикулярно ΠΈΠ½Π΄ΡƒΠΊΡ†ΠΈΠΈ ΠΌΠ°Π³Π½ΠΈΡ‚Π½ΠΎΠ³ΠΎ поля. Найти Ρ€Π°Π±ΠΎΡ‚Ρƒ пСрСмСщСния ΠΏΡ€ΠΎΠ²ΠΎΠ΄Π½ΠΈΠΊΠ° Π·Π° 5 с. Π‘ΠΎΠΏΡ€ΠΎΡ‚ΠΈΠ²Π»Π΅Π½ΠΈΠ΅ ΠΏΡ€ΠΎΠ²ΠΎΠ΄Π½ΠΈΠΊΠ° Ρ€Π°Π²Π½ΠΎ 0,5 Ом.

  • ΠšΠ²Π°Π΄Ρ€Π°Ρ‚Π½Π°Ρ Ρ€Π°ΠΌΠΊΠ° со стороной 10 см, ΠΏΠΎ ΠΊΠΎΡ‚ΠΎΡ€ΠΎΠΉ Ρ‚Π΅Ρ‡Π΅Ρ‚ Ρ‚ΠΎΠΊ силой 10 А, свободно ΡƒΡΡ‚Π°Π½ΠΎΠ²ΠΈΠ»Π°ΡΡŒ Π² ΠΎΠ΄Π½ΠΎΡ€ΠΎΠ΄Π½ΠΎΠΌ ΠΌΠ°Π³Π½ΠΈΡ‚Π½ΠΎΠΌ ΠΏΠΎΠ»Π΅ с ΠΈΠ½Π΄ΡƒΠΊΡ†ΠΈΠ΅ΠΉ 0,2 Π’Π». ΠšΠ°ΠΊΡƒΡŽ Ρ€Π°Π±ΠΎΡ‚Ρƒ Π½ΡƒΠΆΠ½ΠΎ ΡΠΎΠ²Π΅Ρ€ΡˆΠΈΡ‚ΡŒ, Ρ‡Ρ‚ΠΎΠ±Ρ‹ ΠΏΠΎΠ²Π΅Ρ€Π½ΡƒΡ‚ΡŒ Ρ€Π°ΠΌΠΊΡƒ Π½Π° ΡƒΠ³ΠΎΠ» 60Β° Π²ΠΎΠΊΡ€ΡƒΠ³ оси, ΡΠΎΠ²ΠΏΠ°Π΄Π°ΡŽΡ‰Π΅ΠΉ с ΠΎΠ΄Π½ΠΎΠΉ ΠΈΠ· Π΅Π΅ сторон?

  • Π’ΠΈΡ‚ΠΎΠΊ, ΠΏΠΎ ΠΊΠΎΡ‚ΠΎΡ€ΠΎΠΌΡƒ Ρ‚Π΅Ρ‡Π΅Ρ‚ Ρ‚ΠΎΠΊ силой 20 А, свободно установился Π² ΠΎΠ΄Π½ΠΎΡ€ΠΎΠ΄Π½ΠΎΠΌ ΠΌΠ°Π³Π½ΠΈΡ‚Π½ΠΎΠΌ ΠΏΠΎΠ»Π΅ с ΠΈΠ½Π΄ΡƒΠΊΡ†ΠΈΠ΅ΠΉ 0,016 Π’Π». Π”ΠΈΠ°ΠΌΠ΅Ρ‚Ρ€ Π²ΠΈΡ‚ΠΊΠ° 10 см. ΠšΠ°ΠΊΡƒΡŽ Ρ€Π°Π±ΠΎΡ‚Ρƒ Π½ΡƒΠΆΠ½ΠΎ ΡΠΎΠ²Π΅Ρ€ΡˆΠΈΡ‚ΡŒ, Ρ‡Ρ‚ΠΎΠ±Ρ‹ ΠΏΠΎΠ²Π΅Ρ€Π½ΡƒΡ‚ΡŒ Π²ΠΈΡ‚ΠΎΠΊ Π½Π° ΡƒΠ³ΠΎΠ» 90Β° ΠΎΡ‚Π½ΠΎΡΠΈΡ‚Π΅Π»ΡŒΠ½ΠΎ оси, ΡΠΎΠ²ΠΏΠ°Π΄Π°ΡŽΡ‰Π΅ΠΉ с Π΄ΠΈΠ°ΠΌΠ΅Ρ‚Ρ€ΠΎΠΌ?

  • По ΠΊΠΎΠ»ΡŒΡ†Ρƒ, сдСланному ΠΈΠ· Ρ‚ΠΎΠ½ΠΊΠΎΠ³ΠΎ Π³ΠΈΠ±ΠΊΠΎΠ³ΠΎ ΠΏΡ€ΠΎΠ²ΠΎΠ΄Π½ΠΈΠΊΠ° радиусом 10 см, Ρ‚Π΅Ρ‡Π΅Ρ‚ Ρ‚ΠΎΠΊ 100 А. ΠŸΠ΅Ρ€ΠΏΠ΅Π½Π΄ΠΈΠΊΡƒΠ»ΡΡ€Π½ΠΎ плоскости ΠΊΠΎΠ»ΡŒΡ†Π° Π²ΠΎΠ·Π±ΡƒΠΆΠ΄Π΅Π½ΠΎ ΠΌΠ°Π³Π½ΠΈΡ‚Π½ΠΎΠ΅ ΠΏΠΎΠ»Π΅ с ΠΈΠ½Π΄ΡƒΠΊΡ†ΠΈΠ΅ΠΉ 0,1 Π’Π». БобствСнноС ΠΌΠ°Π³Π½ΠΈΡ‚Π½ΠΎΠ΅ ΠΏΠΎΠ»Π΅ ΠΊΠΎΠ»ΡŒΡ†Π° ΠΈ внСшнСС ΠΏΠΎΠ»Π΅ ΡΠΎΠ²ΠΏΠ°Π΄Π°ΡŽΡ‚ ΠΏΠΎ Π½Π°ΠΏΡ€Π°Π²Π»Π΅Π½ΠΈΡŽ. ΠžΠΏΡ€Π΅Π΄Π΅Π»ΠΈΡ‚ΡŒ Ρ€Π°Π±ΠΎΡ‚Ρƒ Π²Π½Π΅ΡˆΠ½ΠΈΡ… сил, ΠΊΠΎΡ‚ΠΎΡ€Ρ‹Π΅, дСйствуя Π½Π° ΠΏΡ€ΠΎΠ²ΠΎΠ΄Π½ΠΈΠΊ, Π΄Π΅Ρ„ΠΎΡ€ΠΌΠΈΡ€ΠΎΠ²Π°Π»ΠΈ Π΅Π³ΠΎ ΠΈ ΠΏΡ€ΠΈΠ΄Π°Π»ΠΈ Ρ„ΠΎΡ€ΠΌΡƒ ΠΊΠ²Π°Π΄Ρ€Π°Ρ‚Π°. Π Π°Π±ΠΎΡ‚ΠΎΠΉ ΠΏΡ€ΠΎΡ‚ΠΈΠ² ΡƒΠΏΡ€ΡƒΠ³ΠΈΡ… сил ΠΏΡ€Π΅Π½Π΅Π±Ρ€Π΅Ρ‡ΡŒ.

  • Π Π°ΠΌΠΊΠ° с Ρ‚ΠΎΠΊΠΎΠΌ 10 А Π²Ρ‹ΠΏΠΎΠ»Π½Π΅Π½Π° ΠΈΠ· Π³ΠΈΠ±ΠΊΠΎΠ³ΠΎ ΠΏΡ€ΠΎΠ²ΠΎΠ΄Π½ΠΈΠΊΠ° ΠΈ ΠΈΠΌΠ΅Π΅Ρ‚ Ρ„ΠΎΡ€ΠΌΡƒ равностороннСго Ρ‚Ρ€Π΅ΡƒΠ³ΠΎΠ»ΡŒΠ½ΠΈΠΊΠ° со стороной 6 см. ΠŸΠ΅Ρ€ΠΏΠ΅Π½Π΄ΠΈΠΊΡƒΠ»ΡΡ€Π½ΠΎ плоскости Ρ€Π°ΠΌΠΊΠΈ Π²ΠΎΠ·Π±ΡƒΠΆΠ΄Π΅Π½ΠΎ ΠΌΠ°Π³Π½ΠΈΡ‚Π½ΠΎΠ΅ ΠΏΠΎΠ»Π΅ с Π½Π°ΠΏΡ€ΡΠΆΠ΅Π½Π½ΠΎΡΡ‚ΡŒΡŽ 200 А/ΠΌ. ΠžΠΏΡ€Π΅Π΄Π΅Π»ΠΈΡ‚ΡŒ Ρ€Π°Π±ΠΎΡ‚Ρƒ Π²Π½Π΅ΡˆΠ½ΠΈΡ… сил, ΠΊΠΎΡ‚ΠΎΡ€Ρ‹Π΅, дСйствуя Π½Π° ΠΏΡ€ΠΎΠ²ΠΎΠ΄Π½ΠΈΠΊ, ΠΏΡ€ΠΈΠ΄Π°Π»ΠΈ ΡƒΠΌΡƒ Ρ„ΠΎΡ€ΠΌΡƒ ΠΊΠ²Π°Π΄Ρ€Π°Ρ‚Π°, Π Π°Π±ΠΎΡ‚ΠΎΠΉ ΠΏΡ€ΠΎΡ‚ΠΈΠ² ΡƒΠΏΡ€ΡƒΠ³ΠΈΡ… сил ΠΏΡ€Π΅Π½Π΅Π±Ρ€Π΅Ρ‡ΡŒ.

  • По ΠΏΡ€ΠΎΠ²ΠΎΠ΄Π½ΠΈΠΊΡƒ, согнутому Π² Π²ΠΈΠ΄Π΅ ΠΊΠ²Π°Π΄Ρ€Π°Ρ‚Π° со стороной 10 см, Ρ‚Π΅Ρ‡Π΅Ρ‚ Ρ‚ΠΎΠΊ 20 А, Π²Π΅Π»ΠΈΡ‡ΠΈΠ½Π° ΠΊΠΎΡ‚ΠΎΡ€ΠΎΠ³ΠΎ поддСрТиваСтся Π½Π΅ΠΈΠ·ΠΌΠ΅Π½Π½ΠΎΠΉ. ΠŸΠ»ΠΎΡΠΊΠΎΡΡ‚ΡŒ ΠΊΠ²Π°Π΄Ρ€Π°Ρ‚Π° составляСт ΡƒΠ³ΠΎΠ» 20Β° с линиями ΠΎΠ΄Π½ΠΎΡ€ΠΎΠ΄Π½ΠΎΠ³ΠΎ ΠΌΠ°Π³Π½ΠΈΡ‚Π½ΠΎΠ³ΠΎ поля Π’ = 0,1 Π’Π». Π’Ρ‹Ρ‡ΠΈΡΠ»ΠΈΡ‚ΡŒ Ρ€Π°Π±ΠΎΡ‚Ρƒ, ΠΊΠΎΡ‚ΠΎΡ€ΡƒΡŽ Π½Π΅ΠΎΠ±Ρ…ΠΎΠ΄ΠΈΠΌΠΎ ΡΠΎΠ²Π΅Ρ€ΡˆΠΈΡ‚ΡŒ для Ρ‚ΠΎΠ³ΠΎ, Ρ‡Ρ‚ΠΎΠ±Ρ‹ ΡƒΠ΄Π°Π»ΠΈΡ‚ΡŒ ΠΏΡ€ΠΎΠ²ΠΎΠ΄Π½ΠΈΠΊ Π·Π° ΠΏΡ€Π΅Π΄Π΅Π»Ρ‹ поля.

  • Плоский ΠΊΠΎΠ½Ρ‚ΡƒΡ€ ΠΏΠ»ΠΎΡ‰Π°Π΄ΡŒΡŽ 300 см2 находится Π² ΠΎΠ΄Π½ΠΎΡ€ΠΎΠ΄Π½ΠΎΠΌ ΠΌΠ°Π³Π½ΠΈΡ‚Π½ΠΎΠΌ ΠΏΠΎΠ»Π΅, индукция ΠΊΠΎΡ‚ΠΎΡ€ΠΎΠ³ΠΎ Ρ€Π°Π²Π½Π° 0,01 Π’Π». ΠŸΠ»ΠΎΡΠΊΠΎΡΡ‚ΡŒ ΠΊΠΎΠ½Ρ‚ΡƒΡ€Π° пСрпСндикулярна линиям поля. Π’ ΠΊΠΎΠ½Ρ‚ΡƒΡ€Π΅ поддСрТиваСтся Π½Π΅ΠΈΠ·ΠΌΠ΅Π½Π½Ρ‹ΠΉ Ρ‚ΠΎΠΊ 1 А. ΠžΠΏΡ€Π΅Π΄Π΅Π»ΠΈΡ‚ΡŒ Ρ€Π°Π±ΠΎΡ‚Ρƒ Π²Π½Π΅ΡˆΠ½ΠΈΡ… сил ΠΏΠΎ ΠΏΠ΅Ρ€Π΅ΠΌΠ΅Ρ‰Π΅Π½ΠΈΡŽ ΠΊΠΎΠ½Ρ‚ΡƒΡ€Π° с Ρ‚ΠΎΠΊΠΎΠΌ Π² ΠΎΠ±Π»Π°ΡΡ‚ΡŒ пространства, Π³Π΄Π΅ ΠΌΠ°Π³Π½ΠΈΡ‚Π½ΠΎΠ΅ ΠΏΠΎΠ»Π΅ отсутствуСт.

  • Π’ΠΈΡ‚ΠΎΠΊ радиусом 10 см, ΠΏΠΎ ΠΊΠΎΡ‚ΠΎΡ€ΠΎΠΌΡƒ Ρ‚Π΅Ρ‡Π΅Ρ‚ Ρ‚ΠΎΠΊ 20 А, ΠΏΠΎΠΌΠ΅Ρ‰Π΅Π½ Π² ΠΌΠ°Π³Π½ΠΈΡ‚Π½ΠΎΠ΅ ΠΏΠΎΠ»Π΅ с ΠΈΠ½Π΄ΡƒΠΊΡ†ΠΈΠ΅ΠΉ 1 Π’Π» Ρ‚Π°ΠΊ, Ρ‡Ρ‚ΠΎ Π΅Π³ΠΎ Π½ΠΎΡ€ΠΌΠ°Π»ΡŒ ΠΎΠ±Ρ€Π°Π·ΡƒΠ΅Ρ‚ ΡƒΠ³ΠΎΠ» 60Β° с Π½Π°Β­ΠΏΡ€Π°Π²Π»Π΅Π½ΠΈΠ΅ΠΌ силовых Π»ΠΈΠ½ΠΈΠΉ. ΠžΠΏΡ€Π΅Π΄Π΅Π»ΠΈΡ‚ΡŒ Ρ€Π°Π±ΠΎΡ‚Ρƒ, ΠΊΠΎΡ‚ΠΎΡ€ΡƒΡŽ Π½ΡƒΠΆΠ½ΠΎ ΡΠΎΠ²Π΅Ρ€ΡˆΠΈΡ‚ΡŒ, Ρ‡Ρ‚ΠΎΠ±Ρ‹ ΡƒΠ΄Π°Π»ΠΈΡ‚ΡŒ Π²ΠΈΡ‚ΠΎΠΊ ΠΈΠ· поля.

  • ΠšΡ€ΡƒΠ³ΠΎΠ²ΠΎΠΉ ΠΊΠΎΠ½Ρ‚ΡƒΡ€ ΠΏΠΎΠΌΠ΅Ρ‰Π΅Π½ Π² ΠΎΠ΄Π½ΠΎΡ€ΠΎΠ΄Π½ΠΎΠ΅ ΠΌΠ°Π³Π½ΠΈΡ‚Π½ΠΎΠ΅ ΠΏΠΎΠ»Π΅ Ρ‚Π°ΠΊ, Ρ‡Ρ‚ΠΎ ΠΏΠ»ΠΎΡΠΊΠΎΡΡ‚ΡŒ ΠΊΠΎΠ½Ρ‚ΡƒΡ€Π° пСрпСндикулярна силовым линиям поля. Π˜Π½Π΄ΡƒΠΊΡ†ΠΈΡ ΠΌΠ°Π³Π½ΠΈΡ‚Π½ΠΎΠ³ΠΎ поля Π’ = 0,2 Π’Π». По ΠΊΠΎΠ½Ρ‚ΡƒΡ€Ρƒ Ρ‚Π΅Ρ‡Π΅Ρ‚ Ρ‚ΠΎΠΊ силой 2 А. Радиус ΠΊΠΎΠ½Ρ‚ΡƒΡ€Π° 2 см. ΠšΠ°ΠΊΡƒΡŽ Ρ€Π°Π±ΠΎΡ‚Ρƒ Π½Π°Π΄ΠΎ ΡΠΎΠ²Π΅Ρ€ΡˆΠΈΡ‚ΡŒ, Ρ‡Ρ‚ΠΎΠ±Ρ‹ ΠΏΠΎΠ²Π΅Ρ€Π½ΡƒΡ‚ΡŒ ΠΊΠΎΠ½Ρ‚ΡƒΡ€ Π½Π° 90Β° Π²ΠΎΠΊΡ€ΡƒΠ³ оси, ΡΠΎΠ²ΠΏΠ°Π΄Π°ΡŽΡ‰Π΅ΠΉ с Π΄ΠΈΠ°ΠΌΠ΅Ρ‚Ρ€ΠΎΠΌ ΠΊΠΎΠ½Ρ‚ΡƒΡ€Π°?

  • Π Π°ΠΌΠΊΠ°, ΠΏΠ»ΠΎΡ‰Π°Π΄ΡŒ ΠΊΠΎΡ‚ΠΎΡ€ΠΎΠΉ 10 см2, вращаСтся Π² ΠΎΠ΄Π½ΠΎΡ€ΠΎΠ΄Π½ΠΎΠΌ ΠΌΠ°Π³Π½ΠΈΡ‚Π½ΠΎΠΌ ΠΏΠΎΠ»Π΅ с частотой 2 ΠΎΠ±/с. Ось вращСния находится Π² плоскости Ρ€Π°ΠΌΠΊΠΈ ΠΈ пСрпСндикулярна силовым линиям ΠΌΠ°Π³Π½ΠΈΡ‚Π½ΠΎΠ³ΠΎ поля, Π½Π°ΠΏΡ€ΡΠΆΠ΅Π½Π½ΠΎΡΡ‚ΡŒ ΠΊΠΎΡ‚ΠΎΡ€ΠΎΠ³ΠΎ 80 кА/ΠΌ. Найти: 1) Π·Π°Π²ΠΈΡΠΈΠΌΠΎΡΡ‚ΡŒ ΠΏΠΎΡ‚ΠΎΠΊΠ° ΠΌΠ°Π³Π½ΠΈΡ‚Π½ΠΎΠΉ ΠΈΠ½Π΄ΡƒΠΊΡ†ΠΈΠΈ, ΠΏΡ€ΠΎΠ½ΠΈΠ·Ρ‹Π²Π°ΡŽΡ‰Π΅Π³ΠΎ Ρ€Π°ΠΌΠΊΡƒ, ΠΎΡ‚ Π²Ρ€Π΅ΠΌΠ΅Π½ΠΈ; 2) наибольшСС Π·Π½Π°Ρ‡Π΅Π½ΠΈΠ΅ Π­Π”Π‘ ΠΈΠ½Π΄ΡƒΠΊΡ†ΠΈΠΈ Π²ΠΎ Π²Ρ€Π°Ρ‰Π°ΡŽΡ‰Π΅ΠΉΡΡ Ρ€Π°ΠΌΠΊΠ΅.

  • ΠžΠΏΡ€Π΅Π΄Π΅Π»ΠΈΡ‚ΡŒ максимальноС Π·Π½Π°Ρ‡Π΅Π½ΠΈΠ΅ Π­Π”Π‘, которая Π²ΠΎΠ·Π½ΠΈΠΊΠ°Π΅Ρ‚ Π² Π·Π°ΠΌΠΊΠ½ΡƒΡ‚ΠΎΠΌ ΠΊΠΎΠ½Ρ‚ΡƒΡ€Π΅ ΠΏΠ»ΠΎΡ‰Π°Π΄ΡŒΡŽ 10 см2, Ρ€Π°Π²Π½ΠΎΠΌΠ΅Ρ€Π½ΠΎ Π²Ρ€Π°Ρ‰Π°ΡŽΡ‰Π΅ΠΌΡΡ с частотой 5 с-1 Π² ΠΌΠ°Π³Π½ΠΈΡ‚Π½ΠΎΠΌ ΠΏΠΎΠ»Π΅ Π½Π°ΠΏΡ€ΡΠΆΠ΅Π½Π½ΠΎΡΡ‚ΡŒΡŽ 10 А/ΠΌ. Ось вращСния Π»Π΅ΠΆΠΈΡ‚ Π² плоско­сти ΠΊΠΎΠ½Ρ‚ΡƒΡ€Π°.

  • Π˜Π½Π΄ΡƒΠΊΡ†ΠΈΡ ΠΌΠ°Π³Π½ΠΈΡ‚Π½ΠΎΠ³ΠΎ поля ΠΌΠ΅ΠΆΠ΄Ρƒ полюсами Π΄Π²ΡƒΡ…ΠΏΠΎΠ»ΡŽΡΠ½ΠΎΠ³ΠΎ Π³Π΅Π½Π΅Ρ€Π°Ρ‚ΠΎΡ€Π° 0,8 Π’Π». Π ΠΎΡ‚ΠΎΡ€ ΠΈΠΌΠ΅Π΅Ρ‚ 100 Π²ΠΈΡ‚ΠΊΠΎΠ² ΠΏΠ»ΠΎΡ‰Π°Π΄ΡŒΡŽ 400 см2. Бколько ΠΎΠ±ΠΎΡ€ΠΎΡ‚ΠΎΠ² Π² ΠΌΠΈΠ½ΡƒΡ‚Ρƒ Π΄Π΅Π»Π°Π΅Ρ‚ ΡΠΊΠΎΡ€ΡŒ, Ссли максимальноС Π·Π½Π°Ρ‡Π΅Π½ΠΈΠ΅ Π­Π”Π‘ ΠΈΠ½Π΄ΡƒΠΊΡ†ΠΈΠΈ Ρ€Π°Π²Π½ΠΎ 200 Π’?

  • Π Π°ΠΌΠΊΠ°, ΠΈΠΌΠ΅ΡŽΡ‰Π°Ρ Ρ„ΠΎΡ€ΠΌΡƒ равностороннСго Ρ‚Ρ€Π΅ΡƒΠ³ΠΎΠ»ΡŒΠ½ΠΈΠΊΠ°, ΠΏΠΎΠΌΠ΅Ρ‰Π΅Π½Π° Π² ΠΎΠ΄Π½ΠΎΡ€ΠΎΠ΄Π½ΠΎΠ΅ ΠΌΠ°Π³Π½ΠΈΡ‚Π½ΠΎΠ΅ ΠΏΠΎΠ»Π΅ с Π½Π°ΠΏΡ€ΡΠΆΠ΅Π½Π½ΠΎΡΡ‚ΡŒΡŽ 6,4Β·104 А/ΠΌ. ΠŸΠ΅Ρ€ΠΏΠ΅Π½Π΄ΠΈΠΊΡƒΠ»ΡΡ€ ΠΊ плоскости Ρ€Π°ΠΌΠΊΠΈ составляСт с Π½Π°ΠΏΡ€Π°Π²Π»Π΅Π½ΠΈΠ΅ΠΌ ΠΌΠ°Π³Π½ΠΈΡ‚Π½ΠΎΠ³ΠΎ поля ΡƒΠ³ΠΎΠ» 30Β°. ΠžΠΏΡ€Π΅Π΄Π΅Π»ΠΈΡ‚ΡŒ Π΄Π»ΠΈΠ½Ρƒ стороны Ρ€Π°ΠΌΠΊΠΈ, Ссли извСстно, Ρ‡Ρ‚ΠΎ срСднСС Π·Π½Π°Ρ‡Π΅Π½ΠΈΠ΅ Π­Π”Π‘, Π²ΠΎΠ·Π½ΠΈΠΊΠ°ΡŽΡ‰Π΅ΠΉ Π² Ρ€Π°ΠΌΠΊΠ΅ ΠΏΡ€ΠΈ Π²Ρ‹ΠΊΠ»ΡŽΡ‡Π΅Π½ΠΈΠΈ поля Π² Ρ‚Π΅Ρ‡Π΅Π½ΠΈΠ΅ 0,03 с, Ρ€Π°Π²Π½ΠΎ 10 ΠΌΠ’.

  • ΠšΠ²Π°Π΄Ρ€Π°Ρ‚Π½Π°Ρ Ρ€Π°ΠΌΠΊΠ° ΠΈΠ· ΠΌΠ΅Π΄Π½ΠΎΠΉ ΠΏΡ€ΠΎΠ²ΠΎΠ»ΠΎΠΊΠΈ сСчСниСм 1 ΠΌΠΌ2 ΠΏΠΎΠΌΠ΅Ρ‰Π΅Π½Π° Π² ΠΌΠ°Π³Π½ΠΈΡ‚Π½ΠΎΠ΅ ΠΏΠΎΠ»Π΅, индукция ΠΊΠΎΡ‚ΠΎΡ€ΠΎΠ³ΠΎ мСняСтся ΠΏΠΎ Π·Π°ΠΊΠΎΠ½Ρƒ Π’ = Π’0 sinΟ‰t, Π³Π΄Π΅ B0 = 0,01 Π’Π»; Ο‰ = 2Ο€/Π’; Π’=0,02 с. ΠŸΠ»ΠΎΡ‰Π°Π΄ΡŒ Ρ€Π°ΠΌΠΊΠΈ 25 см2. ΠŸΠ»ΠΎΡΠΊΠΎΡΡ‚ΡŒ Ρ€Π°ΠΌΠΊΠΈ пСрпСндикулярна Π½Π°ΠΏΡ€Π°Π²Π»Π΅Π½ΠΈΡŽ ΠΌΠ°Π³Π½ΠΈΡ‚Π½ΠΎΠ³ΠΎ поля. Найти Π·Π°Π²ΠΈΡΠΈΠΌΠΎΡΡ‚ΡŒ ΠΎΡ‚ Π²Ρ€Π΅ΠΌΠ΅Π½ΠΈ ΠΈ наибольшСС Π·Π½Π°Ρ‡Π΅Π½ΠΈΠ΅: 1) ΠΌΠ°Π³Π½ΠΈΡ‚Π½ΠΎΠ³ΠΎ ΠΏΠΎΡ‚ΠΎΠΊΠ°, ΠΏΡ€ΠΎΠ½ΠΈΠ·Ρ‹Π²Π°ΡŽΡ‰Π΅Π³ΠΎ Ρ€Π°ΠΌΠΊΡƒ; 2) Π­Π”Π‘ ΠΈΠ½Π΄ΡƒΠΊΡ†ΠΈΠΈ, Π²ΠΎΠ·Π½ΠΈΠΊΠ°ΡŽΡ‰Π΅ΠΉ Π² Ρ€Π°ΠΌΠΊΠ΅; 3) силы Ρ‚ΠΎΠΊΠ°, Ρ‚Π΅ΠΊΡƒΡ‰Π΅Π³ΠΎ ΠΏΠΎ Ρ€Π°ΠΌΠΊΠ΅.

  • ΠšΡ€ΡƒΠ³ΠΎΠ²ΠΎΠΉ ΠΏΠΏΡ€ΠΎΠ²ΠΎΠ»ΠΎΡ‡Π½Ρ‹ΠΉ Π²ΠΈΡ‚ΠΎΠΊ ΠΏΠ»ΠΎΡ‰Π°Π΄ΡŒΡŽ 100 см находится Π² ΠΎΠ΄Π½ΠΎΡ€ΠΎΠ΄Π½ΠΎΠΌ ΠΌΠ°Π³Π½ΠΈΡ‚Π½ΠΎΠΌ ΠΏΠΎΠ»Π΅, индукция ΠΊΠΎΡ‚ΠΎΡ€ΠΎΠ³ΠΎ 1 Π’Π». ΠŸΠ»ΠΎΡΠΊΠΎΡΡ‚ΡŒ Π²ΠΈΡ‚ΠΊΠ° пСрпСндикулярна Π½Π°ΠΏΡ€Π°Π²Π»Π΅Π½ΠΈΡŽ ΠΌΠ°Π³Π½ΠΈΡ‚Π½ΠΎΠ³ΠΎ поля. Π§Π΅ΠΌΡƒ Π±ΡƒΠ΄Π΅Ρ‚ Ρ€Π°Π²Π½ΠΎ срСднСС Π·Π½Π°Ρ‡Π΅Π½ΠΈΠ΅ Π­Π”Π‘ ΠΈΠ½Π΄ΡƒΠΊΡ†ΠΈΠΈ, Π²ΠΎΠ·Π½ΠΈΠΊΠ°ΡŽΡ‰Π΅ΠΉ Π² Π²ΠΈΡ‚ΠΊΠ΅ ΠΏΡ€ΠΈ Π²Ρ‹ΠΊΠ»ΡŽΡ‡Π΅Π½ΠΈΠΈ поля Π² Ρ‚Π΅Ρ‡Π΅Π½ΠΈΠ΅ 0,01 с?

  • ΠšΠ²Π°Π΄Ρ€Π°Ρ‚Π½Π°Ρ Ρ€Π°ΠΌΠΊΠ° со стороной 10 см ΠΏΠΎΠΌΠ΅Ρ‰Π΅Π½Π° Π² ΠΎΠ΄Π½ΠΎΡ€ΠΎΠ΄Π½ΠΎΠ΅ ΠΌΠ°Π³Π½ΠΈΡ‚Π½ΠΎΠ΅ ΠΏΠ°ΠΏΠ΅. ΠΠΎΡ€ΠΌΠ°Π»ΡŒ ΠΊ плоскости Ρ€Π°ΠΌΠΊΠΈ составляСт с линиями ΠΈΠ½Π΄ΡƒΠΊΡ†ΠΈΠΈ ΠΌΠ°Π³Π½ΠΈΡ‚Π½ΠΎΠ³ΠΎ поля ΡƒΠ³ΠΎΠ» 60Β°. Найти ΠΌΠ°Π³Π½ΠΈΡ‚Π½ΡƒΡŽ ΠΈΠ½Π΄ΡƒΠΊΡ†ΠΈΡŽ этого поля, Ссли Π² Ρ€Π°ΠΌΠΊΠ΅ ΠΏΡ€ΠΈ Π²Ρ‹ΠΊΠ»ΡŽΡ‡Π΅Π½ΠΈΠΈ поля Π² Ρ‚Π΅Ρ‡Π΅Π½ΠΈΠ΅ 0,01 с индуцируСтся Π­Π”Π‘ 50 ΠΌΠ’.

  • ΠŸΡ€ΠΎΠ²ΠΎΠ»ΠΎΡ‡Π½Ρ‹ΠΉ Π²ΠΈΡ‚ΠΎΠΊ ΠΏΠ»ΠΎΡ‰Π°Π΄ΡŒΡŽ 1 см2, ΠΈΠΌΠ΅ΡŽΡ‰ΠΈΠΉ сопротивлСниС 104 Ом, пронизываСтся ΠΎΠ΄Π½ΠΎΡ€ΠΎΠ΄Π½Ρ‹ΠΌ ΠΌΠ°Π³Π½ΠΈΡ‚Π½Ρ‹ΠΌ ΠΏΠΎΠ»Π΅ΠΌ, Π»ΠΈΠ½ΠΈΠΈ ΠΈΠ½Π΄ΡƒΠΊΡ†ΠΈΠΈ ΠΊΠΎΡ‚ΠΎΡ€ΠΎΠ³ΠΎ пСрпСндикулярны ΠΊ плоскости Π²ΠΈΡ‚ΠΊΠ°. ΠœΠ°Π³Π½ΠΈΡ‚Π½Π°Ρ индукция измСняСтся со ΡΠΊΠΎΡ€ΠΎΡΡ‚ΡŒΡŽ 0,01 Π’Π»/с. КакоС количСство Ρ‚Π΅ΠΏΠ»ΠΎΡ‚Ρ‹ выдСляСтся Π² Π²ΠΈΡ‚ΠΊΠ΅ Π·Π° Π΅Π΄ΠΈΠ½ΠΈΡ†Ρƒ Π²Ρ€Π΅ΠΌΠ΅Π½ΠΈ?

  • ΠŸΡ€ΡΠΌΠΎΡƒΠ³ΠΎΠ»ΡŒΠ½Ρ‹ΠΉ ΠΊΠΎΠ½Ρ‚ΡƒΡ€ со сторонами 3 см ΠΈ 5 см ΠΈ сопротивлСниСм 1 Ом находится Π² ΠΌΠ°Π³Π½ΠΈΡ‚Π½ΠΎΠΌ ΠΏΠΎΠ»Π΅, ΠΌΠ΅Π½ΡΡŽΡ‰Π΅ΠΌΡΡ ΠΏΠΎ Π·Π°ΠΊΠΎΠ½Ρƒ Π’ = 0,01 t. ΠŸΠ»ΠΎΡΠΊΠΎΡΡ‚ΡŒ ΠΊΠΎΠ½Ρ‚ΡƒΡ€Π° пСрпСндикулярна силовым линиям ΠΌΠ°Π³Π½ΠΈΡ‚Π½ΠΎΠ³ΠΎ поля. Найти Π·Π°ΠΊΠΎΠ½ измСнСния ΠΈΠ½Π΄ΡƒΠΊΡ†ΠΈΠΎΠ½Π½ΠΎΠ³ΠΎ Ρ‚ΠΎΠΊΠ° Π² ΠΊΠΎΠ½Ρ‚ΡƒΡ€Π΅.

  • ΠšΡ€ΡƒΠ³ΠΎΠ²ΠΎΠΉ ΠΊΠΎΠ½Ρ‚ΡƒΡ€ радиусом 5 см ΠΈ сопротивлСниСм 5 Ом находится Π² ΠΎΠ΄Π½ΠΎΡ€ΠΎΠ΄Π½ΠΎΠΌ ΠΌΠ°Π³Π½ΠΈΡ‚Π½ΠΎΠΌ ΠΏΠΎΠ»Π΅, ΠΌΠ΅Π½ΡΡŽΡ‰Π΅ΠΌΡΡ ΠΏΠΎ Π·Π°ΠΊΠΎΠ½Ρƒ Н = 5Β·t4. Π£Π³ΠΎΠ» ΠΌΠ΅ΠΆΠ΄Ρƒ Π½ΠΎΡ€ΠΌΠ°Π»ΡŒΡŽ ΠΊ ΠΊΠΎΠ½Ρ‚ΡƒΡ€Ρƒ ΠΈ силовыми линиями Ρ€Π°Π²Π΅Π½ 30Β°. ΠžΠΏΡ€Π΅Π΄Π΅Π»ΠΈΡ‚ΡŒ Π·Π°ΠΊΠΎΠ½ измСнСния ΠΈΠ½Π΄ΡƒΠΊΡ†ΠΈΠΎΠ½Π½ΠΎΠ³ΠΎ Ρ‚ΠΎΠΊΠ° Π² ΠΊΠΎΠ½Ρ‚ΡƒΡ€Π΅.

  • ΠŸΡ€ΠΎΠ²ΠΎΠ΄ΡΡ‰ΠΈΠΉ ΠΊΠΎΠ½Ρ‚ΡƒΡ€ Π² Π²ΠΈΠ΄Π΅ ΠΊΠ²Π°Π΄Ρ€Π°Ρ‚Π° со стороной 5 см находится Π² ΠΎΠ΄Π½ΠΎΡ€ΠΎΠ΄Π½ΠΎΠΌ ΠΌΠ°Π³Π½ΠΈΡ‚Π½ΠΎΠΌ ΠΏΠΎΠ»Π΅, ΠΌΠ΅Π½ΡΡŽΡ‰Π΅ΠΌΡΡ ΠΏΠΎ Π·Π°ΠΊΠΎΠ½Ρƒ Н = 10 sin2t. Π‘ΠΎΠΏΡ€ΠΎΡ‚ΠΈΠ²Π»Π΅Π½ΠΈΠ΅ ΠΊΠΎΠ½Ρ‚ΡƒΡ€Π° 1 Ом. Π£Π³ΠΎΠ» ΠΌΠ΅ΠΆΠ΄Ρƒ Π½ΠΎΡ€ΠΌΠ°Π»ΡŒΡŽ ΠΊ ΠΊΠΎΠ½Ρ‚ΡƒΡ€Ρƒ ΠΈ силовыми линиями Ρ€Π°Π²Π΅Π½ 45Β°. ΠžΠΏΡ€Π΅Π΄Π΅Π»ΠΈΡ‚ΡŒ Π·Π°ΠΊΠΎΠ½ измСнСния ΠΈΠ½Π΄ΡƒΠΊΡ†ΠΈΠΎΠ½Π½ΠΎΠ³ΠΎ Ρ‚ΠΎΠΊΠ° Π² ΠΊΠΎΠ½Ρ‚ΡƒΡ€Π΅.

  • ΠšΠΎΠ½Ρ‚ΡƒΡ€, ΠΏΠ»ΠΎΡ‰Π°Π΄ΡŒ ΠΊΠΎΡ‚ΠΎΡ€ΠΎΠ³ΠΎ 10 см2 ΠΈ сопротивлСниС 10 Ом, находится Π² ΠΎΠ΄Π½ΠΎΡ€ΠΎΠ΄Π½ΠΎΠΌ ΠΌΠ°Π³Π½ΠΈΡ‚Π½ΠΎΠΌ ΠΏΠΎΠ»Π΅, ΠΌΠ΅Π½ΡΡŽΡ‰Π΅ΠΌΡΡ ΠΏΠΎ Π·Π°ΠΊΠΎΠ½Ρƒ Н = 10Β· t3 .ΠŸΠ»ΠΎΡΠΊΠΎΡΡ‚ΡŒ ΠΊΠΎΠ½Ρ‚ΡƒΡ€Π° пСрпСндикулярна силовым линиям ΠΌΠ°Π³Π½ΠΈΡ‚Π½ΠΎΠ³ΠΎ поля. ΠžΠΏΡ€Π΅Π΄Π΅Π»ΠΈΡ‚ΡŒ Π·Π°ΠΊΠΎΠ½ измСнСния ΠΈΠ½Π΄ΡƒΠΊΡ†ΠΈΠΎΠ½Π½ΠΎΠ³ΠΎ Ρ‚ΠΎΠΊΠ° Π² ΠΊΠΎΠ½Ρ‚ΡƒΡ€Π΅.

  • ΠšΠ²Π°Π΄Ρ€Π°Ρ‚Π½Π°Ρ Ρ€Π°ΠΌΠΊΠ° со стороной 2 см ΠΏΠΎΠΌΠ΅Ρ‰Π΅Π½Π° Π² ΠΎΠ΄Π½ΠΎΡ€ΠΎΠ΄Π½ΠΎΠ΅ ΠΌΠ°Π³Π½ΠΈΡ‚Π½ΠΎΠ΅ Π²ΠΎΠ»Π΅ с ΠΈΠ½Π΄ΡƒΠΊΡ†ΠΈΠ΅ΠΉ 100 Π’Π». ΠŸΠ»ΠΎΡΠΊΠΎΡΡ‚ΡŒ Ρ€Π°ΠΌΠΊΠΈ пСрпСндикулярна ΠΊ линиям ΠΈΠ½Π΄ΡƒΠΊΡ†ΠΈΠΈ поля. Π‘ΠΎΠΏΡ€ΠΎΡ‚ΠΈΠ²Π»Π΅Π½ΠΈΠ΅ Ρ€Π°ΠΌΠΊΠΈ 1 Ом. Какой Ρ‚ΠΎΠΊ ΠΏΡ€ΠΎΡ‚Π΅Ρ‡Π΅Ρ‚ ΠΏΠΎ Ρ€Π°ΠΌΒ­ΠΊΠ΅, Ссли Π΅Π΅ Π²Ρ‹Π΄Π²ΠΈΠ³Π°Ρ‚ΡŒ ΠΈΠ· ΠΌΠ°Π³Π½ΠΈΡ‚Π½ΠΎΠ³ΠΎ поля со ΡΠΊΠΎΡ€ΠΎΡΡ‚ΡŒΡŽ 1 см/с, пСрпСндикулярной ΠΊ линиям ΠΈΠ½Π΄ΡƒΠΊΡ†ΠΈΠΈ? ПолС ΠΈΠΌΠ΅Π΅Ρ‚ Ρ€Π΅Π·ΠΊΠΎ ΠΎΡ‡Π΅Ρ€Ρ‡Π΅Π½Π½Ρ‹Π΅ Π³Ρ€Π°Π½ΠΈΡ†Ρ‹, ΠΈ стороны Ρ€Π°ΠΌΠΊΠΈ ΠΏΠ°Ρ€Π°Π»Π»Π΅Π»ΡŒΠ½Ρ‹ этим Π³Ρ€Π°Π½ΠΈΡ†Π°ΠΌ.

  • Π’ ΠΎΠ΄Π½ΠΎΡ€ΠΎΠ΄Π½ΠΎΠΌ ΠΌΠ°Π³Π½ΠΈΡ‚Π½ΠΎΠΌ ΠΏΠΎΠ»Π΅ с Π½Π°ΠΏΡ€ΡΠΆΠ΅Π½Π½ΠΎΡΡ‚ΡŒΡŽ 1000 А/ΠΌ находится квадратная Ρ€Π°ΠΌΠΊΠ° со стороной 20 см. ΠŸΠ»ΠΎΡΠΊΠΎΡΡ‚ΡŒ Ρ€Π°ΠΌΠΊΠΈ пСрпСндикулярна Π½Π°ΠΏΡ€Π°Π²Π»Π΅Π½ΠΈΡŽ силовых Π»ΠΈΠ½ΠΈΠΉ. Π‘ΠΎΠΏΡ€ΠΎΡ‚ΠΈΠ²Π»Π΅Π½ΠΈΠ΅ Ρ€Π°ΠΌΠΊΠΈ 10 Ом. Какой Ρ‚ΠΎΠΊ ΠΏΠΎΠΉΠ΄Π΅Ρ‚ ΠΏΠΎ Ρ€Π°ΠΌΠΊΠ΅, Ссли Π΅Π΅ Π²Π΄Π²ΠΈΠ³Π°Ρ‚ΡŒ Π² ΠΌΠ°Π³Π½ΠΈΡ‚Π½ΠΎΠ΅ ΠΏΠΎΠ»Π΅ с постоянной ΡΠΊΠΎΡ€ΠΎΡΡ‚ΡŒΡŽ 1 ΠΌ/с пСрпСндикулярно Π½Π°ΠΏΡ€Π°Π²Π»Π΅Π½ΠΈΡŽ силовых Π»ΠΈΠ½ΠΈΠΉ? ПолС ΠΈΠΌΠ΅Π΅Ρ‚ Ρ€Π΅Π·ΠΊΠΎ ΠΎΡ‡Π΅Ρ€Ρ‡Π΅Π½Π½Ρ‹Π΅ Π³Ρ€Π°Π½ΠΈΡ†Ρ‹, ΠΈ Π΄Π²Π΅ стороны Ρ€Π°ΠΌΠΊΠΈ ΠΏΠ°Ρ€Π°Π»Π»Π΅Π»ΡŒΠ½Ρ‹ этой Π³Ρ€Π°Π½ΠΈΡ†Π΅.

  • ΠšΡ€ΡƒΠ³ΠΎΠ²ΠΎΠΉ Π²ΠΈΡ‚ΠΎΠΊ ΠΏΠΎΠΌΠ΅Ρ‰Π΅Π½ Π² ΠΎΠ΄Π½ΠΎΡ€ΠΎΠ΄Π½ΠΎΠ΅ ΠΌΠ°Π³Π½ΠΈΡ‚Π½ΠΎΠ΅ ΠΏΠΎΠ»Π΅. ΠΠΎΡ€ΠΌΠ°Π»ΡŒ ΠΊ плоскости Π²ΠΈΡ‚ΠΊΠ° составляСт с линиями поля ΡƒΠ³ΠΎΠ» 30Β°. Π’Π΅Π»ΠΈΡ‡ΠΈΠ½Π° ΠΈΠ½Π΄ΡƒΠΊΡ†ΠΈΠΈ 0,5 Π’Π». Найти радиус Π²ΠΈΡ‚ΠΊΠ°, Ссли ΠΏΡ€ΠΈ Π²Ρ‹ΠΊΠ»ΡŽΡ‡Π΅Π½ΠΈΠΈ поля Π² Ρ‚Π΅Ρ‡Π΅Π½ΠΈΠ΅ 1 мс индуцируСтся Π­Π”Π‘ 10 ΠΌΠ’.

  • ΠšΡ€ΡƒΠ³ΠΎΠ²ΠΎΠΉ Π²ΠΈΡ‚ΠΎΠΊ ΠΈΠ· Π½ΠΈΠΊΠ΅Π»Π΅Π²ΠΎΠΉ ΠΏΡ€ΠΎΠ²ΠΎΠ»ΠΎΠΊΠΈ сСчСниСм 1 ΠΌΠΌ2 ΠΏΠΎΠΌΠ΅Ρ‰Π΅Π½ Π² ΠΌΠ°Π³Π½ΠΈΡ‚Π½ΠΎΠ΅ ΠΏΠΎΠ»Π΅, Π½Π°ΠΏΡ€ΡΠΆΠ΅Π½Π½ΠΎΡΡ‚ΡŒ ΠΊΠΎΡ‚ΠΎΡ€ΠΎΠ³ΠΎ мСняСтся ΠΏΠΎ Π·Π°ΠΊΠΎΠ½Ρƒ Н = H0 cosΟ‰t, Π³Π΄Π΅ Но = 1000 А/ΠΌ; Ο‰ = 2Ο€/Π’; Π’ = 0,01 с. ΠŸΠ»ΠΎΡ‰Π°Π΄ΡŒ Ρ€Π°ΠΌΠΊΠΈ 36 см2. ΠŸΠ»ΠΎΡΠΊΠΎΡΡ‚ΡŒ Ρ€Π°ΠΌΠΊΠΈ пСрпСндикулярна Π½Π°ΠΏΡ€Π°Π²Π»Π΅Π½ΠΈΡŽ ΠΌΠ°Π³Π½ΠΈΡ‚Π½ΠΎΠ³ΠΎ поля. Найти Π·Π°Π²ΠΈΡΠΈΠΌΠΎΡΡ‚ΡŒ ΠΎΡ‚ Π²Ρ€Π΅ΠΌΠ΅Π½ΠΈ ΠΈ наибольшСС Π·Π½Π°Ρ‡Π΅Π½ΠΈΠ΅: 1) ΠΌΠ°Π³Π½ΠΈΡ‚Π½ΠΎΠ³ΠΎ ΠΏΠΎΡ‚ΠΎΠΊΠ° Ρ‡Π΅Ρ€Π΅Π· Π²ΠΈΡ‚ΠΎΠΊ; 2) Π­Π”Π‘ ΠΈΠ½Π΄ΡƒΠΊΡ†ΠΈΠΈ Π² Π²ΠΈΡ‚ΠΊΠ΅; 3) силы Ρ‚ΠΎΠΊΠ° Π² Π²ΠΈΡ‚ΠΊΠ΅.

  • На столС Π»Π΅ΠΆΠΈΡ‚ ΠΏΡ€ΠΎΠ²ΠΎΠ»ΠΎΡ‡Π½ΠΎΠ΅ ΠΊΠΎΠ»ΡŒΡ†ΠΎ радиусом 10 см. КакоС количСство элСктричСства ΠΏΡ€ΠΎΡ‚Π΅Ρ‡Π΅Ρ‚ ΠΏΠΎ ΠΊΠΎΠ»ΡŒΡ†Ρƒ, Ссли Π΅Π³ΠΎ ΠΏΠ΅Ρ€Π΅Π²Π΅Ρ€Π½ΡƒΡ‚ΡŒ? Π‘ΠΎΠΏΡ€ΠΎΡ‚ΠΈΠ²Π»Π΅Π½ΠΈΠ΅ ΠΊΠΎΠ»ΡŒΡ†Π° 1 Ом. Π’Π΅Ρ€Ρ‚ΠΈΠΊΠ°Π»ΡŒΠ½Π°Ρ ΡΠΎΡΡ‚Π°Π²Π»ΡΡŽΡ‰Π°Ρ ΠΈΠ½Π΄ΡƒΠΊΡ†ΠΈΠΈ ΠΌΠ°Π³Π½ΠΈΡ‚Π½ΠΎΠ³ΠΎ поля Π—Π΅ΠΌΠ»ΠΈ 50 ΠΌΠΊΠ’Π».

  • ΠŸΡ€ΠΎΠ²ΠΎΠ»ΠΎΡ‡Π½Ρ‹ΠΉ Π²ΠΈΡ‚ΠΎΠΊ радиусом 4 см ΠΈ сопротивлСниСм 0,01 Ом находится Π² ΠΎΠ΄Π½ΠΎΡ€ΠΎΠ΄Π½ΠΎΠΌ ΠΌΠ°Π³Π½ΠΈΡ‚Π½ΠΎΠΌ ΠΏΠΎΠ»Π΅ Π΅ ΠΈΠ½Π΄ΡƒΠΊΡ†ΠΈΠ΅ΠΉ 0,04 Π’Π». ΠŸΠ»ΠΎΡΠΊΠΎΡΡ‚ΡŒ Ρ€Π°ΠΌΠΊΠΈ составляСт ΡƒΠ³ΠΎΠ» 30Β° с линиями поля. КакоС количСство элСктричСства ΠΏΡ€ΠΎΡ‚Π΅Ρ‡Π΅Ρ‚ ΠΏΠΎ Π²ΠΈΡ‚ΠΊΡƒ, Ссли ΠΌΠ°Π³Π½ΠΈΡ‚Π½ΠΎΠ΅ ΠΏΠΎΠ»Π΅ Π²Ρ‹ΠΊΠ»ΡŽΡ‡ΠΈΡ‚ΡŒ?

  • ΠšΡ€ΡƒΠ³ΠΎΠ²ΠΎΠΉ ΠΊΠΎΠ½Ρ‚ΡƒΡ€ радиусом 2 см ΠΏΠΎΠΌΠ΅Ρ‰Π΅Π½ Π² ΠΎΠ΄Π½ΠΎΡ€ΠΎΠ΄Π½ΠΎΠ΅ ΠΌΠ°Π³Π½ΠΈΡ‚Π½ΠΎΠ΅ ΠΏΠΎΠ»Π΅, индукция ΠΊΠΎΡ‚ΠΎΡ€ΠΎΠ³ΠΎ 0,2 Π’Π». ΠŸΠ»ΠΎΡΠΊΠΎΡΡ‚ΡŒ ΠΊΠΎΠ½Ρ‚ΡƒΡ€Π° пСрпСндикулярна ΠΊ Π½Π°ΠΏΡ€Π°Π²Π»Π΅Π½ΠΈΡŽ ΠΌΠ°Π³Π½ΠΈΡ‚Π½ΠΎΠ³ΠΎ поля. Π‘ΠΎΠΏΡ€ΠΎΡ‚ΠΈΠ²Π»Π΅Π½ΠΈΠ΅ ΠΊΠΎΠ½Ρ‚ΡƒΡ€Π° 1 Ом. КакоС количСство элСктричСства ΠΏΡ€ΠΎΠΉΠ΄Π΅Ρ‚ Ρ‡Π΅Ρ€Π΅Π· ΠΊΠΎΠ½Ρ‚ΡƒΡ€ ΠΏΡ€ΠΈ ΠΏΠΎΠ²ΠΎΡ€ΠΎΡ‚Π΅ Π° ΡƒΠ³ΠΎΠ» 90Β°?

  • Плоский Π²ΠΈΡ‚ΠΎΠΊ ΠΏΠ»ΠΎΡ‰Π°Π΄ΡŒΡŽ 10 см2 ΠΏΠΎΠΌΠ΅Ρ‰Π΅Π½ Π² ΠΎΠ΄Π½ΠΎΡ€ΠΎΠ΄Π½ΠΎΠ΅ ΠΌΠ°Π³Π½ΠΈΡ‚Π½ΠΎΠ΅ ΠΏΠΎΠ»Π΅ с ΠΈΠ½Π΄ΡƒΠΊΡ†ΠΈΠ΅ΠΉ 0,1 Π’Π», пСрпСндикулярноС ΠΊ. плоскости Π²ΠΈΡ‚ΠΊΠ°. Π‘ΠΎΠΏΡ€ΠΎΡ‚ΠΈΠ²Β­Π»Π΅Π½ΠΈΠ΅ Π²ΠΈΡ‚ΠΊΠ° 1 Ом. Какой заряд ΠΏΡ€ΠΎΡ‚Π΅Ρ‡Π΅Ρ‚ ΠΏΠΎ Π²ΠΈΡ‚ΠΊΡƒ, Ссли ΠΏΠΎΠ»Π΅ Π±ΡƒΠ΄Π΅Ρ‚ ΠΈΡΡ‡Π΅Π·Π°Ρ‚ΡŒ с постоянной ΡΠΊΠΎΡ€ΠΎΡΡ‚ΡŒΡŽ?

  • Π’ΠΎΠ½ΠΊΠΈΠΉ ΠΌΠ΅Π΄Π½Ρ‹ΠΉ ΠΏΡ€ΠΎΠ²ΠΎΠ΄Π½ΠΈΠΊ массой 1 Π³ согнут Π² Π²ΠΈΠ΄Π΅ ΠΊΠ²Π°Π΄Ρ€Π°Ρ‚Π° ΠΈ ΠΊΠΎΠ½Ρ†Ρ‹ Π΅Π³ΠΎ Π·Π°ΠΌΠΊΠ½ΡƒΡ‚Ρ‹. ΠšΠ²Π°Π΄Ρ€Π°Ρ‚ ΠΏΠΎΠΌΠ΅Ρ‰Π΅Π½ Π² ΠΎΠ΄Π½ΠΎΡ€ΠΎΠ΄Π½ΠΎΠ΅ ΠΌΠ°Π³Π½ΠΈΡ‚Π½ΠΎΠ΅ ΠΏΠΎΠ»Π΅ с ΠΈΠ½Π΄ΡƒΠΊΒ­Ρ†ΠΈΠ΅ΠΉ 1 Π’Π» Ρ‚Π°ΠΊ, Ρ‡Ρ‚ΠΎ ΠΏΠ»ΠΎΡΠΊΠΎΡΡ‚ΡŒ Π΅Π³ΠΎ пСрпСндикулярна линиям поля. ΠžΠΏΡ€Π΅Π΄Π΅Π»ΠΈΡ‚ΡŒ количСство элСктричСства, ΠΊΠΎΡ‚ΠΎΡ€ΠΎΠ΅ ΠΏΡ€ΠΎΡ‚Π΅Ρ‡Π΅Ρ‚ ΠΏΠΎ ΠΏΡ€ΠΎΠ²ΠΎΠ΄Π½ΠΈΠΊΡƒ, Ссли ΠΊΠ²Π°Π΄Ρ€Π°Ρ‚, потянув Π·Π° ΠΏΡ€ΠΎΡ‚ΠΈΠ²ΠΎΠΏΠΎΠ»ΠΎΠΆΠ½Ρ‹Π΅ Π²Π΅Ρ€ΡˆΠΈΠ½Ρ‹, Π²Ρ‹Ρ‚ΡΠ½ΡƒΡ‚ΡŒ Π² линию.

  • ΠœΠ°Π³Π½ΠΈΡ‚Π½Ρ‹ΠΉ ΠΏΠΎΡ‚ΠΎΠΊ Ρ‡Π΅Ρ€Π΅Π· ΠΊΠΎΠ½Ρ‚ΡƒΡ€ ΠΈΠ· ΠΏΡ€ΠΎΠ²ΠΎΠ΄Π½ΠΈΠΊΠ° с элСктричСским сопротивлСниСм 2 Ом Ρ€Π°Π²Π½ΠΎΠΌΠ΅Ρ€Π½ΠΎ увСличился ΠΎΡ‚ 0 Π΄ΠΎ 3Β·104 Π’Π±. Какой заряд ΠΏΡ€ΠΈ этом ΠΏΡ€ΠΎΡˆΠ΅Π» Ρ‡Π΅Ρ€Π΅Π· ΠΏΠΎΠΏΠ΅Ρ€Π΅Ρ‡Π½ΠΎΠ΅ сСчСниС ΠΏΡ€ΠΎΠ²ΠΎΠ΄Π½ΠΈΠΊΠ°?

  • ΠœΠ°Π³Π½ΠΈΡ‚Π½Ρ‹ΠΉ ΠΏΠΎΡ‚ΠΎΠΊ Ρ‡Π΅Ρ€Π΅Π· проводящий ΠΊΠΎΠ½Ρ‚ΡƒΡ€ с элСктричСским сопротивлСниСм 1 Ом измСняСтся ΠΏΠΎ Π·Π°ΠΊΠΎΠ½Ρƒ Π€(t) = 10t2 . Какой заряд ΠΏΡ€ΠΎΠΉΠ΄Π΅Ρ‚ Ρ‡Π΅Ρ€Π΅Π· ΠΏΠΎΠΏΠ΅Ρ€Π΅Ρ‡Π½ΠΎΠ΅ сСчСниС ΠΊΠΎΠ½Ρ‚ΡƒΡ€Π° Π·Π° 5 с?

  • ΠŸΡ€ΠΎΠ²ΠΎΠ΄ΡΡ‰ΠΈΠΉ ΠΊΠΎΠ½Ρ‚ΡƒΡ€ ΠΏΠ»ΠΎΡ‰Π°Π΄ΡŒΡŽ 10 см ΠΈ сопротивлСниСм 2 Ом находится Π² ΠΌΠ°Π³Π½ΠΈΡ‚Π½ΠΎΠΌ ΠΏΠΎΠ»Π΅, ΠΈΠ·ΠΌΠ΅Π½ΡΡŽΡ‰Π΅ΠΌΡΡ ΠΏΠΎ Π·Π°ΠΊΠΎΠ½Ρƒ Π’ = 3t. Какой заряд ΠΏΡ€ΠΎΠΉΠ΄Π΅Ρ‚ Ρ‡Π΅Ρ€Π΅Π· ΠΏΠΎΠΏΠ΅Ρ€Π΅Ρ‡Π½ΠΎΠ΅ сСчСниС ΠΊΠΎΠ½Ρ‚ΡƒΡ€Π° Π·Π° 10 с, Ссли ΡƒΠ³ΠΎΠ» ΠΌΠ΅ΠΆΠ΄Ρƒ Π½ΠΎΡ€ΠΌΠ°Π»ΡŒΡŽ ΠΊ ΠΊΠΎΠ½Ρ‚ΡƒΡ€Ρƒ ΠΈ ΠΌΠ°Π³Π½ΠΈΡ‚Π½ΠΎΠΉ ΠΈΠ½Π΄ΡƒΠΊΡ†ΠΈΠ΅ΠΉ Ρ€Π°Π²Π΅Π½ 30Β°?

  • ΠŸΡ€ΠΎΠ²ΠΎΠ»ΠΎΡ‡Π½Ρ‹ΠΉ Π²ΠΈΡ‚ΠΎΠΊ, ΠΈΠΌΠ΅ΡŽΡ‰ΠΈΠΉ ΠΏΠ»ΠΎΡ‰Π°Π΄ΡŒ 0,01 ΠΌ2, Ρ€Π°Π·Ρ€Π΅Π·Π°Π½ Π² Π½Π΅ΠΊΠΎΡ‚ΠΎΡ€ΠΎΠΉ Ρ‚ΠΎΡ‡ΠΊΠ΅, ΠΈ Π² Ρ€Π°Π·Ρ€Π΅Π· Π²ΠΊΠ»ΡŽΡ‡Π΅Π½ кондСнсатор Π΅ΠΌΠΊΠΎΡΡ‚ΡŒΡŽ 10 ΠΌΠΊΠ€. Π’ΠΈΡ‚ΠΎΠΊ ΠΏΠΎΠΌΠ΅Β­Ρ‰Π΅Π½ Π² ΠΎΠ΄Π½ΠΎΡ€ΠΎΠ΄Π½ΠΎΠ΅ ΠΌΠ°Π³Π½ΠΈΡ‚Π½ΠΎΠ΅ ΠΏΠΎΠ»Π΅, Π»ΠΈΠ½ΠΈΠΈ ΠΈΠ½Π΄ΡƒΠΊΡ†ΠΈΠΈ ΠΊΠΎΡ‚ΠΎΡ€ΠΎΠ³ΠΎ пСрпСндикулярны плоскости Π²ΠΈΡ‚ΠΊΠ°, Π˜Π½Π΄ΡƒΠΊΡ†ΠΈΡ ΠΌΠ°Π³Π½ΠΈΡ‚Π½ΠΎΠ³ΠΎ поля Ρ€Π°Π²Π½ΠΎΠΌΠ΅Ρ€Π½ΠΎ измСняСтся Π²ΠΎ Π²Ρ€Π΅ΠΌΠ΅Π½ΠΈ со ΡΠΊΠΎΡ€ΠΎΡΡ‚ΡŒΡŽ 5Β·103 Π’Π»/с. ΠžΠΏΡ€Π΅Π΄Π΅Π»ΠΈΡ‚ΡŒ заряд кондСнсатора.

  • Π’ ΠΎΠ΄Π½ΠΎΡ€ΠΎΠ΄Π½ΠΎΠΌ ΠΌΠ°Π³Π½ΠΈΡ‚Π½ΠΎΠΌ ΠΏΠΎΠ»Π΅ с ΠΈΠ½Π΄ΡƒΠΊΡ†ΠΈΠ΅ΠΉ 0,1 Π’Π» располоТСн плоский ΠΏΡ€ΠΎΠ²ΠΎΠ»ΠΎΡ‡Π½Ρ‹ΠΉ Π²ΠΈΡ‚ΠΎΠΊ Ρ‚Π°ΠΊ, Ρ‡Ρ‚ΠΎ Π΅Π³ΠΎ ΠΏΠ»ΠΎΡΠΊΠΎΡΡ‚ΡŒ пСрпСндикулярна линиямин­дукции, Π’ΠΈΡ‚ΠΎΠΊ Π·Π°ΠΌΠΊΠ½ΡƒΡ‚ Π½Π° Π³Π°Π»ΡŒΠ²Π°Π½ΠΎΠΌΠ΅Ρ‚Ρ€. ΠŸΠΎΠ»Π½Ρ‹ΠΉ заряд, ΠΏΡ€ΠΎΡ‚Π΅ΠΊΡˆΠΈΠΉ Ρ‡Π΅Ρ€Π΅Π· Π³Π°Π»ΡŒΠ²Π°Π½ΠΎΠΌΠ΅Ρ‚Ρ€ ΠΏΡ€ΠΈ ΠΏΠΎΠ²ΠΎΡ€ΠΎΡ‚Π΅ Π²ΠΈΡ‚ΠΊΠ°, Ρ€Π°Π²Π΅Π½ 7,5Β·10-4 Кл. На ΠΊΠ°ΠΊΠΎΠΉ ΡƒΠ³ΠΎΠ» ΠΏΠΎΠ²Π΅Ρ€Π½ΡƒΠ»ΠΈ Π²ΠΈΡ‚ΠΎΠΊ? ΠŸΠ»ΠΎΡ‰Π°Π΄ΡŒ Π²ΠΈΡ‚ΠΊΠ° 0,1 ΠΌ2 , сопротивлСниС Π²ΠΈΡ‚ΠΊΠ° 2 Ом.

  • B ΠΌΠ°Π³Π½ΠΈΡ‚Π½ΠΎΠΌ ΠΏΠΎΠ»Π΅, индукция ΠΊΠΎΡ‚ΠΎΡ€ΠΎΠ³ΠΎ Ρ€Π°Π²Π½Π° 0,1 Π’Π», ΠΏΠΎΠΌΠ΅Ρ‰Π΅Π½Π° квадратная Ρ€Π°ΠΌΠΊΠ° ΠΈΠ· ΠΌΠ΅Π΄Π½ΠΎΠΉ ΠΏΡ€ΠΎΠ²ΠΎΠ»ΠΎΠΊΠΈ сСчСниСм 1 ΠΌΠΌ2, ΠΏΠ»ΠΎΡ‰Π°Π΄ΡŒ Ρ€Π°ΠΌΠΊΠΈ 25 см2 , Π½ΠΎΡ€ΠΌΠ°Π»ΡŒ ΠΊ плоскости Ρ€Π°ΠΌΠΊΠΈ Π½Π°ΠΏΡ€Π°Π²Π»Π΅Π½Π° ΠΏΠΎ силовым линиям поля. КакоС количСство элСктричСства ΠΏΡ€ΠΎΠΉΠ΄Π΅Ρ‚ ΠΏΠΎ ΠΊΠΎΠ½Ρ‚ΡƒΡ€Ρƒ Ρ€Π°ΠΌΠΊΠΈ ΠΏΡ€ΠΈ исчСзновСнии ΠΌΠ°Π³Π½ΠΈΡ‚Π½ΠΎΠ³ΠΎ поля?

  • Π’ ΠΏΡ€ΠΎΠ²ΠΎΠ»ΠΎΡ‡Π½ΠΎΠ΅ ΠΊΠΎΠ»ΡŒΡ†ΠΎ, присоСдинСнноС ΠΊ баллистичСскому Π³Π°Π»ΡŒΠ²Π°Π½ΠΎΒ­ΠΌΠ΅Ρ‚Ρ€Ρƒ, вставили прямой ΠΌΠ°Π³Π½ΠΈΡ‚. По Ρ†Π΅ΠΏΠΈ ΠΏΡ€ΠΎΡˆΠ΅Π» заряд 105 Кл, ΠžΠΏΡ€Π΅Π΄Π΅Β­Π»ΠΈΡ‚ΡŒ ΠΌΠ°Π³Π½ΠΈΡ‚Π½Ρ‹ΠΉ ΠΏΠΎΡ‚ΠΎΠΊ Ρ‡Π΅Ρ€Π΅Π· ΠΏΠΎΠ²Π΅Ρ€Ρ…Π½ΠΎΡΡ‚ΡŒ, ΠΎΠ³Ρ€Π°Π½ΠΈΡ‡Π΅Π½Π½ΡƒΡŽ ΠΊΠΎΠ»ΡŒΡ†ΠΎΠΌ, Ссли сопротивлСниС Ρ†Π΅ΠΏΠΈ Π³Π°Π»ΡŒΠ²Π°Π½ΠΎΠΌΠ΅Ρ‚Ρ€Π° 30 Ом.

  • На расстоянии 1 ΠΌ ΠΎΡ‚ Π΄Π»ΠΈΠ½Π½ΠΎΠ³ΠΎ прямого ΠΏΡ€ΠΎΠ²ΠΎΠ΄Π½ΠΈΠΊΠ° с Ρ‚ΠΎΠΊΠΎΠΌ 103 А рас­полоТСно ΠΊΠΎΠ»ΡŒΡ†ΠΎ радиуса 1 см. ΠšΠΎΠ»ΡŒΡ†ΠΎ располоТСно Ρ‚Π°ΠΊ, Ρ‡Ρ‚ΠΎ ΠΏΠΎΡ‚ΠΎΠΊ, ΠΏΡ€ΠΎΠ½ΠΈΠ·Ρ‹Π²Π°ΡŽΡ‰ΠΈΠΉ ΠΊΠΎΠ»ΡŒΡ†ΠΎ, максималСн. Π§Π΅ΠΌΡƒ Ρ€Π°Π²Π½ΠΎ количСство элСктри­чСства, ΠΊΠΎΡ‚ΠΎΡ€ΠΎΠ΅ ΠΏΡ€ΠΎΡ‚Π΅Ρ‡Π΅Ρ‚ ΠΏΠΎ ΠΊΠΎΠ»ΡŒΡ†Ρƒ, Ссли Ρ‚ΠΎΠΊ Π² ΠΏΡ€ΠΎΠ²ΠΎΠ΄Π½ΠΈΠΊΠ΅ Π±ΡƒΠ΄Π΅Ρ‚ Π²Ρ‹Β­ΠΊΠ»ΡŽΡ‡Π΅Π½? Π‘ΠΎΠΏΡ€ΠΎΡ‚ΠΈΠ²Π»Π΅Π½ΠΈΠ΅ ΠΊΠΎΠ»ΡŒΡ†Π° 10 Ом. ПолС Π² ΠΏΡ€Π΅Π΄Π΅Π»Π°Ρ… ΠΊΠΎΠ»ΡŒΡ†Π° ΡΡ‡ΠΈΡ‚Π°Ρ‚ΡŒ ΠΎΠ΄Π½ΠΎΡ€ΠΎΠ΄Π½Ρ‹ΠΌ.

  • По Π΄Π»ΠΈΠ½Π½ΠΎΠΌΡƒ прямому ΠΏΡ€ΠΎΠ²ΠΎΠ΄Π½ΠΈΠΊΡƒ Ρ‚Π΅Ρ‡Π΅Ρ‚ Ρ‚ΠΎΠΊ. Π’Π±Π»ΠΈΠ·ΠΈ ΠΏΡ€ΠΎΠ²ΠΎΠ΄Π½ΠΈΠΊΠ° распо­лоТСна квадратная Ρ€Π°ΠΌΠΊΠ° ΠΈΠ· Ρ‚ΠΎΠ½ΠΊΠΎΠ³ΠΎ ΠΏΡ€ΠΎΠ²ΠΎΠ΄Π° сопротивлСниСм 0,02 Ом. ΠŸΡ€ΠΎΠ²ΠΎΠ΄Π½ΠΈΠΊ Π»Π΅ΠΆΠΈΡ‚ Π² плоскости Ρ€Π°ΠΌΠΊΠΈ ΠΈ ΠΏΠ°Ρ€Π°Π»Π»Π΅Π»Π΅Π½ Π΄Π²ΡƒΠΌ Π΅Π΅ сторонам, расстояния Π΄ΠΎ ΠΊΠΎΡ‚ΠΎΡ€Ρ‹Ρ… ΠΎΡ‚ ΠΏΡ€ΠΎΠ²ΠΎΠ΄Π° соотвСтствСнно Ρ€Π°Π²Π½Ρ‹ 10 см ΠΈ 20 см. Най­ти силу Ρ‚ΠΎΠΊΠ° Π² ΠΏΡ€ΠΎΠ²ΠΎΠ΄Π½ΠΈΠΊΠ΅, Ссли ΠΏΡ€ΠΈ Π΅Π³ΠΎ Π²Ρ‹ΠΊΠ»ΡŽΡ‡Π΅Π½ΠΈΠΈ Ρ‡Π΅Ρ€Π΅Π· Ρ€Π°ΠΌΠΊΡƒ ΠΏΡ€ΠΎΡ‚Π΅ΠΊΒ­Π»ΠΎ количСство элСктричСства q = 696 Кл.

  • ΠŸΡ€ΠΎΠ²ΠΎΠ΄Π½ΠΈΠΊ MN (рис. 1) с Π΄Π»ΠΈΠ½ΠΎΠΉ Π°ΠΊΡ‚ΠΈΠ²Π½ΠΎΠΉ части 1 ΠΌ ΠΈ сопротивлСниСм 2 Ом находится Π² ΠΎΠ΄Π½ΠΎΡ€ΠΎΠ΄Π½ΠΎΠΌ ΠΌΠ°Π³Π½ΠΈΡ‚Π½ΠΎΠΌ ΠΏΠΎΠ»Π΅ с ΠΈΠ½Π΄ΡƒΠΊΡ†ΠΈΠ΅ΠΉ 1 Π’Π». ΠŸΡ€ΠΎΠ²ΠΎΠ΄Π½ΠΈΠΊ ΠΏΠΎΠ΄ΠΊΠ»ΡŽΡ‡Π΅Π½ ΠΊ источнику с Π­Π”Π‘ 1 Π’ (Π²Π½ΡƒΡ‚Ρ€Π΅Π½Π½ΠΈΠΌ сопротивлСниСм источника ΠΈ сопротивлСниСм подводящих ΠΏΡ€ΠΎΠ²ΠΎΠ΄ΠΎΠ² ΠΏΡ€Π΅Π½Π΅Π±Ρ€Π΅Ρ‡ΡŒ). Какова сила Ρ‚ΠΎΠΊΠ° Π² ΠΏΡ€ΠΎΠ²ΠΎΠ΄Π½ΠΈΠΊΠ΅, Ссли Π°) ΠΏΡ€ΠΎΠ²ΠΎΠ΄Π½ΠΈΠΊ покоится; Π±) ΠΏΡ€ΠΎΠ²ΠΎΠ΄Π½ΠΈΠΊ двиТСтся Π²ΠΏΡ€Π°Π²ΠΎ со ΡΠΊΠΎΡ€ΠΎΡΡ‚ΡŒΡŽ 4 ΠΌ/с?

  • ΠŸΡ€ΠΎΠ²ΠΎΠ΄Π½ΠΈΠΊ MN (рис. 1) с Π΄Π»ΠΈΠ½ΠΎΠΉ Π°ΠΊΡ‚ΠΈΠ²Π½ΠΎΠΉ части 1 ΠΌ ΠΈ сопротивлСниСм 2 Ом находится Π² ΠΎΠ΄Π½ΠΎΡ€ΠΎΠ΄Π½ΠΎΠΌ ΠΌΠ°Π³Π½ΠΈΡ‚Π½ΠΎΠΌ ΠΏΠΎΠ»Π΅ с ΠΈΠ½Π΄ΡƒΠΊΡ†ΠΈΠ΅ΠΉ 1 Π’Π», ΠŸΡ€ΠΎΠ²ΠΎΠ΄Π½ΠΈΠΊ ΠΏΠΎΠ΄ΠΊΠ»ΡŽΡ‡Π΅Π½ ΠΊ источнику с Π­Π”Π‘ 1 Π’ ( Π²Π½ΡƒΡ‚Ρ€Π΅Π½Π½ΠΈΠΌ сопротивлСниСм источника ΠΈ сопротивлСниСм подводящих ΠΏΡ€ΠΎΠ²ΠΎΠ΄ΠΎΠ² ΠΏΡ€Π΅Π½Π΅Π±Ρ€Π΅Ρ‡ΡŒ). Ка­кова сила Ρ‚ΠΎΠΊΠ° Π² ΠΏΡ€ΠΎΠ²ΠΎΠ΄Π½ΠΈΠΊΠ΅, Ссли ΠΏΡ€ΠΎΠ²ΠΎΠ΄Π½ΠΈΠΊ двиТСтся Π²Π»Π΅Π²ΠΎ со ΡΠΊΠΎΡ€ΠΎΡΡ‚ΡŒΡŽ 4 ΠΌ/с?

  • Рис. 1

  • ΠŸΡ€ΠΎΠ²ΠΎΠ΄Π½ΠΈΠΊ MN (рис. 1) с Π΄Π»ΠΈΠ½ΠΎΠΉ Π°ΠΊΡ‚ΠΈΠ²Π½ΠΎΠΉ части 1 ΠΌ ΠΈ сопротивлСниСм 2 Ом находится Π² ΠΎΠ΄Π½ΠΎΡ€ΠΎΠ΄Π½ΠΎΠΌ ΠΌΠ°Π³Π½ΠΈΡ‚Π½ΠΎΠΌ ΠΏΠΎΠ»Π΅ с ΠΈΠ½Π΄ΡƒΠΊΡ†ΠΈΠ΅ΠΉ 1 Π’Π». ΠŸΡ€ΠΎΠ²ΠΎΠ΄Π½ΠΈΠΊ ΠΏΠΎΠ΄ΠΊΠ»ΡŽΡ‡Π΅Π½ ΠΊ источнику с Π­Π”Π‘ 1 Π’ (Π²Π½ΡƒΡ‚Ρ€Π΅Π½Π½ΠΈΠΌ сопротивлСниСм источника ΠΈ сопротивлСниСм подводящих ΠΏΡ€ΠΎΠ²ΠΎΠ΄ΠΎΠ² ΠΏΡ€Π΅Π½Π΅Π±Ρ€Π΅Ρ‡ΡŒ). Π’ ΠΊΠ°ΠΊΠΎΠΌ Π½Π°ΠΏΡ€Π°Π²Π»Π΅Π½ΠΈΠΈ ΠΈ с ΠΊΠ°ΠΊΠΎΠΉ ΡΠΊΠΎΡ€ΠΎΡΡ‚ΡŒΡŽ Π½Π°Π΄ΠΎ ΠΏΠ΅Ρ€Π΅ΠΌΠ΅Ρ‰Π°Ρ‚ΡŒ ΠΏΡ€ΠΎΠ²ΠΎΠ΄Π½ΠΈΠΊ, Ρ‡Ρ‚ΠΎΠ±Ρ‹ Ρ‡Π΅Ρ€Π΅Π· Π½Π΅Π³ΠΎ Π½Π΅ шСл Ρ‚ΠΎΠΊ?

  • Замкнутая ΠΊΠ°Ρ‚ΡƒΡˆΠΊΠ° Π΄ΠΈΠ°ΠΌΠ΅Ρ‚Ρ€ΠΎΠΌ D с числом Π²ΠΈΡ‚ΠΊΠΎΠ² Π½Π° Π΅Π΄ΠΈΠ½ΠΈΡ†Ρƒ Π΄Π»ΠΈΠ½Ρ‹ ΠΊΠ°Ρ‚ΡƒΡˆΠΊΠΈ, Ρ€Π°Π²Π½Ρ‹ΠΌ n, ΠΏΠΎΠΌΠ΅Ρ‰Π΅Π½Π° Π² ΠΎΠ΄Π½ΠΎΡ€ΠΎΠ΄Π½ΠΎΠ΅ ΠΌΠ°Π³Π½ΠΈΡ‚Π½ΠΎΠ΅ ΠΏΠΎΠ»Π΅ с ΠΈΠ½Π΄ΡƒΠΊΡ†ΠΈΠ΅ΠΉ Π’. ΠŸΠ»ΠΎΡΠΊΠΎΡΡ‚ΡŒ ΠΊΠ°Ρ‚ΡƒΡˆΠΊΠΈ пСрпСндикулярна ΠΊ линиям ΠΈΠ½Π΄ΡƒΠΊΡ†ΠΈΠΈ поля. Какой заряд ΠΏΠΎΡ‚Π΅Ρ‡Π΅Ρ‚ ΠΏΠΎ Ρ†Π΅Π½ΠΈ ΠΊΠ°Ρ‚ΡƒΡˆΠΊΠΈ, Ссли Π΅Π΅ ΠΏΠΎΠ²Π΅Ρ€Π½ΡƒΡ‚ΡŒ Π½Π° 180Β°? ΠŸΡ€ΠΎΠ²ΠΎΠ»ΠΎΠΊΠ°, ΠΈΠ· ΠΊΠΎΡ‚ΠΎΡ€ΠΎΠΉ Π½Π°ΠΌΠΎΡ‚Π°Π½Π° ΠΊΠ°Ρ‚ΡƒΡˆΠΊΠ°, ΠΈΠΌΠ΅Π΅Ρ‚ ΠΏΠ»ΠΎΡ‰Π°Π΄ΡŒ сСчСния S ΠΈ ΡƒΠ΄Π΅Π»ΡŒΠ½ΠΎΠ΅ сопротивлСниС Ρ€.

  • Π’ ΠΌΠ°Π³Π½ΠΈΡ‚Π½ΠΎΠΌ ΠΏΠΎΠ»Π΅, индукция ΠΊΠΎΡ‚ΠΎΡ€ΠΎΠ³ΠΎ 0,05 Π’Π», ΠΏΠΎΠΌΠ΅Ρ‰Π΅Π½Π° ΠΊΠ°Ρ‚ΡƒΡˆΠΊΠ°, состоящая ΠΈΠ· 200 Π²ΠΈΡ‚ΠΊΠΎΠ² ΠΏΡ€ΠΎΠ²ΠΎΠ»ΠΎΠΊΠΈ. Π‘ΠΎΠΏΡ€ΠΎΡ‚ΠΈΠ²Π»Π΅Π½ΠΈΠ΅ ΠΊΠ°Ρ‚ΡƒΡˆΠΊΠΈ 40 Ом; ΠΏΠ»ΠΎΡ‰Π°Π΄ΡŒ ΠΏΠΎΠΏΠ΅Ρ€Π΅Ρ‡Π½ΠΎΠ³ΠΎ сСчСния 12 см2. ΠšΠ°Ρ‚ΡƒΡˆΠΊΠ° ΠΏΠΎΠΌΠ΅Ρ‰Π΅Π½Π° Ρ‚Π°ΠΊ, Ρ‡Ρ‚ΠΎ Π΅Π΅ ось составляСт ΡƒΠ³ΠΎΠ» 60Β° с Π½Π°ΠΏΡ€Π°Π²Π»Π΅Π½ΠΈΠ΅ΠΌ ΠΌΠ°Π³Π½ΠΈΡ‚Π½ΠΎΠ³ΠΎ поля. Какой заряд ΠΏΡ€ΠΎΠΉΠ΄Π΅Ρ‚ ΠΏΠΎ ΠΊΠ°Ρ‚ΡƒΡˆΠΊΠ΅ ΠΏΡ€ΠΈ исчСзновСнии ΠΌΠ°Π³Π½ΠΈΡ‚Π½ΠΎΠ³ΠΎ поля?

  • ΠšΠ°Ρ‚ΡƒΡˆΠΊΠ° Π΄ΠΈΠ°ΠΌΠ΅Ρ‚Ρ€ΠΎΠΌ 0,4 ΠΌ находится Π² ΠΏΠ΅Ρ€Π΅ΠΌΠ΅Π½Π½ΠΎΠΌ ΠΌΠ°Π³Π½ΠΈΡ‚Π½ΠΎΠΌ ΠΏΠΎΠ»Π΅. ΠŸΡ€ΠΈ ΠΈΠ·ΠΌΠ΅Π½Π΅Π½ΠΈΠΈ ΠΌΠ°Π³Π½ΠΈΡ‚Π½ΠΎΠ³ΠΎ поля Π½Π° 127,4 Π’Π» Π² Ρ‚Π΅Ρ‡Π΅Π½ΠΈΠΈ 2 с Π² ΠΎΠ±ΠΌΠΎΡ‚ΠΊΠ΅ ΠΊΠ°Ρ‚ΡƒΡˆΠΊΠΈ возбуТдаСтся Π­Π”Π‘ ΠΈΠ½Π΄ΡƒΠΊΡ†ΠΈΠΈ 200 Π’. Бколько Π²ΠΈΡ‚ΠΊΠΎΠ² ΠΈΠΌΠ΅Π΅Ρ‚ ΠΊΠ°Ρ‚ΡƒΡˆΠΊΠ°, Ссли Π²Π΅ΠΊΡ‚ΠΎΡ€ ΠΌΠ°Π³Π½ΠΈΡ‚Π½ΠΎΠΉ ΠΈΠ½Π΄ΡƒΠΊΡ†ΠΈΠΈ Π½Π°ΠΏΡ€Π°Π²Π»Π΅Π½ вдоль оси ΠΊΠ°Ρ‚ΡƒΡˆΠΊΠΈ?

  • ΠšΠ°Ρ‚ΡƒΡˆΠΊΠ° Π΄ΠΈΠ°ΠΌΠ΅Ρ‚Ρ€ΠΎΠΌ 10 см, состоящая ΠΈΠ· 750 Π²ΠΈΡ‚ΠΊΠΎΠ² ΠΏΡ€ΠΎΠ²ΠΎΠ»ΠΎΠΊΠΈ, находится Π² ΠΌΠ°Π³Π½ΠΈΡ‚Π½ΠΎΠΌ ΠΏΠΎΠ»Π΅. Найти ΡΡ€Π΅Π΄Π½ΡŽΡŽ Π­Π”Π‘ ΠΈΠ½Π΄ΡƒΠΊΡ†ΠΈΠΈ, Π²ΠΎΠ·Π½ΠΈΠΊΠ°ΡŽΡ‰ΡƒΡŽ Π² этой ΠΊΠ°Ρ‚ΡƒΡˆΠΊΠ΅, Ссли индукция ΠΌΠ°Π³Π½ΠΈΡ‚Π½ΠΎΠ³ΠΎ поля увСличиваСтся Π² Ρ‚Π΅Ρ‡Π΅Β­Π½ΠΈΠ΅ 0,1 с ΠΎΡ‚ 2 Π΄ΠΎ 8 Π’Π». Π’Π΅ΠΊΡ‚ΠΎΡ€ ΠΌΠ°Π³Π½ΠΈΡ‚Π½ΠΎΠΉ ΠΈΠ½Π΄ΡƒΠΊΡ†ΠΈΠΈ Π½Π°ΠΏΡ€Π°Π²Π»Π΅Π½ вдоль оси ΠΊΠ°Ρ‚ΡƒΡˆΠΊΠΈ.

  • ΠšΠ°Ρ‚ΡƒΡˆΠΊΠ° Π΄ΠΈΠ°ΠΌΠ΅Ρ‚Ρ€ΠΎΠΌ 10 см, ΠΈΠΌΠ΅ΡŽΡ‰Π°Ρ 500 Π²ΠΈΡ‚ΠΊΠΎΠ², находится Π² ΠΌΠ°Π³Π½ΠΈΡ‚Π½ΠΎΠΌ ΠΏΠΎΠ»Π΅. Π§Π΅ΠΌΡƒ Π±ΡƒΠ΄Π΅Ρ‚ Ρ€Π°Π²Π½ΠΎ срСднСС Π·Π½Π°Ρ‡Π΅Π½ΠΈΠ΅ Π­Π”Π‘ ΠΈΠ½Π΄ΡƒΠΊΡ†ΠΈΠΈ Π² этой ΠΊΠ°Ρ‚ΡƒΡˆΠΊΠ΅, Ссли индукция ΠΌΠ°Π³Π½ΠΈΡ‚Π½ΠΎΠ³ΠΎ поля увСличиваСтся Π² Ρ‚Π΅Ρ‡Π΅Π½ΠΈΠΈ 0,1 с ΠΎΡ‚ 0 Π΄ΠΎ 2 Π’Π». Π’Π΅ΠΊΡ‚ΠΎΡ€ ΠΌΠ°Π³Π½ΠΈΡ‚Π½ΠΎΠΉ ΠΈΠ½Π΄ΡƒΠΊΡ†ΠΈΠΈ Π½Π°ΠΏΡ€Π°Π²Π»Π΅Π½ вдоль оси ΠΊΠ°Ρ‚ΡƒΡˆΠΊΠΈ.

  • B ΠΎΠ΄Π½ΠΎΡ€ΠΎΠ΄Π½ΠΎΠΌ ΠΌΠ°Π³Π½ΠΈΡ‚Π½ΠΎΠΌ ΠΏΠΎΠ»Π΅, индукция ΠΊΠΎΡ‚ΠΎΡ€ΠΎΠ³ΠΎ 0,1 Π’Π», вращаСтся ΠΊΠ°Ρ‚ΡƒΡˆΠΊΠ°, состоящая ΠΈΠ· 200 Π²ΠΈΡ‚ΠΊΠΎΠ². Ось вращСния ΠΊΠ°Ρ‚ΡƒΡˆΠΊΠΈ пСрпСндикулярна ΠΊ Π΅Π΅ оси ΠΈ ΠΊ Π½Π°ΠΏΡ€Π°Π²Π»Π΅Π½ΠΈΡŽ поля. ΠŸΠ΅Ρ€ΠΈΠΎΠ΄ обращСния ΠΊΠ°Ρ‚ΡƒΡˆΠΊΠΈ 0,2 с, ΠΏΠ»ΠΎΡ‰Π°Π΄ΡŒ ΠΏΠΎΠΏΠ΅Ρ€Π΅Ρ‡Π½ΠΎΠ³ΠΎ сСчСния ΠΊΠ°Ρ‚ΡƒΡˆΠΊΠΈ 4 см2. Найти ΠΌΠ°ΠΊΡΠΈΠΌΠ°Π»ΡŒΠ½ΡƒΡŽ Π­Π”Π‘ ΠΈΠ½Π΄ΡƒΠΊΡ†ΠΈΠΈ Π²ΠΎ Π²Ρ€Π°Ρ‰Π°ΡŽΡ‰Π΅ΠΉΡΡ ΠΊΠ°Ρ‚ΡƒΡˆΠΊΠ΅.

  • Π’ ΠΎΠ΄Π½ΠΎΡ€ΠΎΠ΄Π½ΠΎΠΌ ΠΌΠ°Π³Π½ΠΈΡ‚Π½ΠΎΠΌ ΠΏΠΎΠ»Π΅, индукция ΠΊΠΎΡ‚ΠΎΡ€ΠΎΠ³ΠΎ 0,1 Π’Π», Ρ€Π°Π²Π½ΠΎΠΌΠ΅Ρ€Π½ΠΎ, вращаСтся ΠΊΠ°Ρ‚ΡƒΡˆΠΊΠ°, состоящая ΠΈΠ· 100 Π²ΠΈΡ‚ΠΊΠΎΠ² ΠΏΡ€ΠΎΠ²ΠΎΠ»ΠΎΠΊΠΈ. ΠšΠ°Ρ‚ΡƒΡˆΠΊΠ° Π΄Π΅Π»Π°Π΅Ρ‚ 5 ΠΎΠ±/с. ΠŸΠ»ΠΎΡ‰Π°Π΄ΡŒ ΠΏΠΎΠΏΠ΅Ρ€Π΅Ρ‡Π½ΠΎΠ³ΠΎ сСчСния ΠΊΠ°Ρ‚ΡƒΡˆΠΊΠΈ 100 см2. Ось вращСния ΠΊΠ°Ρ‚ΡƒΡˆΠΊΠΈ пСрпСндикулярна оси ΠΊΠ°Ρ‚ΡƒΡˆΠΊΠΈ ΠΈ Π½Π°ΠΏΡ€Π°Π²Π»Π΅Π½ΠΈΡŽ ΠΌΠ°Π³Π½ΠΈΡ‚Π½ΠΎΠ³ΠΎ ΠΏΠΎΒ­Π»Π°. Найти ΠΌΠ°ΠΊΡΠΈΠΌΠ°Π»ΡŒΠ½ΡƒΡŽ Π­Π”Π‘ ΠΈΠ½Π΄ΡƒΠΊΡ†ΠΈΠΈ Π²ΠΎ Π²Ρ€Π°Ρ‰Π°ΡŽΡ‰Π΅ΠΉΡΡ ΠΊΠ°Ρ‚ΡƒΡˆΠΊΠ΅.

  • ΠšΠΎΡ€ΠΎΡ‚ΠΊΠΎΠ·Π°ΠΌΠΊΠ½ΡƒΡ‚Π°Ρ ΠΊΠ°Ρ‚ΡƒΡˆΠΊΠ°, состоящая ΠΈΠ· 1000 Π²ΠΈΡ‚ΠΊΠΎΠ² ΠΏΡ€ΠΎΠ²ΠΎΠ»ΠΎΠΊΠΈ, помСшСна Π² ΠΌΠ°Π³Π½ΠΈΡ‚Π½ΠΎΠ΅ ΠΏΠΎΠ»Π΅, Π»ΠΈΠ½ΠΈΠΈ ΠΈΠ½Π΄ΡƒΠΊΡ†ΠΈΠΈ ΠΊΠΎΡ‚ΠΎΡ€ΠΎΠ³ΠΎ Π½Π°ΠΏΡ€Π°Π²Π»Π΅Π½Ρ‹ вдоль оси ΡΠ»ΡƒΡˆΠΊΠΈ. ΠŸΠ»ΠΎΡ‰Π°Π΄ΡŒ ΠΏΠΎΠΏΠ΅Ρ€Π΅Ρ‡Π½ΠΎΠ³ΠΎ сСчСния ΠΊΠ°Ρ‚ΡƒΡˆΠΊΠΈ 40 см2, Π΅Π΅ сопротивлСниС 160 Ом. Найти ΠΌΠΎΡ‰Π½ΠΎΡΡ‚ΡŒ Ρ‚Π΅ΠΏΠ»ΠΎΠ²Ρ‹Ρ… ΠΏΠΎΡ‚Π΅Ρ€ΡŒ, Ссли индукция ΠΌΠ°Π³Π½ΠΈΡ‚Π½ΠΎΠ³ΠΎ по­ля Ρ€Π°Π²Π½ΠΎΠΌΠ΅Ρ€Π½ΠΎ измСняСтся Π²ΠΎ Π²Ρ€Π΅ΠΌΠ΅Π½ΠΈ со ΡΠΊΠΎΡ€ΠΎΡΡ‚ΡŒΡŽ 10-3 Π’Π»/с.

  • ΠšΠ°Ρ‚ΡƒΡˆΠΊΠ° ΠΈΠΌΠ΅Π΅Ρ‚ 1000 Π²ΠΈΡ‚ΠΊΠΎΠ² ΠΏΡ€ΠΎΠ²ΠΎΠ»ΠΎΠΊΠΈ ΠΈ находится Π² ΠΏΠ΅Ρ€Π΅ΠΌΠ΅Π½Π½ΠΎΠΌ ΠΌΠ°Π³Β­Π½ΠΈΡ‚Π½ΠΎΠΌ ΠΏΠΎΠ»Π΅. ΠŸΡ€ΠΈ ΠΈΠ·ΠΌΠ΅Π½Π΅Π½ΠΈΠΈ ΠΈΠ½Π΄ΡƒΠΊΡ†ΠΈΠΈ поля с 50 Π΄ΠΎ 100 Π’Π» Π² Ρ‚Π΅Ρ‡Π΅Π½ΠΈΠΈ 5 с Π² ΠΎΠ±ΠΌΠΎΡ‚ΠΊΠ΅ ΠΊΠ°Ρ‚ΡƒΡˆΠΊΠΈ возбуТдаСтся Π­Π”Π‘ ΠΈΠ½Π΄ΡƒΠΊΡ†ΠΈΠΈ 200 Π’. Каков Π΄ΠΈΠ°ΠΌΠ΅Ρ‚Ρ€ ΠΊΠ°Ρ‚ΡƒΡˆΠΊΠΈ, Ссли Π²Π΅ΠΊΡ‚ΠΎΡ€ ΠΌΠ°Π³Π½ΠΈΡ‚Π½ΠΎΠΉ ΠΈΠ½Π΄ΡƒΠΊΡ†ΠΈΠΈ Π½Π°ΠΏΡ€Π°Π²Π»Π΅Π½ вдоль оси ΠΊΠ°Ρ‚ΡƒΡˆΠΊΠΈ?

  • ΠšΠ°Ρ‚ΡƒΡˆΠΊΠ°, намотанная Π½Π° Π½Π΅ΠΌΠ°Π³Π½ΠΈΡ‚Π½Ρ‹ΠΉ каркас Π² Ρ„ΠΎΡ€ΠΌΠ΅ Ρ†ΠΈΠ»ΠΈΠ½Π΄Ρ€Π°, ΠΈΠΌΠ΅Π΅Ρ‚ 750 Π²ΠΈΡ‚ΠΊΠΎΠ² ΠΈ ΠΈΠ½Π΄ΡƒΠΊΡ‚ΠΈΠ²Π½ΠΎΡΡ‚ΡŒ 25 ΠΌΠ“Π½. Π§Ρ‚ΠΎΠ±Ρ‹ ΡƒΠ²Π΅Π»ΠΈΡ‡ΠΈΡ‚ΡŒ ΠΈΠ½Π΄ΡƒΠΊΡ‚ΠΈΠ²Π½ΠΎΡΡ‚ΡŒ ΠΊΠ°Ρ‚ΡƒΡˆΠΊΠΈ Π΄ΠΎ 36 ΠΌΠ“Π½, ΠΎΠ±ΠΌΠΎΡ‚ΠΊΡƒ с ΠΊΠ°Ρ‚ΡƒΡˆΠΊΠΈ сняли ΠΈ Π·Π°ΠΌΠ΅Π½ΠΈΠ»ΠΈ ΠΎΠ±ΠΌΠΎΡ‚ΠΊΠΎΠΉ ΠΈΠ· Π±ΠΎΠ»Π΅Π΅ Ρ‚ΠΎΠ½ΠΊΠΎΠΉ ΠΏΡ€ΠΎΠ²ΠΎΠ»ΠΎΠΊΠΈ с Ρ‚Π°ΠΊΠΈΠΌ расчСтом, Ρ‡Ρ‚ΠΎΠ±Ρ‹ Π΄Π»ΠΈΠ½Π° ΠΊΠ°Ρ‚ΡƒΡˆΠΊΠΈ ΠΎΡΡ‚Π°Β­Π»Π°ΡΡŒ ΠΏΡ€Π΅ΠΆΠ½Π΅ΠΉ. Бколько Π²ΠΈΡ‚ΠΊΠΎΠ² оказалось Π² ΠΊΠ°Ρ‚ΡƒΡˆΠΊΠ΅ послС ΠΏΠ΅Ρ€Π΅ΠΌΠΎΡ‚ΠΊΠΈ?

  • Бколько Π²ΠΈΡ‚ΠΊΠΎΠ² ΠΏΡ€ΠΎΠ²ΠΎΠ»ΠΎΠΊΠΈ Π΄ΠΈΠ°ΠΌΠ΅Ρ‚Ρ€ΠΎΠΌ 0,4 ΠΌΠΌ с изоляциСй Π½ΠΈΡ‡Ρ‚ΠΎΠΆΠ½ΠΎΠΉ Ρ‚ΠΎΠ»Ρ‰ΠΈΠ½Ρ‹ Π½ΡƒΠΆΠ½ΠΎ Π½Π°ΠΌΠΎΡ‚Π°Ρ‚ΡŒ Π½Π° ΠΊΠ°Ρ€Ρ‚ΠΎΠ½Π½Ρ‹ΠΉ Ρ†ΠΈΠ»ΠΈΠ½Π΄Ρ€ Π΄ΠΈΠ°ΠΌΠ΅Ρ‚Ρ€ΠΎΠΌ 2 см, Ρ‡Ρ‚ΠΎΠ±Ρ‹ ΠΏΠΎΠ»ΡƒΡ‡ΠΈΡ‚ΡŒ ΠΎΠ΄Π½ΠΎΡΠ»ΠΎΠΉΠ½ΡƒΡŽ ΠΊΠ°Ρ‚ΡƒΡˆΠΊΡƒ с ΠΈΠ½Π΄ΡƒΠΊΡ‚ΠΈΠ²Π½ΠΎΡΡ‚ΡŒΡŽ 1 ΠΌΠ“Π½? Π’ΠΈΡ‚ΠΊΠΈ Π²ΠΏΠ»ΠΎΡ‚Β­Π½ΡƒΡŽ ΠΏΡ€ΠΈΠ»Π΅Π³Π°ΡŽΡ‚ Π΄Ρ€ΡƒΠ³ ΠΊ Π΄Ρ€ΡƒΠ³Ρƒ.

  • Найти ΠΈΠ½Π΄ΡƒΠΊΡ‚ΠΈΠ²Π½ΠΎΡΡ‚ΡŒ солСноида, ΠΏΠΎΠ»ΡƒΡ‡Π΅Π½Π½ΠΎΠ³ΠΎ ΠΏΡ€ΠΈ Π½Π°ΠΌΠΎΡ‚ΠΊΠ΅ ΠΏΡ€ΠΎΠ²ΠΎΠ΄Π° Π΄Π»ΠΈΠ½ΠΎΠΉ 10 ΠΌ Π½Π° цилиндричСский ΠΆΠ΅Π»Π΅Π·Π½Ρ‹ΠΉ ΡΡ‚Π΅Ρ€ΠΆΠ΅Π½ΡŒ Π΄Π»ΠΈΠ½ΠΎΠΉ 10 см. ΠžΡ‚Π½ΠΎΡΠΈΡ‚Π΅Π»ΡŒΠ½ΡƒΡŽ ΠΌΠ°Π³Π½ΠΈΡ‚Π½ΡƒΡŽ ΠΏΡ€ΠΎΠ½ΠΈΡ†Π°Π΅ΠΌΠΎΡΡ‚ΡŒ ΠΆΠ΅Π»Π΅Π·Π° ΠΏΡ€ΠΈΠ½ΡΡ‚ΡŒ Ρ€Π°Π²Π½ΠΎΠΉ 400.

  • Π˜Π½Π΄ΡƒΠΊΡ‚ΠΈΠ²Π½ΠΎΡΡ‚ΡŒ солСноида, Π½Π°ΠΌΠΎΡ‚Π°Π½Π½ΠΎΠ³ΠΎ Π² ΠΎΠ΄ΠΈΠ½ слой Π½Π° Π½Π΅ΠΌΠ°Π³Π½ΠΈΡ‚Π½Ρ‹ΠΉ каркас, 1,6 ΠΌΠ“Π½, Π”Π»ΠΈΠ½Π° солСноида 1 ΠΌ, сСчСниС Ρ€Π°Π²Π½ΠΎ 20 см2. Бколько Π²ΠΈΡ‚ΠΊΠΎΠ² приходится Π½Π° ΠΊΠ°ΠΆΠ΄Ρ‹ΠΉ сантимСтр Π΄Π»ΠΈΠ½Ρ‹ солСноида?

  • Бколько Π²ΠΈΡ‚ΠΊΠΎΠ² ΠΏΡ€ΠΎΠ²ΠΎΠ»ΠΎΠΊΠΈ Π΄ΠΈΠ°ΠΌΠ΅Ρ‚Ρ€ΠΎΠΌ 0,6 ΠΌΠΌ ΠΈΠΌΠ΅Π΅Ρ‚ однослойная ΠΎΠ±ΠΌΠΎΡ‚ΠΊΠ° ΠΊΠ°Ρ‚ΡƒΡˆΠΊΠΈ, ΠΈΠ½Π΄ΡƒΠΊΡ‚ΠΈΠ²Π½ΠΎΡΡ‚ΡŒ ΠΊΠΎΡ‚ΠΎΡ€ΠΎΠΉ 1 ΠΌΠ“Π½ ΠΈ Π΄ΠΈΠ°ΠΌΠ΅Ρ‚Ρ€ 4 см? Π’ΠΈΡ‚ΠΊΠΈ ΠΏΠ»ΠΎΡ‚Π½ΠΎ ΠΏΡ€ΠΈΠ»Π΅Π³Π°ΡŽΡ‚ Π΄Ρ€ΡƒΠ³ ΠΊ Π΄Ρ€ΡƒΠ³Ρƒ.

  • Π§Π΅Ρ€Π΅Π· ΠΊΠ°Ρ‚ΡƒΡˆΠΊΡƒ радиусом R = 2 см, ΡΠΎΠ΄Π΅Ρ€ΠΆΠ°Ρ‰ΡƒΡŽ N = 500 Π²ΠΈΡ‚ΠΊΠΎΠ², ΠΏΡ€ΠΎΡ…ΠΎΠ΄ΠΈΡ‚ постоянный Ρ‚ΠΎΠΊ I = 5 А. ΠžΠΏΡ€Π΅Π΄Π΅Π»ΠΈΡ‚ΡŒ ΠΈΠ½Π΄ΡƒΠΊΡ‚ΠΈΠ²Π½ΠΎΡΡ‚ΡŒ ΠΊΠ°Ρ‚ΡƒΡˆΠΊΠΈ, Ссли Π½Π°ΠΏΡ€ΡΠΆΠ΅Π½Π½ΠΎΡΡ‚ΡŒ ΠΌΠ°Π³Π½ΠΈΡ‚Π½ΠΎΠ³ΠΎ поля Π² Π΅Π΅ Ρ†Π΅Π½Ρ‚Ρ€Π΅ Н= 104 А/ΠΌ.

  • ΠžΠ±ΠΌΠΎΡ‚ΠΊΠ° солСноида состоит ΠΈΠ· N Π²ΠΈΡ‚ΠΊΠΎΠ² ΠΌΠ΅Π΄Π½ΠΎΠΉ ΠΏΡ€ΠΎΠ²ΠΎΠ»ΠΎΠΊΠΈ, ΠΏΠΎΠΏΠ΅Ρ€Π΅Ρ‡Π½ΠΎΠ΅ Β«Ρ‚Π΅Ρ‡Π΅Π½ΠΈΠ΅ ΠΊΠΎΡ‚ΠΎΡ€ΠΎΠΉ S=1 ΠΌΠΌ2. Π”Π»ΠΈΠ½Π° солСноида I =25 см; Π΅Π³ΠΎ сопротивлС­ниС R = 0,2 Ом. Найти ΠΈΠ½Π΄ΡƒΠΊΡ‚ΠΈΠ²Π½ΠΎΡΡ‚ΡŒ L солСноида.

  • ΠšΠ°Ρ‚ΡƒΡˆΠΊΠ° Π΄Π»ΠΈΠ½ΠΎΠΉ 1 = 20 см ΠΈ Π΄ΠΈΠ°ΠΌΠ΅Ρ‚Ρ€ΠΎΠΌ D = 3 см ΠΈΠΌΠ΅Π΅Ρ‚ N = 400 Π²ΠΈΡ‚ΠΊΠΎΠ². По ΠΊΠ°Ρ‚ΡƒΡˆΠΊΠ΅ ΠΈΠ΄Π΅Ρ‚ Ρ‚ΠΎΠΊ I = 2А. Найти ΠΈΠ½Π΄ΡƒΠΊΡ‚ΠΈΠ²Π½ΠΎΡΡ‚ΡŒ L ΠΊΠ°Ρ‚ΡƒΡˆΠΊΠΈ ΠΈ ΠΌΠ°Π³Π½ΠΈΡ‚Β­Π½Ρ‹ΠΉ ΠΏΠΎΡ‚ΠΎΠΊ Π€, ΠΏΡ€ΠΎΠ½ΠΈΠ·Ρ‹Π²Π°ΡŽΡ‰ΠΈΠΉ ΠΏΠ»ΠΎΡ‰Π°Π΄ΡŒ Π΅Π΅ ΠΏΠΎΠΏΠ΅Ρ€Π΅Ρ‡Π½ΠΎΠ³ΠΎ сСчСния.

  • Бколько Π²ΠΈΡ‚ΠΊΠΎΠ² ΠΏΡ€ΠΎΠ²ΠΎΠ»ΠΎΠΊΠΈ Π΄ΠΈΠ°ΠΌΠ΅Ρ‚Ρ€ΠΎΠΌ d = 0,6 ΠΌΠΌ ΠΈΠΌΠ΅Π΅Ρ‚ однослойная ΠΎΠ±ΠΌΠΎΡ‚ΠΊΠ° ΠΊΠ°Ρ‚ΡƒΡˆΠΊΠΈ, ΠΈΠ½Π΄ΡƒΠΊΡ‚ΠΈΠ²Π½ΠΎΡΡ‚ΡŒ ΠΊΠΎΡ‚ΠΎΡ€ΠΎΠΉ L = 1 ΠΌΠ“Π½ ΠΈ Π΄ΠΈΠ°ΠΌΠ΅Ρ‚Ρ€ D= 4 см? Π’ΠΈΡ‚ΠΊΠΈ ΠΏΠ»ΠΎΡ‚Π½ΠΎ ΠΏΡ€ΠΈΠ»Π΅Π³Π°ΡŽΡ‚ Π΄Ρ€ΡƒΠ³ ΠΊ Π΄Ρ€ΡƒΠ³Ρƒ.

  • Бколько Π²ΠΈΡ‚ΠΊΠΎΠ² ΠΈΠΌΠ΅Π΅Ρ‚ ΠΊΠ°Ρ‚ΡƒΡˆΠΊΠ° с ΠΈΠ½Π΄ΡƒΠΊΡ‚ΠΈΠ²Π½ΠΎΡΡ‚ΡŒΡŽ 10 ΠΌΠ“Π½, Ссли ΠΏΡ€ΠΈ силС Ρ‚ΠΎΠΊΠ° 2А ΠΌΠ°Π³Π½ΠΈΡ‚Π½Ρ‹ΠΉ ΠΏΠΎΡ‚ΠΎΠΊ сквозь ΠΎΠ΄ΠΈΠ½ Π²ΠΈΡ‚ΠΎΠΊ ΠΊΠ°Ρ‚ΡƒΡˆΠΊΠΈ Ρ€Π°Π²Π΅Π½ 5 ΠΌΠΊΠ’Π±?

  • Бколько Π²ΠΈΡ‚ΠΊΠΎΠ² ΠΈΠΌΠ΅Π΅Ρ‚ ΠΊΠ°Ρ‚ΡƒΡˆΠΊΠ°, ΠΈΠ½Π΄ΡƒΠΊΡ‚ΠΈΠ²Π½ΠΎΡΡ‚ΡŒ ΠΊΠΎΡ‚ΠΎΡ€ΠΎΠΉ Ρ€Π°Π²Π½Π° L = 1 ΠΌΠ“Π½, Ссли ΠΏΡ€ΠΈ Ρ‚ΠΎΠΊΠ΅ Π² I = 1 А ΠΌΠ°Π³Π½ΠΈΡ‚Π½Ρ‹ΠΉ ΠΏΠΎΡ‚ΠΎΠΊ сквозь ΠΎΠ΄ΠΈΠ½ Π²ΠΈΡ‚ΠΎΠΊ ΠΊΠ°Ρ‚ΡƒΡˆΠΊΠΈ Ρ€Π°Π²Π΅Π½ Π€ = 2 ΠΌΠΊΠ’Π±?

  • ΠšΠ°Ρ‚ΡƒΡˆΠΊΠ° Π΄Π»ΠΈΠ½ΠΎΠΉ 0,5 ΠΌ ΠΈ Π΄ΠΈΠ°ΠΌΠ΅Ρ‚Ρ€ΠΎΠΌ 2 см ΠΈΠΌΠ΅Π΅Ρ‚ 600 Π²ΠΈΡ‚ΠΊΠΎΠ². По ΠΊΠ°Ρ‚ΡƒΡˆΠΊΠ΅ ΠΈΠ΄Π΅Ρ‚ Ρ‚ΠΎΠΊ 1 А. Найти ΠΈΠ½Π΄ΡƒΠΊΡ‚ΠΈΠ²Π½ΠΎΡΡ‚ΡŒ ΠΊΠ°Ρ‚ΡƒΡˆΠΊΠΈ ΠΈ ΠΌΠ°Π³Π½ΠΈΡ‚Π½Ρ‹ΠΉ ΠΏΠΎΡ‚ΠΎΠΊ, ΠΏΡ€ΠΎΠ½ΠΈΒ­Π·Ρ‹Π²Π°ΡŽΡ‰ΠΈΠΉ Π΅Π΅ ΠΏΠΎΠΏΠ΅Ρ€Π΅Ρ‡Π½ΠΎΠ΅ сСчСниС.

  • Π’ солСноидС, ΠΈΠ½Π΄ΡƒΠΊΡ‚ΠΈΠ²Π½ΠΎΡΡ‚ΡŒ ΠΊΠΎΡ‚ΠΎΡ€ΠΎΠ³ΠΎ 0,4 Π“Π½ ΠΈ ΠΏΠ»ΠΎΡ‰Π°Π΄ΡŒ ΠΏΠΎΠΏΠ΅Ρ€Π΅Ρ‡Π½ΠΎΠ³ΠΎ сСчСния 10 см2, сила Ρ‚ΠΎΠΊΠ° Ρ€Π°Π²Π½Π° 0,5 А. Какова индукция поля Π²Π½ΡƒΡ‚Ρ€ΠΈ со­лСноида, Ссли ΠΎΠ½ содСрТит 100 Π²ΠΈΡ‚ΠΊΠΎΠ²? ПолС ΡΡ‡ΠΈΡ‚Π°Ρ‚ΡŒ ΠΎΠ΄Π½ΠΎΡ€ΠΎΠ΄Π½Ρ‹ΠΌ.

  • На ΠΊΠ°Ρ€Ρ‚ΠΎΠ½Π½Ρ‹ΠΉ каркас Π΄Π»ΠΈΠ½ΠΎΠΉ 50 см ΠΈ ΠΏΠ»ΠΎΡ‰Π°Π΄ΡŒΡŽ сСчСния 4 см2 Π½Π°ΠΌΠΎΡ‚Π°Π½ Π² ΠΎΠ΄ΠΈΠ½ слой ΠΏΡ€ΠΎΠ²ΠΎΠ΄ Π΄ΠΈΠ°ΠΌΠ΅Ρ‚Ρ€ΠΎΠΌ 0,2 ΠΌΠΌ Ρ‚Π°ΠΊ, Ρ‡Ρ‚ΠΎ Π²ΠΈΡ‚ΠΊΠΈ ΠΏΠ»ΠΎΡ‚Π½ΠΎ ΠΏΡ€ΠΈΠ»Π΅Π³Π°ΡŽΡ‚ Π΄Ρ€ΡƒΠ³ ΠΊ Π΄Ρ€ΡƒΠ³Ρƒ. Π’Ρ‹Ρ‡ΠΈΡΠ»ΠΈΡ‚ΡŒ ΠΈΠ½Π΄ΡƒΠΊΡ‚ΠΈΠ²Π½ΠΎΡΡ‚ΡŒ ΠΏΠΎΠ»ΡƒΡ‡ΠΈΠ²ΡˆΠ΅Π³ΠΎΡΡ солСноида.

  • ΠžΠΏΡ€Π΅Π΄Π΅Π»ΠΈΡ‚ΡŒ ΠΈΠ½Π΄ΡƒΠΊΡ‚ΠΈΠ²Π½ΠΎΡΡ‚ΡŒ Π΄Π²ΡƒΡ…ΠΏΡ€ΠΎΠ²ΠΎΠ΄Π½ΠΎΠΉ Π»ΠΈΠ½ΠΈΠΈ Π½Π° участкС Π΄Π»ΠΈΠ½ΠΎΠΉ 1 ΠΊΠΌ. Радиус ΠΏΡ€ΠΎΠ²ΠΎΠ΄Π° 1 ΠΌΠΌ, расстояниС ΠΌΠ΅ΠΆΠ΄Ρƒ осСвыми линиями 0,4 ΠΌ. (Π£Ρ‡Π΅ΡΡ‚ΡŒ Ρ‚ΠΎΠ»ΡŒΠΊΠΎ Π²Π½ΡƒΡ‚Ρ€Π΅Π½Π½ΠΈΠΉ ΠΏΠΎΡ‚ΠΎΠΊ, Ρ‚.Π΅. ΠΏΠΎΡ‚ΠΎΠΊ, ΠΏΡ€ΠΎΠ½ΠΈΠ·Ρ‹Π²Π°ΡŽΡ‰ΠΈΠΉ ΠΊΠΎΠ½Ρ‚ΡƒΡ€, ΠΎΠ³Ρ€Π°Π½ΠΈΡ‡Π΅Π½Π½Ρ‹ΠΉ ΠΏΡ€ΠΎΠ²ΠΎΠ΄Π°ΠΌΠΈ).

  • На каркас Π΄ΠΈΠ°ΠΌΠ΅Ρ‚Ρ€ΠΎΠΌ 0,1 ΠΌ Π½Π°ΠΌΠΎΡ‚Π°Π½ солСноид, содСрТащий 500 Π²ΠΈΡ‚ΠΊΠΎΠ². ΠŸΡ€ΠΈ ΠΏΠΎΠ΄ΠΊΠ»ΡŽΡ‡Π΅Π½ΠΈΠΈ солСноида ΠΊ аккумулятору с Π­Π”Π‘ 12 Π’ Ρ‡Π΅Ρ€Π΅Π· 0,001 с Ρ‚ΠΎΠΊ Π² Ρ†Π΅Π½ΠΈ достигаСт 2 А. ΠžΠΏΡ€Π΅Π΄Π΅Π»ΠΈΡ‚ΡŒ Π΄Π»ΠΈΠ½Ρƒ солСноида, Ссли Π΅Π³ΠΎ сопротивлСниС 3 Ом, Π° сопротивлСниСм аккумулятора ΠΈ ΠΏΡ€ΠΎΠ²ΠΎΠ΄ΠΎΠ² ΠΌΠΎΠΆΠ½ΠΎ ΠΏΡ€Π΅Π½Π΅Π±Ρ€Π΅Ρ‡ΡŒ.

  • Π‘ΠΎΠ»Π΅Π½ΠΎΠΈΠ΄ Π΄ΠΈΠ°ΠΌΠ΅Ρ‚Ρ€ΠΎΠΌ 3 см ΠΈ содСрТащий 300 Π²ΠΈΡ‚ΠΊΠΎΠ² ΠΏΠΎΠ΄ΠΊΠ»ΡŽΡ‡Π°ΡŽΡ‚ ΠΊ Π±Π°Ρ‚Π°Ρ€Π΅Π΅ с Π²Π½ΡƒΡ‚Ρ€Π΅Π½Π½ΠΈΠΌ сопротивлСниСм 1 Ом. ΠŸΡ€ΠΈ этом Ρ‡Π΅Ρ€Π΅Π· 0,01 с Ρ‚ΠΎΠΊ Π² Ρ†Π΅ΠΏΠΈ достигаСт 1 А. Π‘ΠΎΠΏΡ€ΠΎΡ‚ΠΈΠ²Π»Π΅Π½ΠΈΠ΅ солСноида 10 Ом. Π•Π³ΠΎ Π΄Π»ΠΈΠ½Π° 0,4 ΠΌ, Π° сопротивлСниСм ΠΏΡ€ΠΎΠ²ΠΎΠ΄ΠΎΠ² ΠΌΠΎΠΆΠ½ΠΎ ΠΏΡ€Π΅Π½Π΅Π±Ρ€Π΅Ρ‡ΡŒ. Найти Π­Π”Π‘ Π±Π°Ρ‚Π°Ρ€Π΅ΠΈ.

  • ΠšΠ°Ρ‚ΡƒΡˆΠΊΠ° сопротивлСниСм 0,5 Ом с ΠΈΠ½Π΄ΡƒΠΊΡ‚ΠΈΠ²Π½ΠΎΡΡ‚ΡŒΡŽ 4 ΠΌΠ“Π½ ΠΏΠ°Ρ€Π°Π»Π»Π΅Π»ΡŒΠ½ΠΎ соСдинСна с ΠΏΡ€ΠΎΠ²ΠΎΠ΄Π½ΠΈΠΊΠΎΠΌ сопротивлСниСм 2,5 Ом, ΠΏΠΎ ΠΊΠΎΡ‚ΠΎΡ€ΠΎΠΌΡƒ Ρ‚Π΅Ρ‡Π΅Ρ‚ постоянный Ρ‚ΠΎΠΊ 1 А. ΠžΠΏΡ€Π΅Π΄Π΅Π»ΠΈΡ‚ΡŒ количСство элСктричСства, ΠΊΠΎΡ‚ΠΎΡ€ΠΎΠ΅ Π±ΡƒΒ­Π΄Π΅Ρ‚ ΠΈΠ½Π΄ΡƒΡ†ΠΈΡ€ΠΎΠ²Π°Π½ΠΎ Π² ΠΊΠ°Ρ‚ΡƒΡˆΠΊΠ΅ ΠΏΡ€ΠΈ Ρ€Π°Π·ΠΌΡ‹ΠΊΠ°Π½ΠΈΠΈ Ρ†Π΅ΠΏΠΈ ΠΊΠ»ΡŽΡ‡ΠΎΠΌ К (рис. 2).

  • ΠžΠ±ΠΌΠΎΡ‚ΠΊΠ° солСноида с Π½Π΅ΠΌΠ°Π³Π½ΠΈΡ‚Π½Ρ‹ΠΌ сСрдСчником ΠΈΠΌΠ΅Π΅Ρ‚ 10 Π²ΠΈΡ‚ΠΊΠΎΠ² Π½Π° ΠΊΠ°ΠΆΠ΄Ρ‹ΠΉ сантимСтр Π΄Π»ΠΈΠ½Ρ‹. Π§Π΅ΠΌΡƒ Ρ€Π°Π²Π½Π° ΠΏΠ»ΠΎΡ‚Π½ΠΎΡΡ‚ΡŒ энСргии ΠΌΠ°Π³Π½ΠΈΡ‚Π½ΠΎΠ³ΠΎ поля ΠΏΡ€ΠΈ силС Ρ‚ΠΎΠΊΠ° 16 А?

  • ΠžΠΏΡ€Π΅Π΄Π΅Π»ΠΈΡ‚ΡŒ ΡΠ½Π΅Ρ€Π³ΠΈΡŽ ΠΌΠ°Π³Π½ΠΈΡ‚Π½ΠΎΠ³ΠΎ поля солСноида, содСрТащСго 500 Π²ΠΈΡ‚ΠΊΠΎΠ², ΠΊΠΎΡ‚ΠΎΡ€Ρ‹Π΅ Π½Π°ΠΌΠΎΡ‚Π°Π½Ρ‹ Π½Π° ΠΊΠ°Ρ€Ρ‚ΠΎΠ½Π½Ρ‹ΠΉ каркас радиусом 2 см ΠΈ Π΄Π»ΠΈΒ­ΠΌΠΎΠΉ 0,5 ΠΌ, Ссли ΠΏΠΎ Π½Π΅ΠΌΡƒ Ρ‚Π΅Ρ‡Π΅Ρ‚ Ρ‚ΠΎΠΊ 5 А.

  • ΠžΠ±ΠΌΠΎΡ‚ΠΊΠ° солСноида содСрТит 10 Π²ΠΈΡ‚ΠΊΠΎΠ² Π½Π° ΠΊΠ°ΠΆΠ΄Ρ‹ΠΉ сантимСтр Π΄Π»ΠΈΠ½Ρ‹. Π‘Π΅Ρ€Π΄Π΅Ρ‡Π½ΠΈΠΊ Π½Π΅ΠΌΠ°Π³Π½ΠΈΡ‚Π½Ρ‹ΠΉ. ΠŸΡ€ΠΈ ΠΊΠ°ΠΊΠΎΠΉ силС Ρ‚ΠΎΠΊΠ° ΠΏΠ»ΠΎΡ‚Π½ΠΎΡΡ‚ΡŒ энСргии ΠΌΠ°Π³Β­Π½ΠΈΡ‚Π½ΠΎΠ³ΠΎ поля Ρ€Π°Π²Π½Π° 1 Π”ΠΆ/ΠΌ3?

  • Π‘ΠΎΠ»Π΅Π½ΠΎΠΈΠ΄ содСрТит 1000 Π²ΠΈΡ‚ΠΊΠΎΠ². Π‘ΠΈΠ»Π° Ρ‚ΠΎΠΊΠ° Π² ΠΎΠ±ΠΌΠΎΡ‚ΠΊΠ΅ солСноида 1 А, ΠΌΠ°Π³Π½ΠΈΡ‚Π½Ρ‹ΠΉ ΠΏΠΎΡ‚ΠΎΠΊ Ρ‡Π΅Ρ€Π΅Π· ΠΎΠ΄ΠΈΠ½ Π²ΠΈΡ‚ΠΎΠΊ Ρ€Π°Π²Π΅Π½ 0,01 Π’Π±. Π’Ρ‹Ρ‡ΠΈΡΠ»ΠΈΡ‚ΡŒ ΡΠ½Π΅Ρ€Π³ΠΈΡŽ ΠΌΠ°Π³Π½ΠΈΡ‚Π½ΠΎΠ³ΠΎ поля.

  • Π‘ΠΎΠ»Π΅Π½ΠΎΠΈΠ΄ Π΄Π»ΠΈΠ½ΠΎΠΉ l = 50 см ΠΈ ΠΏΠ»ΠΎΡ‰Π°Π΄ΡŒΡŽ ΠΏΠΎΠΏΠ΅Ρ€Π΅Ρ‡Π½ΠΎΠ³ΠΎ сСчСния S = 2 см2 ΠΈΠΌΠ΅Π΅Ρ‚ ΠΈΠ½Π΄ΡƒΠΊΡ‚ΠΈΠ²Π½ΠΎΡΡ‚ΡŒ L = 0,2 ΠΌΠΊΠ“Π½. ΠŸΡ€ΠΈ ΠΊΠ°ΠΊΠΎΠΌ Ρ‚ΠΎΠΊΠ΅ I объСмная ΠΏΠ»ΠΎΡ‚Β­Π½ΠΎΡΡ‚ΡŒ энСргии ΠΌΠ°Π³Π½ΠΈΡ‚Π½ΠΎΠ³ΠΎ поля Π²Π½ΡƒΡ‚Ρ€ΠΈ солСноида Ο‰ΠΎ = 1 ΠΌΠ”ΠΆ/ΠΌ3?

  • На Π½Π΅ΠΌΠ°Π³Π½ΠΈΡ‚Π½Ρ‹ΠΉ Ρ†ΠΈΠ»ΠΈΠ½Π΄Ρ€ Π΄Π»ΠΈΠ½ΠΎΠΉ 50 см ΠΈ сСчСниСм 2 см2 Π½Π°ΠΌΠΎΡ‚Π°Π½ Π² ΠΎΠ΄ΠΈΠ½ слой ΠΏΡ€ΠΎΠ²ΠΎΠ΄ Ρ‚Π°ΠΊ, Ρ‡Ρ‚ΠΎ Π½Π° ΠΊΠ°ΠΆΠ΄Ρ‹ΠΉ сантимСтр Π΄Π»ΠΈΠ½Ρ‹ стСрТня прихо­дится 20 Π²ΠΈΡ‚ΠΊΠΎΠ². ΠžΠΏΡ€Π΅Π΄Π΅Π»ΠΈΡ‚ΡŒ ΡΠ½Π΅Ρ€Π³ΠΈΡŽ ΠΌΠ°Π³Π½ΠΈΡ‚Π½ΠΎΠ³ΠΎ поля солСноида, Ссли ΠΏΠΎ Π½Π΅ΠΌΡƒ Ρ‚Π΅Ρ‡Π΅Ρ‚ Ρ‚ΠΎΠΊ 1 А.

  • Π’Π½ΡƒΡ‚Ρ€ΠΈ солСноида Π΄Π»ΠΈΠ½ΠΎΠΉ 75 см ΠΈ сСчСниСм 5 см2 индукция ΠΌΠ°Π³Π½ΠΈΡ‚Π½ΠΎΒ­Π³ΠΎ поля Ρ€Π°Π²Π½Π° 0,02 Π’Π», На ΠΊΠ°ΠΆΠ΄Ρ‹ΠΉ сантимСтр Π΄Π»ΠΈΠ½Ρ‹ солСноида прихо­дится 25 Π²ΠΈΡ‚ΠΊΠΎΠ². ΠžΠΏΡ€Π΅Π΄Π΅Π»ΠΈΡ‚ΡŒ ΡΠ½Π΅Ρ€Π³ΠΈΡŽ ΠΌΠ°Π³Π½ΠΈΡ‚Π½ΠΎΠ³ΠΎ поля солСноида.

  • КакоС число Π²ΠΈΡ‚ΠΊΠΎΠ² ΠΈΠΌΠ΅Π΅Ρ‚ солСноид Π΄Π»ΠΈΠ½ΠΎΠΉ 0,4 ΠΌ ΠΈ Π΄ΠΈΠ°ΠΌΠ΅Ρ‚Ρ€ΠΎΠΌ 4 см, Ссли ΠΏΡ€ΠΈ Ρ‚ΠΎΠΊΠ΅ 2 А энСргия ΠΌΠ°Π³Π½ΠΈΡ‚Π½ΠΎΠ³ΠΎ поля Ρ€Π°Π²Π½Π° 105 Π”ΠΆ?

  • Π‘ΠΎΠ»Π΅Π½ΠΎΠΈΠ΄ содСрТит 1000 Π²ΠΈΡ‚ΠΊΠΎΠ², Π΅Π³ΠΎ Π΄ΠΈΠ°ΠΌΠ΅Ρ‚Ρ€ 3 см. Какова Π΄Π»ΠΈΠ½Π° солС­ноида, Ссли ΠΏΡ€ΠΈ Ρ‚ΠΎΠΊΠ΅ 10 А энСргия ΠΌΠ°Π³Π½ΠΈΡ‚Π½ΠΎΠ³ΠΎ поля составляСт 0,01 Π”ΠΆ?

  • Π’Π½ΡƒΡ‚Ρ€ΠΈ Π΄Π»ΠΈΠ½Π½ΠΎΠ³ΠΎ солСноида Π½Π°ΠΏΡ€ΡΠΆΠ΅Π½Π½ΠΎΡΡ‚ΡŒ ΠΌΠ°Π³Π½ΠΈΡ‚Π½ΠΎΠ³ΠΎ поля Ρ€Π°Π²Π½Π° 1000 А/ΠΌ. Число Π²ΠΈΡ‚ΠΊΠΎΠ², приходящССся Π½Π° ΠΊΠ°ΠΆΠ΄Ρ‹ΠΉ сантимСтр Π΄Π»ΠΈΠ½Ρ‹ со­лСноида, n = 10 см. Π§Π΅ΠΌΡƒ Ρ€Π°Π²Π½Π° ΠΏΠ»ΠΎΡ‚Π½ΠΎΡΡ‚ΡŒ энСргии ΠΌΠ°Π³Π½ΠΈΡ‚Π½ΠΎΠ³ΠΎ поля Π² солСноидС?

  • Π‘ΠΎΠ»Π΅Π½ΠΎΠΈΠ΄ содСрТит 100 Π²ΠΈΡ‚ΠΊΠΎΠ², Π΅Π³ΠΎ Π΄Π»ΠΈΠ½Π° 1 ΠΌ. Найти радиус солСнои­да, Ссли ΠΏΡ€ΠΈ Ρ‚ΠΎΠΊΠ΅ 2 А энСргия ΠΌΠ°Π³Π½ΠΈΡ‚Π½ΠΎΠ³ΠΎ поля составляСт 104 Π”ΠΆ.

  • ΠšΠΎΠ½Ρ‚ΡƒΡ€ ΠΈΠΌΠ΅Π΅Ρ‚ сопротивлСниС R = 2 Ом ΠΈ ΠΈΠ½Π΄ΡƒΠΊΡ‚ΠΈΠ²Π½ΠΎΡΡ‚ΡŒ L = 0,2 Π“Π½. ΠŸΠΎΡΡ‚Ρ€ΠΎΠΈΡ‚ΡŒ Π³Ρ€Π°Ρ„ΠΈΠΊ зависимости Ρ‚ΠΎΠΊΠ° I Π² ΠΊΠΎΠ½Ρ‚ΡƒΡ€Π΅ ΠΎΡ‚ Π²Ρ€Π΅ΠΌΠ΅Π½ΠΈ t, ΠΏΡ€ΠΎΡˆΠ΅Π΄Β­ΡˆΠ΅Π³ΠΎ с ΠΌΠΎΠΌΠ΅Π½Ρ‚Π° Π²ΠΊΠ»ΡŽΡ‡Π΅Π½ΠΈΡ Π² Ρ†Π΅ΠΏΡŒ Π­Π”Π‘, для ΠΈΠ½Ρ‚Π΅Ρ€Π²Π°Π»Π° 0≀ t ≀ 0,5 с Ρ‡Π΅Ρ€Π΅Π· ΠΊΠ°ΠΆΠ΄ΡƒΡŽ 0,1 с. По оси ΠΎΡ€Π΄ΠΈΠ½Π°Ρ‚ ΠΎΡ‚ΠΊΠ»Π°Π΄Ρ‹Π²Π°Ρ‚ΡŒ ΠΎΡ‚Π½ΠΎΡˆΠ΅Π½ΠΈΠ΅ Π½Π°Ρ€Π°ΡΡ‚Π°ΡŽΡ‰Π΅Π³ΠΎ Ρ‚ΠΎΠΊΠ° I ΠΊ ΠΊΠΎΠ½Π΅Ρ‡Π½ΠΎΠΌΡƒ Ρ‚ΠΎΠΊΡƒ I0.

  • Π”Π²Π΅ ΠΊΠ°Ρ‚ΡƒΡˆΠΊΠΈ ΠΈΠΌΠ΅ΡŽΡ‚ Π²Π·Π°ΠΈΠΌΠ½ΡƒΡŽ ΠΈΠ½Π΄ΡƒΠΊΡ‚ΠΈΠ²Π½ΠΎΡΡ‚ΡŒ L12 = 5ΠΌΠ“Π½. Π’ ΠΏΠ΅Ρ€Π²ΠΎΠΉ ΠΊΠ°Β­Ρ‚ΡƒΡˆΠΊΠ΅ Ρ‚ΠΎΠΊ измСняСтся ΠΏΠΎ Π·Π°ΠΊΠΎΠ½Ρƒ I = IosinΟ‰t, Π³Π΄Π΅ Io = 10 A, Ο‰ = 2Ο€/Π’ ΠΈ Π’ = 0,02 с. Найти Π·Π°Π²ΠΈΡΠΈΠΌΠΎΡΡ‚ΡŒ ΠΎΡ‚ Π²Ρ€Π΅ΠΌΠ΅Π½ΠΈ t Π­Π”Π‘ Ξ΅2, ΠΈΠ½Π΄ΡƒΡ†ΠΈΡ€ΡƒΠ΅ΠΌΠΎΠΉ Π²ΠΎ Π²Ρ‚ΠΎΒ­Ρ€ΠΎΠΉ ΠΊΠ°Ρ‚ΡƒΡˆΠΊΠ΅, ΠΈ наибольшСС Π·Π½Π°Ρ‡Π΅Π½ΠΈΠ΅ этой Π­Π”Π‘.

  • ΠŸΠΎΡ‚ΠΎΠΊ ΠΌΠ°Π³Π½ΠΈΡ‚Π½ΠΎΠΉ ΠΈΠ½Π΄ΡƒΠΊΡ†ΠΈΠΈ Ρ‡Π΅Ρ€Π΅Π· ΠΏΠ»ΠΎΡ‰Π°Π΄ΡŒ ΠΏΠΎΠΏΠ΅Ρ€Π΅Ρ‡Π½ΠΎΠ³ΠΎ сСчСния ΠΊΠ°Β­Ρ‚ΡƒΡˆΠΊΠΈ, ΠΈΠΌΠ΅ΡŽΡ‰Π΅ΠΉ 1000 Π²ΠΈΡ‚ΠΊΠΎΠ², измСнился Π½Π° 0,002 Π’Π± Π² Ρ€Π΅Π·ΡƒΠ»ΡŒΡ‚Π°Ρ‚Π΅ измС­нСния Ρ‚ΠΎΠΊΠ° Π² ΠΊΠ°Ρ‚ΡƒΡˆΠΊΠ΅ с 4 Π΄ΠΎ 20 А. ΠžΠΏΡ€Π΅Π΄Π΅Π»ΠΈΡ‚ΡŒ коэффициСнт Π²Π·Π°ΠΈΠΌΠ½ΠΎΠΉ ΠΈΠ½Π΄ΡƒΠΊΡ†ΠΈΠΈ ΠΊΠ°Ρ‚ΡƒΡˆΠ΅ΠΊ.

  • Π”Π²Π΅ ΠΊΠ°Ρ‚ΡƒΡˆΠΊΠΈ располоТСны Π½Π° нСбольшом расстоянии ΠΎΠ΄Π½Π° ΠΎΡ‚ Π΄Ρ€ΡƒΠ³ΠΎΠΉ. Когда сила Ρ‚ΠΎΠΊΠ° Π² ΠΏΠ΅Ρ€Π²ΠΎΠΉ ΠΊΠ°Ρ‚ΡƒΡˆΠΊΠ΅ измСняСтся с быстротой 5 А/с, Π²ΠΎ Π²Ρ‚ΠΎΒ­Ρ€ΠΎΠΉ ΠΊΠ°Ρ‚ΡƒΡˆΠΊΠ΅ Π²ΠΎΠ·Π½ΠΈΠΊΠ°Π΅Ρ‚ Π­Π”Π‘ ΠΈΠ½Π΄ΡƒΠΊΡ†ΠΈΠΈ 0,1 Π’. ΠžΠΏΡ€Π΅Π΄Π΅Π»ΠΈΡ‚ΡŒ коэффициСнт Π²Π·Π°ΠΈΠΌΠ½ΠΎΠΉ ΠΈΠ½Π΄ΡƒΠΊΡ†ΠΈΠΈ ΠΊΠ°Ρ‚ΡƒΡˆΠ΅ΠΊ.

  • Π˜Π½Π΄ΡƒΠΊΡ‚ΠΈΠ²Π½ΠΎΡΡ‚ΡŒ ΠΊΠ°Ρ‚ΡƒΡˆΠΊΠΈ 2 ΠΌΠ“Π½. Π’ΠΎΠΊ частотой 50 Π“Ρ†, ΠΏΡ€ΠΎΡ‚Π΅ΠΊΠ°ΡŽΡ‰ΠΈΠΉ ΠΏΠΎ ΠΊΠ°Ρ‚ΡƒΡˆΠΊΠ΅, измСняСтся ΠΏΠΎ ΡΠΈΠ½ΡƒΡΠΎΠΈΠ΄Π°Π»ΡŒΠ½ΠΎΠΌΡƒ Π·Π°ΠΊΠΎΠ½Ρƒ. Π§Π΅ΠΌΡƒ Ρ€Π°Π²Π½ΠΎ срСднСС Π·Π½Π°Ρ‡Π΅Π½ΠΈΠ΅ Π­Π”Π‘ самоиндукции, Π²ΠΎΠ·Π½ΠΈΠΊΠ°ΡŽΡ‰Π΅ΠΉ Π·Π° ΠΈΠ½Ρ‚Π΅Ρ€Π²Π°Π» Π²Ρ€Π΅ΠΌΠ΅Π½ΠΈ, Π² Ρ‚Π΅Β­Ρ‡Π΅Π½ΠΈΠ΅ ΠΊΠΎΡ‚ΠΎΡ€ΠΎΠ³ΠΎ Ρ‚ΠΎΠΊ Π² ΠΊΠ°Ρ‚ΡƒΡˆΠΊΠ΅ измСняСтся ΠΎΡ‚ минимального Π΄ΠΎ макси­мального значСния? АмплитудноС Π·Π½Π°Ρ‡Π΅Π½ΠΈΠ΅ силы Ρ‚ΠΎΠΊΠ° 10 А.

  • Π‘ΠΎΠ»Π΅Π½ΠΎΠΈΠ΄ содСрТит 1000 Π²ΠΈΡ‚ΠΊΠΎΠ². Π‘Π΅Ρ‡Π΅Π½ΠΈΠ΅ солСноида 10 см2. По ΠΎΠ±Β­ΠΌΠΎΡ‚ΠΊΠ΅ Ρ‚Π΅Ρ‡Π΅Ρ‚ Ρ‚ΠΎΠΊ, ΡΠΎΠ·Π΄Π°ΡŽΡ‰ΠΈΠΉ ΠΏΠΎΠ»Π΅ с ΠΈΠ½Π΄ΡƒΠΊΡ†ΠΈΠ΅ΠΉ 1,5 Π’Π». Найти срСднСС Π·Π½Π°Ρ‡Π΅Π½ΠΈΠ΅ Π­Π”Π‘, которая Π²ΠΎΠ·Π½ΠΈΠΊΠ°Π΅Ρ‚ Π² солСноидС, Ссли Ρ‚ΠΎΠΊ ΡƒΠΌΠ΅Π½ΡŒΡˆΠΈΡ‚ΡΡ Π΄ΠΎ нуля Π·Π° 500 мкс.

  • ΠžΠ±ΠΌΠΎΡ‚ΠΊΠ° солСноида состоит ΠΈΠ· ΠΎΠ΄Π½ΠΎΠ³ΠΎ слоя ΠΏΠ»ΠΎΡ‚Π½ΠΎ ΠΏΡ€ΠΈΠ»Π΅Π³Π°ΡŽΡ‰ΠΈΡ… Π΄Ρ€ΡƒΠ³ ΠΊ Π΄Ρ€ΡƒΠ³Ρƒ Π²ΠΈΡ‚ΠΊΠΎΠ² ΠΌΠ΅Π΄Π½ΠΎΠ³ΠΎ ΠΏΡ€ΠΎΠ²ΠΎΠ΄Π°. Π”ΠΈΠ°ΠΌΠ΅Ρ‚Ρ€ ΠΏΡ€ΠΎΠ²ΠΎΠ΄Π° 0,2 ΠΌΠΌ, Π΄ΠΈΠ°ΠΌΠ΅Ρ‚Ρ€ со­лСноида 5 см. По солСноиду Ρ‚Π΅Ρ‡Π΅Ρ‚ Ρ‚ΠΎΠΊ 1 А. ΠžΠΏΡ€Π΅Π΄Π΅Π»ΠΈΡ‚ΡŒ, ΠΊΠ°ΠΊΠΎΠ΅ количСст­во элСктричСства ΠΏΡ€ΠΎΡ‚Π΅Ρ‡Π΅Ρ‚ Ρ‡Π΅Ρ€Π΅Π· ΠΎΠ±ΠΌΠΎΡ‚ΠΊΡƒ, Ссли ΠΊΠΎΠ½Ρ†Ρ‹ Π΅Π΅ Π·Π°ΠΌΠΊΠ½ΡƒΡ‚ΡŒ Π½Π°ΠΊΠΎΒ­Ρ€ΠΎΡ‚ΠΊΠΎ. Π’ΠΎΠ»Ρ‰ΠΈΠ½ΠΎΠΉ изоляции ΠΏΡ€Π΅Π½Π΅Π±Ρ€Π΅Ρ‡ΡŒ.

  • Π”Π²Π΅ ΠΊΠ°Ρ‚ΡƒΡˆΠΊΠΈ Π½Π°ΠΌΠΎΡ‚Π°Π½Ρ‹ Π½Π° ΠΎΠ±Ρ‰ΠΈΠΉ сСрдСчник. Π˜Π½Π΄ΡƒΠΊΡ‚ΠΈΠ²Π½ΠΎΡΡ‚ΡŒ ΠΏΠ΅Ρ€Π²ΠΎΠΉ ΠΊΠ°Β­Ρ‚ΡƒΡˆΠΊΠΈ 0,2 Π“Π½, Π²Ρ‚ΠΎΡ€ΠΎΠΉ — 0,8 Π“Π½. Π‘ΠΎΠΏΡ€ΠΎΡ‚ΠΈΠ²Π»Π΅Π½ΠΈΠ΅ Π²Ρ‚ΠΎΡ€ΠΎΠΉ ΠΊΠ°Ρ‚ΡƒΡˆΠΊΠΈ 600 Ом. Какой Ρ‚ΠΎΠΊ ΠΏΠΎΡ‚Π΅Ρ‡Π΅Ρ‚ ΠΏΠΎ Π²Ρ‚ΠΎΡ€ΠΎΠΉ ΠΊΠ°Ρ‚ΡƒΡˆΠΊΠ΅, Ссли Ρ‚ΠΎΠΊ I1= 0,3 А, Ρ‚Π΅ΠΊΡƒΡ‰ΠΈΠΉ Π² ΠΏΠ΅Ρ€Β­Π²ΠΎΠΉ ΠΊΠ°Ρ‚ΡƒΡˆΠΊΠ΅, Π²Ρ‹ΠΊΠ»ΡŽΡ‡ΠΈΡ‚ΡŒ Π² Ρ‚Π΅Ρ‡Π΅Π½ΠΈΠ΅ 1 мс ?

  • 1 .Π’ΠΎΠ»ΡŒΠΊΠ΅Π½ΡˆΡ‚Π΅ΠΉΠ½ B.C. Π‘Π±ΠΎΡ€Π½ΠΈΠΊ Π·Π°Π΄Π°Ρ‡ ΠΏΠΎ ΠΎΠ±Ρ‰Π΅ΠΌΡƒ курсу Ρ„ΠΈΠ·ΠΈΠΊΠΈ. — М.: Нау­ка. 1985Π³.- 550 с.

    2.Π§Π΅Ρ€Ρ‚ΠΎΠ² А.Π“., Π’ΠΎΡ€ΠΎΠ±ΡŒΠ΅Π² А.А.,. Π—Π°Π΄Π°Ρ‡Π½ΠΈΠΊ ΠΏΠΎ Ρ„ΠΈΠ·ΠΈΠΊΠ΅. — М.: Π’Ρ‹ΡΡˆ. шк. 1973. — 512 с.

    3. Π”Π΅Ρ‚Π»Π°Ρ„ А.А. ΠšΡƒΡ€Ρ Ρ„ΠΈΠ·ΠΈΠΊΠΈ, Ρ‚. 2. — М.: Наука. 2010. — 455с.

    studfiles.net